Você está na página 1de 250

Sara Raposo

Carlos Pires

FILOSOFIA
11.º ANO

www.duvida
metodica11 .te.pt
Caros colegas,
Este Manual teve como ponto de partida o • Relembra ideias-chave – síntese dos aspetos
blogue «Dúvida Metódica», criado por nós em fundamentais de cada um dos capítulos;
2008. Nessa altura a nossa intenção foi mos- • Discute – debate, a propósito de situações con-
trar como a Filosofia pode ser uma disciplina cretas apresentadas em imagens ou textos,
apelativa e útil. Essa mesma intenção também dos problemas filosóficos estudados, tendo em
nos guiou agora ao conceber este Manual. vista o posicionamento pessoal e crítico dos
E, tal como acontecia no blogue, diversos alunos (que podem trabalhar individualmente,
recursos disponibilizados foram aplicados na em pares ou em grupo alargado);
sala de aula com os nossos alunos e devem • Visiona – pequenos vídeos explicativos, filmes
muito ao que aprendemos na interação com ou documentários acerca dos problemas de
eles. cada capítulo, acompanhados de um guião e
Há, como é natural, muitas formas de enten- de propostas de resolução;
der o ensino da Filosofia, mas em todas elas se • Analisa textos – textos, numa linguagem
valoriza o pensamento crítico e a discussão de acessível aos alunos, acompanhados de um
problemas. Para nós, como autores e profes- guião com questões de análise e aplicação;
sores, estas são ideias básicas em que se tem • Aprender lá fora – sugestões de atividades,
alicerçado o nosso trabalho. Preocupámo-nos de natureza interdisciplinar e de aplicação,
também com a utilização de uma linguagem realizáveis fora da sala de aula. Podem tam-
clara e acessível – que torne o discurso facil- bém ser integradas no âmbito da Cidadania
mente inteligível – e com a análise e discussão e Desenvolvimento e/ou de uma DAC;
de casos concretos a partir das teorias estuda-
• Põe-te à prova – questões relativas à totalida-
das, evidenciando a aplicabilidade da Filosofia
de dos conteúdos de cada capítulo, para que
às questões da vida. o aluno possa avaliar os seus conhecimentos.
Este Manual encontra-se organizado por rubri-
cas, idênticas em todos os capítulos, exceto no Três das ideias que nos orientaram na constru-
5 (tema/problema) devido à natureza peculiar ção deste Manual são:
dos conteúdos e dos objetivos pretendidos. As • apresentar, de forma sistemática, instrumen-
rubricas são as seguintes: tos facilitadores da compreensão: exemplos
• Situação inicial – um caso concreto apelati- ilustrativos, esquemas, quadros comparati-
vo que visa captar a atenção dos alunos; as vos, imagens e notas explicativas para alu-
questões orientadoras, sem grandes pres- nos e professores;
supostos, pretendem promover o interesse • utilizar, em todos os capítulos, recursos e ati-
pelo problema em causa e a reflexão; vidades promotoras da reflexão e do debate;
• Exercita – em diversos momentos de cada • proporcionar uma quantidade significativa
capítulo são propostos exercícios de aplica- de materiais didáticos a alunos e professo-
ção e de revisão dos conteúdos, com caráter res, distribuídos de forma organizada e equi-
formativo, para que o aluno verifique se está librada pelo Manual, Caderno do Aluno e
ou não a acompanhar a matéria; Dossiê do Professor.

© Texto | Dúvida Metódica, 11.º ano


O Caderno do Aluno teve como objetivo prin- por capítulo;
cipal ajudar os alunos a estudar melhor, explo- • Notas explicativas para todos os capítulos
rando de forma autónoma outros recursos e do Manual que esclarecem, desenvolvem ou
informações, além dos sugeridos no Manual, e aprofundam certos conteúdos filosóficos;
fornecendo algumas sugestões práticas e infor-
• Tipologia de questões: características e
mações (como fazer um ensaio argumentativo
exemplificação dos vários tipos de itens;
ou responder a diferentes tipos de perguntas em
Filosofia). Contém 20 fichas de trabalho acom- • Relembra Ideias-chave de 10.º ano para aju-
panhadas de propostas de resolução; textos dar os alunos a rever mais facilmente os
complementares e informações (para o aluno conteúdos do Exame Nacional de Filosofia e
explorar de forma autónoma, de acordo com os facilitar a sua preparação;
seus interesses); indicações para aceder à Aula • Outra Maneira de Preparar os Exames: um
Digital e a informação sobre os recursos dis- exemplo, que será complementado em Aula
ponibilizados aos alunos (por exemplo: Áudio- Digital;
-resumos e Códigos QR); resumo das ideias-chave • Prova com a estrutura da Prova de Exame
do 10.º ano; duas provas com a estrutura do Nacional;
Exame Nacional com respostas; sugestões de
• Avaliação por competências: exemplos;
Bibliografia e Webgrafia (pensadas para alunos
do ensino secundário que querem saber mais). • Bibliografia e Webgrafia que permitem apro-
fundar e pesquisar outras informações sobre
os temas de todos os capítulos;
Pensamos que os materiais do Dossiê do
• Em Aula Digital, serão disponibilizados dois
Professor devem ter uma natureza, sobretudo,
temas/problemas alternativos ao do Manual:
prática. Por isso, apresentamos uma grande
Guerra justa e Tecnociência;
diversidade de sugestões de atividades e recur-
sos (incidindo em competências diferentes) para • Em Aula Digital, serão disponibilizadas plani-
implementar com os alunos na sala de aula e ficações (anual e por tema/capítulo).
para dar aos professores possibilidades alternati-
vas relativamente aos instrumentos de avaliação: Quanto aos instrumentos de avaliação, procu-
rámos apresentar um vasto leque de tarefas
• Questões-aula + Propostas de resolução;
que apelassem a competências de natureza
• Banco de questões + Propostas de resolução;
diferente, com graus de complexidade varia-
• Testes + Propostas de resolução + Guião de dos e abrangendo competências especifica-
estudo para os testes; mente filosóficas, como a problematização,
• Ensaio filosófico; a conceptualização e a argumentação (de
• Como articular Cidadania e Desenvolvimen- acordo com o que se encontra definido nas
to, Filosofia e outras disciplinas? – quadro Aprendizagens Essenciais), bem como as áreas
semelhante ao do projeto do 10.º ano, que de competências elencadas no Perfil dos
será disponibilizado em Aula Digital; Alunos à Saída da escolaridade Obrigatória,
• Textos complementares para todos os capí- entre outras, «O pensamento crítico e criativo»
tulos do Manual, complementados por outros e «O desenvolvimento pessoal e autonomia».
textos a disponibilizar em Aula Digital; Nas Questões-aula (duas a seis por cada um
• Guião de recursos multimédia e Roteiro de dos 4 capítulos, num total de 15), as tarefas pro-
utilização da Aula Digital; postas podem ser realizadas individualmente
• Respostas de todas as rubricas do manual: ou em grupo e adaptadas ao perfil das turmas
Situação inicial; Exercita; Discute; Visiona; e dos alunos. No Banco de questões existe,
Analisa textos e Põe-te à prova, organizadas para cada capítulo, uma grande quantidade e

© Texto | Dúvida Metódica, 11.º ano


diversidade de recursos, de modo a que os • Banco de esquemas (em versão interativa)
colegas tenham margem de escolha. e também de imagens e cartoons
Incluímos também um teste por capítulo, (disponibilizam-se todas as imagens
além de um teste diagnóstico (que permite a por capítulo e respetivas questões de
revisão de conteúdos do 10.º ano aplicáveis exploração, para apoiar a apresentação
no 11.º ano). dos conteúdos e momentos de debate na
São apresentadas Propostas de resolução sala de aula);
para todas as atividades e, no caso dos tes- • Vídeos curtos e trailers de filmes
tes, também guiões de estudo para os alu- (legendados em português) da rubrica
nos. Este procedimento pode ser adotado em Visiona;
qualquer atividade e não apenas nos testes.
• Recursos dos códigos QR do manual;
Por exemplo, no caso do Ensaio filosófico,
• Áudio-resumos dos principais conteúdos
também faz sentido comunicar previamente
de todos os capítulos;
aos alunos as regras de elaboração e os cri-
térios de avaliação. • Testes interativos (com 10 questões de
resposta fechada e com correção auto-
Quanto aos recursos multimédia disponíveis mática, incidindo nos principais conteúdos
online, na Aula Digital (para informações de cada capítulo);
mais detalhadas por capítulo, consultar o • Kahoot (1 por capítulo).
guião de recursos multimédia), destacamos:
• Animações: de todas as ocorrências da Os recursos aqui apresentados resultam
rubrica Situação Inicial; de passagens de em grande medida das aprendizagens que
textos; de exemplos; de um argumento e fomos fazendo com os nossos alunos e cole-
de um dilema moral; gas, ao longo de vários anos. Com esta parti-
lha esperamos contribuir para que cada um
• Jogo: Penso, logo acerto – jogo em que são
de vós descubra a melhor forma de tornar as
apresentadas questões e alternativas de
aulas estimulantes e pedagogicamente efi-
resposta fechada, e no qual o aluno pode
cazes. Dito por outras palavras: esperamos
aceder, como ajuda, a dicas de filósofos;
ajudar a fazer aquilo que todos nós, professo-
• Polígrafo filosófico: animações interativas,
res e autores, procuramos fazer diariamente
para todos os capítulos, que consistem
desde que nos dedicámos a esta profissão.
na apresentação de afirmações de
Contamos com o vosso feedback para conti-
diferentes filósofos, que serão validadas
nuar a aprender.
ou desmentidas pelo polígrafo filosófico;
Votos de um bom trabalho!
• Apresentações, em formato PowerPoint,
dos principais conteúdos dos capítulos; Os autores

© Texto | Dúvida Metódica, 11.º ano


Índice
1. Questões-aula + Propostas de resolução . . . . . . . . . . . . . . . . . . . . . . . . . . . . . . . . . . . . . . . . . . . . . . . . . . . . . . 7
Cap. 1 ǝ,ŔĕŜťøİĺĩĺČĕÑǝĺŜŔŗĺêĩøİÑŜñÑñøƧıĕîçĺLjñÑŔĺŜŜĕêĕĩĕñÑñø
øñÑĺŗĕČøİñĺëĺıđøëĕİøıťĺ . . . . . . . . . . . . . . . . . . . . . . . . . . . . . . . . . . . . . . . . . . . . . . . . . . . . . . . 9
Cap. 2 ǝ>ĕĩĺŜĺƧÑñÑëĕüıëĕÑǝĺŜŔŗĺêĩøİÑŜñÑñøİÑŗëÑîçĺLjñĺİùťĺñĺLj
ñÑøžĺĩūîçĺøñÑĺêģøťĕžĕñÑñøñÑëĕüıëĕÑ . . . . . . . . . . . . . . . . . . . . . . . . . . . . . . . . . . . . . . . . . . . . 16
Cap. 3 ǝ>ĕĩĺŜĺƧÑñÑÑŗťøǝĺŔŗĺêĩøİÑñÑñøƧıĕîçĺñøÑŗťø . . . . . . . . . . . . . . . . . . . . . . . . . . . . . . . . . . . 24
Cap. 4 ǝ>ĕĩĺŜĺƧÑñÑŗøĩĕČĕçĺǝĺŔŗĺêĩøİÑñÑøƄĕŜťüıëĕÑñø#øūŜ. . . . . . . . . . . . . . . . . . . . . . . . . . . . . . 27

Propostas de resolução . . . . . . . . . . . . . . . . . . . . . . . . . . . . . . . . . . . . . . . . . . . . . . . . . . . . . . . . . . . . . . . . . . . . . . . 29

2. Banco de questões + Propostas de resolução. . . . . . . . . . . . . . . . . . . . . . . . . . . . . . . . . . . . . . . . . . . . . . . . . . 35


Cap. 1 ǝ,ŔĕŜťøİĺĩĺČĕÑǝĺŜŔŗĺêĩøİÑŜñÑñøƧıĕîçĺLjñÑŔĺŜŜĕêĕĩĕñÑñø
øñÑĺŗĕČøİñĺëĺıđøëĕİøıťĺ . . . . . . . . . . . . . . . . . . . . . . . . . . . . . . . . . . . . . . . . . . . . . . . . . . . . . . . 37
Cap. 2 ǝ>ĕĩĺŜĺƧÑñÑëĕüıëĕÑǝĺŜŔŗĺêĩøİÑŜñÑñøİÑŗëÑîçĺLjñĺİùťĺñĺLj
ñÑøžĺĩūîçĺøñÑĺêģøťĕžĕñÑñøñÑëĕüıëĕÑ . . . . . . . . . . . . . . . . . . . . . . . . . . . . . . . . . . . . . . . . . . . . 50
Cap. 3 ǝ>ĕĩĺŜĺƧÑñÑÑŗťøǝĺŔŗĺêĩøİÑñÑñøƧıĕîçĺñøÑŗťø . . . . . . . . . . . . . . . . . . . . . . . . . . . . . . . . . . . 64
Cap. 4 ǝ>ĕĩĺŜĺƧÑñÑŗøĩĕČĕçĺǝĺŔŗĺêĩøİÑñÑøƄĕŜťüıëĕÑñø#øūŜ. . . . . . . . . . . . . . . . . . . . . . . . . . . . . . 71

Propostas de resolução . . . . . . . . . . . . . . . . . . . . . . . . . . . . . . . . . . . . . . . . . . . . . . . . . . . . . . . . . . . . . . . . . . . . . . . 79

3. Testes + Propostas de resolução + Guiões de estudo . . . . . . . . . . . . . . . . . . . . . . . . . . . . . . . . . . . . . . . . . . 83


Teste 1 –øŜťøñĕÑČıĻŜťĕëĺ . . . . . . . . . . . . . . . . . . . . . . . . . . . . . . . . . . . . . . . . . . . . . . . . . . . . . . . . . . . . . . . . . . . . 85
…ŗĺŔĺŜťÑñøŗøŜĺĩūîçĺñĺťøŜťøñĕÑČıĻŜťĕëĺ . . . . . . . . . . . . . . . . . . . . . . . . . . . . . . . . . . . . . . . . . . . . . . . . . . 89
Teste 2 Epistemologia . . . . . . . . . . . . . . . . . . . . . . . . . . . . . . . . . . . . . . . . . . . . . . . . . . . . . . . . . . . . . . . . . . . . . . . 91
…ŗĺŔĺŜťÑñøŗøŜĺĩūîçĺñĺťøŜťøƱ . . . . . . . . . . . . . . . . . . . . . . . . . . . . . . . . . . . . . . . . . . . . . . . . . . . . . . . . . . . . . 93
Teste 3>ĕĩĺŜĺƧÑñÑëĕüıëĕÑ. . . . . . . . . . . . . . . . . . . . . . . . . . . . . . . . . . . . . . . . . . . . . . . . . . . . . . . . . . . . . . . . . . . 95
…ŗĺŔĺŜťÑñøŗøŜĺĩūîçĺñĺťøŜťøƲ . . . . . . . . . . . . . . . . . . . . . . . . . . . . . . . . . . . . . . . . . . . . . . . . . . . . . . . . . . . . . 99
Teste 4>ĕĩĺŜĺƧÑñÑÑŗťø . . . . . . . . . . . . . . . . . . . . . . . . . . . . . . . . . . . . . . . . . . . . . . . . . . . . . . . . . . . . . . . . . . . . . . 101
…ŗĺŔĺŜťÑñøŗøŜĺĩūîçĺñĺťøŜťøƳ . . . . . . . . . . . . . . . . . . . . . . . . . . . . . . . . . . . . . . . . . . . . . . . . . . . . . . . . . . . . . 103
Teste 5>ĕĩĺŜĺƧÑñÑŗøĩĕČĕçĺ . . . . . . . . . . . . . . . . . . . . . . . . . . . . . . . . . . . . . . . . . . . . . . . . . . . . . . . . . . . . . . . . . . 105
…ŗĺŔĺŜťÑñøŗøŜĺĩūîçĺñĺťøŜťøƴ . . . . . . . . . . . . . . . . . . . . . . . . . . . . . . . . . . . . . . . . . . . . . . . . . . . . . . . . . . . . . 107

Guiões de estudo . . . . . . . . . . . . . . . . . . . . . . . . . . . . . . . . . . . . . . . . . . . . . . . . . . . . . . . . . . . . . . . . . . . . . . . . . . . . . 108

ƳǍ,ıŜÑĕĺƧĩĺŜĻƧëĺ . . . . . . . . . . . . . . . . . . . . . . . . . . . . . . . . . . . . . . . . . . . . . . . . . . . . . . . . . . . . . . . . . . . . . . . . . . . . . . 113

5. Cidadania e Desenvolvimento . . . . . . . . . . . . . . . . . . . . . . . . . . . . . . . . . . . . . . . . . . . . . . . . . . . . . . . . . . . . . . . . 119


ĺİĺÑŗťĕëūĩÑŗĕñÑñÑıĕÑø#øŜøıžĺĩžĕİøıťĺLj>ĕĩĺŜĺƧÑøĺūťŗÑŜñĕŜĕëĕŔĩĕıÑŜǎǚøİūĩÑ#ĕČĕťÑĩǛ
ŔŗøıñøŗYÒ>ĺŗÑǚøİūĩÑ#ĕČĕťÑĩǛ
6. Textos complementares . . . . . . . . . . . . . . . . . . . . . . . . . . . . . . . . . . . . . . . . . . . . . . . . . . . . . . . . . . . . . . . . . . . . . . 121
Capítulo 1ǝ,ŔĕŜťøİĺĩĺČĕÑǝĺŜŔŗĺêĩøİÑŜñÑñøƧıĕîçĺLjñÑŔĺŜŜĕêĕĩĕñÑñøøñÑĺŗĕČøİ
ñĺëĺıđøëĕİøıťĺ . . . . . . . . . . . . . . . . . . . . . . . . . . . . . . . . . . . . . . . . . . . . . . . . . . . . . . . . . . . . . . . . . . . . . . . . . . . . 122
Capítulo 2ǝ>ĕĩĺŜĺƧÑñÑëĕüıëĕÑǝĺŜŔŗĺêĩøİÑŜñÑñøİÑŗëÑîçĺLjñĺİùťĺñĺLjñÑøžĺĩūîçĺ
øñÑĺêģøťĕžĕñÑñøñÑëĕüıëĕÑ . . . . . . . . . . . . . . . . . . . . . . . . . . . . . . . . . . . . . . . . . . . . . . . . . . . . . . . . . . . . . . . . . 128
Capítulo 3ǝ>ĕĩĺŜĺƧÑñÑÑŗťøǝĺŔŗĺêĩøİÑñÑñøƧıĕîçĺñøÑŗťø . . . . . . . . . . . . . . . . . . . . . . . . . . . . . . 132
Capítulo 4ǝ>ĕĩĺŜĺƧÑñÑŗøĩĕČĕçĺǝĺŔŗĺêĩøİÑñÑøƄĕŜťüıëĕÑñø#øūŜ . . . . . . . . . . . . . . . . . . . . . . . . . 134
Capítulo 5ǝ–øİÑǓŔŗĺêĩøİÑLJĩøČĕťĕİĕñÑñøİĺŗÑĩñÑøƄŔøŗĕİøıťÑîçĺÑıĕİÑĩ . . . . . . . . . . . . . . . 135

© Texto | #ŬžĕñÑaøťĻñĕëÑ, 11.º ano


7. Ensino Digital . . . . . . . . . . . . . . . . . . . . . . . . . . . . . . . . . . . . . . . . . . . . . . . . . . . . . . . . . . . . . . . . . . . . . . . . . . . . . . . . 141
Guião de recursos digitais do projeto Dúvida Metódica . . . . . . . . . . . . . . . . . . . . . . . . . . . . . . . . . . . . . . . 142
Roteiro Aula Digital . . . . . . . . . . . . . . . . . . . . . . . . . . . . . . . . . . . . . . . . . . . . . . . . . . . . . . . . . . . . . . . . . . . . . . . . . . . 151

8. Respostas das atividades de todas as rubricas do Manual . . . . . . . . . . . . . . . . . . . . . . . . . . . . . . . . . . . . . 165


Cap. 1ǝ,ŔĕŜťøİĺĩĺČĕÑǝĺŜŔŗĺêĩøİÑŜñÑñøƧıĕîçĺLjñÑŔĺŜŜĕêĕĩĕñÑñø
øñÑĺŗĕČøİñĺëĺıđøëĕİøıťĺ
ĕťūÑîçĺĕıĕëĕÑĩ . . . . . . . . . . . . . . . . . . . . . . . . . . . . . . . . . . . . . . . . . . . . . . . . . . . . . . . . . . . . . . . . . . . . . . . . . . . . . . . 167
Exercita . . . . . . . . . . . . . . . . . . . . . . . . . . . . . . . . . . . . . . . . . . . . . . . . . . . . . . . . . . . . . . . . . . . . . . . . . . . . . . . . . . . . . . 167
Discute . . . . . . . . . . . . . . . . . . . . . . . . . . . . . . . . . . . . . . . . . . . . . . . . . . . . . . . . . . . . . . . . . . . . . . . . . . . . . . . . . . . . . . . 169
¯ĕŜĕĺıÑ . . . . . . . . . . . . . . . . . . . . . . . . . . . . . . . . . . . . . . . . . . . . . . . . . . . . . . . . . . . . . . . . . . . . . . . . . . . . . . . . . . . . . . . 170
ıÑĩĕŜÑťøƄťĺŜ . . . . . . . . . . . . . . . . . . . . . . . . . . . . . . . . . . . . . . . . . . . . . . . . . . . . . . . . . . . . . . . . . . . . . . . . . . . . . . . . 170
Põe-te à prova . . . . . . . . . . . . . . . . . . . . . . . . . . . . . . . . . . . . . . . . . . . . . . . . . . . . . . . . . . . . . . . . . . . . . . . . . . . . . . . 171

Cap. 2ǝ>ĕĩĺŜĺƧÑñÑëĕüıëĕÑǝĺŜŔŗĺêĩøİÑŜñÑñøİÑŗëÑîçĺLjñĺİùťĺñĺLjñÑøžĺĩūîçĺ
øñÑĺêģøťĕžĕñÑñøñÑëĕüıëĕÑ
2.1kŜŔŗĺêĩøİÑŜñÑñøİÑŗëÑîçĺøñĺİùťĺñĺ
ĕťūÑîçĺĕıĕëĕÑĩ . . . . . . . . . . . . . . . . . . . . . . . . . . . . . . . . . . . . . . . . . . . . . . . . . . . . . . . . . . . . . . . . . . . . . . . . . . . . . . 172
Exercita . . . . . . . . . . . . . . . . . . . . . . . . . . . . . . . . . . . . . . . . . . . . . . . . . . . . . . . . . . . . . . . . . . . . . . . . . . . . . . . . . . . . . 172
2.2kŜŔŗĺêĩøİÑŜñÑøžĺĩūîçĺøñÑĺêģøťĕžĕñÑñøñÑëĕüıëĕÑ. . . . . . . . . . . . . . . . . . . . . . . . . . . . . . . . . . . 173
ĕťūÑîçĺĕıĕëĕÑĩ . . . . . . . . . . . . . . . . . . . . . . . . . . . . . . . . . . . . . . . . . . . . . . . . . . . . . . . . . . . . . . . . . . . . . . . . . . . . . . 173
Exercita . . . . . . . . . . . . . . . . . . . . . . . . . . . . . . . . . . . . . . . . . . . . . . . . . . . . . . . . . . . . . . . . . . . . . . . . . . . . . . . . . . . . . . 173
Discute . . . . . . . . . . . . . . . . . . . . . . . . . . . . . . . . . . . . . . . . . . . . . . . . . . . . . . . . . . . . . . . . . . . . . . . . . . . . . . . . . . . . . . . 174
¯ĕŜĕĺıÑ . . . . . . . . . . . . . . . . . . . . . . . . . . . . . . . . . . . . . . . . . . . . . . . . . . . . . . . . . . . . . . . . . . . . . . . . . . . . . . . . . . . . . . . 175
ıÑĩĕŜÑťøƄťĺŜ . . . . . . . . . . . . . . . . . . . . . . . . . . . . . . . . . . . . . . . . . . . . . . . . . . . . . . . . . . . . . . . . . . . . . . . . . . . . . . . . 176
Põe-te à prova . . . . . . . . . . . . . . . . . . . . . . . . . . . . . . . . . . . . . . . . . . . . . . . . . . . . . . . . . . . . . . . . . . . . . . . . . . . . . . . 177

Cap. 3ǝ>ĕĩĺŜĺƧÑñÑÑŗťøǝĺŔŗĺêĩøİÑñÑñøƧıĕîçĺñøÑŗťø
ĕťūÑîçĺĕıĕëĕÑĩ . . . . . . . . . . . . . . . . . . . . . . . . . . . . . . . . . . . . . . . . . . . . . . . . . . . . . . . . . . . . . . . . . . . . . . . . . . . . . . . 178
Exercita . . . . . . . . . . . . . . . . . . . . . . . . . . . . . . . . . . . . . . . . . . . . . . . . . . . . . . . . . . . . . . . . . . . . . . . . . . . . . . . . . . . . . . 178
Discute . . . . . . . . . . . . . . . . . . . . . . . . . . . . . . . . . . . . . . . . . . . . . . . . . . . . . . . . . . . . . . . . . . . . . . . . . . . . . . . . . . . . . . . 179
¯ĕŜĕĺıÑ . . . . . . . . . . . . . . . . . . . . . . . . . . . . . . . . . . . . . . . . . . . . . . . . . . . . . . . . . . . . . . . . . . . . . . . . . . . . . . . . . . . . . . . 180
ıÑĩĕŜÑťøƄťĺŜ . . . . . . . . . . . . . . . . . . . . . . . . . . . . . . . . . . . . . . . . . . . . . . . . . . . . . . . . . . . . . . . . . . . . . . . . . . . . . . . . 181
Põe-te à prova . . . . . . . . . . . . . . . . . . . . . . . . . . . . . . . . . . . . . . . . . . . . . . . . . . . . . . . . . . . . . . . . . . . . . . . . . . . . . . . 182

Cap. 4ǝ>ĕĩĺŜĺƧÑñÑŗøĩĕČĕçĺǝĺŔŗĺêĩøİÑñÑøƄĕŜťüıëĕÑñø#øūŜ
ĕťūÑîçĺĕıĕëĕÑĩ . . . . . . . . . . . . . . . . . . . . . . . . . . . . . . . . . . . . . . . . . . . . . . . . . . . . . . . . . . . . . . . . . . . . . . . . . . . . . . . 183
Exercita . . . . . . . . . . . . . . . . . . . . . . . . . . . . . . . . . . . . . . . . . . . . . . . . . . . . . . . . . . . . . . . . . . . . . . . . . . . . . . . . . . . . . . 183
Discute . . . . . . . . . . . . . . . . . . . . . . . . . . . . . . . . . . . . . . . . . . . . . . . . . . . . . . . . . . . . . . . . . . . . . . . . . . . . . . . . . . . . . . . 185
¯ĕŜĕĺıÑ . . . . . . . . . . . . . . . . . . . . . . . . . . . . . . . . . . . . . . . . . . . . . . . . . . . . . . . . . . . . . . . . . . . . . . . . . . . . . . . . . . . . . . . 186
ıÑĩĕŜÑťøƄťĺŜ . . . . . . . . . . . . . . . . . . . . . . . . . . . . . . . . . . . . . . . . . . . . . . . . . . . . . . . . . . . . . . . . . . . . . . . . . . . . . . . . 187
Põe-te à prova . . . . . . . . . . . . . . . . . . . . . . . . . . . . . . . . . . . . . . . . . . . . . . . . . . . . . . . . . . . . . . . . . . . . . . . . . . . . . . . 187

9. Outros. . . . . . . . . . . . . . . . . . . . . . . . . . . . . . . . . . . . . . . . . . . . . . . . . . . . . . . . . . . . . . . . . . . . . . . . . . . . . . . . . . . . . . . . 189
bĺťÑŜøƄŔĩĕëÑťĕžÑŜǚŗøİĕŜŜŒøŜñĺİÑıūÑĩǛ . . . . . . . . . . . . . . . . . . . . . . . . . . . . . . . . . . . . . . . . . . . . . . . . . . . 191
Tipologia de questões . . . . . . . . . . . . . . . . . . . . . . . . . . . . . . . . . . . . . . . . . . . . . . . . . . . . . . . . . . . . . . . . . . . . . . 197
ˆøĩøİêŗÑĕñøĕÑŜǞëđÑžøñĺưƯǍƪÑıĺ . . . . . . . . . . . . . . . . . . . . . . . . . . . . . . . . . . . . . . . . . . . . . . . . . . . . . . . . . . 199
kūťŗÑaÑıøĕŗÑñø…ŗøŔÑŗÑŗĺŜ,ƄÑİøŜLJūİøƄøİŔĩĺ . . . . . . . . . . . . . . . . . . . . . . . . . . . . . . . . . . . . . . . . 213
…ŗĺžÑëĺİÑøŜťŗūťūŗÑñĺ,ƄÑİøbÑëĕĺıÑĩñø>ĕĩĺŜĺƧÑ . . . . . . . . . . . . . . . . . . . . . . . . . . . . . . . . . . . . . 215
ˆøŜĺĩūîçĺñÑ…ŗĺžÑëĺİÑøŜťŗūťūŗÑñĺ,ƄÑİøbÑëĕĺıÑĩñø>ĕĩĺŜĺƧÑ . . . . . . . . . . . . . . . . . . . . . . . 218
žÑĩĕÑîçĺŔĺŗëĺİŔøťüıëĕÑŜLJøƄøİŔĩĺŜ . . . . . . . . . . . . . . . . . . . . . . . . . . . . . . . . . . . . . . . . . . . . . . . . . . . . . 219
–øİÑŜǓŔŗĺêĩøİÑŜÑĩťøŗıÑťĕžĺŜLJ–øëıĺëĕüıëĕÑǚøİūĩÑ#ĕČĕťÑĩǛ
…ĩÑıĕƧëÑîŒøŜǚøİūĩÑ#ĕČĕťÑĩǛ
ĕêĩĕĺČŗÑƧÑø°øêČŗÑƧÑ. . . . . . . . . . . . . . . . . . . . . . . . . . . . . . . . . . . . . . . . . . . . . . . . . . . . . . . . . . . . . . . . . . . . 237

© Texto | Dúvida Metódica, 11.º ano


Questões-aula
QUESTÕES-AULA E
PROPOSTAS DE RESOLUÇÃO

Cada Questão-aula é constituída por três a cinco itens


que avaliam várias competências filosóficas.
São apresentadas quinze Questões-aula no total:
seis no capítulo 1, cinco no capítulo 2, duas no
capítulo 3 e duas no capítulo 4.
Esta atividade pode ser utilizada para diversificar
os instrumentos de avaliação, da forma que o
professor entender mais adequada do ponto de vista
pedagógico. Pode ser aplicada individualmente ou em
grupo. A duração e o número de questões podem ser
adaptados, nomeadamente ao perfil da turma.

© Texto | Dúvida Metódica, 11.º ano


1 Epistemologia – os problemas da definição, da possibilidade e da origem do conhecimento

QUESTÃO-AULA 1

1 . Depois de observar as imagens e ler o texto, responde às questões apresentadas.

«Imaginem uma brilhante neurocientista chamada Mary. Ela vive num quarto pre-
to e branco, só lê livros a preto e branco e os ecrãs que tem são só a preto e branco.
Apesar de nunca ter visto cores, Mary é uma especialista na visão a cores e sabe tudo
o que já foi descoberto sobre isso na Física e na Biologia. Sabe como as diferentes
ondas de comprimento da luz estimulam três tipos de células cónicas na retina e
sabe como os sinais elétricos viajam pelo nervo ótico até ao cérebro. Aí, criam pa-
drões de atividade neural que correspondem às muitas cores que quase todos os se-
res humanos distinguem. Imaginem, agora, que um dia o ecrã a preto e branco de
Mary se avaria e aparece uma maçã verde. Pela primeira vez, ela tem a experiência
de uma coisa que conhece há anos. Será que aprende uma coisa nova? Há alguma
coisa na perceção da cor que não tenha sido captada por todo o seu saber?»
A experiência mental «O Quarto de Mary», do filósofo Frank Jackson,
in https://ed.ted.com/lessons/mary-s-room-a-philosophical-thought-experiment-eleanor-nelsen#watch
(adaptado)

1.1 Antes de ver no ecrã do seu computador uma maçã verde, que tipo de conhecimento
possuía a Mary acerca das cores? Justifica.
1.2 Podemos afirmar que a Mary, ao ver a maçã verde, conseguiu ter um tipo de
conhecimento diferente sobre as cores? Porquê?

2 . Tendo em conta a definição tradicional de conhecimento, analisa as alíneas A a F e


responde às questões apresentadas.
A. Existem vídeos no Youtube em que algumas pessoas dizem ter conhecimento de
que a Terra é plana e até explicam porquê.
B. Certos povos sabem, há já vários séculos, que existem espíritos malignos.
C. Muitos jornalistas, de todo o mundo, informaram (no dia 24 de fevereiro de 2022)
que um país chamado Rússia invadiu um país chamado Ucrânia.
D. O Manuel sabia, antes de atirar uma moeda ao ar, que esta ia cair com a coroa
para cima (o que viria a acontecer de facto).
E. Certos cientistas conhecem a composição química das bombas atómicas e
conseguem explicar os malefícios provocados pela sua utilização numa guerra.
F. A Maria, mesmo antes do jogo de futebol com a sua equipa favorita ter começado,
sabe que esta vai ganhar (o que acabou por suceder).

2.1 Em que alíneas se pode afirmar que o sujeito possui conhecimento? Justifica.

2.2 Em que alíneas se pode afirmar que o sujeito não possui conhecimento? Justifica.

Editável e fotocopiável © Texto | Dúvida Metódica, 11.º ano 9


1 Epistemologia – os problemas da definição, da possibilidade e da origem do conhecimento

QUESTÃO-AULA 2
Após o visionamento do vídeo, da autoria do filósofo inglês Nigel Warburton, responde às
questões apresentadas.

«Aparência e realidade»
Duração: 5:23 Legendado em português

1 . Dá dois exemplos que mostrem a falibilidade dos nossos sentidos.

2 . Qual é o ponto de vista de Platão, expresso na «Alegoria da caverna», acerca do


conhecimento empírico? Descartes concorda com ele? Porquê?

3 . O argumento do sonho não nos permite duvidar de que tipo de conhecimento?

4 . A experiência mental do «cérebro numa cuba» é comparável a que argumento cartesiano?


Porquê?

5 . Descartes considera que conseguiu ultrapassar a dúvida? Porquê?

10 Editável e fotocopiável © Texto | Dúvida Metódica, 11.º ano


1 Epistemologia – os problemas da definição, da possibilidade e da origem do conhecimento

QUESTÃO-AULA 3
As três figuras seguintes procuram representar uma parte do percurso da dúvida metódica, de
acordo com a descrição feita por Descartes.

1 . Analisa com atenção as figuras 1, 2 e 3 e responde às questões apresentadas.

Deus

Mesa 2+2=4
Génio maligno
Cogito

Namorada Sereia

FIG. 1

Deus

Mesa 2+2=4
Cogito
Génio maligno

Namorada Sereia

FIG. 2

Editável e fotocopiável © Texto | Dúvida Metódica, 11.º ano 11


1 Epistemologia – os problemas da definição, da possibilidade e da origem do conhecimento

Deus

Mesa 2+2=4
Génio maligno Cogito

Namorada Sereia

FIG. 3

1.1 Por que razão, na figura 1, todas as ideias, com exceção do cogito, se encontram
tracejadas?

1.2 Por que razão, na figura 2, a ideia de Deus já não se encontra tracejada?

1.3 Porque é que, na figura 3, o génio maligno surge traçado com um X?

1.4 Porque é que, na figura 3, ideias como «2 + 2 = 4» e «Mesa» não se encontram


tracejadas, ao contrário da ideia de «Sereia», que, além de tracejada, surge traçada
com um X?

12 Editável e fotocopiável © Texto | Dúvida Metódica, 11.º ano


1 Epistemologia – os problemas da definição, da possibilidade e da origem do conhecimento

QUESTÃO-AULA 4
Após o visionamento do vídeo sobre Descartes «Podemos estar certos de alguma coisa?»,
responde às questões apresentadas.

«Podemos estar certos de alguma coisa?»


Duração: 2:44 Legendado em português

1 . Ao utilizar a dúvida como método, qual foi o objetivo de Descartes?

2 . Esclarece, a partir do exemplo dado no vídeo, o argumento dos sentidos enganadores.

3 . Identifica a ideia central do argumento do sonho e indica as crenças que este permite pôr
em causa.

4 . A dúvida cartesiana torna-se mais radical com o argumento do génio maligno. Explica
porquê.

5 . O génio maligno, segundo Descartes, não pode enganar o sujeito acerca de quê?
Justifica.

Editável e fotocopiável © Texto | Dúvida Metódica, 11.º ano 13


1 Epistemologia – os problemas da definição, da possibilidade e da origem do conhecimento

QUESTÃO-AULA 5
Criar filhos, à maneira de Hume

Analisa com atenção a banda desenhada e responde às questões apresentadas.

https://existentialcomics.com/ (adaptado)

1 . Que inferências indutivas são realizadas pela criança da banda desenhada.


2 . Tal como a criança da banda desenhada, também tu fazes inferências indutivas no teu dia
a dia. Dá dois exemplos.
3 . Segundo David Hume, as inferências indutivas têm uma base racional? Porquê?
4 . O pai da criança representa, naturalmente, Hume. Qual era a posição deste filósofo
acerca da origem do conhecimento? Justifica.

14 Editável e fotocopiável © Texto | Dúvida Metódica, 11.º ano


1 Epistemologia – os problemas da definição, da possibilidade e da origem do conhecimento

QUESTÃO-AULA 6
Após o visionamento do vídeo, sobre a teoria do conhecimento de Hume, responde às
questões apresentadas.

«Introdução a Hume: Ideias fundamentais de Ensaio Sobre o Entendimento Humano»


Duração: 11:34 Legendado em português

1 . A partir de dois exemplos, explica como se distinguem as impressões das ideias.

2 . Dá dois exemplos de questões de facto.

3 . Segundo Hume, apenas temos experiência de «conjunções constantes» entre os


fenómenos e não de «relações necessárias». Quais são as implicações desta tese para a
ciência?

4 . Identifica o problema, formulado por Hume, que os filósofos depois dele continuaram,
e ainda continuam, a debater.

5 . Por que motivo, de acordo com Hume, «o hábito deve ser o grande guia da vida humana»?
Consideras que Hume tem razão? Porquê?

Editável e fotocopiável © Texto | Dúvida Metódica, 11.º ano 15


2 Filosofia da ciência – os problemas da demarcação, do método, da evolução e da objetividade da ciência

QUESTÃO-AULA 1
1 . Considera o texto e a figura que se seguem.
«Desde 1834 que se sabia que uma molécula de benzeno é constituída por seis áto-
mos de carbono e seis átomos de hidrogénio. Mas, faltava determinar a sua estru-
tura, de uma forma que explicasse o seu comportamento químico.
O químico Friedrich August Kekulé Von Stradonitz descobriu essa estrutura, em
1865. O modo como fez a descoberta é curioso. Um dia estava a escrever acerca
do benzeno, mas adormeceu na cadeira e teve um sonho. Nesse sonho, ele via os
átomos como que dançando à sua frente e formando cadeias (isto é, filas ou se-
quências). Longas cadeias que começaram a torcer-se como cobras. De repente,
uma das cobras mordeu a sua própria cauda.
Kekulé acordou e, ao recordar a imagem da cobra a morder a sua própria cauda,
ocorreu-lhe de repente que a disposição dos átomos na molécula do benzeno po-
deria ser semelhante, ou seja, que poderia ter uma forma hexagonal. [Um hexágo-
no é um polígono de seis lados.]
Mais tarde testou-se a hipótese e constatou-se que é de facto assim.»
«O Sonho de Kekulé e a Descoberta do Benzeno»,
in https://www.preparaenem.com/quimica/o-sonho-kekule-descoberta-benzeno.htm
(adaptado)

FIG. 1

1.1 De acordo com a perspetiva falsificacionista, os procedimentos descritos inserem-se


em que etapas do método científico?
1.2 Um defensor da perspetiva indutivista, concordaria com a explicação apresentada
para a criação da hipótese? Porquê?

2 . De acordo com o falsificacionismo, qual é o papel das inferências indutivas no método


científico?

3 . De acordo com Popper, o facto de uma ideia científica surgir a partir de um sonho,
compromete a sua objetividade? Porquê?

4 . Enunciados científicos universais como «todas as moléculas de benzeno têm uma forma
hexagonal» serão, segundo Popper, verificáveis? Justifica.

16 Editável e fotocopiável © Texto | Dúvida Metódica, 11.º ano


2 Filosofia da ciência – os problemas da demarcação, do método, da evolução e da objetividade da ciência

QUESTÃO-AULA 2
Analisa com atenção a banda desenhada e responde às questões apresentadas.

Editável e fotocopiável © Texto | Dúvida Metódica, 11.º ano cont. 17


2 Filosofia da ciência – os problemas da demarcação, do método, da evolução e da objetividade da ciência

https://existentialcomics.com/ (adaptado)

1 . Segundo Popper, que razões se podem apresentar para considerar a psicanálise como
uma pseudociência? Responde, transcrevendo duas passagens da banda desenhada.
2 . De acordo com Popper, qual é a principal razão que se pode apresentar para considerar
a teoria da relatividade de Einstein como científica? Responde, transcrevendo uma
passagem da banda desenhada.
3 . Do ponto de vista popperiano, a verdade ou a falsidade das teorias científicas pode ser
provada? Porquê? Concordas com Popper? Porquê?

18 Editável e fotocopiável © Texto | Dúvida Metódica, 11.º ano


2 Filosofia da ciência – os problemas da demarcação, do método, da evolução e da objetividade da ciência

QUESTÃO-AULA 3
1 . Lê o artigo da revista Visão e responde às seguintes questões.

«A experiência de Eddington impressionou o filósofo da ciência mais famoso não


só do século XX, mas porventura dos séculos em geral. Karl Popper, pois. A teoria
de Einstein tinha sido sujeita a um teste rigoroso, por isso muito arriscado, e saído
dele incólume. Este procedimento metodológico, salienta Popper, contrasta com
os artifícios próprios da pseudociência, que permitem “encaixar” numa teoria
qualquer descoberta empírica concebível.
Como a teoria de Einstein se saiu bem num teste muito arriscado, isso, embora
de forma alguma a comprove, há de tê-la confirmado, não? – Não! — diz Popper. –
Não?! – Não — diz ainda Popper —, esse sucesso experimental corroborou a teoria.
Só isso.
Há agora que aclarar os termos “confirmação” e “corroboração”, que na sua aceção
comum são sinónimos ou quase. (…) “Confirmação”: uma teoria é confirmada por
dados empíricos quando esses dados a apoiam num certo grau. A confirmação,
portanto, é algo positivo. Já a corroboração é somente a ausência de algo negativo:
uma teoria corroborada, para Popper, é uma teoria que, tendo sido sujeita a testes
empíricos dignos desse nome, não foi refutada ou falsificada. Se uma teoria tiver
sido submetida a muitos testes rigorosos e tiver sobrevivido a todos eles, terá um
elevado grau de corroboração — o que até pode parecer uma coisa boa (a palavra
ilude), mas na verdade não passa da ausência de uma coisa má, a refutação. (…)
O facto de a palavra “corroboração” soar a algo positivo, mesmo que modesta-
mente positivo, leva muitas pessoas (incluindo muitos cientistas) a não perceber
a perspetiva falsificacionista de Popper (…). Pois o falsificacionismo diz-nos que a
observação desempenha um papel inteiramente negativo ou eliminativo na ava-
liação de teorias científicas: através da observação, podemos refutar uma teoria
científica, mas nunca confirmá-la, por pouco que seja. Por muitos sucessos expe-
rimentais que uma teoria acumule, nunca poderemos dizer que, provavelmente,
ela é verdadeira. Esses sucessos limitam-se a manter viva a mera possibilidade de
ela ser verdadeira.»
Pedro Galvão, in Visão, https://visao.sapo.pt/opiniao/2019-11-05-O-que-diria-Popper-/
?fbclid=IwAR03MRuVaJmwBKw67BbJ8U1v3ASJu1IdD0JIII7eDZswAb2htGaRlEhWAc0
(consultado em 21/12/2021)

1.1 Uma previsão efetuada pela teoria da relatividade geral foi testada através de uma
experiência durante o eclipse de 1919. Segundo Popper, qual é a diferença entre esse
procedimento e os «artifícios» da pseudociência?

1.2 Distingue, a partir das ideias apresentadas no texto, a «confirmação» da


«corroboração» das teorias científicas.

1.3 Se adotássemos um ponto de vista crítico em relação ao falsificacionismo, como


argumentaríamos para defender que esta teoria não permite considerar as teorias
científicas como sendo confiáveis?

Editável e fotocopiável © Texto | Dúvida Metódica, 11.º ano 19


2 Filosofia da ciência – os problemas da demarcação, do método, da evolução e da objetividade da ciência

QUESTÃO-AULA 4
1 . Lê atentamente o texto seguinte e responde à questão apresentada.
«O geocentrismo é um modelo cosmológico muito antigo. Na Antiguidade era
muito raro que alguém discordasse dessa teoria. Aristóteles (384-322 a.C.), um dos
mais importantes filósofos da história, defendeu-a.
Ptolomeu (90-168 d.C.), famoso astrónomo e matemático grego, no livro Almagesto,
deu a forma final à teoria, afirmando que a Terra estava parada no centro do univer- 1
so e que os restantes corpos celestes, incluindo o Sol, giravam à sua volta. Os corpos
celestes mais próximos da Terra reconhecidos pela teoria geocêntrica eram a Lua,
Marte, Vénus, Mercúrio, Júpiter, Saturno e o Sol. Depois ficavam as estrelas mais lon-
gínquas (Ptolomeu e outros astrónomos identificaram cerca de mil).
Esses corpos celestes descreviam, segundo a teoria geocêntrica, órbitras circula-
res em torno da Terra, pois associava-se o círculo à perfeição e supunha-se que no 2
mundo supralunar tudo era perfeito.
Considerava-se também que essa era uma descrição completa e não havia mais
3
corpos celestes, pelo menos próximo da Terra.
Ptolomeu observou as estrelas, registou os seus brilhos, estabeleceu normas de
previsão de eclipses e procurou descrever o movimento dos planetas.
A teoria geocêntrica foi utilizada por árabes e europeus durante séculos. Com base 4
nela elaboraram calendários e previram eclipses. Foi também usada na orientação
de navios no mar.
A 4 de julho de 1054 observou-se em muitos pontos da Terra uma supernova na ne-
bulosa de Caranguejo. Os chineses, os astecas e os maias (que desconheciam a teo-
ria geocêntrica) observaram-na e deixaram registos escritos do facto. Certamente
5
que alguns astrónomos europeus (todos seguidores de Ptolomeu) também a obser-
varam, mas não deixaram nada escrito sobre o assunto, como se não lhe tivessem
atribuído importância, como se fosse algo pontual e pouco significativo.
Percebemos melhor esse silêncio se nos lembrarmos de que, à luz do geocentris-
mo, não era suposto existirem novidades no céu.
Copérnico (1473-1543) estudou os movimentos de Marte, planeta cujas mudan-
ças de grandeza e brilho não poderiam provir da sua suposta trajetória à volta da
Terra. Percebeu depois que esses fenómenos irregulares Marte se podiam expli-
car caso se admitisse que este orbitava em torno do Sol e não da Terra.
Para os defensores do geocentrismo, essas irregularidades não constituíam refu-
tações da teoria, mas apenas problemas pontuais e pouco significativos, pelo que 6
rejeitaram prontamente a teoria de Copérnico.
Contudo, as investigações de outros cientistas – como Tycho Brahe, Kepler e
Galileu – mostraram que havia vários outros problemas com a teoria geocêntrica. 7
Eram demasiados problemas para poderem ser ignorados. A teoria geocêntrica
começou a perder credibilidade.
Galileu (1564-1642) desenvolveu a sua atividade científica opondo-se constante-
mente ao geocentrismo. Construiu telescópios e com eles descobriu montanhas
na Lua e manchas no Sol, mostrando assim que não eram perfeitamente esféricos,
como se julgava. Descobriu também satélites de Júpiter e de Saturno e muito mais 8
estrelas do que as admitidas pelo geocentrismo, mostrando assim que a descrição
que a teoria fazia dos céus não estava completa.
Galileu desenvolveu as ideias de Copérnico e defendeu o heliocentrismo: era em
torno do Sol que giravam a Terra e os outros planetas.

20 Editável e fotocopiável © Texto | Dúvida Metódica, 11.º ano


2 Filosofia da ciência – os problemas da demarcação, do método, da evolução e da objetividade da ciência

Perante essas descobertas, os defensores do geocentrismo foram sendo cada vez


mais em menor número e o heliocentrismo começou a ter uma aceitação cada vez 9
mais generalizada.»
Adaptado de Wikipédia (geocentrismo, Ptolomeu,
Nicoláu Copérnico, heliocentrismo – consultado em 02/04/2022)

Os principais conceitos da teoria de Thomas Kuhn estão assinalados com números.


Identifica os números que correspondem aos conceitos seguintes (atenção: uma alínea
pode corresponder a mais do que um número).
A. Paradigma:
B. Pressuposto filosófico envolvido no paradigma geocêntrico:
C. Ciência normal:
D. Anomalia:
E. Crise científica:
F. Ciência extraordinária:
G. Revolução científica:

2 . Lê o seguinte artigo do jornal The Guardian e responde às questões apresentadas.

Os egos dos cientistas são «Os cientistas progrediriam mais


um obstáculo ao progresso, rapidamente na resolução dos problemas
segundo uma das pioneiras do mundo se aprendessem a pôr os egos
da vacina contra a covid-19 de lado e colaborassem mais uns com os
outros, de acordo com a investigadora-
-chefe da vacina Pfizer/BioNTech contra
a covid-19. Katalin Karikó, vice-presidente
Fotografia: Zoltán Balogh/EPA

para as terapias de substituição de RNA


da BioNTech, sediada na Alemanha, foi
durante décadas alvo de humilhações e
de ceticismo e, finalmente, expulsa do
seu laboratório enquanto desenvolvia a
FIG. 1 A bioquímica húngara Katalin Karikó tecnologia que tornou possíveis as vacinas
representada num mural de uma parede de da Pfizer e da Moderna.»
um apartamento em Budapeste.
Ian Sample, in The Guardian, https://www.theguardian.
com/world/2021/sep/10/scientists-egos-key-barrier-to-
progress-covid-vaccine-pioneer-katalin-kariko
(consultado em 02/12/2021)

2.1 Identifica, segundo a teoria de Kuhn, o tipo de fatores que são referidos no artigo.
2.2 De acordo com Kuhn, a ciência será inteiramente objetiva? Porquê?
Concordas com Kuhn? Porquê?

Editável e fotocopiável © Texto | Dúvida Metódica, 11.º ano 21


2 Filosofia da ciência – os problemas da demarcação, do método, da evolução e da objetividade da ciência

QUESTÃO-AULA 5
1 . Preenche, de acordo com a teoria de Popper, o esquema seguinte.

B.
decorre da _________________
de erros;

O progresso C.
constitui um ________________
cientifico é A. de conhecimento;
_________________ porque

D.
permite uma _______________
à verdade.

2 . Preenche, de acordo com a teoria de Kuhn, o esquema seguinte.

a resolução de
A.
__________________________

os objetivos dos a consolidação do


Na ciência
normal, cientistas são: B.
__________________________

tornar o paradigma estabeleci-


C.
do mais _____________ através
da sua aplicação na solução
D.
de ________________________.

22 Editável e fotocopiável © Texto | Dúvida Metódica, 11.º ano


2 Filosofia da ciência – os problemas da demarcação, do método, da evolução e da objetividade da ciência

3 . Analisa o cartoon seguinte, cujos personagens são Galileu, um rei e membros do clero.

«Bem, isto muda tudo.»

3.1 A afirmação proferida pelo rei ao olhar para o globo terrestre refere-se a que
mudança? Como designa Kuhn o período em que essa mudança ocorre?

3.2 Como se pode explicar, segundo a teoria de Kuhn, a afirmação: «… isto muda tudo».

3.3 Na perspetiva de Kuhn, as alterações radicais ocorridas ao longo da história da ciência


correspondem a um progresso cumulativo? Porquê?

Editável e fotocopiável © Texto | Dúvida Metódica, 11.º ano 23


3 Filosofia da arte – o problema da definição de arte

QUESTÃO-AULA 1
1 . Analisa com atenção as figuras seguintes.

FIG. 1 Fotografia de Henri Cartier Bresson FIG. 2 Arte rupestre nas grutas de Lascaux, em França

FIG. 3 Guernica (1937), do pintor espanhol Pablo Picasso. O quadro representa o bombardeamento da cidade basca de
Guernica, em 1937, durante a guerra civil espanhola.

1.1 De acordo com a teoria representacional, as figuras 1 e 3 podem ser consideradas


obras de arte? Justifica.
1.2 A figura 2 poderá constituir um contraexemplo em relação a que teoria da arte?
Justifica.

2 . Segundo algumas teorias, as características definidoras da arte são intrínsecas às


obras. Segundo outras teorias, são contextuais. Dá um exemplo de cada uma dessas
características, indicando também a respetiva teoria.

3 . O que significa afirmar que uma teoria da arte é classificativa e não avaliativa?

24 Editável e fotocopiável © Texto | Dúvida Metódica, 11.º ano


3 Filosofia da arte – o problema da definição de arte

QUESTÃO-AULA 2
1 . Visiona o vídeo seguinte e responde às questões apresentadas.

«Frida Kahlo: a mulher por detrás da lenda»


Iseult Gillespie
Duração: 3:55 Legendado em português

1.1 Dá três exemplos de temáticas que a pintora Frida Kahlo tenha abordado nas suas
obras.
1.2 Indica duas razões que permitam relacionar algumas obras de Frida Kahlo (como, por
exemplo, A Coluna Partida) com a teoria expressivista da arte.

2 . Visiona o vídeo, observa as figuras e responde às questões apresentadas.

«O brilhantismo caótico de Jean-Michel Basquiat»


Jordana Moore Saggese
Duração: 4:32 Legendado em português

Editável e fotocopiável © Texto | Dúvida Metódica, 11.º ano 25


3 Filosofia da arte – o problema da definição de arte

FIG. 1 Head of a Fryer, de Jean-Michel Basquiat (1982) FIG. 2 Jean-Michel Basquiat começou por desenhar
graffitis nas ruas. Entre 1977 e 1980, assinava-os
como SAMO.

2.1 Dá três exemplos de temáticas que o artista Jean-Michel Basquiat tenha abordado nas
suas obras.
2.2 Indica duas razões que permitam relacionar algumas obras de Basquiat (como, por
exemplo, Head of a Fryer) com a teoria institucional da arte.

3 . Escolhe uma obra, de um dos artistas dos vídeos anteriores, e elabora uma apreciação
crítica, salientando aspetos que mais te agradaram/desagradaram e relacionando essa
obra com pelo menos uma das teorias estudadas acerca da definição de arte.

26 Editável e fotocopiável © Texto | Dúvida Metódica, 11.º ano


4 Filosofia da religião – o problema da existência de Deus

QUESTÃO-AULA 1
1 . Descreve duas características comuns aos argumentos cosmológico e teleológico.

2 . Em que argumento, a favor da existência de Deus, se recorre à ideia de regressão causal e


com que objetivo?

3 . Analisa o seguinte cartoon.

3.1 Esclarece o pressuposto, acerca do atributo da existência, implícito no argumento


ontológico.

3.2 Que implicações tem a aplicação das ideias contidas no argumento ontológico ao
«namorado marciano»?

3.3 A ideia de «ser mais grandioso» será coerente? Responde a esta questão de modo a
obteres uma objeção ao argumento ontológico.

Editável e fotocopiável © Texto | Dúvida Metódica, 11.º ano 27


4 Filosofia da religião – o problema da existência de Deus

QUESTÃO-AULA 2
1 . Lê o texto e responde às perguntas que se seguem.

«Se eu te oferecesse uma aposta grátis para uma oportunidade de ganhares uma
fortuna, aceitarias, certo? Se tivesses de dizer apenas algumas palavras para uma
oportunidade no paraíso eterno, pareceria um bom negócio?
O filósofo francês do século XVII Blaise Pascal pensava que sim, e que todos nós
devíamos acreditar em Deus por esta razão.
(…) O que é menos conhecido sobre a aposta de Pascal é que ele também pensava
que existem benefícios reais que advêm de ser religioso aqui na Terra, tais como a
felicidade existencial e um sentido de comunidade.
Para aqueles que possam desafiar: “Isto não é uma crença genuína”, a resposta de
Pascal era que se pode simplesmente “fingir até que se consiga”. Ao fingires que
és suficientemente religioso – através de rituais e liturgia, por exemplo –, um dia
serás efetivamente religioso (…).
Portanto, diz os versos, canta as canções e reza a Deus todas as noites. Despendendo
apenas alguns minutos por dia, receberás o melhor boletim de apostas de todos os
tempos.»
Jonny Thomson, Mini Filosofia, O Pequeno Livro das Grandes Ideias,
Minoutauro, Lisboa, 2021, pp. 159-60

1.1 Explica o que pretendia Pascal com o argumento da aposta.

1.2 Qual é a crítica ao argumento da aposta referida no texto?

1.3 Concordas com a resposta dada por Pascal a essa crítica? Porquê?

2 . Identifica a que tipo de mal se referem as figuras seguintes. Como é que um teísta
explicaria a existência destes males?

FIG. 1 Mulher vítima de ataques aéreos russos na cidade FIG. 2 Erupção de vulcão em Eyjafjallajökull, na Islândia
de Kharkiv, na Ucrânia (24 de fevereiro de 2022). (2010).

28 Editável e fotocopiável © Texto | Dúvida Metódica, 11.º ano


PROPOSTAS DE RESOLUÇÃO

Capítulo 1 ter esta crença, apesar de esta ter encontrado um princípio


Epistemologia – os problemas se ter revelado verdadeira. indubitável: o cogito
da definição, da possibilidade Acertaram na verdade por mero (ou penso, logo existo).
e da origem do conhecimento acaso. As crenças do sujeito podem
ser todas ilusórias, devido à
QUESTÃO-AULA 2 ação do génio maligno, mas se
QUESTÃO-AULA 1 o sujeito duvida, então pensa
1. Exemplos: as alucinações e o
1.1 Conhecimento proposicional. holograma do punhal. e existe. Logo, o génio maligno
A Mary conhecia as cores 2. Na «Alegoria da caverna», não pode enganar o sujeito
apenas teoricamente, isto Platão considera que os acerca do facto de este ser
é, conhecia as proposições sentidos são enganadores e dão um ser pensante, ainda que os
que explicam o fenómeno da acesso somente às «sombras». conteúdos do seu pensamento
visão das cores do ponto de A fonte do conhecimento possam ser todos falsos.
vista da Física e da Biologia. verdadeiro é a razão ou
Por exemplo, a Mary «sabe pensamento, é este que nos QUESTÃO-AULA 3
como as diferentes ondas de permite compreender a 1.1 Porque as crenças nestas ideias
comprimento da luz estimulam natureza ilusória das sombras se encontram suspensas devido
três tipos de células cónicas na (do conhecimento sensorial) e à dúvida introduzida pelo
retina e sabe como os sinais ter acesso à realidade (o que se argumento do génio maligno.
elétricos viajam pelo nervo encontra fora da caverna). 1.2 Porque, nesta etapa do percurso
ótico até ao cérebro. Aí, criam Descartes concorda com da dúvida metódica, Descartes
padrões de atividade neural Platão quanto ao facto de a considerou já ter provado a
que correspondem às muitas informação sensorial não ser existência de Deus.
cores que quase todos os seres confiável e na valorização 1.3 Porque se existe um Deus
humanos distinguem.» da razão enquanto fonte de perfeito, não pode existir um
1.2 Sim, conseguiu ter conhecimento. Deus enganador, pois um
conhecimento por contacto. Descartes começa por duvidar dos atributos de Deus é a
Ela foi capaz de apreender, do conhecimento sensorial
bondade; portanto, este não
diretamente através dos (a posteriori), através dos
nos enganaria em relação a
sentidos, a cor verde da maçã argumentos dos sentidos
tudo. Assim sendo, a hipótese
que surgiu no ecrã do seu enganadores e do argumento
do génio maligno (ou Deus
computador. do sonho, defendendo que
enganador) é afastada.
2.1 As alíneas C e E. Em ambas as crenças empíricas não
1.4 A ideia de sereia é falsa,
as alíneas, de acordo com se encontram devidamente
como mostram quer a razão
a definição tradicional de justificadas. Por outro lado, na
quer a experiência. Deus
conhecimento, os sujeitos filosofia cartesiana, apenas as
garante que se raciocinarmos
possuem crenças verdadeiras ideias conhecidas por intuição
cuidadosamente e se
justificadas (acreditam em racional (como o cogito e a
observarmos as coisas com
algo que é verdadeiro e têm ideia de Deus) são as mais
atenção não nos enganaremos
boas razões a sustentar as suas certas.
e obteremos ideias verdadeiras.
crenças). Portanto, encontram- 3. Do conhecimento a priori / dos
-se reunidas, em simultâneo, conhecimentos da matemática,
as três condições necessárias e QUESTÃO-AULA 4
como 2 + 2 = 4, que são iguais
suficientes para a existência de quer nos sonhos, quer na 1. Encontrar uma certeza
conhecimento. realidade. inabalável / um princípio
2.2 Nas alíneas A, B, D e F. Nas 4. Ao argumento cartesiano indubitável.
alíneas A e B, as pessoas têm do génio maligno (ou do 2. Exemplo: ao colocarmos
crenças falsas e as justificações Deus enganador). Ambos os uma vara dentro de água,
que apresentam são argumentos – o «cérebro numa percecionamo-la como
inadequadas; por isso, faltam cuba» e o génio maligno – estando partida, quando na
as condições necessárias da correspondem a experiências realidade isso não acontece.
verdade e da justificação. mentais que exploram a No argumento dos sentidos
Nas alíneas D e F falta a possibilidade de o sujeito ser enganadores, Descartes
condição necessária da enganado em relação a tudo, defende que se algumas
justificação, já que os sujeitos incluindo os seus raciocínios. informações sensoriais são
não possuíam boas razões para 5. Sim, pois Descartes pensa falsas; assim, não podemos

Editável e fotocopiável © Texto | Dúvida Metódica, 11.º ano 29


PROPOSTAS DE RESOLUÇÃO

garantir que os sentidos não nos presentes do Pai Natal. várias ideias, como a de ouro
enganem sempre e estes não Portanto, todas as crianças e de montanha, e, através da
são confiáveis como fonte de bem-comportadas recebem imaginação, forma-se a ideia de
conhecimento. presentes do Pai Natal. uma montanha de ouro.
3. Ideia central do argumento 2. Dois exemplos de inferências 2. Exemplos de duas questões de
do sonho: se os sonhos podem realizadas no dia a dia: facto:
reproduzir a realidade, como • As laranjas que comi até hoje • A água apagou o fogo.
é que podemos distinguir o eram doces. Logo, todas as • Hoje, o Sol nasceu.
estado de vigília do sonho laranjas são doces. 3. A tese de que apenas temos
(se estamos acordados ou a • Observei que o Sol se pôs experiência de «conjunções
sonhar)? Descartes argumenta todos os dias. Portanto, constantes» tem consequências
que, se não existe um critério amanhã também irei observar céticas para a ciência. Uma
para fazer esta distinção, então o pôr do Sol. parte significativa da atividade
tudo pode ser um sonho. 3. Não. As inferências indutivas científica consiste na procura
Crenças que o argumento do (a generalização e a previsão) de causas. Muitas leis científicas
sonho permite pôr em causa: não se encontram justificadas pressupõem o estabelecimento
crenças a posteriori. Por racionalmente porque de relações causais. Ora, para
exemplo: podemos julgar que concluímos algo que ultrapassa
Hume, as relações causais
estamos a jogar um jogo em a experiência, baseando-nos
baseiam-se no hábito, ou seja,
que destruímos o castelo do na ideia de que a natureza
numa crença subjetiva, sem
Drácula e a cabeça de uma funciona de forma regular:
justificação racional. Por isso,
medusa e, afinal, nada disso ser princípio da uniformidade
se Hume tiver razão, as teorias
real. da natureza. Dado que este
científicas que se referem a
4. O argumento do génio maligno, princípio não é justificável
relações causais não constituem
uma situação hipotética a priori (por ser uma questão
conhecimento.
proposta por Descartes, de facto) nem a posteriori, já
4. O problema da indução.
explora a possibilidade de que se baseia num raciocínio
todas as crenças poderem ser indutivo (e justificá-lo desse
5. Quer no caso da causalidade
quer no caso da indução,
falsas, levando a que a dúvida modo envolveria uma petição
possa ser estendida a todos de princípio), então as Hume concluiu que fazemos
os conteúdos do pensamento inferências indutivas não têm inferências que não se baseiam
do sujeito, incluindo (além dos justificação racional. Esta é uma nem em princípios a priori nem
conhecimentos a posteriori) os conclusão cética. em experiências externas (das
conhecimentos a priori. 4. É uma posição empirista. próprias coisas), mas sim numa
5. Não o pode enganar acerca da Considera que todas as ideias experiência interna e subjetiva:
crença no cogito (penso, logo são originadas a partir dos o costume ou hábito de esperar
existo). Tal acontece porque sentidos (mesmo as mais que o futuro seja semelhante ao
para duvidar, o sujeito tem de abstratas como Deus) e, por passado e que certas coisas se
pensar e para pensar, o sujeito isso, não há ideias inatas. Os sigam a outras.
tem de existir. Ainda que o sentidos são a principal fonte O aluno poderá concordar
génio possa enganar o sujeito do conhecimento e o seu limite. com Hume, alegando (por
acerca de todos os conteúdos exemplo) que é o costume que
do seu pensamento, não é QUESTÃO-AULA 6 nos permite manter as nossas
possível colocar em causa o 1. As impressões são provenientes crenças indutivas e causais,
cogito, a existência do sujeito dos sentidos e podem ser já que não as conseguimos
como um ser pensante. externas (respeitantes a objetos justificar racionalmente, mas
captados pelos cinco sentidos) também não podemos rejeitá-
QUESTÃO-AULA 5 ou internas (como o desejo de -las devido à sua utilidade
1. Inferências indutivas: comer). As ideias são cópias, prática.
• O meu pai disse-me mil menos vivas e intensas, das O aluno poderá não concordar
vezes para colocar os copos impressões. Por exemplo: a com Hume, alegando (por
no armário. Logo, amanhã partir da impressão (complexa) exemplo) que basear as nossas
voltarei a colocá-los de novo de árvore forma-se a ideia crenças indutivas e causais no
armário. (complexa) de árvore. Existem costume é insatisfatório e que
• Até hoje, os meninos que se também ideias complexas que estas podem ser racionalmente
portaram bem receberam resultam da combinação de justificadas.

30 Editável e fotocopiável © Texto | Dúvida Metódica, 11.º ano


Capítulo 2 3. Não, porque as teorias 1.2 A confirmação de uma teoria
Filosofia da ciência – os problemas científicas apenas podem ser refere-se ao facto de esta se
da demarcação, do método, falsificadas (refutadas) ou encontrar fundamentada em
da evolução e da objetividade corroboradas. A corroboração certas evidências empíricas,
da ciência significa que as teorias são tendo um maior ou menor
aceites provisoriamente, grau de probabilidade de
QUESTÃO-AULA 1 enquanto resistirem às ser verdadeira; depende da
1.1 Inserem-se nas seguintes tentativas de falsificação (ou amostra. A corroboração diz
etapas: seja, aos testes). Segundo respeito ao facto de uma
• Elaboração das hipóteses ou Popper, a verdade é um teoria ter sido submetida a
conjeturas. princípio regulador da atividade testes e não ter sido falsificada
• Submissão da hipótese a científica, um ideal a alcançar / refutada até ao momento,
e não pode ser aplicado ao podendo vir a sê-lo no futuro.
testes.
conteúdo das teorias.
1.2 Não. Segundo os indutivistas, Assim sendo, podemos eliminar
Se o aluno concordar com as teorias falsas, mas nunca
as hipóteses científicas são
Popper, poderá alegar (por
construídas a partir dos confirmar a verdade das teorias.
exemplo) que, por muitos testes
dados observados, através de 1.3 Poderíamos argumentar que,
bem-sucedidos que façamos
generalizações. se Popper tivesse razão, então
a uma teoria, nunca podemos
2. Nenhum. Os cientistas não não teríamos boas razões
garantir que um dia não surjam
recorrem ao raciocínio indutivo, para confiar não só nas teorias
resultados que a falsifiquem.
só utilizam o raciocínio científicas, como também
Se o aluno discordar de Popper,
dedutivo. nas aplicações destas – que
poderá alegar (por exemplo)
3. Não. Para Popper, a permitem as tecnologias que
que, embora não se consiga
objetividade das hipóteses utilizamos no dia a dia (por
comprovar conclusiva e
depende de critérios lógicos, exemplo, computadores,
definitivamente a verdade de
dos resultados dos testes automóveis e telemóveis) –, já
uma teoria, um certo número de
empíricos e da submissão à que estas poderiam revelar-se
testes bem-sucedidos que lhe
discussão crítica. Estes critérios falsas no futuro. Então, porque
façamos podem tornar muito
são partilhados pelos cientistas havemos de acreditar nelas?
improvável a sua falsidade e
e aplicados de uma forma
isso ser suficiente para afirmar a
imparcial. QUESTÃO-AULA 4
sua verdade.
4. Não. Enunciados universais
1.
(como «todas as moléculas
QUESTÃO-AULA 3 A. Paradigma – 1 e 3
de benzeno têm uma forma
hexagonal») referem-se a todos 1.1 A diferença entre esse B. Pressuposto filosófico
procedimento científico e os envolvido no paradigma
os casos – do presente, passado
e futuro – e, assim sendo, dada «artifícios» da pseudociência geocêntrico – 2
a sua infinidade, nunca serão é a seguinte: para as teorias C. Ciência normal – 4
verificáveis na sua totalidade. serem consideradas científicas D. Anomalia – 5 e 6
Mas são, alega Popper, têm de ser sujeitas a testes E. Crise científica – 7
falsificáveis. exigentes e rigorosos que visam F. Ciência extraordinária – 8
falsificá-las. Se resistirem, como G. Revolução científica – 9
QUESTÃO-AULA 2 aconteceu com a teoria de 2.1 Fatores extracientíficos /
Einstein, são aceites até que não objetivos: subjetivos e
1. Segundo Popper, a psicanálise
novas tentativas de falsificação sociológicos, como o facto de,
não pode ser considerada uma
sejam bem-sucedidas. Este do ponto de vista individual,
ciência porque:
• pretende ser capaz «de procedimento metodológico os cientistas procurarem
explicar seja o que for»; difere dos «artifícios» usados obter reconhecimento social
• «por não fazeres qualquer na pseudociência: não faz e prestígio com as suas
previsão acerca do futuro, previsões específicas e, por descobertas, o que acaba por
nada pode falsificar a tua isso, não é suscetível de interferir no trabalho de equipa
teoria.» ser testada. Além disso, as necessário à realização de
2. A principal razão é que «fez explicações vagas tornam descobertas científicas eficazes.
previsões muito objetivas que possível que qualquer facto seja 2.2 Não, a ciência não é
poderiam falsificar a teoria por ilusoriamente encarado como inteiramente objetiva. Existem
completo.» uma confirmação da teoria. critérios objetivos, partilhados

Editável e fotocopiável © Texto | Dúvida Metódica, 11.º ano 31


PROPOSTAS DE RESOLUÇÃO

pela comunidade científica que os paradigmas são (pode ser apenas algo
– como a simplicidade e a incomensuráveis e a escolhido e recolhido pelo
abrangência, entre outros –, substituição do antigo pelo artista) e for considerado arte
que os cientistas podem aplicar novo paradigma significar por um membro do mundo da
na escolha dos paradigmas. uma alteração radical da visão arte. Teoria institucional.
Contudo, a interpretação do mundo e da forma como 3. Afirmar que uma teoria da arte
destes e a valorização que os cientistas explicavam os é classificativa e não avaliativa
cada cientista faz de cada fenómenos. significa que apenas pretende
um deles depende de fatores 3.3 Não. As revoluções científicas dizer o que é e o que não é arte,
subjetivos, sendo alguns não constituem um progresso sem pretender distinguir entre a
destes fatores exteriores à cumulativo (este só existe no boa e a má arte.
ciência (interesses monetários, período da ciência normal),
princípios ideológicos, aspetos dado que não é possível QUESTÃO-AULA 2
psicológicos diversos, etc.). comparar o novo paradigma 1.1 Exemplos de temáticas
Se o aluno concordar com com o anterior. Estes são abordadas:
Kuhn, poderá alegar (por incomensuráveis e, por isso, • o estado físico/mental da
exemplo) que ao longo da não podemos dizer que um é artista e as suas experiências
história da ciência existem melhor do que outro ou que o de vida (a deficiência física e o
muitos exemplos inegáveis atual se encontra mais próximo casamento, por exemplo);
de cientistas que se deixaram da verdade. • a cultura indígena mexicana;
levar pelos mais diversos • naturezas-mortas.
fatores subjetivos e que isso 1.2 Duas razões que justificam
Capítulo 3
determinou muitas vezes a a relação da obra referida
sua adesão a certas teorias em Filosofia da arte – o problema da
de Frida Kahlo com a teoria
detrimento de outras. definição de arte
expressivista:
Se o aluno não concordar • nessa pintura, a artista
com Kuhn, poderá alegar (por QUESTÃO-AULA 1 exprime e clarifica algumas
exemplo) que, apesar de ao 1.1 Sim, ambas as figuras podem emoções, relacionadas com o
longo da história da ciência ser consideradas obras de seu estado físico e psicológico,
existirem bastantes casos de arte, porque, segundo a que antes poderia não saber
cientistas que se deixaram levar teoria representacional, identificar, ganhando assim
por fatores subjetivos, esses a representação pode ser consciência e compreensão
casos são menos numerosos imitativa (quando visa ser delas;
do que os sugeridos por semelhante à coisa imitada), • além disso, a obra também
Kuhn e que, por outro lado, como é o caso da figura 1, ou permite ao público clarificar
as motivações iniciais dos simbólica (quando representa as suas emoções, promover
cientistas são pouco relevantes sem ser semelhante), o que o autoconhecimento
e que o aspeto decisivo são acontece na figura 3 (no quadro e não apenas o mero
os testes (que têm de ser em que Picasso representa entretenimento.
reconhecidos pela comunidade o horror da guerra civil 2.1 Exemplos de temáticas
científica) a que as teorias são espanhola). abordadas:
submetidas. 1.2 À teoria histórica. De acordo • a identidade e a opressão dos
com esta perspetiva, algo é arte, marginalizados, por exemplo:
QUESTÃO-AULA 5 caso o autor tenha a intenção prisioneiros ou escravos
1. A. cumulativo; B. correção; de que isso seja visto como africanos trazidos para a
C. acréscimo; D. aproximação. o foram as obras anteriores. América;
2. A. enigmas; B. paradigma Isso torna difícil explicar a • as obras de outros artistas
dominante; C. abrangente; existência das primeiras obras relevantes na história da arte;
D. problemas novos. de arte. • a representação do interior do
3.1 À mudança do paradigma 2. Exemplos: corpo.
geocêntrico para o paradigma • Caraterísticas intrínsecas 2.2 Duas razões que justificam a
heliocêntrico. Para Kuhn às obras de arte – todas as relação da referida obra de
corresponde a uma revolução obras de arte possuem forma Jean-Michel Basquiat com a
científica. significante. Teoria formalista. teoria institucional da arte:
3.2 Pode explicar-se pelo • Caraterísticas contextuais – segundo esta teoria, uma
facto de Kuhn considerar algo é arte se for um artefacto condição para algo ser arte

32 Editável e fotocopiável © Texto | Dúvida Metódica, 11.º ano


é ser um artefacto (pode 3.1 O pressuposto é que a dia, a realizar rituais (ir à missa
ser apenas algo escolhido e existência é uma qualidade e rezar, por exemplo).
recolhido pelo artista). Ora, entre outras (como a 1.2 A aposta de Pascal não permite
esta obra é um artefacto (uma omnisciência, omnipotência, uma crença genuína.
caixa num suporte em madeira). etc.) que Deus deve possuir para 1.3 A esta objeção Pascal responde
Outra razão é ser reconhecido ser perfeito. que se nos comportarmos como
pelo «mundo da arte» (artistas, 3.2 O cartoon do namorado crentes poderemos desenvolver
críticos, historiadores de arte, marciano é uma versão uma crença genuína na
galeristas e o próprio público) e humorística da crítica de existência de Deus.
isso sucedeu com esta obra. Gaunilo. Como tal, constitui Se o aluno concordar com
3. Na sua resposta o aluno deverá: uma redução ao absurdo e Pascal, poderá alegar (por
• identificar claramente a obra procura mostrar que, usando exemplo) que a aposta de
escolhida; a estrutura do argumento Pascal é um procedimento
• descrever os aspetos que mais ontológico, podemos construir adequado: dada a nossa finitude
lhe agradam / desagradam, argumentos que «provam» as / imperfeição / ignorância
justificando a sua escolha; coisas absurdas que quisermos,
somos incapazes de provar a
• explicar a relação dessa desde que as definamos como
obra de arte com as teorias sua existência e, por isso, temos
sendo as mais perfeitas em que
estudadas acerca da definição de apostar nela; por outro lado,
se pode pensar – no caso, o
de arte. ao comportarmo-nos como
namorado perfeito.
crentes revelamos de facto
3.3 A ideia de «ser mais grandioso»
um compromisso religioso que
não é coerente, pois podemos
Capítulo 4 conduzirá ao desenvolvimento
sempre conceber um ser ainda
Filosofia da religião – o problema de uma fé sincera.
mais grandioso. Do mesmo
da existência de Deus modo, não faria sentido Se o aluno não concordar com
identificar um número como Pascal, poderá alegar (por
sendo o maior número, pois exemplo) que a aposta de
QUESTÃO-AULA 1
podemos sempre continuar a Pascal pressupõe erradamente
1. Duas características comuns: que a fé pode depender de
• ambos partem de premissas contar.
uma decisão voluntária, o que
a posteriori (baseadas em
QUESTÃO-AULA 2 promove comportamentos
informações que a experiência
1.1 Com o argumento da aposta, religiosos não sinceros e
nos dá);
Pascal pretendia mostrar interesseiros.
• a conclusão defendida por
ambos é idêntica: Deus existe / que a crença em Deus é mais 2. A figura 1 refere-se ao mal
são argumentos teístas. vantajosa do que a descrença. moral e a figura 2 ao mal
2. No argumento cosmológico. Se apostarmos que Deus existe, natural.
A ideia de regressão causal os ganhos são superiores às Um teísta justificaria a
salienta que na natureza perdas. Podemos ganhar, no existência destes dois tipos
as coisas não se causam a futuro, o paraíso eterno e, no de mal, dizendo que a sua
si próprias e a cadeia de presente, felicidade e sentido de existência permite a edificação
causas não pode regredir pertença à comunidade. Face a do caráter ou, então, que não
infinitamente. Assim sendo, benefícios tão significativos, as sabemos o suficiente para
o universo teve de ter uma perdas são irrelevantes: gastar considerar o mal injustificado.
primeira causa, que é Deus. parte do nosso tempo, no dia a

Editável e fotocopiável © Texto | Dúvida Metódica, 11.º ano 33


Banco
de Questões
BANCO DE QUESTÕES
E PROPOSTAS
DE RESOLUÇÃO

© Texto | Dúvida Metódica, 11.º ano


1 Epistemologia

GRUPO I
(Questões de resposta fechada)

Indica a alternativa correta.

1 . Qual é o problema epistemológico?


A. Será que Deus existe?
B. Qual é o bem supremo?
C. Os valores são relativos ou objetivos?
D. O que é uma justificação adequada?

2 . Considera as seguintes frases e seleciona a opção que as avalia corretamente:


i) Saber que há estrelas maiores do que o Sol.
ii) Conhecer a melhor amiga da nossa avó.
iii) Saber descascar batatas.
A. i) Conhecimento proposicional. ii) Conhecimento prático. iii) Conhecimento por
contacto.
B. i) Conhecimento por contacto. ii) Conhecimento proposicional. iii) Conhecimento
prático.
C. i) Conhecimento prático. ii) Conhecimento proposicional. iii) Conhecimento por
contacto.
D. i) Conhecimento proposicional. ii) Conhecimento por contacto. iii) Conhecimento
prático.

3 . Qual é a frase falsa?


A. O conhecimento proposicional é o conhecimento de ideias verdadeiras.
B. O conhecimento proposicional consiste num saber fazer.
C. O conhecimento por contacto é o conhecimento direto de pessoas, lugares e coisas.
D. O conhecimento prático é o conhecimento de determinadas aptidões.

4 . Qual é a situação em que o sujeito e o objeto coincidem?


A. O Júlio pensa na Júlia e tenta perceber quais são as suas intenções.
B. O Júlio pensa no futuro da humanidade.
C. O Júlio descobre quais são os gostos da Júlia.
D. O Júlio pensa em si próprio e tenta perceber porque se sente ansioso.

5 . Se o Gonçalo sabe que a Susana está doente, então


A. talvez a Susana não esteja doente.
B. o Gonçalo conhece a Susana e viu-a doente.
C. a Susana está doente.
D. talvez a Susana esteja doente.

6 . De acordo com a definição tradicional de conhecimento, uma pessoa sabe que P se


A. acredita justificadamente em P e P é uma proposição verdadeira.
B. tem uma crença muito forte na verdade de P.
C. duvida fortemente de P, embora P seja uma proposição verdadeira e justificada.
D. julga que conhece P.

Editável e fotocopiável © Texto | Dúvida Metódica, 11.º ano 37


1 Epistemologia

7 . Uma crença não é


A. um conhecimento.
B. o assentimento de que uma ideia é verdadeira.
C. um estado mental de adesão a uma determinada ideia.
D. a aceitação da verdade de uma ideia.

8 . Analisa o exemplo seguinte.


No dia do teste de Filosofia, a Matilde, uma aluna habitualmente muito responsável e
cumpridora, colocou maquilhagem para parecer pálida e na aula de Português fingiu
convincentemente vários sintomas de doença: tossiu, espirrou e queixou-se de frio e dores.
Disse que sentia um grande mal-estar e que não se conseguia concentrar. Os colegas e o
professor de Português disseram-lhe: «Estás doente e deves ir para casa. Fazes o teste de
Filosofia noutro dia. Nós explicamos a situação à professora de Filosofia.»
Os colegas e o professor da Matilde
A. formaram a crença falsa e não justificada de que ela estava doente.
B. formaram a crença falsa, mas justificada de que ela estava doente.
C. formaram a crença verdadeira, mas não justificada de que ela estava doente.
D. sabiam que ela estava doente.

9 . Analisa o exemplo seguinte.


«O André está na fila para comprar um bilhete de cinema, pensou o José ao passar ao
largo e observar um indivíduo de chapéu e com um vistoso cachecol, iguais aos usados
habitualmente pelo André. Mas depois ocorreu-lhe que não devia ser o André, pois ele não
era fã de cinema.»
Supõe que era realmente o André que estava na fila. Nesse caso, o que inequivocamente
faltava ao José para ter conhecimento?
A. A crença.
B. A verdade.
C. A justificação.
D. Sorte.

10 . De acordo com a análise tradicional do conhecimento, qual é a frase falsa?


A. Pode ser racional acreditar numa falsidade.
B. Algumas crenças falsas são justificadas, nomeadamente se a falsidade não resulta da
falta de empenho do sujeito.
C. É possível conhecer uma ideia falsa.
D. Conhecemos uma ideia se essa ideia é verdadeira e acreditamos fundadamente nela.

11 . O Pancrácio sabe que a sida é provocada por um espírito maligno, pois a sua crença
acerca das causas da doença baseia-se nas tradições da sociedade em que foi educado.
Esta atribuição de conhecimento é
A. correta, pois ele tem uma crença justificada e, além disso, esta é verdadeira.
B. incorreta, pois ele tem uma crença falsa embora adequadamente justificada.
C. correta, pois ele tem uma crença verdadeira embora pouco justificada.
D. incorreta, pois ele tem uma crença falsa e inadequadamente justificada.

38 Editável e fotocopiável © Texto | Dúvida Metódica, 11.º ano


1 Epistemologia

12 . Nos contraexemplos de Gettier à definição tradicional de conhecimento, o sujeito não


possui conhecimento, pois a
A. justificação que tem é infalível.
B. justificação que tem é ineficaz, na medida em que não incide no aspeto que torna a
crença verdadeira.
C. justificação que tem é ineficaz, na medida em que incide no mesmo aspeto que torna a
crença verdadeira.
D. sua crença é, sem que ele possa suspeitar disso, falsa.

13 . Nos contraexemplos de Gettier à definição tradicional de conhecimento, a justificação,


apesar de plausível, não evita
A. o acaso epistémico, ou seja, a possibilidade de a crença ser falsa.
B. o acaso epistémico, ou seja, acertar na verdade por sorte.
C. que tenhamos apenas duas das condições necessárias para ter conhecimento.
D. que tenhamos apenas uma das condições necessárias para ter conhecimento.

14 . Qual é a frase falsa?


A. Os céticos radicais defendem que devemos suspender todas as crenças.
B. Um exemplo de ceticismo moderado é a ideia de que na apreciação estética de obras
de arte não existe conhecimento, mas sim mera opinião.
C. Um exemplo de ceticismo moderado é a ideia de que devemos suspender a crença na
existência do mundo exterior, bem como todas as crenças associadas a essa.
D. O ceticismo, seja radical seja moderado, caracteriza-se pelo questionamento crítico de
crenças que muitas pessoas aceitam.

15 . Segundo o argumento cético dos enganos sensoriais,


A. para justificar uma crença é preciso recorrer a outra crença e assim sucessivamente.
B. a divergência de opiniões sobre todos os assuntos é irremediável.
C. não podemos garantir que os nossos sentidos não nos enganam sempre.
D. é contraditório garantir que nada se sabe.

16 . As justificações que damos precisam elas mesmas de ser justificadas e por isso…
Um cético radical continuaria a frase dizendo:
A. existe uma regressão infinita na justificação que só termina quando se adquire
conhecimento bem fundamentado.
B. temos de pesquisar cuidadosamente para diminuir a probabilidade de erro.
C. temos de encontrar as crenças básicas que justificam sem precisar – devido à sua
enorme simplicidade e evidência – de ser justificadas por outras.
D. não se produz nenhuma justificação com valor epistémico.

17 . Segundo o argumento cético da regressão infinita da justificação,


A. para justificar uma crença é preciso recorrer a outra crença e assim sucessivamente.
B. a divergência de opiniões sobre todos os assuntos é irremediável.
C. não podemos garantir que os nossos sentidos não nos enganam sempre.
D. é contraditório garantir que nada se sabe.

Editável e fotocopiável © Texto | Dúvida Metódica, 11.º ano 39


1 Epistemologia

18 . Descartes não procurou


A. estabelecer os fundamentos do conhecimento.
B. refutar o ceticismo.
C. desvalorizar a importância da matemática.
D. mostrar que podemos conhecer o mundo.

19 . A dúvida cartesiana não é


A. voluntária e hiperbólica.
B. teórica e provisória.
C. psicológica e prática.
D. metódica e intencional.

20 . Imagina que decides submeter as tuas ideias ao teste da dúvida proposto por Descartes.
Qual das ideias seguintes seria a mais resistente à dúvida?
A. Tenho duas orelhas.
B. Além, ao fundo da rua, há um banco.
C. A Terra é maior do que a Lua.
D. P implica P.

21 . Imagina que decides submeter as tuas ideias ao teste da dúvida proposto por Descartes.
Qual das ideias seguintes seria a menos resistente à dúvida?
A. Tenho duas pernas.
B. 66 + 6 = 72.
C. Um quadrado tem quatro lados iguais.
D. Eu sou, eu existo.

22 . A dúvida metódica não é


A. uma dúvida psicologicamente sentida.
B. um método para atingir a certeza.
C. o procedimento de tomar como falsas as crenças que suscitem a mais pequena dúvida.
D. um exame mental que visa distinguir as crenças duvidosas das crenças indubitáveis.

23 . Descartes procurava um fundamento indubitável para o conhecimento. A certa altura,


concluiu que não podia encontrá-lo na Matemática, devido
A. à possibilidade de os sentidos nos enganarem.
B. ao perigo de uma regressão infinita da justificação.
C. à hipótese de a vida ser um sonho.
D. à hipótese de existir um Deus Enganador.

24 . Segundo Descartes, o cogito é indubitável, pois


A. envolve a mesma certeza que a matemática.
B. é óbvio que a existência é uma condição suficiente do pensamento.
C. Deus garante a sua veracidade.
D. negar a sua veracidade seria contraditório.

40 Editável e fotocopiável © Texto | Dúvida Metódica, 11.º ano


1 Epistemologia

25 . Segundo Descartes,
A. a justificação do conhecimento é sempre empírica.
B. só temos conhecimentos a priori.
C. na base de todos os conhecimentos, incluindo os empíricos, estão conhecimentos a
priori ou racionais, como o cogito e a existência de Deus.
D. Na base, ou fundamento, de todos os conhecimentos, incluindo os racionais, estão
conhecimentos a posteriori, como o cogito e a existência de Deus.

26 . O argumento cartesiano do sonho permite duvidar


A. das crenças matemáticas e de outras crenças a priori.
B. das crenças matemáticas, mas não de outras crenças a priori.
C. de todas as crenças.
D. das crenças empíricas.

27 . A dúvida cartesiana também se aplica às crenças a priori.


O argumento que permite pôr em causa as crenças a priori é o argumento
A. das ilusões dos sentidos.
B. do sonho.
C. do génio maligno.
D. da marca.

28 . Para Descartes, o critério de verdade é


A. a informação dos sentidos.
B. a clareza e distinção das ideias.
C. a razão.
D. Deus.

29 . Vejo e sinto as minhas mãos. Logo, as minhas mãos existem.


Poderá Descartes aceitar este argumento antes de concluir que Deus existe?
A. Não, pois as afirmações a priori são especialmente incertas e precisam de um
fundamento muito forte.
B. Não, pois quer o argumento do sonho quer o argumento do génio maligno permitem
duvidar da conclusão desse argumento.
C. Sim, pois a clareza e a distinção de uma ideia provam que ela é verdadeira.
D. Sim, pois é muito improvável que os sentidos nos enganem nessas circunstâncias.

30 . No percurso da dúvida metódica a chamada «necessidade de Deus» ocorre porque


A. Descartes precisa de uma garantia para passar do cogito à afirmação de que muitas
outras crenças são verdadeiras.
B. Descartes precisa de uma garantia para justificar a religião e o papel desta na
sociedade.
C. sem a ideia de que existe um Deus bom o cogito não é uma crença indubitável.
D. Descartes procurava uma certeza e a existência de Deus é a primeira que encontra.

Editável e fotocopiável © Texto | Dúvida Metódica, 11.º ano 41


1 Epistemologia

31 . Descartes abandona a hipótese do génio maligno porque


A. nunca acreditou que tal ser existisse.
B. esse ser não o poderia enganar acerca da sua própria existência.
C. essa hipótese é incompatível com a dúvida metódica.
D. um tal ser é incompatível com a existência do Deus bondoso e omnipotente.

32 . Segundo Descartes, a bondade divina não garante que


A. todas as nossas ideias são verdadeiras.
B. se pensarmos de modo organizado e não nos precipitarmos ao tirar conclusões
descobriremos a verdade.
C. o critério das ideias claras e distintas nos leva a fazer afirmações verdadeiras.
D. se usarmos bem as nossas capacidades cognitivas evitaremos o erro.

33 . Por considerar que provou a existência de um Deus bom e não enganador, Descartes
recupera muitas crenças que tinham sido suspensas pela dúvida metódica. Por exemplo,
A. a crença de que nesta vida não podemos ter certezas, a crença de que a folha de papel
que segura na mão existe e a crença de que é um mamífero.
B. a crença de que 3 + 3 = 6, a crença de que a folha de papel que segura na mão existe e
a crença de que não tem corpo.
C. a crença de que 2 + 2 = 4, a crença de que existem sereias e a crença de que é um
filósofo e matemático.
D. a crença de que 1 + 1 = 2, a crença de que a folha de papel que segura na mão existe e
a crença de que é um mamífero.

34 . Tendo em conta a filosofia cartesiana, analisa as frases seguintes e seleciona a alínea que
avalia corretamente cada uma delas:
i) A causa da ideia de Deus tem de ser perfeita.
ii) A causa deve possuir pelo menos tanta realidade como o efeito.
iii) Recusar a existência de Deus não implica admitir que um efeito possa ser superior à
sua causa.
iv) Uma coisa é criar a ideia de um ser perfeito e outra muito diferente seria criar um ser
perfeito.
v) Um ser imperfeito não poderia ter criado a ideia de perfeição.
A sequência correta é:
A. i) Argumento da marca. ii) Crítica ao argumento da marca. iii) Argumento da marca.
iv) Crítica ao argumento da marca. v) Argumento da marca.
B. i) Argumento da marca. ii) Crítica ao argumento da marca. iii) Argumento da marca.
iv) Crítica ao argumento da marca. v) Crítica ao argumento da marca.
C. i) Argumento da marca. ii) Argumento da marca. iii) Crítica ao argumento da marca.
iv) Crítica ao argumento da marca. v) Argumento da marca.
D. i) Crítica ao argumento da marca. ii) Argumento da marca. iii) Argumento da marca.
iv) Crítica ao argumento da marca. v) Argumento da marca.

35 . Qual é, segundo Descartes, a ideia inata?


A. Laranja.
B. Caderno.
C. Substância.
D. Fantasma.

42 Editável e fotocopiável © Texto | Dúvida Metódica, 11.º ano


1 Epistemologia

36 . Qual é, segundo Descartes, a ideia adventícia?


A. Círculo.
B. Veado.
C. Deus.
D. Demónio.

37 . Qual é, segundo Descartes, a ideia factícia?


A. Verdade.
B. Peixe.
C. Pentágono.
D. Sátiro.

38 . De acordo com Descartes,


A. não há conhecimento a priori substancial.
B. o conhecimento mais rigoroso é o empírico.
C. podemos formar conhecimentos a priori acerca do mundo.
D. o conhecimento humano é a priori e não a posteriori.

39 . A alínea que não corresponde a uma crítica à filosofia cartesiana é:


A. O conhecimento humano é como um edifício que precisa de novos alicerces.
B. Não é possível provar a existência de Deus a partir da análise da ideia de Deus.
C. Para perceber que pensar implica existir é preciso perceber várias outras ideias.
D. A causa da ideia de Deus pode ser o próprio sujeito.

40 . Quanto às ideias inatas, os empiristas pensam que


A. se estas existissem, seriam conhecidas pelas crianças, mas isso não sucede.
B. estas existem, mas são como sementes e precisam de ser desenvolvidas e explicitadas.
C. estas existem, mas não podem ser conhecidas.
D. estas existem e toda a gente as conhece e usa.

41 . Ver uma prancha de surf…


De acordo com Hume, neste exemplo está em causa uma
A. impressão simples.
B. impressão complexa.
C. ideia simples.
D. ideia complexa.

42 . A guerra voltou à Europa...


De acordo com Hume, neste exemplo, ao falar-se de guerra, é referida uma
A. impressão simples.
B. impressão complexa.
C. ideia simples.
D. ideia complexa.

Editável e fotocopiável © Texto | Dúvida Metódica, 11.º ano 43


1 Epistemologia

43 . Aquele vermelho é tão vivo! De acordo com Hume, neste exemplo está em causa uma
A. impressão simples.
B. impressão complexa.
C. ideia simples.
D. ideia complexa.

44 . Sinto pena da Francelina…


De acordo com Hume, a menção à pena refere-se a uma impressão
A. interna ou sensação.
B. interna ou sentimento.
C. externa ou sentimento.
D. externa ou sensação.

45 . Segundo Hume, as perceções podem-se dividir em duas classes principais:


A. impressões e ideias.
B. impressões simples e complexas.
C. ideias simples e complexas.
D. impressões internas e externas.

46 . Hume não diz que


A. é falso que existam ideias inatas.
B. as ideias são cópias das impressões.
C. as ideias são menos nítidas do que as impressões.
D. qualquer ideia que não derive diretamente de uma impressão é falsa.

47 . De acordo com Hume, a questão de facto é:


A. O livre-arbítrio existe ou não existe.
B. A covid-19 é uma doença extremamente contagiosa.
C. Os sólidos ocupam espaço.
D. Um cego total não vê.

48 . De acordo com Hume, um exemplo de verdade necessária é:


A. metade de cento e dez é igual à soma de trinta mais vinte e cinco.
B. no inverno chove mais do que no verão.
C. as folhas da laranjeira são verdes.
D. Lisboa é maior do que Quarteira.

49 . De acordo com Hume, as relações de ideias são


A. empíricas, certas e a priori.
B. a posteriori, certas e necessárias.
C. a priori, meramente analíticas e necessárias.
D. a priori, meramente analíticas e contingentes.

50 . De acordo com Hume, as questões de facto são


A. prováveis, contingentes e a priori.
B. apenas prováveis, substanciais e a posteriori.
C. a posteriori, substanciais e meramente analíticas.
D. contingentes, certas e a posteriori.

44 Editável e fotocopiável © Texto | Dúvida Metódica, 11.º ano


1 Epistemologia

51 . A verdade de uma afirmação é contingente quando ela


A. é provavelmente falsa.
B. é verdadeira, mas podia ser falsa.
C. tem de ser verdadeira.
D. é discutível e controversa.

52 . Hume considera que o conhecimento a priori


A. é substancial, na medida em que é rigoroso e envolve certeza.
B. é substancial, na medida em que consiste numa explicitação de ideias adquiridas
previamente.
C. não é substancial, na medida em que não nos diz algo novo acerca do mundo.
D. não é substancial, na medida em que se baseia na experiência e esta pode ser
enganadora.

53 . Para Hume, a ideia que geralmente as pessoas têm da causalidade é ilusória, pois a
causalidade na verdade é
A. uma relação real.
B. uma conexão necessária entre causa e efeito.
C. um hábito mental nosso que projetamos na natureza.
D. uma relação de ideias e não uma questão de facto.

54 . Lê o texto.
«Quando lanço um pedaço de madeira seca numa lareira, o meu espírito é imediatamente
levado a conceber que ele vai aumentar as chamas, não que as vai extinguir. Esta
transição de pensamento da causa para o efeito não procede da razão. Se não tivesse
assistido muitas vezes a essa situação não pensaria que as chamas iriam aumentar.»

David Hume, Tratados Filosóficos I – Investigação sobre o Entendimento Humano, INCM, Lisboa, 2002, p. 49

Com estas palavras, David Hume pretende dizer exatamente que a


A. causalidade tem uma explicação psicológica.
B. explicação da causalidade envolve o hábito.
C. causalidade não pode ser compreendida a posteriori.
D. causalidade não pode ser compreendida a priori.

55 . Segundo Hume, na causalidade aquilo de que temos realmente experiência é


A. a conexão necessária entre dois fenómenos.
B. um fenómeno a acontecer a seguir a outro.
C. um fenómeno a causar outro.
D. vários fenómenos a provocarem outros fenómenos.

56 . Segundo Hume, por conjugação constante entende-se o


A. facto de a um fenómeno A se seguir habitualmente um fenómeno B.
B. facto de o fenómeno A causar o fenómeno B.
C. facto de não haver relação entre o fenómeno A e o fenómeno B.
D. mesmo que conexão necessária.

Editável e fotocopiável © Texto | Dúvida Metódica, 11.º ano 45


1 Epistemologia

57 . Segundo Hume, a ideia de conexão necessária nasce de


A. uma ideia inata.
B. uma impressão sensorial.
C. uma experiência do mundo que depois é projetada internamente.
D. um sentimento, que depois é projetado no mundo externo.

58 . Lê o texto.
«O senhor Hume tem defendido que só temos esta noção de causa: algo que é anterior
ao efeito e que, de acordo com a experiência, foi seguido constantemente pelo efeito.
Seguir-se-ia desta definição de causa que a noite é a causa do dia e o dia a causa da
noite. Pois, desde o começo do mundo, não houve coisas que se tenham sucedido mais
constantemente. Seguir-se-ia também desta definição que tudo o que seja singular na sua
natureza, ou que seja a primeira coisa do seu género, não pode ter uma causa.»
Thomas Reid, Essays on the Active Powers of Man, IAVE, Exame Nacional de Filosofia, 2017, 2.ª Fase

Neste texto, Thomas Reid defende que


A. a causalidade é equivalente à conjunção constante.
B. não existem conjunções constantes.
C. não existem conexões necessárias entre causas e efeitos.
D. a causalidade não é equivalente à conjunção constante.

59 . Há uma objeção à teoria de Hume acerca da causalidade que diz:


A. a Matemática fornece conhecimento substancial acerca do mundo.
B. só temos experiência das conjunções constantes.
C. a ideia de que a causalidade é a conjunção constante de certos fenómenos e não uma
conexão real entre fenómenos tem consequências absurdas.
D. não pode haver nada na mente que não tenha passado pelos sentidos.

60 . Analisa as afirmações seguintes:


«Que o Sol não se há de levantar amanhã não é uma proposição menos inteligível e não
implica maior contradição do que a afirmação de que ele se levantará.»
David Hume, Investigação sobre o Entendimento Humano, INCM, Lisboa, 2002, p. 42

Ao exprimir-se deste modo, Hume quer dizer que


A. é provável que o Sol não nasça amanhã.
B. não se pode mostrar que a frase «o Sol não nascerá amanhã» é falsa.
C. a frase «o Sol não nascerá amanhã» não é uma contradição.
D. não conseguimos imaginar uma situação em que o Sol não nasça amanhã.
porque
i. se o Sol nascerá ou não amanhã é uma questão de facto e, portanto, a verdade ou
falsidade das afirmações sobre esse assunto é necessária.
ii. se o Sol nascerá ou não amanhã é uma questão de facto e, portanto, a verdade ou
falsidade das afirmações sobre esse assunto é contingente.
iii. se o Sol nascerá ou não amanhã é uma relação de ideias e, portanto, a verdade ou
falsidade das afirmações sobre esse assunto é necessária.
iv. se o Sol nascerá ou não amanhã é uma relação de ideias e, portanto, a verdade ou
falsidade das afirmações sobre esse assunto é contingente.

46 Editável e fotocopiável © Texto | Dúvida Metódica, 11.º ano


1 Epistemologia

61 . De acordo com Hume, se uma proposição não pode ser refutada por meio da experiência
é uma
A. relação de ideias.
B. questão de facto.
C. verdade inata.
D. ilusão.

62 . Segundo Hume, as inferências indutivas pressupõem


A. a regularidade da natureza.
B. o futuro.
C. que a natureza muda constantemente.
D. que a natureza nunca muda.

63 . Segundo Hume, o princípio da uniformidade da natureza consiste na ideia de que


A. a indução é muito útil.
B. na natureza não há mudanças.
C. na natureza tudo é uniforme, ou seja, igual.
D. a natureza é regular.

64 . Segundo Hume, o princípio da uniformidade da natureza


A. pode ser justificado a priori, pois é uma relação de ideias e o pensamento é uma fonte
de conhecimento.
B. não pode ser justificado a priori, mas pode ser justificado a posteriori.
C. não pode ser justificado a priori, pois é uma questão de facto.
D. pode ser justificado a priori, pois podemos imaginar uma natureza que não é regular.

65 . Segundo Hume, o princípio da uniformidade da natureza


A. não pode ser justificado a posteriori, pois isso envolveria uma petição de princípio.
B. não pode ser justificado a priori, pois isso envolveria uma petição de princípio.
C. não é necessário para justificar a indução.
D. contrariamente à indução, não pode ser justificado racionalmente.

66 . Para Hume, a tentativa de justificar a posteriori a ideia de que a natureza é regular levaria
ao raciocínio de que a natureza
A. é regular, logo é confiável.
B. é regular, logo a indução está justificada.
C. tem sido regular, logo é provável que continue a ser regular.
D. não tem sido regular, logo é provável que continue a não ser regular.

67 . Segundo Hume, se for verdadeiro o princípio da uniformidade da natureza é uma verdade


A. necessária.
B. contingente.
C. a priori.
D. lógica.

Editável e fotocopiável © Texto | Dúvida Metódica, 11.º ano 47


1 Epistemologia

68 . Segundo Hume, a ideia de conexão necessária não é


A. a cópia de um sentimento.
B. o resultado de um hábito do sujeito.
C. o resultado de uma projeção psicológica efetuada pelo sujeito.
D. o resultado direto de uma experiência externa.

69 . Lê o texto seguinte.
«Quando olhamos para os objetos exteriores à nossa volta (…) nunca somos capazes de
identificar, num caso singular, qualquer poder ou conexão necessária, qualquer qualidade
que ligue o efeito à causa e torne o primeiro uma consequência infalível da segunda.
Constatamos apenas que um efeito realmente se segue à causa. O impulso da primeira
bola de bilhar é seguido pelo movimento da segunda, e isso é tudo o que é dado aos
nossos sentidos externos.»
David Hume, ibid., pp. 77-78

No excerto transcrito, Hume


A. primeiro afirma que podemos ver as conexões necessárias e depois nega que possamos
ver as conjunções constantes.
B. primeiro afirma que vemos unicamente as conjunções constantes e depois nega que
possamos ver as conexões necessárias.
C. primeiro nega que possamos ver as conexões necessárias e depois afirma que vemos
unicamente as conjunções constantes.
D. primeiro nega que possamos ver as conjunções constantes e depois afirma que
podemos ver as conexões necessárias.

70 . Identifica a frase que não constitui uma objeção a Hume:


A. Não nos devemos esquecer que a negação de uma verdade necessária é contraditória
e não apenas falsa.
B. O facto de se poder perceber a priori que 3496 × 778 = 2 719 888 não implica que esse
resultado seja uma trivialidade linguística.
C. O conhecimento matemático pode não envolver o recurso à experiência e ainda assim
constituir uma explicação do mundo.
D. Se o conhecimento matemático é aplicável ao mundo, então é um conhecimento do
mundo.

GRUPO II
(Outras questões)

1 . O Jacinto sabe que 20 + 20 = 31.


No exemplo apresentado, a palavra «sabe» foi incorretamente usada. Substitui-a por outra
palavra mais adequada.

2 . Uma crítica habitual ao ceticismo radical é que este é incoerente.


Será que o ceticismo moderado também incorre nessa incoerência? Porquê?
3 . Eu tenho olhos verdes.
Para rejeitar essa crença, Descartes necessitaria de recorrer ao argumento do génio
maligno? Porquê?

48 Editável e fotocopiável © Texto | Dúvida Metódica, 11.º ano


1 Epistemologia

4 . Qual é o critério de verdade proposto por Descartes?

5 . Recorrendo apenas ao pensamento não conseguimos conhecer o mundo.


Descartes concordaria com essa ideia? Porquê?

6 . O que são, segundo Hume, as perceções? E as impressões?

7 . Segundo Hume, a ideia de lobisomem deriva ou não da experiência? Porquê?

8 . Uma esfera não tem arestas.


De acordo com Hume, precisamos da experiência para justificar essa proposição? Porquê?

9 . Hume pensava que o costume é o grande guia da vida humana.


Que razões tinha ele para pensar desse modo?

10 . Hume considerou o ceticismo irrefutável.


Significa isso que, na sua perspetiva, os seres humanos não têm nenhum conhecimento?
Porquê?

Editável e fotocopiável © Texto | Dúvida Metódica, 11.º ano 49


2 Filosofia da ciência

GRUPO I
(Questões de resposta fechada)

Indica a alternativa correta.

1 . A Filosofia da ciência
A. estuda a história da ciência.
B. estuda problemas filosóficos colocados pela ciência.
C. é uma ciência social e humana, como a Psicologia e a Sociologia.
D. estuda o modo como as pessoas da atualidade encaram a ciência e os seus riscos.

2 . Geralmente o senso comum não inclui


A. saberes práticos.
B. superstições.
C. crenças decorrentes da experiência de vida.
D. saberes expressos numa linguagem rigorosa.

3 . Qual é a afirmação falsa?


A. Do senso comum fazem parte conhecimentos muito úteis na vida quotidiana.
B. O senso comum apoia-se na experiência de vida e não na experiência científica.
C. O senso comum implica investigações, estudos efetuados metodicamente.
D. Com os recursos do senso comum não seria possível descobrir que a luz do Sol leva
8,33 minutos a chegar à Terra.

4 . O senso comum é adquirido


A. nos livros.
B. graças à experiência de vida.
C. cientificamente.
D. graças à reflexão.

5 . O problema da demarcação pode formular-se do seguinte modo:


A. Como se pode diferenciar o positivismo do falsificacionismo?
B. Como se pode diferenciar uma teoria científica verdadeira de uma teoria científica falsa?
C. Como se pode diferenciar uma teoria científica de uma teoria que não é científica?
D. Como se pode diferenciar a filosofia e a pseudociência?

6 . Qual das seguintes questões apresenta o problema da demarcação?


A. Como se pode diferenciar uma hipótese verdadeira de uma hipótese falsa?
B. Qual é a diferença entre «teorias comprovadas» e «teorias corroboradas»?
C. Onde é que se deve traçar a fronteira entre a ciência e as outras tentativas de explicar
o mundo?
D. Porque é que algumas pessoas valorizam mais as explicações científicas e outras
preferem explicações pseudocientíficas?

50 Editável e fotocopiável © Texto | Dúvida Metódica, 11.º ano


2 Filosofia da ciência

7 . A Filosofia
A. não é uma pseudociência, pois não procura reproduzir os procedimentos da ciência.
B. não é uma ciência, embora tente usar o método científico.
C. é uma disciplina científica, pois visa conhecer o mundo.
D. é uma pseudociência, já que os problemas filosóficos não têm respostas aceites por
todos os especialistas.

8 . Uma pseudociência é uma teoria


A. científica falsa.
B. científica que é alvo de debate.
C. que não pretende ser científica e não compete com a ciência na explicação dos
fenómenos.
D. que não recorre a métodos científicos, mas que tenta competir com a ciência na
explicação dos fenómenos.

9 . Segundo o critério da verificação, uma teoria é científica se


A. é verdadeira ou falsa.
B. consiste em afirmações irrefutáveis.
C. consiste em afirmações empiricamente falsificáveis.
D. consiste em afirmações cuja verdade ou falsidade é possível estabelecer
empiricamente de modo conclusivo.

10 . Uma objeção ao positivismo alega que


A. as leis da natureza não são falsificáveis, pois exprimem-se através de frases universais.
B. a verificabilidade não serve como critério de cientificidade, pois se a utilizarmos
seremos levados a considerar que as leis da natureza não são científicas, o que é muito
implausível.
C. a verificabilidade não serve como critério de cientificidade, pois se a utilizarmos
seremos levados a considerar que as leis da natureza são científicas, o que é muito
implausível.
D. as leis da natureza não são verificáveis, pois exprimem-se através de frases
particulares.

11 . Popper considera que a astrologia é uma pseudociência, pois as suas afirmações


A. são sempre falsas.
B. não são falsificáveis, ou seja, são tão vagas que é difícil submetê-las a testes empíricos.
C. nunca são falsas.
D. não são falsificáveis, ou seja, são verdades objetivas e comprovadas.

12 . Para ser falsificável uma teoria não pode


A. ser corroborada pelos factos.
B. ter fundamento ou justificação.
C. estar blindada contra o fracasso empírico.
D. explicitar ela própria situações cuja ocorrência implique a sua falsidade.

Editável e fotocopiável © Texto | Dúvida Metódica, 11.º ano 51


2 Filosofia da ciência

13 . Segundo Popper, se uma teoria é científica, então


A. pode ser comprovada pela observação.
B. pode ser refutada através de experiências.
C. foi refutada pela observação.
D. será necessariamente falsificada pela experiência.

14 . Indica qual das seguintes afirmações é empiricamente falsificável.


A. As baleias vivem no mar.
B. Os fantasmas podem ser perigosos.
C. Alguns répteis voam.
D. Os fantasmas são perigosos.

15 . Segundo o falsificacionismo, uma condição necessária da cientificidade de uma teoria é


ser falsificável, ou seja, poder
A. ser conclusivamente confirmada através de testes empíricos.
B. revelar-se verdadeira em condições observacionais estipuladas pelo cientista.
C. revelar-se falsa em condições observacionais estipuladas pelo cientista.
D. perceber-se facilmente a sua falsidade.

16 . Segundo Karl Popper, para uma teoria ser científica a falsificabilidade


A. é uma condição suficiente e necessária.
B. não é nem condição suficiente nem necessária.
C. é uma condição suficiente mas não necessária.
D. é uma condição necessária mas não suficiente.

17 . Segundo Karl Popper,


A. a falsificabilidade é uma condição necessária da ciência e isso significa que qualquer
teoria científica tem de ser falsificável.
B. a falsificabilidade é uma condição suficiente da ciência e isso significa que qualquer
teoria científica tem de ser falsificável.
C. a falsificabilidade é uma condição necessária da ciência e isso significa que qualquer
teoria falsificável é científica.
D. a falsificabilidade é uma condição suficiente da ciência e isso significa que qualquer
teoria falsificável é científica.

18 . Popper entende que todas as proposições científicas são falsificáveis. E será que todas as
proposições falsificáveis são científicas?

Ao responder a esta questão Popper não diria:


A. Se uma proposição for científica, será também falsificável, mas uma proposição
falsificável pode não ser científica.
B. Para serem científicas, as proposições além de serem falsificáveis têm de ter poder
explicativo.
C. Para serem científicas, as proposições, além de serem testáveis por meio de tentativas
de refutação, têm de propor explicações relevantes para problemas empíricos.
D. Se a falsificabilidade é uma condição necessária da cientificidade, então também é
uma condição suficiente e nesse caso qualquer proposição falsificável será científica.

52 Editável e fotocopiável © Texto | Dúvida Metódica, 11.º ano


2 Filosofia da ciência

19 . De acordo com a perspetiva falsificacionista sobre a ciência, todas as teorias


A. falsificáveis são científicas.
B. científicas são falsificáveis.
C. falsificáveis são falsificadas.
D. científicas são falsificadas.

20 . De acordo com Popper, para que uma teoria seja aceite pelos cientistas não é requerido
que
A. seja falsificável, para que possa ser submetida a testes experimentais.
B. seja submetida a testes experimentais orientados para a tentativa de falsificação da
teoria e não para a sua confirmação.
C. resista às tentativas de falsificação.
D. tenha um grau de refutabilidade pequeno.

21 . De acordo com Popper, uma boa teoria científica


A. é empiricamente irrefutável.
B. tem um grau de falsificabilidade elevado.
C. tem um grau de falsificabilidade pequeno.
D. não necessita de confrontação empírica.

22 . Considera as seguintes frases e seleciona a alínea que, segundo Popper, as avalia


corretamente.
1. Nenhum planeta tem luz própria.
2. Os astros influenciam imenso o destino humano.
A. 1 e 2 são falsificáveis.
B. Nem 1 nem 2 são falsificáveis.
C. 1 é falsificável e 2 não é falsificável.
D. 1 não é falsificável e 2 é falsificável.

23 . 1. No próximo mês haverá um pequeno terramoto em Portugal.


2. No próximo mês haverá um pequeno terramoto em Lisboa.

Segundo Popper,
A. 1 é mais falsificável que 2.
B. 2 é mais falsificável que 1.
C. O grau de falsificabilidade de 1 e 2 é semelhante.
D. Nem 1 nem 2 são falsificáveis.

24 . Segundo Popper, quanto maior é o grau de falsificabilidade de uma teoria


A. menos corroborada ela está.
B. menos científica ela é.
C. menos conteúdo empírico ela tem.
D. mais conteúdo empírico ela tem.

Editável e fotocopiável © Texto | Dúvida Metódica, 11.º ano 53


2 Filosofia da ciência

25 . Todos os professores de Filosofia têm mais de dois metros e meio de altura.


Esta afirmação
A. não é científica, pois não é falsificável.
B. não é científica, apesar de ser falsificável.
C. é científica, pois é falsificável.
D. não é científica apenas porque é falsa.

26 . O indutivismo é
A. uma teoria filosófica acerca do modo como a ciência procede.
B. uma teoria científica acerca do modo como a ciência procede.
C. o conjunto das etapas que um cientista percorre para fazer as suas descobertas.
D. o método usado pela maioria dos cientistas.

27 . Segundo o indutivismo, a submissão das hipóteses científicas a testes empíricos tem o


objetivo de as
A. corroborar.
B. tentar refutar.
C. refutar.
D. confirmar.

28 . O indutivismo considera que uma investigação científica deve


A. assentar na experiência do mundo.
B. assentar no conhecimento anterior.
C. principiar com hipóteses obtidas indutivamente.
D. principiar com testes muito rigorosos.

29 . Uma objeção ao indutivismo é a ideia de que


A. as experiências servem para confirmar as hipóteses.
B. muitas teorias científicas referem entidades inexistentes.
C. a recolha de dados deve ser o início da investigação.
D. muitas teorias científicas referem entidades inobserváveis.

30 . Uma das objeções à perspetiva indutivista do método científico é a seguinte:


A. sem conhecer à partida algumas teorias, os dados empíricos raramente são
compreensíveis.
B. registar e classificar muitos dados empíricos é indispensável para formular boas
teorias.
C. a indução, apesar de ser apenas provável, é confiável.
D. os cientistas devem esforçar-se por interpretar os dados de modo objetivo.

31 . A observação pura não é possível.


Esta afirmação faz parte de uma objeção ao
A. falsificacionismo.
B. positivismo.
C. indutivismo.
D. historicismo.

54 Editável e fotocopiável © Texto | Dúvida Metódica, 11.º ano


2 Filosofia da ciência

32 . Os cientistas admitem que uma parte do universo é constituída por matéria negra.
Esta não é visível, podendo apenas ser inferida através dos seus efeitos gravitacionais
sobre a matéria visível (estrelas, galáxias, etc.).

Este caso permite fazer uma objeção à perspetiva


A. indutivista do método científico, pois se há coisas inobserváveis estudadas pela ciência
não é possível que as investigações científicas comecem pela observação.
B. indutivista do método científico, pois mostra que a observação pura não é possível e
que qualquer observação pressupõe teorias prévias.
C. falsificacionista do método científico, pois se há coisas inobserváveis estudadas pela
ciência não é possível formular hipóteses falsificáveis acerca delas.
D. falsificacionista do método científico, pois mostra que a observação pura não é possível
e que qualquer observação pressupõe teorias prévias.

33 . Uma conjetura é uma


A. hipótese.
B. teoria confirmada.
C. teoria corroborada.
D. hipótese que não é falsificável.

34 . Segundo Popper, a criação de uma teoria é


A. uma generalização hipotética destinada a tentar resolver um problema.
B. uma generalização destinada a tentar resolver um problema.
C. uma consequência da observação.
D. fruto da criatividade intelectual.

35 . Segundo Popper, uma teoria corroborada é uma teoria que


A. é verdadeira.
B. está falsificada.
C. está indutivamente comprovada.
D. até ao momento resistiu às tentativas de falsificação.

36 . De acordo com Popper, a característica distintiva de uma teoria corroborada é que


A. é falsificável.
B. é verdadeira.
C. é verosímil.
D. está provada.

37 . Corroboração significa:
A. confirmação conclusiva.
B. provado como verdadeiro.
C. a hipótese foi submetida a testes e ainda não foi refutada.
D. a hipótese ainda não foi submetida a testes e ainda não foi falsificada.

Editável e fotocopiável © Texto | Dúvida Metódica, 11.º ano 55


2 Filosofia da ciência

38 . Segundo Popper, o facto de a indução ser injustificável


A. torna a ciência uma atividade irracional.
B. não tem importância para a ciência, pois a indução não desempenha nesta nenhum
papel.
C. não é razão para rejeitar o indutivismo.
D. torna implausível o falsificacionismo.

39 . Uma das objeções à perspetiva de Popper acerca do método científico é a seguinte:


A. em ciência nunca se pode fazer uma observação pura.
B. muitas explicações científicas não têm origem na observação.
C. os cientistas trabalham principalmente com o objetivo de confirmar as suas teorias.
D. os cientistas têm uma atitude crítica em relação às suas teorias.

40 . Lê o texto:
«Nunca se pode provar nem afirmar que uma teoria científica é verdadeira. Quando muito,
pode provar-se que é falsa – se se realizar um teste cujos resultados sejam contrários às
suas previsões».
Jorge Buescu, O Mistério do Bilhete de Identidade e Outras Histórias,
9.ª edição, Gradiva, Lisboa, 2004, p. 13 (adaptado)

Esta frase enquadra-se na perspetiva


A. indutivista.
B. falsificacionista.
C. positivista.
D. historicista.

41 . Lê o texto:
«Aquilo em que nós acreditamos (…) não é que a teoria de Newton ou a de Einstein sejam
verdadeiras, mas sim boas aproximações à verdade, podendo ser superadas por outras
melhores.»
Karl Popper, O Realismo e o Objetivo da Ciência,
D. Quixote, Lisboa, 1997, p. XX

Popper quer dizer que as teorias de Newton e de Einstein


A. são falsas.
B. são provavelmente verdadeiras.
C. estão corroboradas.
D. são provavelmente falsas.

42 . Tanto Descartes como Popper consideram que a submissão das nossas crenças ou opiniões
a um severo exame crítico é um aspeto central do método de procura da verdade. Porém,
Descartes e Popper divergem quanto aos resultados da aplicação desse método, pois
A. não concordam que a maneira mais adequada de procurar a verdade e o
conhecimento seja começar por pôr em causa as nossas crenças ou opiniões.
B. Descartes admite encontrar verdades definitivas, ao passo que Popper defende que as
teorias científicas (empíricas) são apenas aproximações à verdade, que poderão vir a
ser revistas.
C. Descartes valoriza mais a observação que Popper.
D. Descartes preocupava-se com a filosofia e Popper com a ciência.

56 Editável e fotocopiável © Texto | Dúvida Metódica, 11.º ano


2 Filosofia da ciência

43 . Segundo Popper, a afirmação correta é:


A. se uma teoria é conjetural, não é objetiva.
B. uma teoria para ser objetiva tem de ser verdadeira.
C. as hipóteses científicas podem ter surgido através de um procedimento racional
e objetivo, mas depois são testadas de modo irracional e subjetivo.
D. as hipóteses científicas podem ter surgido de modo irracional e subjetivo, mas depois
são testadas de modo racional e objetivo.

44 . De acordo com Popper, a ciência é objetiva porque


A. as descobertas científicas são verdades confirmadas.
B. as teorias que passam com sucesso os testes de falsificabilidade são verdadeiras.
C. a avaliação das teorias é feita com base em testes imparciais.
D. os cientistas são pessoas sérias.

45 . De acordo com Popper, a ciência progride pela substituição de


A. teorias falsas por teorias verdadeiras.
B. teorias falsificadas por teorias que resistem aos testes.
C. teorias falsas por outras teorias falsas.
D. teorias infalsificáveis por teorias falsificáveis.

46 . Acerca da evolução da ciência Popper não diria que


A. a ciência evolui através da acumulação de certezas e confirmações.
B. para que a ciência evolua é preciso ter uma atitude crítica relativamente às teorias
científicas.
C. corrigir erros e substituir teorias falsas faz a ciência aproximar-se da verdade.
D. as teorias científicas não devem ser encaradas como dogmas, por muito corroboradas
que estejam.

47 . Popper fez uma comparação entre a evolução da ciência e o processo de seleção natural.
O facto de serem comparáveis tem a seguinte implicação:
A. as teorias mais aptas e bem-sucedidas ficam conclusivamente comprovadas após
superarem testes rigorosos e exigentes.
B. uma vez que todas as teorias científicas, mesmo as mais recentes, podem conter erros,
o progresso da ciência é, em grande medida, uma ilusão.
C. a comunidade científica não tem razões para aceitar uma teoria em vez de outra, uma
vez que mesmo as teorias que não foram falsificadas podem conter erros.
D. a comunidade científica aceita trabalhar com as teorias que superaram testes rigorosos
e exigentes e põe de lado – ou, pelo menos, impõe-lhes alterações substanciais – as
teorias que «chumbaram» nesses testes.

48 . Suposições teóricas gerais, e leis e técnicas para a sua aplicação adotadas por uma
comunidade científica específica.
Segundo Kuhn, esta frase diz respeito ao conceito de
A. paradigma.
B. ciência normal.
C. ciência extraordinária.
D. revolução científica.

Editável e fotocopiável © Texto | Dúvida Metódica, 11.º ano 57


2 Filosofia da ciência

49 . Os cientistas aplicam uma teoria tentando explicar aspetos relevantes do mundo.


Segundo Kuhn, trata-se do conceito de
A. paradigma.
B. ciência normal.
C. ciência extraordinária.
D. revolução científica.

50 . Comparação e discussão de modelos teóricos alternativos.


Segundo Kuhn, trata-se do conceito de
A. paradigma.
B. ciência extraordinária.
C. crise.
D. revolução científica.

51 . Acumulação de dificuldades explicativas.


Segundo Kuhn, trata-se do conceito de
A. anomalia.
B. ciência extraordinária.
C. crise.
D. revolução científica.

52 . Substituição do modelo teórico aceite pela comunidade científica.


Segundo Kuhn, trata-se do conceito de
A. paradigma.
B. ciência extraordinária.
C. crise.
D. revolução científica.

53 . Teoria da relatividade, proposta por Einstein e aceite pela comunidade científica atual.
Segundo Kuhn, é um exemplo de
A. paradigma.
B. ciência normal.
C. ciência extraordinária.
D. revolução científica.

54 . Kuhn considera que há períodos de consenso e períodos de divergência na comunidade


científica. O fim de um período de consenso e a consequente entrada num período de
divergência devem-se
A. à acumulação de anomalias.
B. à resolução de enigmas.
C. à atitude crítica própria da ciência normal.
D. à evolução da ciência.

58 Editável e fotocopiável © Texto | Dúvida Metódica, 11.º ano


2 Filosofia da ciência

55 . Lê o texto:

«Considere-se (…) os homens que chamaram louco a Copérnico por este proclamar
que a Terra se movia. Eles não estavam simplesmente errados, nem completamente
errados. Para eles, a ideia de posição fixa fazia parte do significado de “Terra”. (...) De
modo correspondente, a inovação de Copérnico não se limitava a mover a Terra. Era,
em vez disso, todo um novo modo de olhar para os problemas da física e da astronomia,
um modo de olhar que mudava necessariamente o significado quer de “Terra”, quer de
“movimento”».
Thomas Kuhn, A Estrutura das Revoluções Científicas,
Guerra & Paz, Lisboa, 2009, p. 205

Para Kuhn, exemplos como o do excerto transcrito apoiam a ideia de que paradigmas
diferentes são
A. extraordinários.
B. comparáveis.
C. incomensuráveis.
D. revolucionários.

56 . Segundo Kuhn, quando uma comunidade científica se dedica sobretudo à resolução de


enigmas, a ciência encontra-se num período
A. revolucionário.
B. não paradigmático.
C. de ciência extraordinária.
D. de ciência normal.

57 . Segundo Kuhn, existem períodos de ciência normal, durante os quais


A. se registam progressos cumulativos.
B. diversos paradigmas competem entre si.
C. os cientistas procuram a falsificação de teorias.
D. é impossível descobrir-se anomalias.

58 . Segundo Kuhn,
A. as revoluções científicas são frequentes na história da ciência.
B. anomalias frequentes podem originar um período de crise numa determinada ciência.
C. uma simples anomalia é suficiente para derrubar um paradigma.
D. a «ciência normal» desenvolve-se à margem de qualquer paradigma.

59 . Kuhn defende que


A. não existe qualquer forma de progresso científico.
B. a ciência permite descobrir como é realmente a natureza.
C. cada teoria representa melhor a realidade do que as teorias anteriores.
D. o desenvolvimento da ciência não é uma aproximação à verdade objetiva.

60 . Segundo Kuhn, uma revolução científica ocorre porque


A. na ciência não há progresso em direção à verdade.
B. ocorre uma competição e mudança de paradigma.
C. os paradigmas são incomensuráveis.
D. a ciência normal é mais frequente do que a ciência extraordinária.

Editável e fotocopiável © Texto | Dúvida Metódica, 11.º ano 59


2 Filosofia da ciência

61 . Lê o texto:

«Uma das coisas que uma comunidade científica adquire através de um paradigma
(enquanto este estiver em vigor) é um critério para escolher problemas que sejam
solucionáveis. Em boa medida, estes são os únicos problemas que a comunidade considera
como científicos ou merecedores de atenção.»
Thomas Kuhn, ibid., p. 87

Os problemas a que o excerto transcrito se refere são


A. anomalias e ocorrem em períodos de ciência normal.
B. enigmas e ocorrem em períodos de ciência normal.
C. enigmas e ocorrem em períodos de ciência extraordinária.
D. anomalias e ocorrem em períodos de ciência extraordinária.

62 . A substituição da teoria geocêntrica de Ptolomeu pela teoria heliocêntrica de Copérnico é


um exemplo de:
A. ciência extraordinária.
B. revolução científica
C. crise.
D. anomalia.

63 . Segundo Kuhn, a ciência normal é praticada com o objetivo de


A. consolidar o antigo paradigma e aplicá-lo a novos domínios.
B. derrubar um paradigma e resolver anomalias.
C. consolidar o paradigma dominante e aplicá-lo a novos domínios.
D. tentar falsificar o paradigma dominante.

64 . Do ponto de vista de Kuhn, os cientistas, ao preferirem uma teoria científica a outra,


utilizam critérios como por exemplo:
A. racionalidade e objetividade.
B. simplicidade e exatidão.
C. inconsistência e exatidão.
D. objetividade e exatidão.

65 . De acordo com Kuhn, as revoluções científicas ocorrem quando


A. há uma anomalia.
B. há uma crise.
C. a ciência normal é substituída pela ciência extraordinária.
D. um paradigma dominante é substituído por um novo paradigma.

66 . Segundo Kuhn, a ciência


A. é objetiva.
B. não é objetiva.
C. tem uma objetividade limitada.
D. tem a discussão crítica como uma característica essencial.

60 Editável e fotocopiável © Texto | Dúvida Metódica, 11.º ano


2 Filosofia da ciência

67 . Dizer, como Kuhn, que os paradigmas são incomensuráveis, não significa que
A. cada paradigma permite uma visão do mundo que é singular e profundamente
diferente da visão permitida pelos outros paradigmas.
B. cada paradigma «fala» uma linguagem diferente dos outros.
C. os paradigmas são incomparáveis.
D. um paradigma é melhor do que outro.

68 . Uma das objeções à teoria de Kuhn rejeita a incomensurabilidade dos paradigmas, dado
que
A. há teorias científicas que permitem realizações técnicas que outras não permitem e,
portanto, é plausível que sejam melhores do que elas.
B. o novo paradigma evita as anomalias do paradigma anterior.
C. a maior parte dos cientistas acaba por escolher um dos paradigmas.
D. no período de ciência extraordinária ocorrem discussões intensas.

69 . Conjunto de teorias e regras metodológicas que serve de matriz a uma dada disciplina
científica durante um certo período de tempo e que estipula como devem ser conduzidas
as investigações.
De acordo com Kuhn, trata-se de
A. um paradigma.
B. um puzzle.
C. uma revolução científica.
D. um progresso científico.

70 . Mudança radical na direção de uma disciplina científica, caracterizada pela alteração das
suas teorias fundamentais e modos de investigar.
De acordo com Kuhn, trata-se de
A. uma crise.
B. uma anomalia.
C. uma revolução científica.
D. um momento de ciência extraordinária.

Editável e fotocopiável © Texto | Dúvida Metódica, 11.º ano 61


2 Filosofia da ciência

GRUPO II
(Outras questões)

1 . Dá três exemplos de problemas científicos e indica as ciências que os estudam.


2 . Dá um exemplo de uma afirmação que
A. não seja nem verificável nem falsificável.
B. seja verificável mas não falsificável.
C. seja falsificável mas não verificável.

3 . Existe uma perspetiva filosófica que descreve o método científico do modo a seguir
apresentado, mas com as etapas ordenadas de forma diferente.
Identifica essa perspetiva e ordena corretamente as etapas.
A. Caso a conjetura não seja refutada em nenhuma das experiências, é provisoriamente
aceite como a melhor teoria disponível.
B. A hipótese é testada por meio de tentativas de falsificação, ou seja, de testes
experimentais severos concebidos para identificar possíveis erros.
C. Os cientistas propõem uma conjetura, uma tentativa de solução.
D. Os cientistas deparam-se com um problema.

4 . Existe uma perspetiva filosófica que descreve o método científico do modo a seguir
apresentado, mas com as etapas ordenadas de forma diferente.
Identifica essa perspetiva e ordena corretamente as etapas.
A. Os cientistas propõem uma hipótese explicativa.
B. Os cientistas conduzem observações cuidadosas e deparam-se com um fenómeno que
não sabem explicar.
C. Caso muitas experiências sejam favoráveis à hipótese esta é declarada verdadeira e
considera-se que está provada.
D. Os cientistas realizam experiências para averiguar se a sua suposição é verdadeira ou
falsa.

5 . Será que Descartes e Karl Popper valorizavam a certeza do mesmo modo? Porquê?

6 . «A ciência começa com a observação, afirma Francis Bacon. Proponho-me substituir esta
fórmula baconiana por outra. A ciência começa com ________________.»
Karl Popper, O Mito do Contexto,
Edições 70, Lisboa, 2009, p. 161

Completa a frase de Popper usando apenas uma ou duas palavras.

7 . Qual das seguintes afirmações é mais falsificável? Porquê?


A. A leitura de livros de poesia melhora a capacidade de expressão escrita.
B. A leitura de livros de literatura melhora a capacidade de expressão escrita.

8 . De acordo com uma objeção que se faz a Karl Popper, se este tivesse razão não seria
racional da nossa parte andar de avião. Explica porquê.

62 Editável e fotocopiável © Texto | Dúvida Metódica, 11.º ano


2 Filosofia da ciência

9 . Identifica, de acordo com a teoria de Thomas Kuhn, o conceito referido em cada uma das
seguintes frases.
A. Algo que não tem explicação ou justificação à luz de uma teoria aceite, sendo
frequentemente ignorado.
B. Muitos enigmas são facilmente resolvidos, pois surgem respostas no âmbito do próprio
paradigma através da pesquisa científica.
C. Abandono de um conjunto articulado de conceitos e regras metodológicas e sua
substituição por outro conjunto.
D. Realização científica de caráter exemplar que propõe uma certa visão de mundo, isto é,
um conjunto sistemático de ideias para explicar como é o mundo.
E. Resolução frequente de enigmas ou «quebra-cabeças».
F. Criação e debate de teorias alternativas.
G. Um conjunto de anomalias é considerado grave se põe em causa componentes
fundamentais do paradigma vigente e persistentemente resiste a uma solução
por intermédio dos recursos que esse paradigma põe à disposição dos cientistas,
provocando um ambiente de dúvida.

10 . A teoria da relatividade de Einstein é tida em conta pelos cientistas quando planeiam a


aterragem de uma nave em Marte. A teoria geocêntrica, pelo contrário, não tem qualquer
papel nesse planeamento.
Com base neste exemplo, podemos criticar a teoria de Karl Popper ou a teoria de Thomas
Kuhn? Porquê?

Editável e fotocopiável © Texto | Dúvida Metódica, 11.º ano 63


3 Filosofia da arte

GRUPO I
(Questões de resposta fechada)

Indica a alternativa correta.

1 . O problema da definição da arte pode formular-se através da seguinte questão:


A. O que é uma obra de arte com valor?
B. O que distingue as várias artes umas das outras?
C. O que leva as pessoas a criar arte?
D. O que é a arte?

2 . Para uma definição de arte ser uma boa definição, basta


A. permitir distinguir o que é arte do que não é arte.
B. permitir distinguir a boa arte da má arte.
C. aplicar-se a todas as obras de arte.
D. aplicar-se só às obras de arte.

3 . «Um livro é um objeto feito de papel.»


A explicação mais completa para o facto de esta não ser uma boa definição de livro é que
A. nem todos os livros são feitos de papel.
B. nem todos os objetos feitos de papel são livros.
C. não apresenta nem condições necessárias nem condições suficientes.
D. ignora desenvolvimentos recentes, como os ebooks.

4 . Considera as seguintes frases e seleciona a alínea que as avalia corretamente:


1. Segundo as teorias essencialistas, há determinadas características que estão presentes
em todas as obras de arte e que constituem a natureza da arte.
2. Segundo as teorias não-essencialistas não há características intrínsecas comuns a
todas as obras de arte.
3. Tanto as teorias essencialistas como as teorias não-essencialistas reconhecem
condições necessárias e suficientes definidoras da arte.
A. As três frases são falsas.
B. As três frases são verdadeiras.
C. As frases 1 e 2 são verdadeiras. A frase 3 é falsa.
D. As frases 1 e 3 são verdadeiras. A frase 2 é falsa.

5 . «A pintura de um centauro representa algo, apesar de não existirem centauros.»


Esta frase relaciona-se explicitamente com a tese de que a essência da arte é
A. representar simbolicamente o mundo.
B. imitar a natureza.
C. exprimir os sentimentos do artista acerca do mundo.
D. apresentar determinadas formas capazes de emocionar esteticamente o recetor.

6 . «A pintura é como um espelho que se coloca diante dos objetos.»


Esta metáfora relaciona-se com a tese de que a essência da arte é
A. representar simbolicamente o mundo.
B. imitar a natureza.
C. exprimir os sentimentos do artista acerca do mundo.
D. apresentar determinadas formas capazes de emocionar esteticamente o recetor.

64 Editável e fotocopiável © Texto | Dúvida Metódica, 11.º ano


3 Filosofia da arte

7 . Lê o texto seguinte.
«O facto óbvio é que uma imagem, para representar um objeto, tem de ser um símbolo
deste, tem de estar em seu lugar, referir-se a ele; e nenhum grau de semelhança é
suficiente para estabelecer a relação de referência exigida. Nem a semelhança é
necessária para a referência; quase tudo pode estar no lugar de tudo.»
Nelson Goodman, Linguagens da Arte,
Gradiva, Lisboa, 2006, p. 37

O autor do excerto transcrito


A. critica a tese de que a arte deve ser imitativa e defende a tese de que a arte consiste
numa representação simbólica.
B. defende a tese de que a arte deve ser imitativa e critica a tese de que a arte consiste
numa representação simbólica.
C. defende quer a tese de que a arte deve ser imitativa quer a tese de que a arte consiste
numa representação simbólica.
D. critica quer a tese de que a arte deve ser imitativa quer a tese de que a arte consiste
numa representação simbólica.

8 . Lê o texto seguinte.
«[DN] – O que quer transmitir com este novo trabalho [o álbum musical The Solution is
Resteless]?
[Joan] – Não escrevo canções a pensar que quero dizer coisas. Escrevo-as para expressar o
que sinto e como me sinto.»
In Diário de Notícias, entrevista com a cantora Joan As Police Woman,
19/02/2022

A resposta da cantora pode enquadrar-se na teoria


A. formalista da arte.
B. representacional da arte.
C. expressivista da arte.
D. institucional da arte.

9 . Lê o texto seguinte.
«Para os ________ , a arte é para o mundo interior das emoções como a ciência para o
mundo exterior. A ciência tem como objeto eventos físicos, enquanto a arte tem como
objeto as emoções humanas que ela exprime.»
Cláudio F. Costa, «Teorias da arte»,
in Crítica – https://criticanarede.com/est_tarte.html
(consultado em 24/11/2021)

O espaço em branco, no excerto transcrito, deve ser preenchido com a seguinte palavra:
A. formalistas.
B. intencionalistas.
C. historicistas.
D. expressivistas.

Editável e fotocopiável © Texto | Dúvida Metódica, 11.º ano 65


3 Filosofia da arte

10 . Lê o texto seguinte.
«Antes de o artista produzir a sua obra ele ainda não possui a emoção (…) que a sua obra
produzirá na audiência e em si mesmo. O que ele possui é uma “excitação emocional”, um
sentimento indefinido e incompreensível.»
Cláudio F. Costa, ibid. (adaptado)

O excerto transcrito refere-se a uma tese de


A. R. G. Collingwood.
B. Clive Bell.
C. George Dickie.
D. Jerrold Levinson.

11 . Lê o texto seguinte.
«Antes de o artista produzir a sua obra ele ainda não possui a emoção (…) que a sua obra
produzirá na audiência e em si mesmo. O que ele possui é uma “excitação emocional”, um
sentimento indefinido e incompreensível.”
Cláudio F. Costa, ibid. (adaptado)

A frase que, de acordo com a teoria expressivista de R. G. Collingwood, pode continuar


este texto é:
A. ao criar a obra, o artista contagia a audiência com esse sentimento indefinido.
B. ao criar a obra, o artista clarifica esse sentimento, dando a oportunidade à audiência
para fazer o mesmo.
C. o artista e a sua obra serão depois recebidos no mundo da arte.
D. ao criar a obra, o artista tenta representar o mundo de modo realista para, assim,
clarificar esse sentimento.

12 . Lê o texto seguinte.
«Todos sabemos perfeitamente bem que a arte não é ofício; e tudo o que desejo fazer é
relembrar ao leitor as diferenças bem conhecidas que separam as duas coisas.»
R. G. Collingwood, «Os Princípios da Arte»,
in Crítica – https://criticanarede.com/principiosdaarte.html
(consultado em 24/11/2021)

Uma das diferenças referidas por Collingwood é que:


A. na arte, a distinção entre meios e fins é crucial e no ofício pode nem existir.
B. no ofício, a distinção entre meios e fins é crucial e na arte pode nem existir.
C. a distinção entre meios e fins é importante quer na arte quer nos ofícios, mas na arte é
essencial.
D. a distinção entre meios e fins é muito importante quer na arte quer nos ofícios, mas nos
ofícios é completamente crucial.

13 . Segundo a teoria formalista, a emoção estética é uma emoção


A. especial, que só temos quando estamos diante de uma obra de arte.
B. especial, que temos diante da natureza ou de obras de arte.
C. sentida pelo artista e comunicada ao recetor da obra.
D. que temos diante de qualquer coisa bela.

66 Editável e fotocopiável © Texto | Dúvida Metódica, 11.º ano


3 Filosofia da arte

14 . Um defensor da teoria formalista da arte não diria que a forma significante


A. provoca a emoção estética.
B. é diferente da forma física dos objetos.
C. é uma certa relação entre as partes da obra.
D. depende do rigor do conteúdo representado ou expresso.

15 . Esta música prende a nossa atenção devido à sua harmonia e equilíbrio.


Esta afirmação enquadra-se na teoria
A. da arte como representação.
B. da arte como expressão.
C. formalista da arte.
D. institucional da arte.

16 . Definir o conceito de arte recorrendo à noção de mundo da arte é um círculo vicioso.


Esta afirmação contém uma objeção à teoria
A. da arte como expressão.
B. formalista da arte.
C. institucional da arte.
D. histórica da arte.

17 . Uma objeção à teoria formalista sustenta que esta é circular, pois


A. o conceito de forma significante, que é fundamental na teoria, é extremamente vago e
impreciso.
B. considera que a forma é mais importante do que o conteúdo.
C. aplica-se mais facilmente a alguns tipos de arte que a outros.
D. explica os conceitos de «forma significante» e «emoção estética» remetendo um para o
outro.

18 . A arte não é uma categoria unificada de coisas, não podendo por isso ser definida.
Esta tese é defendida
A. pela teoria institucional da arte.
B. pela teoria histórica da arte.
C. pelo ceticismo acerca da definição de arte.
D. pela teoria expressivista da arte.

19 . Lê o texto seguinte.
«Muitas teorias erraram ao tentar definir o conceito de arte em termos de condições
necessárias e suficientes, isto é, tratando “arte” como um conceito fechado.»
Célia Teixeira, «Disputas acerca da arte»,
in Crítica – https://criticanarede.com/filos_est.html
(consultado em 24/11/2021 e adaptado)

As teorias referidas no excerto transcrito são


A. as teorias essencialistas.
B. as teorias não-essencialistas.
C. as teorias essencialistas e o ceticismo acerca da definição de arte.
D. quer as teorias essencialistas quer as teorias não-essencialistas.

Editável e fotocopiável © Texto | Dúvida Metódica, 11.º ano 67


3 Filosofia da arte

20 . Lê o texto seguinte.
«O mundo da arte é uma instituição social, em nome da qual certos indivíduos com a
autoridade relevante atuam de modo a conferir o estatuto de “candidato a apreciação”
a alguns aspetos de certos artefactos, que contam como obras de arte em virtude deste
procedimento.»
Kathleen Stock, «A definição da arte»,
in Crítica – https://criticanarede.com/defarte.html
(consultado em 24/11/2021 e adaptado)

Esta tese arte é defendida pela teoria


A. formalista.
B. institucional.
C. histórica.
D. expressivista.

21 . Analisa as frases seguintes:


i) Como é que um certo objeto adquire o estatuto de obra de arte?
ii) Quais são as características de um certo objeto que o fazem ser uma obra de arte?

Seleciona a alternativa correta.


A. i) Teorias essencialistas. ii) Teorias não-essencialistas.
B. i) Teorias não-essencialistas. ii) Teorias essencialistas.
C. i) Teorias não-essencialistas. ii) Ceticismo acerca da definição de arte.
D. i) Ceticismo acerca da definição de arte. ii) Teorias não-essencialistas.

22 . Lê o texto seguinte.
«X é uma obra de arte no momento de tempo t se, e só se, a pessoa que tem um direito
de posse sobre X tenciona que X seja encarado “do modo (ou modos), seja ele qual for,
em que os objetos na extensão de ‘obra de arte’ antes de t são ou foram correta ou
comummente encarados”.»
Kathleen Stock, «A definição da arte»,
in Crítica – https://criticanarede.com/defarte.html
(consultado em 24/11/2021)

A definição de arte apresentada no excerto transcrito é proposta pela teoria


A. formalista.
B. expressivista.
C. histórica.
D. institucional.

23 . A definição historicista de arte assenta num critério recursivo, ou seja, considera que
A. na arte a continuidade é mais importante do que a diversidade.
B. um objeto é arte na medida em que se assemelha às obras de arte anteriores.
C. propostas radicalmente novas não devem ser consideradas artísticas.
D. um objeto é arte na medida em que é para ser encarado como o foram as obras de arte
do passado.

68 Editável e fotocopiável © Texto | Dúvida Metódica, 11.º ano


3 Filosofia da arte

24 . Os autores de falsificações de obras de arte têm a intenção séria de que estas sejam
olhadas do mesmo modo que as obras de arte anteriores. Mas, apesar disso, estas não são
realmente obras de arte.
Esta é uma objeção à teoria
A. da arte como expressão.
B. formalista da arte.
C. institucional da arte.
D. histórica da arte.

25 . Para a teoria formalista


A. a arte tem de representar alguma coisa: trata-se de uma característica necessária.
B. a representação não é essencial: uma obra de arte tanto pode representar algo como
não representar.
C. a representação artística deve ser acompanhada pelos sentimentos do artista.
D. o conteúdo de uma obra de arte é mais importante do que a sua forma.

26 . O caráter criativo e inovador da arte faz com que aquilo que é chamado arte vá mudando
incessantemente. Por isso, Morris Weitz defende que
A. o conceito de «arte» é um conceito aberto.
B. não faz sentido usar a palavra «arte».
C. devemos ser especialmente cuidadosos ao procurar definir a arte.
D. as características definidoras da arte não são intrínsecas, mas contextuais.

27 . A teoria institucional não defende que


A. as características distintivas das obras de arte são propriedades relacionais.
B. o contexto social em que obras de arte são apreciadas é fundamental.
C. uma obra de arte tem de ser um artefacto.
D. as características distintivas das obras de arte existem nas próprias obras.

28 . Para a teoria institucional, um artefacto é algo


A. produzido pelo próprio artista.
B. produzido ou modificado pelo artista.
C. produzido ou escolhido pelo artista.
D. escolhido pelo artista.

29 . Do mundo da arte não fazem parte os


A. artistas e os galeristas.
B. críticos de arte e o público.
C. historiadores de arte.
D. políticos.

30 . A teoria institucional é uma teoria


A. classificativa e não-avaliativa.
B. avaliativa e não-classificativa.
C. que pretende dizer o que é e não é arte e distinguir a boa e a má arte.
D. que apenas pretende distinguir a boa e a má arte.

Editável e fotocopiável © Texto | Dúvida Metódica, 11.º ano 69


3 Filosofia da arte

31 . Obras que habitualmente são consideradas artísticas, mas cujos autores nada têm a ver
com o meio artístico, podem ser consideradas contraexemplos à teoria
A. expressivista.
B. formalista.
C. representacional.
D. institucional.

32 . Para a teoria histórica, para uma obra ser arte é necessário que o autor
A. tenha a intenção de a inserir numa tradição histórica.
B. tenha a intenção de que esta seja bela.
C. tenha a intenção de que esta seja expressiva.
D. consiga com ela despertar determinadas emoções no público.

33 . Jerrold Levinson sublinha a importância da intenção, pois


A. sem esta a semelhança com as obras de arte do passado poderia ser ocasional.
B. a arte é tudo aquilo que os artistas querem que seja arte.
C. a intencionalidade é uma característica necessária das ações humanas.
D. a intencionalidade é um sentimento que o artista deve expressar.

34 . Os graffitis feitos em paredes alheias são contraexemplos à teoria histórica, pois


A. não procuram ser belos.
B. mostram que o direito de propriedade não é uma condição necessária para haver arte.
C. não se relacionam com a história da arte.
D. não revelam nenhuma intenção artística.

35 . Segundo a teoria histórica, algo é arte se for visto como o eram as obras anteriores, e as
anteriores como as anteriores, e assim sucessivamente. Isso suscita uma objeção a essa
teoria, pois
A. se é assim, não se consegue explicar como é que a arte tem mudado tanto ao longo da
história.
B. se é assim, não se consegue explicar a existência da primeira obra de arte.
C. as obras de arte mais antigas eram piores do que as mais modernas.
D. não é possível descobrir factos históricos que comprovem a teoria.

GRUPO II
(Outras questões)

1 . Indica uma semelhança e uma diferença entre a teoria institucional e a teoria formalista
da arte.
2 . De acordo com a teoria expressivista de Collingwood, qual deverá ser o principal objetivo
do artista ao criar uma obra de arte?

3 . Como responde Weitz à pergunta: é possível definir arte? Porquê?

4 . Esclarece, de acordo com a teoria institucional, quem faz parte do mundo da arte e
apresenta uma objeção a essa teoria que incida nesse conceito.

5 . Segundo a teoria histórica, um artista pode transformar em arte algo que não lhe
pertence? Justifica.

70 Editável e fotocopiável © Texto | Dúvida Metódica, 11.º ano


4 Filosofia da religião

GRUPO I
(Questões de resposta fechada)

Indica a alternativa correta.

1 . Qual é a questão que pertence à filosofia da religião?


A. O que é a fé?
B. O que é uma crença?
C. O que é o conhecimento?
D. O que é uma boa justificação?

2 . Uma religião politeísta é uma religião


A. cujos crentes pensam de maneira muito diferente.
B. sem deuses.
C. com vários deuses.
D. só com um Deus.

3 . Uma religião monoteísta é uma religião


A. com vários deuses.
B. só com um Deus.
C. cujos crentes pensam de maneira muito semelhante.
D. com poucos crentes.

4 . Chama-se henoteísmo à crença


A. numa pluralidade de seres divinos ou deuses dignos de adoração.
B. de que existem diversos deuses, mas apenas um é digno da nossa adoração.
C. de que o universo foi criado por um ser muito poderoso que deve ser reverenciado.
D. de que Deus é uma energia criadora.

5 . O ateísmo defende que


A. não existe Deus, mas existem deuses.
B. as crenças religiosas são verdadeiras.
C. não há nenhuma divindade.
D. não se pode garantir nem que Deus existe nem que Deus não existe.

6 . Qual é a característica que não faz parte do conceito teísta de Deus?


A. Bondade.
B. Eternidade.
C. Omnisciência.
D. Impessoalidade.

7 . Segundo o teísmo, Deus não é


A. um criador indiferente às criaturas.
B. omnipresente e criador do universo.
C. todo-poderoso e moralmente perfeito.
D. um ser pessoal, embora radicalmente diferente dos seres humanos.

Editável e fotocopiável © Texto | Dúvida Metódica, 11.º ano 71


4 Filosofia da religião

8 . Lê o texto seguinte.

«Não consigo conceber um Deus pessoal que tenha influência direta nas ações dos
indivíduos ou que julgue as criaturas que criou. A minha religiosidade consiste numa
humilde admiração pelo espírito infinitamente superior que se revela no pouco que
conseguimos compreender sobre o mundo passível de ser conhecido. Essa convicção
profundamente emocional da presença de um poder superior racional, que se revela
nesse universo incompreensível forma a minha ideia de Deus.»

Esta afirmação acerca de Deus, atribuída ao físico Albert Einstein, manifesta uma
conceção
A. teísta.
B. deísta.
C. politeísta.
D. cristã.

9 . Qual é a religião que não é monoteísta?


A. Judaísmo.
B. Cristianismo.
C. Islamismo.
D. Religião da Grécia Antiga.

10 . Nenhuma coisa do mundo pode ser causa de si própria.


Esta frase é uma premissa do argumento
A. do desígnio.
B. da aposta.
C. cosmológico.
D. ontológico.

11 . Uma coisa não pode simplesmente existir: tem de haver algo que a provocou.
Esse facto ilustra explicitamente a ideia de que
A. na natureza todo o acontecimento tem uma causa.
B. nenhum acontecimento é a causa de si mesmo.
C. na natureza não há cadeias infinitas de causa/efeito.
D. tem de existir uma causa primeira.

12 . O Rui não é o seu próprio pai. Esse facto ilustra especificamente a ideia de que
A. tem de existir uma causa primeira
B. nenhum acontecimento é a causa de si mesmo.
C. na natureza não há cadeias infinitas de causa/efeito.
D. na natureza todo o acontecimento tem uma causa.

13 . Não pode haver uma regressão infinita das causas.


Esta frase é uma premissa do argumento
A. do desígnio.
B. da aposta.
C. cosmológico.
D. ontológico.

72 Editável e fotocopiável © Texto | Dúvida Metódica, 11.º ano


4 Filosofia da religião

14 . De acordo com o argumento cosmológico, todas as coisas situadas no espaço e no tempo


têm causas anteriores
A. e, portanto, Deus também tem uma causa anterior a ele mesmo.
B. mas Deus não tem, pois está para além do espaço e do tempo.
C. e isso é contraditório com o facto de se dizer que Deus é incausado.
D. mas Deus não tem, apesar de se situar no espaço e no tempo.

15 . Dizer «X tem um caráter teleológico» significa que X


A. é objetivo e não subjetivo.
B. é o final do processo e não o seu início.
C. está organizado em função de um fim a atingir.
D. tem um desígnio consciente.

16 . Se encontramos a frase «É verdade que a verdade não existe» desenhada na areia da


praia, a hipótese mais plausível é que essa frase
A. está ali e pronto, não sendo necessário invocar nenhum tipo de causa para a explicar.
B. foi formada pela ação do vento.
C. se formou devido a um acaso desconhecido qualquer.
D. foi escrita por um ser que lá esteve antes de nós e que sabe escrever.

17 . É inequívoco que o argumento teleológico pressupõe que


A. na natureza existe ordem, ou seja, uma articulação harmoniosa das diferentes partes.
B. na natureza existe ordem, ou seja, desígnios isolados para cada coisa.
C. Deus existe.
D. todos os animais têm alguma forma de inteligência na medida em que os seus corpos e
comportamentos têm explicações teleológicas.

18 . Qual dos seguintes argumentos é uma versão (distinta da versão de Tomás de Aquino) do
argumento teleológico?
A. Objetos artificiais, como os relógios, e coisas naturais, como os seres vivos, requerem
causas anteriores. Essas causas, por sua vez, também requerem causas. Esse processo
causal é longo, mas não pode ser interminável. Consequentemente, esse processo teve
uma origem que não requer nenhuma causa e a essa origem chamamos Deus.
B. Um relógio é constituído por inúmeras peças que trabalham articuladamente para
dar as horas e, como tal, foi intencionalmente feito por um relojoeiro. Os seres vivos
também são constituídos por diversos órgãos e o seu funcionamento conjunto permite
que esses seres realizem as suas atividades. Como tal, é implausível que os seres
vivos tenham surgido por acaso e é provável que tenham tido um criador que os fez
intencionalmente, ou seja, Deus.
C. Deus é, por definição, o ser supremo. Se não existisse, não seria supremo.
Consequentemente, é contraditório dizer que Deus não existe.
D. No universo existem coisas contingentes, que existem, mas podiam não existir. Se todas
as causas fossem contingentem não se conseguiria explicar a existência dessas coisas.
Consequentemente, tem de existir um ser necessário, a que chamamos Deus, que é a
causa derradeira de tudo o mais.

Editável e fotocopiável © Texto | Dúvida Metódica, 11.º ano 73


4 Filosofia da religião

19 . Tomás de Aquino, na sua versão do argumento teleológico, não alega que


A. há desígnio nas plantas e animais apesar da sua falta de consciência ou compreensão.
B. se X é desprovido de inteligência, mas funciona de acordo com uma certa finalidade, a
origem desta é algo de exterior a X.
C. os corpos naturais atuam segundo processos teleológicos que os levam a obter os
melhores resultados.
D. as coisas naturais desprovidas de inteligência alcançam as suas finalidades de modo
acidental.

20 . Lê o texto seguinte.
«Aqueles que aceitam este argumento afirmam que para onde quer que olhemos,
sobretudo tratando-se da natureza – quer olhemos para árvores, falésias, animais ou seja
o que for –, encontramos cada vez mais indícios que confirmam a existência de Deus, uma
vez que se trata de coisas bastante intrincadas e ajustadas entre si.»
Nigel Warburton, Elementos Básicos de Filosofia,
Gradiva, Lisboa, p. XX (adaptado)

No excerto transcrito, o argumento referido é


A. o argumento cosmológico.
B. o argumento ontológico.
C. o argumento teleológico.
D. a aposta de Pascal.

21 . «Achar que o mundo não tem um criador é o mesmo que afirmar que um dicionário é o
resultado de uma explosão numa tipografia.»
Tendo em conta o que sabemos sobre os dicionários, esta afirmação, atribuída ao político
e cientista Benjamin Franklin, enquadra-se no argumento
A. do desígnio.
B. da aposta.
C. cosmológico.
D. ontológico.

22 . Qual das seguintes ideias não pode fazer parte de uma objeção ao argumento
teleológico?
A. É defensável que este argumento é uma petição de princípio, pois a existência de Deus
é disfarçadamente pressuposta nas premissas.
B. Mesmo que a explicação da ordem existente na natureza implique a existência de um
ser inteligente capaz de a criar, nada prova que esse ser seja omnisciente e sumamente
bom.
C. Tentar provar a existência de Deus apenas com base no conceito de Deus e sem ter em
conta informação a posteriori é uma tarefa condenada ao fracasso.
D. As coisas naturais e as coisas artificiais são comparáveis em certos aspetos, mas
existem entre elas diferenças tão grandes que, o facto destas últimas implicarem
necessariamente um criador consciente e capaz de agir intencionalmente, não significa
que as primeiras também tenham de o ter.

74 Editável e fotocopiável © Texto | Dúvida Metódica, 11.º ano


4 Filosofia da religião

23 . Qual das seguintes frases é uma premissa do argumento ontológico?


A. Não conseguimos pensar num ser mais perfeito do que Deus.
B. As maravilhas da natureza não se devem ao acaso.
C. Não há uma cadeia causal que regrida infinitamente.
D. Se Deus existe, estaremos melhor sendo crentes do que não sendo.

24 . Segundo o argumento ontológico, a possibilidade de se conceber um ser com todas as


perfeições
A. prova que Deus existe no pensamento.
B. implica a existência real desse ser.
C. mostra que o pensamento humano tem um alcance infinito.
D. mostra que a existência não é um predicado.

25 . Lê o texto seguinte.
« _____________ baseia-se em exemplos de sofrimento intenso, em seres humanos
ou animais, que aparentemente não servem qualquer propósito benéfico. (…). Por que
permitiria então Deus que isto acontecesse quando, se existe, podia tê-lo impedido com
tanta facilidade?»
William L. Rowe, Introdução à Filosofia da Religião,
Verbo, Lisboa, p. XX

No excerto transcrito, o espaço em branco deve ser preenchido com a expressão


A. A natureza humana.
B. O problema do mal.
C. A existência humana.
D. A grandeza divina.

26 . O facto de existir muito sofrimento no mundo é difícil de conciliar com a existência de um


Deus bondoso e todo-poderoso.
Esta afirmação faz parte
A. do chamado «problema do mal».
B. de uma resposta ao «problema do mal».
C. do argumento do desígnio.
D. de uma crítica ao argumento do desígnio.

27 . Considera as seguintes frases:


1. Se acreditarmos em Deus e este existir, o nosso ganho – a felicidade eterna – é infinito.
2. Se acreditarmos em Deus e este não existir, a nossa perda é pequena.
3. Se não acreditarmos em Deus e este existir, a nossa perda será enorme.
4. Se não acreditarmos em Deus e este não existir, o nosso ganho será enorme.

Pascal concordava com


A. todas as frases.
B. apenas com a 1 e a 4.
C. apenas com a 1 e a 3.
D. apenas com a 1, a 2 e a 3.

Editável e fotocopiável © Texto | Dúvida Metódica, 11.º ano 75


4 Filosofia da religião

28 . A ideia fundamental do fideísmo é que os assuntos religiosos, como a existência de Deus,


A. não podem ser justificados por meio de argumentos, mas apenas pela fé.
B. não podem ser justificados pela fé, mas apenas por meio de argumentos.
C. não fazem qualquer sentido.
D. devem ser comprovados por provas racionais.

29 . Para Pascal «apostar» na existência de Deus é um procedimento


A. apenas intelectual.
B. apenas prático.
C. intelectual (defender a existência de Deus) e prático (participar em cerimónias
religiosas).
D. intelectual (participar em cerimónias religiosas) e prático (defender a existência de
Deus).

30 . Pascal não disse que


A. a fé é Deus conhecido pelo coração e não pela razão.
B. o coração tem razões que a razão desconhece.
C. se há um Deus, ele é infinitamente incompreensível.
D. todas as religiões monoteístas têm o mesmo valor.

31 . A teoria da evolução das espécies, de Darwin, relaciona-se com o argumento do desígnio por
A. mostrar cientificamente a força desse argumento.
B. explicar a ordem natural sem recorrer à hipótese de um criador divino, o que permite
criticar o argumento.
C. explicar a ordem natural sem recorrer à hipótese de um criador divino, o que permite
defender o argumento.
D. explicar a existência de Deus através de mecanismos naturais.

32 . Lê o texto seguinte.
«É plausível argumentar que o universo tem vários “defeitos de conceção”: por exemplo, o
olho humano tem uma tendência para a miopia e para criar cataratas com a idade – o que
dificilmente pode ser considerado a obra de um Criador todo-poderoso que desejasse criar
o melhor mundo possível.»
Nigel Warburton, Elementos Básicos de Filosofia,
Gradiva, Lisboa, p. XX (adaptado)

As considerações do excerto transcrito podem enquadrar-se numa objeção ao argumento


A. do desígnio.
B. da aposta.
C. cosmológico.
D. ontológico.

33 . O facto de na nossa vida existirem dificuldades permite-nos desenvolver o nosso valor.


Esta frase faz parte
A. do chamado «problema do mal».
B. de uma resposta ao «problema do mal».
C. do argumento da aposta.
D. de uma objeção ao argumento da aposta.

76 Editável e fotocopiável © Texto | Dúvida Metódica, 11.º ano


4 Filosofia da religião

34 . Qual é a frase em que não se tenta compatibilizar a existência de mal e a existência do


Deus teísta?
A. O mal resulta de escolhas humanas livres e é o «preço» que temos de pagar pela
existência de livre-arbítrio.
B. Não sabemos o suficiente para garantir que o mal existente não tem qualquer
propósito.
C. Sem mal não existiriam muitas das virtudes humanas.
D. Um Deus omnipotente e omnisciente conseguiria criar um mundo em que não existisse
mal e os seres humanos fossem livres e responsáveis.

35 . Qual é a frase em que não se apresenta uma objeção ao argumento da aposta de Pascal?
A. Se compararmos os argumentos a favor e contra a existência de Deus constamos que
existe uma espécie de empate.
B. Um Deus perfeito talvez não ficasse contente com o facto de a crença na sua existência
resultar de um cálculo de interesses.
C. As crenças não são voluntárias e, por isso, não podemos decidir acreditar em Deus.
D. A «aposta de Pascal» promove uma religiosidade pouco sincera.

GRUPO II
(Outras questões)

1 . Dá dois exemplos de problemas estudados na Filosofia da religião.


2 . Justifica a seguinte afirmação: os argumentos cosmológico e teleológico são argumentos
a posteriori.

3 . Qual é, segundo o argumento ontológico, a consequência de não admitirmos que o


predicado da existência está incluído nos atributos divinos?

4 . Descreve o objetivo de uma teodiceia.

5 . De acordo com Pascal, se «apostarmos» na existência de Deus, podemos ganhar o quê?

Editável e fotocopiável © Texto | Dúvida Metódica, 11.º ano 77


PROPOSTAS DE RESOLUÇÃO

Capítulo 1 possível obter conhecimento a pode dizer-se que este é um guia


Epistemologia priori substancial (ou seja, acerca para nós.
do mundo) – como, por exemplo, 10. Não. De acordo com Hume,
GRUPO I
o conhecimento do cogito, da os seres humanos, apesar
1. D 25. C 49. C
existência de Deus e até de leis de saberem menos do que
2. D 26. D 50. B
da Física. geralmente julgam, têm
3. B 27. C 51. B
6. Para Hume, as perceções são os alguns conhecimentos: têm
4. D 28. B 52. C
conteúdos mentais: impressões e conhecimentos relativos a
5. C 29. B 53. C
ideias. As impressões são obtidas relações de ideias e a questões
6. A 30. A 54. D
pelos sentidos ao experienciar de facto ligadas à experiência
7. A 31. D 55. B
os seus objetos específicos. imediata. Essa é uma das razões
8. B 32. A 56. A
Podem ser externas (as sensações que levaram Hume a declarar-se
9. A 33. D 57. D
visuais, auditivas, olfativas, etc.) cético moderado e não radical.
10. C 34. C 58. D
ou internas (os sentimentos e os
11. D 35. C 59. C
desejos).
12. B 36. B 60. C/ii Capítulo 2
7. Sim, deriva. Todas as ideias
13. D 37. D 61. A
derivam da experiência (ou seja, Filosofia da ciência
14. C 38. C 62. A
são cópias das impressões). A
15. C 39. A 63. D GRUPO I
ideia de lobisomem é uma ideia
16. D 40. A 64. C 1. B. 25. B. 49. B.
complexa e imaginária, pelo
17. A 41. B 65. A 2. D. 26. A. 50. C.
que não é a cópia direta de uma
18. C 42. D 66. C 3. C. 27. D. 51. C.
impressão: resulta da mistura
19. C 43. A 67. B 4. B. 28. A. 52. D.
pela imaginação de outras
20. D 44. B 68. D 5. C. 29. D. 53. A.
ideias (homem e lobo) que, essas
21. A 45. A 69. C 6. C. 30. A. 54. A.
sim, derivam diretamente de
22. A 46. D 70. A 7. A. 31. C. 55. C.
impressões.
23. D 47. B 8. D. 32. A. 56. D.
8. Não precisamos da experiência
24. D 48. A 9. D. 33. A. 57. A.
para fazer essa justificação e
10. B. 34. D. 58. B.
GRUPO II perceber que a proposição é
11. B. 35. D. 59. D.
1. Pensa / julga / acredita. verdadeira. Trata-se de uma
12. C. 36. C. 60. B.
2. Não. O ceticismo radical incorre relação de ideias e pode ser
13. B. 37. C. 61. B.
(segundo os seus críticos) numa conhecida a priori, bastando
14. A. 38. B. 62. B.
incoerência, pois defende que analisar os conceitos usados.
15. C. 39. C. 63. C.
não sabemos nada e ao mesmo (Os conceitos de esfera e aresta
16. D. 40. B. 64. B.
tempo pretende saber algo derivam da experiência, mas
17. A. 41. C. 65. D.
(que não sabemos nada). Ora, o agora que já os possuímos
18. D. 42. B. 66. C.
ceticismo moderado duvida da não precisamos de consultar
19. B. 43. D. 67. D.
possibilidade do conhecimento novamente a experiência para
20. D. 44. C. 68. A.
em áreas específicas, pelo que a fazer a justificação referida.)
21. B. 45. B. 69. A.
sua posição não é atingida pela 9. Hume pensava que não
22. C. 46. A. 70. C.
dúvida que suscita. conseguimos justificar
23. B. 47. D.
3. Não. O argumento do génio racionalmente as inferências
24. D. 48. A.
maligno é apenas necessário indutivas e causais, nem através
para rejeitar as crenças a de procedimentos a priori nem GRUPO II
priori, como é o caso das através de procedimentos 1. Como se formaram os
crenças matemáticas. A crença a posteriori. Para ele, tais continentes? (Geologia). Como
apresentada é empírica e, por inferências baseiam-se na se formam os buracos negros?
isso, basta o argumento do sonho experiência, mas não na (Física / Astronomia) Como
para a rejeitar (posso sonhar experiência externa. Trata-se da surgiram os primeiros seres
que tenho olhos verdes e na experiência interna: o costume ou vivos? (Biologia)
realidade não os ter). hábito de esperar que o futuro se 2. A: Quem tem uma aura pura
4. É o critério da clareza e da assemelhe ao passado e o efeito não se deixa afetar pelo caos
distinção: se uma ideia é clara e se siga à causa. Essas inferências exterior. B: Alguns alunos perdem
distinta, então é verdadeira. são fundamentais na vida prática a concentração ao fim de 10
5. Descartes não concordaria com e, como se baseiam no costume, minutos. C: O movimento dos
essa ideia. Na sua opinião, é

Editável e fotocopiável © Texto | Dúvida Metódica, 11.º ano 79


PROPOSTAS DE RESOLUÇÃO

planetas é determinado pela 9. A: Anomalia. B: Ciência normal. 3. Weitz considera que não é
gravidade. C: Revolução científica. possível definir de forma
3. A perspetiva é o falsificacionismo. D: Paradigma. E: Ciência normal. adequada a arte, já que as várias
1: D. 2: C. 3: B. 4: A. F: Ciência extraordinária. G: Crise. teorias se contrariam umas às
4. A perspetiva é o indutivismo. 10. A teoria de Kuhn, pois este outras e são alvo de objeções
1: B. 2: A. 3: D. 4: C. defende que os paradigmas e contraexemplos. A arte é um
5. Não. Descartes procurou uma são incomensuráveis, não se conceito aberto, não possui
crença indubitável (ou seja, podendo dizer que um é melhor características necessárias e
totalmente certa) para com do que outro e que a mudança suficientes que possam ser
ela poder fundamentar o de um para outro constitui captadas por uma definição, por
conhecimento e obter outras um progresso científico. Ora, isso é um conceito indefinível.
certezas. Mas Popper pensava se a teoria da relatividade 4. Do «mundo da arte» fazem parte
que uma afirmação ou teoria permite realizações práticas artistas, críticos, historiadores
que se apresente como sendo que teorias anteriores, como o de arte, galeristas e o próprio
totalmente certa não pode geocentrismo, não permitem, é público.
constituir uma explicação implausível não reconhecer que Objeção à teoria institucional:
séria e relevante do mundo. a teoria da relatividade é melhor o conceito de «mundo da arte»
Para Popper, as afirmações (= explica melhor o mundo) do é vago, pois não sabemos
e as teorias não devem ser que outras e, portanto, constitui exatamente quais são as suas
indubitáveis, mas sim falsificáveis um progresso relativamente a regras, o que é legítimo ou
– devem ser tentativas ousadas elas. não um membro fazer, etc.
de explicar o mundo e, por isso, Esta teoria lida mal com obras
correm o risco de falhar. Por consideradas artísticas, mas
outro lado, Popper pensava que Capítulo 3 cujos autores não fazem parte do
a ciência não pode aspirar à mundo da arte: intuitivamente,
certeza: mesmo que uma teoria Filosofia da arte não parece certo que se tornem
resista a inúmeros testes, nunca GRUPO I arte apenas quando alguém do
se pode garantir que um dia não 1. D 13. A 25. B mundo da arte as reconhece (e,
se vai revelar falsa. Em suma: 2. A 14. D 26. A portanto, tais obras constituem
para Descartes a certeza era 3. C 15. C 27. D contraexemplos).
uma virtude epistemológica, mas 4. B 16. C 28. C 5. Não. De acordo com esta teoria,
para Popper a certeza não é uma 5. A 17. D 29. D um objeto só é uma obra de
virtude epistemológica. 6. B 18. C 30. A arte, se o artista possui direito
6. problemas / um problema. 7. A 19. D 31. D de propriedade sobre este
7. A afirmação mais falsificável é 8. C 20. B 32. A e, além disso, tem ou teve a
a B: trata-se de uma afirmação 9. D 21. B 33. A firme intenção de que este seja
mais geral e abrangente e, por 10. A 22. C 34. C encarado como as obras de arte
isso, corre um risco maior de 11. B 23. D 35. B anteriores foram encaradas.
falhar. Dito de outro modo: os 12. B 24. D A referência ao direito de
casos que podem falsificar A propriedade tem o objetivo de
falsificam também B e pode GRUPO II impossibilitar que um artista
haver casos que falsificam B e 1. Uma semelhança: ambas as possa, sem autorização dos
não falsificam A. teorias da arte (a formalista e a proprietários, tentar transformar
8. Para Popper, uma teoria institucional) consideram que em arte coisas que não lhe
científica nunca deixa de ser há características necessárias e pertençam.
uma mera hipótese, por muito suficientes definidoras da arte.
corroborada que esteja. Nunca Uma diferença: para a teoria
estamos em condições de dizer formalista, as condições
Capítulo 4
que é uma teoria verdadeira. necessárias e suficientes são algo
Mas se fosse assim não seria que é intrínseco às obras de arte Filosofia da religião
racional confiar nessa teoria. (a forma significante), enquanto
GRUPO I
Se não tivéssemos boas razões para a teoria institucional
1. A 9. D 17. A
para considerar verdadeiras as são algo que é contextual ou
2. C 10. C 18. B
teorias científicas subjacentes relacional (o reconhecimento
3. B 11. A 19. D
ao funcionamento dos aviões pelo «mundo da arte»).
4. B 12. B 20. C
estaríamos a ser irracionais 2. Principal objetivo do artista ao
5. C 13. C 21. A
ao usá-los. Mas isso é muito criar uma obra de arte: expressar
6. D 14. B 22. C
implausível e, por isso, é provável e clarificar emoções, ganhando
7. A 15. C 23. A
que Popper esteja enganado. consciência e compreensão de
8. B 16. D 24. B
certas emoções particulares.

80 Editável e fotocopiável © Texto | Dúvida Metódica, 11.º ano


PROPOSTAS DE RESOLUÇÃO

25. B 29. C 33. B Argumento cosmológico: parte 3. Deus não seria perfeito, já que
26. A 30. D 34. D da constatação de que existem lhe faltaria um predicado (ou
27. D 31. B 35. A coisas no universo e estas não se qualidade): a existência.
28. A 32. A causam a si próprias, têm causas O argumento ontológico
anteriores. pressupõe que seria incoerente
GRUPO II
Argumento teleológico: parte ter a ideia de Deus e não
1. Dois problemas (por exemplo):
da constatação de que as reconhecer a sua existência.
Será que Deus existe?
coisas existentes, apesar da sua 4. Objetivo de uma teodiceia:
O que é a fé?
complexidade, funcionam de apresentar razões que expliquem
2. São ambos a posteriori porque o
forma ordenada, harmoniosa e o facto de Deus permitir a
conteúdo das premissas se baseia
com uma determinada finalidade existência do mal no mundo.
em informações retiradas da
(ou desígnio). 5. A vida eterna.
experiência.

Editável e fotocopiável © Texto | Dúvida Metódica, 11.º ano 81


Testes
TESTES,
PROPOSTAS DE RESOLUÇÃO
E GUIÕES DE ESTUDO

O Teste 1 é um teste diagnóstico. A sua aplicação


possibilita a revisão de alguns conteúdos do 10.º ano
que também irão ser utilizados no 11.º ano.
Os Testes 2 a 5 dizem respeito aos capítulos 1, 2, 3 e 4
do manual.
Cada professor fará as adaptações que considerar
pedagogicamente adequadas, aumentando ou
diminuindo o número de itens, tendo em conta
a duração das suas aulas (blocos de 45/50 ou
90/100 minutos), o perfil dos alunos e o momento
da aplicação. Assim sendo, não são apresentadas
cotações para os itens.
Caso se pretenda incluir nos testes mais ou menos
itens ou combinar temas/problemas de capítulos
diferentes, poderão ser utilizados alguns dos que se
encontram nas Questões-aula e no Banco de questões.
Os Guiões de Estudo para os testes destinam-se aos
alunos.

© Texto | Dúvida Metódica, 11.º ano


1 Teste diagnóstico

Teste 1
Teste diagnóstico

(Revisão de alguns conteúdos do 10.º ano aplicáveis no 11.º ano)

1 . Estabelece as correspondências corretas entre as alíneas e os argumentos de 1 a 10,


apresentados no quadro seguinte.

A. Negação dupla. B. Modus ponens. C. Modus tollens. D. Contraposição.


E. Silogismo hipotético. F. Silogismo disjuntivo. G. Lei de De Morgan, negação da
disjunção. H. Lei de De Morgan, negação da conjunção. I. Falácia da afirmação
da consequente. J. Falácia da negação da antecedente.

Não é verdade que Deus e o diabo existem.


1
Logo, Deus não existe ou o diabo não existe.

Se és sincero, não dizes mentiras.


2 Não dizes mentiras.
Logo, és sincero.

Se Deus existe, então não há mal no mundo e a vida humana tem sentido.
Contudo, é falso que não haja mal no mundo e que a vida humana tenha
3
sentido.
Logo, Deus não existe.

Se matar é sempre errado, então a eutanásia não é moralmente permissível.


4 Sendo assim, caso a eutanásia seja moralmente permissível, matar não é
sempre errado.

O argumento X convenceu os defensores da paz ou os apoiantes da guerra.


5 O argumento X não convenceu os apoiantes da guerra.
Logo, o argumento X convenceu os defensores da paz.

É falso que a vida não tem sentido.


6
Logo, a vida tem sentido.

É falso que o aborto ou a eutanásia sejam impermissíveis.


7
Por isso, nem o aborto nem a eutanásia são impermissíveis.

Se me amas, não me mentes.


8 Não me amas.
Logo, mentes-me.

No caso de ouvires música clássica frequentemente, ficarás mais inteligente.


E se fores mais inteligente, terás melhores classificações nos trabalhos esco-
9
lares. Portanto, no caso de ouvires essa música com frequência, terás melho-
res classificações nos trabalhos escolares.

Se o argumento X é sólido, então não tem conclusão falsa.


10 Ora, esse argumento é sólido.
Por isso, a sua conclusão não é falsa.

Editável e fotocopiável © Texto | Dúvida Metódica, 11.º ano 85


1 Teste diagnóstico

2 . «Se existem verdades morais universais, então os valores morais não são relativos.»
Se esta frase for conclusão de uma contraposição, qual será a premissa?

3 . Considera a frase a seguir apresentada como premissa de um modus tollens e completa o


argumento.
«Se a intenção não tem valor moral, então Kant não tinha razão.»

4 . Constrói um modus ponens que inclua, como premissa, a frase «A miséria é injusta».

5 . Estabelece as correspondências corretas entre as alíneas e os argumentos não dedutivos


de 1 a 10, apresentados no quadro seguinte. Uma alínea pode aplicar-se a mais do que um
número.

A. Generalização. B. Previsão. C. Argumento por analogia. D. Argumento de autoridade.

O planeta Filosófico, a 20 anos-luz da Terra, tem água, oxigénio, zonas com


1 temperaturas amenas e estações diferentes ao longo do ano.
Por isso, tal como sucede na Terra, talvez nele existam seres vivos.

Gostei de todos os livros de David Lodge que li.


2 Por isso, aposto que também gostarei do livro que ele vai publicar
no próximo ano.

A Organização Mundial de Saúde, num comunicado feito em março de 2020,


recomendou a lavagem frequente das mãos, de modo a evitar o contágio
3
por organismos infeciosos.
Por isso, devemos lavar frequentemente as mãos.

Em muitos aspetos, o mundo é como uma máquina.


4 Ora, as máquinas são criadas por seres inteligentes.
Logo, o mundo foi criado por um ser inteligente.

Os seres humanos que ficaram mais de dois meses sem comer morreram.
5 Logo, nenhum ser humano consegue ficar mais de dois meses sem comer e
permanecer vivo.

Os gatos costumam ronronar.


6
Por isso, o gato que tenho aqui comigo é bem capaz de começar a ronronar.

Conheço alguns futebolistas profissionais e todos eles gostam de passear na


7 praia.
Por isso, qualquer futebolista profissional gosta de passear na praia.

A Natália e a Teresa usam jeans e sapatos de ténis.


8 A Natália gosta de dormir.
Logo, a Teresa também gosta de dormir.

86 Editável e fotocopiável © Texto | Dúvida Metódica, 11.º ano


1 Teste diagnóstico

6 . Estabelece as correspondências corretas entre as alíneas e as falácias de 1 a 11,


apresentadas no quadro seguinte. Uma letra pode aplicar-se a mais do que um número.

A. Derrapagem. B. Falso dilema. C. Falácia ad hominem. D. Espantalho. E. Apelo ao povo.


F. Apelo à ignorância. G. Falsa relação causal. H. Petição de princípio.

Na semana passada, na praia Lógica Tropical, descobriu-se uma coisa muito


boa: o consumo de gelados aumentou. Infelizmente, depois descobriu-se
1 também uma coisa muito má: alguém furtou as redes de voleibol dos cam-
pos da praia. Por isso, é preciso parar imediatamente com a venda de gela-
dos nessa praia, para evitar mais furtos das redes de voleibol.

Einstein tinha dúvidas acerca da veracidade da mecânica quântica. Contudo,


não nos podemos esquecer que esse grande cientista foi infiel à mulher e
2
tratou de modo displicente um filho doente. Por isso, as suas dúvidas acerca
da mecânica quântica não tinham qualquer razão de ser.

Se reconhecermos alguns direitos aos animais, por exemplo o direito de


não sofrerem desnecessariamente e não serem torturados, talvez tenhamos
3
também de lhes reconhecer o direito à vida e… adeus ao bife com batatas
fritas! Depois, bem… ainda acabamos a conceder-lhes o direito de voto.

Nenhuma pessoa, incluindo os filósofos, conseguiu estabelecer inequivoca-


4 mente a objetividade da ética. Sendo assim, é óbvio que a ética não é obje-
tiva.

O meu pai diz que mentir é errado e que não devo dizer mentiras. Só que
5 eu já o surpreendi a mentir diversas vezes. A conclusão que retiro daí é que,
afinal, mentir é correto.

6 Os seres humanos têm livre-arbítrio, pois são livres.

O cantor Zé da Viola vendeu milhões de discos e os seus concertos estão


7 sempre cheios. As pessoas estão sempre a elogiá-lo! Por isso, é claro que a
sua música é boa.

Aprecias filmes violentos? Então a violência não te incomoda. Mas, se a vio-


lência não te incomoda, não és uma pessoa sensível. Mas… se não és uma
8
pessoa sensível… está visto que acabas por maltratar os outros. E se maltra-
tas os outros, és mau. Portanto, se aprecias filmes violentos, és mau. Mau!

Existe um ser criador do mundo, omnipotente, omnisciente e infinitamente


9
bom. Logo, Deus existe.

Tu és a favor das guerras ou és contra as guerras. Mas… eu sei, eu conheço-te:


10
tu não és a favor das guerras. Logo, és contra as guerras.

O Hugo diz que os animais têm direitos pois são seres sencientes, ou seja,
11 que podem sofrer. Eu não concordo com ele! Não concordo, pois é absurdo
dizer que os animais são superiores aos seres humanos.

Editável e fotocopiável © Texto | Dúvida Metódica, 11.º ano 87


1 Teste diagnóstico

7 . O que é um argumento por analogia?

8 . Qual é o argumento não-dedutivo cuja utilidade deriva do facto de muitas vezes


precisarmos de confiar no que outras pessoas nos dizem?

9 . Que falácia cometeremos se, ao argumentar, concluirmos que qualquer guerra é injusta
com base no exemplo de uma única guerra?

10 . Como podemos mostrar que a conclusão de uma generalização é falsa?

11 . Lê o seguinte texto:

«Os males do mundo devem-se tanto a deficiências morais quanto à falta de inteligência.
Mas a humanidade não descobriu até agora qualquer método para erradicar as deficiências
morais. (…) A inteligência, pelo contrário, é facilmente aperfeiçoada através de métodos que
todos os educadores competentes conhecem. Sendo assim, até que se descubra um método
para ensinar a virtude, o progresso terá de ser alcançado através do aperfeiçoamento da
inteligência, e não da moral.»
Bertrand Russel, citado por Anthony Weston, A Arte de Argumentar,
Gradiva, Lisboa, 1996, p. 22

11.1 Qual é a tese defendida no texto?

11.2 Que razões invoca o autor para justificar a sua tese?

88 Editável e fotocopiável © Texto | Dúvida Metódica, 11.º ano


PROPOSTAS
PROPOSTA DE
DERESOLUÇÃO
RESOLUÇÃODO
DOTESTE
TESTEDIAGNÓSTICO
1

Teste diagnóstico 6.
(Revisão de alguns conteúdos do 10.º ano aplicáveis 1. G 5. C 9. H
no 11.º ano) 2. C 6. H 10. B
3. A 7. E 11. D
1. 4. F 8. A
1. H 6. A
2. I 7. G 7. Um argumento por analogia é um argumento em
3. C 8. J que se defende que, se duas coisas são semelhantes
4. D 9. E em alguns aspetos mais evidentes, é provável que
5. F 10. B também sejam semelhantes noutros aspetos menos
evidentes. Em Filosofia, os argumentos por analogia
2. A premissa será: Se os valores morais são relativos, são bastante usados.
então não existem verdades morais universais.
8. É o argumento de autoridade.
3. Se a intenção não tem valor moral, então Kant não
tinha razão. 9. Falácia da generalização precipitada.
Kant tinha razão.
Logo, a intenção tem valor moral. 10. Apresentando um contraexemplo.

4. Eis uma possibilidade: 11.


Se a miséria é injusta, então devemos organizar 11.1
a sociedade de modo a impedir a existência de A tese é: o progresso terá de ser alcançado através
miséria. do aperfeiçoamento da inteligência, e não da moral.
A miséria é injusta. 11.2
Logo, devemos organizar a sociedade de modo a As razões são:
impedir a existência de miséria. • A humanidade ainda não descobriu nenhum
método para erradicar as deficiências morais.
5. • A inteligência é facilmente aperfeiçoada através
1. C 4. C 7. A de métodos que os educadores competentes
2. B 5. A 8. C conhecem.
3. D 6. B

89
2 Epistemologia – os problemas da definição, da possibilidade e da origem do conhecimento

Teste 2
GRUPO I

Seleciona a alternativa correta.

1 . Qual é o conhecimento que pode ser obtido a priori?


A. Trezentos e setenta e nove é menor do que duzentos e dois mais cento e oitenta e nove.
B. O planeta Terra tem um único satélite natural, a Lua.
C. Júpiter é o maior planeta do sistema solar.
D. As coisas físicas são constituídas por átomos.

2 . A conclusão que um cético radical obtém a partir da premissa «Os sentidos por vezes
enganam-nos» é:
A. A experiência é a principal fonte do conhecimento humano.
B. Raramente podemos confiar nos sentidos.
C. Podemos garantir que os sentidos nem sempre nos enganam.
D. Não podemos garantir que os sentidos não nos enganam sempre.

3 . De acordo com Descartes, a dúvida metódica faz-nos rejeitar as crenças


A. a priori antes das crenças a posteriori.
B. a priori antes das crenças empíricas.
C. empíricas antes das crenças a priori.
D. empíricas antes das crenças a posteriori.

4 . Analisa as frases que se seguem quanto à posição de David Hume sobre a indução.
1. Aquilo em que acreditamos depende, muitas vezes, de inferências indutivas.
2. As inferências indutivas encontram-se racionalmente justificadas.
3. As inferências indutivas baseiam-se no hábito.
4. Através da indução é possível alcançar conclusões garantidamente verdadeiras.
Deve afirmar-se que
A. 2 e 3 são corretas; 1 e 4 são incorretas.
B. 1 e 3 são corretas; 2 e 4 são incorretas.
C. 2 é correta; 1, 3 e 4 são incorretas.
D. 1, 2 e 3 são corretas; 4 é incorreta.

5 . Segundo o filósofo David Hume, a ideia de um certo sabor amargo é originada


A. por uma impressão simples.
B. por uma impressão complexa.
C. por uma ideia complexa.
D. pela imaginação.

6 . David Hume defende que


A. todas as ideias são originadas por perceções.
B. algumas ideias são originadas por perceções.
C. todas as ideias e impressões são perceções.
D. nenhuma ideia é originada por impressões.

Editável e fotocopiável © Texto | Dúvida Metódica, 11.º ano 91


2 Epistemologia – os problemas da definição, da possibilidade e da origem do conhecimento

GRUPO II

1 . Como se explica, no percurso da dúvida cartesiana, a posição solipsista do eu pensante em


que, num determinado momento, o cogito se encontra?

2 . Depois de considerar que provou a existência de Deus, que consequências retira


Descartes?

3 . Como explicam Descartes e Hume a formação da ideia


do ser representado na figura (um centauro)?

4 . Considera as alíneas seguintes.


A. 12 * 5 = 60
B. A Maria é mais alta do que a Joana.
C. A = A
D. Nos Países Baixos, muitas pessoas sabem falar inglês.

4.1. Quais são as alíneas que correspondem, segundo Hume, a questões de facto? E quais
as que correspondem a relações de ideias?
4.2. Indica, de acordo com Hume, duas características opostas das questões de facto e das
relações de ideias.

5 . Como justifica Hume a discrepância existente entre as conclusões céticas que obteve nas
suas reflexões e o modo como entende a vida prática?

6 . A conceção de causalidade defendida por Hume foi alvo de diversas críticas, em particular
de Thomas Reid. Concordas com essas objeções ou com as ideias defendidas por Hume?
Porquê?

Ao justificar a tua opinião deves:


– explicar as críticas de Thomas Reid;
– apresentar a tua posição;
– argumentar a favor da posição defendida.

92 Editável e fotocopiável © Texto | Dúvida Metódica, 11.º ano


PROPOSTA DE RESOLUÇÃO DO TESTE 2

Epistemologia – os problemas da definição, da 4.


possibilidade e da origem do conhecimento 4.1 Alíneas que correspondem a questões de facto: B e D.
Alíneas que correspondem a relações de ideias: A e C.
4.2 Questões de facto: a sua verdade é contingente e o
GRUPO I
seu conhecimento é substancial / Relações de ideias:
1. A 3. C 5. A
a sua verdade é necessária e o seu conhecimento
2. D 4. B 6. C
não é substancial.

GRUPO II 5. Hume reconhece que, do ponto de vista teórico,


1. A dúvida cartesiana é progressiva. Descartes os céticos têm razão, já que algumas das nossas
começa por colocar em causa as crenças empíricas, ideias (por exemplo relacionadas com as inferências
com o argumento dos sentidos enganadores e a indutivas e as relações causais) não têm justificação
seguir com o argumento do sonho. Contudo, para racional. Mas, ele não defende que deixemos de
conseguir duvidar das crenças a priori, ele recorre ao fazer raciocínios indutivos ou de acreditar que
argumento do génio maligno (ou Deus enganador), existem relações causais reais entre os fenómenos
uma experiência mental que lhe permitirá pensar nem sequer que deixemos de fazer inferências
que todas as ideias podem ser falsas, à exceção do causais, apesar da experiência não nos mostrar
cogito (penso, logo existo). Mesmo que este seja um (na perspetiva dele) essa relação. Se não for assim,
princípio indubitável, o eu pensante não pode ter a nossa vida tornar-se-ia muito difícil ou mesmo
a certeza de mais nada, pois o génio maligno pode impossível. Além disso, temos uma tendência
enganá-lo acerca de tudo, encontrando-se, por isso, psicológica para fazer inferências indutivas e
numa posição solipsista em que, exceto o cogito, acreditar na causalidade, isso é algo que faz parte da
tudo pode ser falso (a existência do corpo, dos outros nossa natureza. Por isso, devemos continuar a fazê-lo,
ou da realidade exterior, por exemplo). apesar de não termos justificação racional.

2. Descartes pensa que, se existe um Deus perfeito, 6. Se o aluno concordar com as críticas de Thomas Reid
e um dos seus atributos é a bondade, então ele e discordar de Hume, poderá alegar (por exemplo)
não pode querer enganar o sujeito. Deste modo, a que a conceção de causalidade de Hume, entendida
hipótese do génio maligno (ou Deus enganador) é como conjunção constante, conduz a consequências
afastada. Na filosofia cartesiana, Deus é o garante de absurdas, pois pode existir conjunção constante sem
que as ideias claras e distintas, que o sujeito possui, que exista uma relação causal (como a sucessão
são verdadeiras. Isso torna possível a recuperação entre o dia e a noite) e relação causal sem que
de muitas crenças que tinham sido suspensas pela que exista conjunção constante (como é o caso da
dúvida metódica. explosão que deu origem ao universo, segundo a
teoria do Big Bang).
3. Tanto Descartes como Hume consideram que a ideia Se o aluno discordar das críticas de Thomas Reid
de centauro resulta da imaginação. Para Descartes, e concordar com Hume, poderá alegar que de
trata-se de uma ideia factícia formada a partir de facto a nossa experiência da causalidade se reduz
outras ideias (como a de homem e a de cavalo). à observação de conjunções constantes (e não
Segundo Hume, centauro é uma ideia complexa, de conexões necessárias) e que devido a isso nos
originada pela junção de várias ideias e que não habituámos a esperar que a certos fenómenos se
corresponde a uma impressão externa. sigam outros.

93
3 Filosofia da ciência – os problemas da demarcação, do método, da evolução e da objetividade da ciência

Teste 3
GRUPO I

Seleciona a alternativa correta.

1 . Um dos problemas estudados pela Filosofia da ciência é o seguinte:


A. Qual é o procedimento subjacente à elaboração das hipóteses científicas?
B. Na Química, o método experimental pode ser utilizado para estudar que tipo de
fenómenos?
C. Como é que os físicos descobriram as partículas subatómicas?
D. Quais são as metodologias utilizáveis na área da Psicologia social?

2 . Segundo o método das conjeturas e refutações, o raciocínio dedutivo é utilizado na etapa


que corresponde à
A. formulação do problema.
B. elaboração das conjeturas.
C. submissão da conjetura a testes.
D. formulação das leis científicas.

3 . Lê o texto.
«A intuição chega-lhe uma noite, no parque por trás do Instituto de Copenhaga. O jovem
Werner Heisenberg passeia pensativo no parque. O parque está às escuras (é em 1925). Há
apenas um ou outro fraco candeeiro que lança para baixo uma pequena bola de luz. As
bolas de luz estão separadas por longos espaços de escuridão. De repente, Heisenberg vê
passar um homem. Aliás, na realidade não o vê passar: vê-o aparecer por baixo de um can-
deeiro, desaparecer nas trevas e pouco depois reaparecer ao pé de outro candeeiro, e logo
a seguir voltar a desaparecer nas trevas. E assim por diante (…) até desaparecer na noite.
Heisenberg pensa: (…) o homem é um objeto grande, grosso e pesado, e os objetos grandes
não aparecem e desaparecem assim…
Mas, o que sabemos nós dos eletrões? Este é o clique de Heisenberg. Regressa a casa e
mergulha nos cálculos. Emergirá algum tempo depois com uma teoria desconcertante: uma
descrição fundamental do movimento das partículas em que estas não são descritas por
meio da sua posição a cada momento, mas só com a posição em certos instantes (…) em que
interagem com qualquer outra coisa.»
Carlo Rovelli, A Realidade Não É o que Parece,
Contraponto, Lisboa, 2019, pp. 106-108 (adaptado)

Esta descrição do modo como Heisenberg descobriu algumas ideias fundamentais da


teoria da mecânica quântica, acentua o papel da imaginação e, por isso, ilustra a tese da
perspetiva
A. historicista do método científico.
B. falsificacionista do método científico.
C. indutivista do método científico.
D. positivista do método científico.

Editável e fotocopiável © Texto | Dúvida Metódica, 11.º ano 95


3 Filosofia da ciência – os problemas da demarcação, do método, da evolução e da objetividade da ciência

4 . Popper e Kuhn concordam quanto à


A. interferência decisiva de fatores externos – ideológicos e sociológicos – na objetividade
da ciência.
B. maior aproximação das teorias atuais à verdade.
C. existência, na história da ciência, de um progresso que é sempre contínuo e cumulativo.
D. existência de critérios objetivos na escolha das teorias científicas.

5 . Segundo Kuhn, o que nos impede de considerar um paradigma como superior a outro é o
facto de
A. cada um deles apresentar diferentes conceções do mundo.
B. não serem incomensuráveis.
C. ambos se encontrarem próximos da verdade.
D. a escolha entre ambos ser feita de forma racional.

6 . «A observação é sempre seletiva. Requer um objeto determinado, uma tarefa definida, um


interesse, um ponto de vista, um problema.»
Esta afirmação constitui uma objeção à conceção do método defendida
A. tanto pelo indutivismo como pelo falsificacionismo.
B. pelo historicismo.
C. pelo falsificacionismo.
D. pelo indutivismo.

GRUPO II

1 . Lê o texto e responde às questões a seguir apresentadas.


«(…) O princípio da verificação [foi] defendido pelos chamados positivistas lógicos a partir
da década de 1920. A ideia aproximada era a de que para fazer uma afirmação com signi-
ficado acerca do mundo temos de dizer algo que possa ser verificado ou falsificado pela
observação. A intenção era incluir afirmações científicas e excluir as não científicas. Os posi-
tivistas lógicos fizeram numerosas tentativas perspicazes de encontrar uma versão precisa
do princípio da verificação. Todas fracassaram (…).»
Timothy Williamson, Filosofar, da Curiosidade Comum ao Raciocínio Lógico,
Gradiva, Lisboa, 2019, p. 116 (adaptado)

1.1 De acordo com os positivistas lógicos, como é que se pode fazer a distinção entre as
teorias científicas e as não científicas? Na tua resposta, integra informação do texto.
1.2 Indica uma razão lógica que, segundo Popper, tenha levado o princípio da verificação
a fracassar enquanto critério de demarcação.
1.3 Qual foi o critério de cientificidade, proposto por Popper, para diferenciar a ciência da
não ciência?

96 Editável e fotocopiável © Texto | Dúvida Metódica, 11.º ano


3 Filosofia da ciência – os problemas da demarcação, do método, da evolução e da objetividade da ciência

2 . Lê o texto e responde às questões a seguir apresentadas.

«Sugiro, portanto, que Sir Karl Popper caracterizou todo o empreendimento científico em
termos que só se aplicam às suas partes ocasionalmente revolucionárias. A sua ênfase
é natural e comum: as proezas de um Copérnico ou de Einstein atraem mais (…); Sir Karl
não seria o primeiro a considerar erradamente o que eu chamo ciência normal como um
empreendimento intrinsecamente desinteressante. Não obstante, provavelmente nem a
ciência nem o desenvolvimento do conhecimento se podem compreender se a investigação
for vista apenas através das revoluções que ocasionalmente produz. (…) É a ciência normal
que revela tanto os pontos a testar como a maneira de os testar. Ou, ainda, os profissionais
são educados para a prática normal da ciência e não para a extraordinária.

Em certo sentido, virando ao contrário o ponto de vista de Sir Karl, é precisamente o


abandono do discurso crítico que assinala a transição para a ciência. Uma vez que um
campo fez tal transição, o discurso crítico só reaparece em momentos de crise, quando as
bases do campo estão outra vez em perigo. Só quando têm de escolher entre teorias rivais
é que os cientistas se comportam como filósofos.»
Thomas Kuhn, A Tensão Essencial, Edições 70, Lisboa,
2009, pp. 311-313 (adaptado)

2.1 Distingue, do ponto de vista de Kuhn, a ciência normal da ciência extraordinária.


2.2 O que entende Kuhn por revolução científica?
2.3 No texto, Kuhn crítica a importância que Popper atribui à crítica no desenvolvimento
da ciência.

Na tua opinião, quem é que terá razão? Porquê?

Ao justificar a tua opinião deves:


– comparar as perspetivas de Popper e Kuhn;
– apresentar a tua posição;
– argumentar a favor da posição defendida.

Editável e fotocopiável © Texto | Dúvida Metódica, 11.º ano 97


PROPOSTAS
PROPOSTA DE
DERESOLUÇÃO
RESOLUÇÃODO
DOTESTE
TESTE31

Filosofia da ciência – os problemas da demarcação, do 2.2 A revolução científica corresponde à mudança de


método, da evolução e da objetividade da ciência paradigma. O novo é incomensurável com o que
vigorou anteriormente.
2.3 Se o aluno considerar que Popper tem razão poderá
GRUPO I
(por exemplo) alegar que a crítica é fundamental
1. A 3. B 5. A
na prática científica e que não ocorre apenas no
2. C 4. D 6. D
trabalho de grandes cientistas como Copérnico
e Einstein, mas também no trabalho diário dos
GRUPO II cientistas anónimos, pois significa que os cientistas
1. estão atentos a eventuais erros e avaliam os
1.1 Através do princípio da verificação, segundo o qual resultados que vão obtendo. Se não tivessem uma
a verdade ou falsidade dos enunciados científicos atitude crítica, dificilmente a sua atividade poderia
pode ser estabelecida através da experiência: «para ser considerada científica.
fazer uma afirmação com significado acerca do Se o aluno considerar que Kuhn tem razão poderá
mundo temos de dizer algo que possa ser verificado (por exemplo) alegar que a crítica é de facto
ou falsificado pela observação». importante em certos momentos da ciência (ou seja,
1.2 As leis e teorias correspondem a enunciados na ciência extraordinária) quando há confronto de
universais. Estes referem-se a todos os casos – do teorias alternativas. Como aconteceu, no caso de
presente, passado e futuro – e, assim sendo, dada cientistas especialmente dotados, como Copérnico e
a sua infinidade, não podem ser verificados, isto é, Einstein, que levam a cabo inovações fundamentais.
a sua verdade não pode ser estabelecida de modo Todavia, na maior parte do tempo (ou seja, na
conclusivo. ciência normal) a atividade científica é mais rotineira
1.3 O critério de falsificabilidade. e não envolve discussões críticas, mas sim aplicações
da teoria (ou seja, do paradigma) aceite pela
2. comunidade científica.
2.1 No período da ciência normal, o paradigma é
aplicado na resolução de enigmas ou puzzles.
Os cientistas visam consolidá-lo e torná-lo mais
abrangente. Adotam uma atitude conservadora,
pois resistem às mudanças que colocam em causa
a tradição. No período da ciência extraordinária,
o paradigma dominante passa a ser discutido,
contestado e procuram-se modelos explicativos
alternativos, emergindo daí um paradigma rival.
Nesse período, os cientistas têm uma atitude crítica,
o que não acontece na ciência normal.

99
4 Filosofia da arte – o problema da definição de arte

Teste 4
GRUPO I

Seleciona a alternativa correta.

1 . A Filosofia da arte não estuda


A. a definição de arte.
B. o valor da arte enquanto arte.
C. a história da arte.
D. critérios distintivos entre boa e má arte.

2 . Uma boa definição de arte não deve


A. aplicar-se a todas as obras de arte.
B. aplicar-se apenas às obras de arte.
C. aplicar-se a todas as obras de arte e só a elas.
D. limitar-se a distinguir entre a boa e a má arte.

3 . «Aquela obra não se percebe: retrata o quê, afinal?»


Esta opinião pressupõe a teoria
A. representacional.
B. expressivista.
C. formalista.
D. histórica.

4 . A teoria histórica pretende ser


A. normativa.
B. classificativa.
C. normativa e classificativa.
D. uma explicação das obras de arte mais antigas.

5 . Segundo Levinson, a intenção do artista de que a sua obra seja vista como foram vistas as
obras do passado deve ser
A. breve e passageira.
B. acidental.
C. séria e firme.
D. ignorada por quem vê a obra.

6 . O mundo da arte é
A. um movimento artístico.
B. uma teoria sobre a arte.
C. uma obra de arte.
D. uma instituição social.

Editável e fotocopiável © Texto | Dúvida Metódica, 11.º ano 101


4 Filosofia da arte – o problema da definição de arte

GRUPO II

1 . Se pensarmos que uma obra de arte pode não ser uma imitação realista da realidade,
temos de rejeitar a teoria representacional? Porquê?

2 . A teoria expressivista, na formulação de Collingwood, implica que uma obra seja artística
sempre que desperte certos sentimentos no público? Porquê?

3 . Qual é, segundo a teoria formalista, a relação entre a forma significante e a emoção


estética?

4 . O que significa dizer que o conceito de arte é aberto e não fechado?

5 . Porque é que a teoria histórica não é uma teoria essencialista?

6 . «Os artistas determinam o que é ou não é passível de ser considerado arte.»


Pedro Cabrita Reis, artista plástico

Concordas com a tese de Pedro Cabrita Reis? Porquê?

102 Editável e fotocopiável © Texto | Dúvida Metódica, 11.º ano


PROPOSTA DE RESOLUÇÃO DO TESTE 4

Filosofia da arte – o problema da definição de arte 5. A teoria histórica não é uma teoria essencialista, pois
considera que aquilo que todas as obras de arte têm
GRUPO I em comum não é algo que faça parte das próprias
1. C 3. A 5. C obras (não é uma essência), mas sim algo que faz
2. D 4. B 6. D parte do contexto em que surgem: a intenção dos
artistas de que sejam vistas como o foram as obras
GRUPO II anteriores.
1.
Não temos de rejeitar a teoria representacional, pois 6. Caso o aluno concorde que os artistas determinam
as imitações realistas não são o único tipo possível de o que é ou não é passível de ser considerado arte,
representação. Pode tratar-se, por exemplo, de uma pode defender a sua perspetiva através da teoria
representação simbólica. institucional.
Se o aluno discordar, poderá justificar a sua
2. Não. Na «arte» do entretenimento (que é uma forma discordância, apresentando objeções à teoria
de ofício e não genuína arte) os criadores seguem institucional ou apresentando uma das teorias
uma espécie de receita para emocionar (fazer rir, por essencialistas.
exemplo) o público.

3. A forma significante provoca a emoção estética.

4. O conceito de arte é considerado aberto por alguns


autores, na medida em que tem mudado ao longo da
história e é expectável que continue a mudar.
Os artistas podem a qualquer momento decidir que
a arte deve incluir coisas diferentes de tudo o que foi
feito até essa altura.

Editável e fotocopiável © Texto | Dúvida Metódica, 11.º ano 103


5 Filosofia da religião – o problema da existência de Deus

Teste 5
GRUPO I

Seleciona a alternativa correta.

1 . O monoteísmo implica
A. a crença num Deus único.
B. o teísmo.
C. a defesa de uma religião que valorize mais a razão do que a fé.
D. a inexistência de mal no mundo.

2 . Qual destas questões apresenta o problema filosófico da existência de Deus?


A. Poderá Deus ser o fundamento da moralidade?
B. Temos ou não boas razões para afirmar que o conceito de Deus refere um ser real?
C. Amar Deus é mais importante do que amar a família?
D. Temos ou não boas razões para defender a importância social da religião?

3 . O mundo não teve uma origem ou começo.


Esta frase faz parte de uma objeção ao argumento
A. do mal.
B. cosmológico.
C. ontológico.
D. teológico.

4 . A complexidade existente nas coisas naturais não é intencional.


Esta frase faz parte de uma objeção ao argumento
A. cosmológico.
B. ontológico.
C. da aposta.
D. teleológico.

5 . O mal existente no mundo visa a edificação do caráter.


Esta frase faz parte
A. do argumento do desígnio.
B. do problema do mal.
C. de uma resposta ao problema do mal.
D. de uma objeção ao argumento teleológico.

6 . Pascal pensava que


A. os argumentos a favor da existência de Deus são claramente melhores do que os
argumentos contra a existência de Deus.
B. os argumentos contra existência de Deus são claramente melhores do que os
argumentos a favor da existência de Deus.
C. há uma espécie de empate entre os argumentos a favor da existência de Deus e os
argumentos contra a existência de Deus.
D. a crença em Deus é ilegítima.

Editável e fotocopiável © Texto | Dúvida Metódica, 11.º ano 105


5 Filosofia da religião – o problema da existência de Deus

GRUPO II

1 . Qual é o argumento a favor da existência de Deus que pode incluir nas suas premissas, e
sem mais especificações, a ideia de que existem coisas? Porquê?

2 . Em que medida se pode dizer que uma máscara cirúrgica tem um caráter teleológico?

3 . Na versão do argumento teleológico apresentada por Tomás de Aquino existe uma


analogia. Que analogia é essa?

4 . Se definirmos Deus como o ser supremo, seremos logicamente obrigados a admitir a sua
existência? Porquê?

5 . De que modo o atributo divino da omnisciência pode ser referido na formulação do


problema do mal?

6 . Uma crença genuína não pode ser voluntária. Como é que, partindo desta ideia, podemos
criticar o argumento da aposta de Pascal?

106 Editável e fotocopiável © Texto | Dúvida Metódica, 11.º ano


PROPOSTA DE RESOLUÇÃO DO TESTE 5

Filosofia da religião – o problema da existência de 4. Se o aluno defender que a definição de Deus como
Deus o ser supremo (ou seja, como o ser mais grandioso
do que o qual nada pode ser pensado) nos obriga
logicamente a admitir a sua existência, poderá
GRUPO I justificar a sua posição através do argumento
1. A 3. B 5. C ontológico e tentar mostrar que definir Deus desse
2. B 4. D 6. C modo e depois negar-lhe a existência é incoerente.
Se o aluno defender que a definição de Deus como o
GRUPO II ser supremo não nos obriga logicamente a admitir a
1. O argumento cosmológico, pois procura mostrar que sua existência, poderá justificar a sua posição através
o facto de existirem coisas implica que exista uma de uma das objeções ao argumento ontológico,
causa primeira. (O argumento teleológico obrigaria a alegando que o conceito de um ser maior do que
especificar essas coisas, de modo que fossem coisas o qual nada pode ser pensado é inadequado, pois
dotadas de desígnio.) podemos sempre conceber um ainda mais grandioso;
ou alegando que o argumento ontológico tem
2. As máscaras cirúrgicas têm um caráter teleológico, consequências absurdas.
pois foram propositadamente concebidas para
executar essa função e a sua forma ajusta-se a essa 5. Se Deus é omnisciente (ou seja, sabe tudo), então sabe
função (ou finalidade). que o mal vai ocorrer (e sendo omnipotente poderia
impedi-lo).
3. As coisas naturais estão direcionadas para atingir
certas finalidades tal como a flecha se dirige para o 6. Se uma pessoa acredita sinceramente em algo, então
alvo. Tal como a flecha é direcionada pelo arqueiro, essa crença não resulta de uma decisão, mas sim de
as coisas naturais são direcionadas por Deus. uma descoberta. Por isso, mesmo que alguém pense
que a crença em Deus é vantajosa isso não será
suficiente para que comece realmente a acreditar
em Deus.

107
Guiões de estudo

FILOSOFIA – 11.º ANO / TESTE N.º ___ / GUIÃO DE ESTUDO


Agrupamento de Escolas / Escola:
Ano letivo: / Data: / Duração: / Professor/a:

Conteúdos das
Que tipo de questões
Aprendizagens No final deste capítulo devo conseguir...
poderei encontrar?
Essenciais

Módulo IV – • Formular os problemas epistemológicos da definição, Questões de resposta:


O conhecimento da possibilidade e da origem do conhecimento. • restrita;
e a racionalidade • Explicar a definição tradicional do conhecimento. • curta;
científica e • Distinguir o conhecimento a priori do conhecimento • extensa.
tecnológica a posteriori. Questões de resposta
• Caracterizar o ceticismo (moderado e radical), fechada.
1. Filosofia do
esclarecendo em que consiste o desafio cético.
conhecimento (ou
• Explicar a resposta racionalista de Descartes ao
Epistemologia): Onde posso estudar?
desafio cético e o percurso da dúvida metódica.
os problemas
• Aplicar os argumentos cartesianos para duvidar à
da possibilidade Manual
análise e discussão de exemplos reais e imaginários.
e da origem do • (pág. 6 a pág. 73)
• Caracterizar o primeiro princípio da filosofia
conhecimento
cartesiana (o cogito).
Aula Digital
1.1 A definição • Explicar como pretende Descartes ter provado a
• Testes interativos
tradicional do existência de Deus e a importância deste no sistema
• Vídeos explicativos
conhecimento cartesiano.
(legendados em português)
• Explicar a resposta cartesiana ao problema da origem
1.2 O desafio cético • Animações
do conhecimento.
• Jogo: «Penso, logo acerto»
1.3 A resposta • Discutir objeções à teoria de Descartes.
• Polígrafo filosófico
racionalista de • Distinguir, segundo a teoria empirista de Hume, as
Descartes impressões das ideias e as relações de ideias das
Caderno do Aluno
1.4 A resposta questões de facto.
• Fichas de trabalho
empirista de Hume • Esclarecer a perspetiva de Hume sobre a causalidade.
(e propostas de resolução)
• Explicar em que consiste o problema da indução.
1.5 A comparação • Textos complementares
• Esclarecer as razões que levaram Hume a defender
entre as duas o ceticismo moderado.
teorias explicativas Bom trabalho!
• Discutir objeções à teoria de Hume.
do conhecimento • Comparar as perspetivas de Descartes e de Hume
quanto aos problemas da possibilidade e da origem
do conhecimento.
• Avaliar logicamente os argumentos a favor e contra as
várias teorias.
• Avaliar criticamente situações que envolvam
os problemas da possibilidade e da origem do
conhecimento, apresentando uma posição pessoal
fundamentada.

Conceitos principais: Epistemologia; definição


tradicional do conhecimento (crença, verdade
e justificação); razão/experiência; racionalismo/
/empirismo; conhecimento a priori e a posteriori;
ceticismo (moderado e radical); dúvida metódica;
argumentos dos enganos sensoriais, do sonho e do
génio maligno; cogito; ideias (inatas, adventícias
e factícias); argumento da marca; círculo cartesiano;
perceção/impressão/ideia; relações de ideias/questões
de facto; causalidade; conjunção constante/conexão
necessária; hábito; inferências indutivas; problema
da indução; princípio da uniformidade da natureza.

108 Editável e fotocopiável © Texto | Dúvida Metódica, 11.º ano


Guiões de estudo

FILOSOFIA – 11.º ANO / TESTE N.º ___ / GUIÃO DE ESTUDO


Agrupamento de Escolas / Escola:
Ano letivo: / Data: / Duração: / Professor/a:

Conteúdos das
Que tipo de questões
Aprendizagens No final deste capítulo devo conseguir...
poderei encontrar?
Essenciais

2. O estatuto do • Conhecer exemplos de problemas estudados pela Questões de resposta:


conhecimento Filosofia da ciência. • restrita;
científico – Filosofia • Esclarecer a relevância dos problemas estudados no • curta;
da ciência âmbito da Filosofia da ciência. • extensa.
• Conhecer algumas características distintivas da Questões de resposta
2.1 O problema
ciência. fechada.
da demarcação:
• Formular o problema da demarcação.
as perspetivas do
• Esclarecer, do ponto de vista do positivismo lógico, em
positivismo lógico Onde posso estudar?
que consistem a verificação e a confirmação.
e de Popper
• Discutir críticas à perspetiva dos positivistas lógicos.
Manual
2.2 O problema • Esclarecer a resposta de Popper ao problema da
• (pág. 86 a pág. 153)
do método: demarcação.
as perspetivas • Distinguir as teorias falsificáveis das teorias
Aula Digital
indutivista falsificadas.
• Testes interativos
e falsificacionista • Diferenciar, do ponto de vista de Popper, a ciência da
• Vídeos explicativos
pseudociência.
2.3 Os problemas (legendados em português)
• Formular o problema do método.
da evolução e • Animações
• Caracterizar a perspetiva indutivista do método
da objetividade • Jogo: «Penso, logo acerto»
científico.
da ciência: as • Polígrafo filosófico
• Discutir críticas ao indutivismo.
perspetivas de
• Caracterizar o método das conjeturas e refutações.
Popper e de Kuhn Caderno do Aluno
• Justificar a resposta de Popper ao problema da
• Fichas de trabalho
indução formulado por Hume.
(e propostas de resolução)
• Discutir críticas ao falsificacionismo.
• Textos complementares
• Aplicar as teorias estudadas sobre os problemas da
demarcação e do método científico à análise de casos
Bom trabalho!
reais ou imaginários.
• Avaliar logicamente os argumentos a favor e contra as
várias teorias.
• Avaliar criticamente situações concretas que
envolvam os problemas da demarcação e do método,
apresentando uma posição pessoal fundamentada.
• Formular o problema da evolução da ciência.
• Formular o problema da objetividade da ciência.
• Explicar a analogia feita por Popper entre a evolução
da ciência e o processo de seleção natural.
• Esclarecer em que medida, segundo Popper, a
verdade é um princípio regulador.
• Explicitar em que consiste a tese de Popper de que na
ciência existe um progresso cumulativo.
• Explicar os argumentos de Popper a favor da
objetividade e da racionalidade da ciência.
• Discutir objeções à perspetiva de Popper.
• Caracterizar as noções utilizadas por Kuhn para
descrever o processo de evolução da ciência:
paradigma, ciência normal, enigma, anomalia, crise,
ciência extraordinária e revolução científica.

Editável e fotocopiável © Texto | Dúvida Metódica, 11.º ano 109


Guiões de estudo

FILOSOFIA – 11.º ANO / TESTE N.º ___ / GUIÃO DE ESTUDO


Agrupamento de Escolas / Escola:
Ano letivo: / Data: / Duração: / Professor/a:

Conteúdos das
Que tipo de questões
Aprendizagens No final deste capítulo devo conseguir...
poderei encontrar?
Essenciais

• Identificar os critérios que, de acordo com Kuhn,


levam os cientistas, nas revoluções científicas, a
escolher um paradigma em detrimento de outro.
• Explicar a tese da incomensurabilidade dos
paradigmas e as suas implicações.
• Esclarecer a posição de Kuhn quanto ao problema da
evolução da ciência.
• Esclarecer a posição de Kuhn quanto ao problema da
objetividade da ciência.
• Discutir objeções à teoria de Kuhn.
• Comparar as perspetivas de Kuhn e de Popper em
relação à objetividade e à evolução da ciência.
• Aplicar as teorias de Popper e de Kuhn à análise de
casos concretos ocorridos ao longo da história da
ciência.
• Avaliar logicamente os argumentos a favor e contra as
várias teorias.
• Avaliar criticamente as teorias de Kuhn e Popper e
defender uma posição pessoal acerca dos problemas
da evolução e da objetividade da ciência, defendendo
uma posição pessoal fundamentada.

Conceitos principais: Filosofia da ciência;


ciência; demarcação; verificação; confirmação;
pseudociência; método científico; indutivismo;
indução; falsificacionismo; conjetura; problema da
indução; falsificabilidade; falsificação; progresso
cumulativo; seleção natural; verdade; objetividade;
critérios objetivos; racionalidade; paradigma; ciência
normal; enigmas; anomalias; ciência extraordinária;
revolução científica; incomensurabilidade; critérios
de cientificidade objetivos e subjetivos; progresso não
cumulativo.

110 Editável e fotocopiável © Texto | Dúvida Metódica, 11.º ano


Guiões de estudo

FILOSOFIA – 11.º ANO / TESTE N.º ___ / GUIÃO DE ESTUDO


Agrupamento de Escolas / Escola:
Ano letivo: / Data: / Duração: / Professor/a:

Conteúdos das
Que tipo de questões
Aprendizagens No final deste capítulo devo conseguir...
poderei encontrar?
Essenciais

3. Análise e • Dar exemplos de diferentes tipos de manifestações Questões de resposta:


compreensão da artísticas. • restrita;
experiência estética • Esclarecer o problema da definição de arte. • curta;
– Filosofia da arte • Justificar a relevância do problema da definição de • extensa.
arte. Questões de resposta
3.1 O problema da • Explicar o que seria uma boa definição de arte. fechada.
definição de arte • Distinguir as teorias essencialistas das teorias
3.1.1 As teorias não-essencialistas.
• Compreender as teses das teorias essencialistas, Onde posso estudar?
essencialistas:
a representacional, os respetivos argumentos e as objeções.
• Compreender as teses das teorias não-essencialistas, Manual
a expressivista e a
os respetivos argumentos e as objeções. • (pág. 166 a pág. 211)
formalista
• Explicar a posição cética relativamente à definição de
3.1.2 As teorias arte. Aula Digital
não-essencialistas: • Compreender as teses das teorias institucional e • Testes interativos
a institucional histórica, e os respetivos argumentos e objeções. • Vídeos explicativos
e a histórica • Apresentar e/ou reconhecer exemplos ilustrativos das (legendados em português)
várias teorias e contraexemplos às mesmas. • Animações
• Comparar as teorias estudadas. • Jogo: «Penso, logo acerto»
• Discutir as definições de arte apresentadas pelas • Polígrafo filosófico
diferentes teorias estudadas.
• Avaliar logicamente os argumentos a favor e contra as Caderno do Aluno
várias teorias. • Fichas de trabalho
• Avaliar criticamente situações que envolvam o
(e propostas de resolução)
problema da definição de arte, apresentando uma
• Textos complementares
posição pessoal fundamentada.

Conceitos principais: Filosofia da arte; definição; Bom trabalho!


condições necessárias e suficientes; teorias
essencialistas; representação; expressão; forma
significante; emoção estética; teorias não-essencialistas;
artefacto; mundo da arte.

Editável e fotocopiável © Texto | Dúvida Metódica, 11.º ano 111


Guiões de estudo

FILOSOFIA – 11.º ANO / TESTE N.º ___ / GUIÃO DE ESTUDO


Agrupamento de Escolas / Escola:
Ano letivo: / Data: / Duração: / Professor/a:

Conteúdos das
Que tipo de questões
Aprendizagens No final deste capítulo devo conseguir...
poderei encontrar?
Essenciais

4. Análise e • Formular o problema da existência de Deus, Questões de resposta:


compreensão justificando a sua relevância. • restrita;
da experiência • Conhecer o fenómeno da diversidade religiosa. • curta;
religiosa – Filosofia • Caracterizar o teísmo e distinguir diferentes • extensa.
da religião conceções de Deus. Questões de resposta
• Explicar os argumentos a favor do teísmo: fechada.
4.1 O problema da
o argumento cosmológico, o argumento teleológico
existência de Deus
e o argumento ontológico.
Onde posso estudar?
4.1.1 Argumentos • Apresentar objeções aos argumentos teístas acerca da
teístas: existência de Deus.
Manual
cosmológico, • Esclarecer em que consiste o problema do mal.
• (pág. 226 a pág. 263)
teleológico • Discutir criticamente diferentes respostas ao
e ontológico problema do mal.
Aula Digital
• Explicar a aposta de Pascal.
4.1.2 O fideísmo • Testes interativos
• Apresentar objeções à argumentação de Pascal a
de Pascal • Vídeos explicativos
favor da crença religiosa.
(legendados em português)
4.1.3 O argumento • Conhecer outras formulações mais atuais de alguns
• Animações
do mal dos argumentos estudados.
• Jogo: «Penso, logo acerto»
• Comparar as várias posições, os argumentos e as
• Polígrafo filosófico
objeções estudadas quanto ao problema da existência
de Deus.
Caderno do Aluno
• Avaliar logicamente os argumentos a favor e contra as
• Fichas de trabalho
várias teorias.
(e propostas de resolução)
• Avaliar criticamente situações que envolvam o
• Textos complementares
problema da existência de Deus, apresentando uma
posição pessoal fundamentada.
Bom trabalho!
Conceitos principais: Filosofia da religião; Deus;
teísmo; deísmo; argumento cosmológico; argumento
teleológico; argumento ontológico; problema do mal;
fideísmo; aposta de Pascal.

112 Editável e fotocopiável © Texto | Dúvida Metódica, 11.º ano


Ensaio
ENSAIO FILOSÓFICO

Editável e fotocopiável © Texto | Dúvida Metódica, 11.º ano


O ensaio filosófico

«O ensaio filosófico

1 O que é um ensaio filosófico? 3 Como escolher o título do ensaio?


Um ensaio filosófico é um texto argumentativo em A melhor maneira de intitular o ensaio é apresen-
que se defende uma posição sobre um determi- tar o mais claramente possível o problema que se
nado problema filosófico. Uma vez que a melhor vai tratar. E a melhor maneira de o fazer é colo-
maneira de formular um problema é fazer uma car uma pergunta. Exemplos de títulos de ensaios
pergunta, o objetivo de um ensaio filosófico é res- podem ser:
ponder a uma pergunta e defender essa resposta,
«Será que os animais têm direitos?»
oferecendo argumentos e refutando as objeções.
«É a existência do mal compatível com a
existência de Deus?» (…)
2 O que se espera que um estudante
mostre ao escrever um ensaio?
4 Como se prepara um ensaio?
Um ensaio deve mostrar que o seu autor sabe
relacionar o problema com as teorias e argumen- Leia criticamente os textos indicados pelo pro-
tos em causa. (…) fessor que tratam o tema proposto. Nessa leitura,
deve procurar identificar as teses em confronto e
Num ensaio, o estudante não pode limitar-se a os argumentos que as sustentam. Deve ainda pro-
dar a sua opinião. Tem também de avançar com curar assegurar-se de que compreende correta-
argumentos e de responder aos argumentos con- mente o que está em causa.
trários. (…)
Uma boa ideia é discutir os problemas e os argu-
mentos com os outros. (…)

5 Como se deve estruturar um ensaio?


Habitualmente, um ensaio tem três partes: a intro-
dução, o corpo do ensaio e a conclusão. (…)
Tenha em mente que, num ensaio, apesar de a
introdução ser a primeira coisa que se lê, é geral-
mente a última a ser escrita; isto porque só depois
da redação final é possível ter uma visão de con-
junto do ensaio.
O ensaio deve ser estruturado de acordo com as
seguintes nove regras:

Editável e fotocopiável © Texto | Dúvida Metódica, 11.º ano 115


O ensaio filosófico

1. Formule o problema 5. Neste momento, deve apresentar a sua posi-


ção. Isso deve ser feito mostrando qual é a pro-
2. Diga qual o objetivo do ensaio Introdução posição que irá ser defendida. (…)
3. Mostre a importância 6. Deve apresentar cuidadosamente os argumen-
do problema
tos a favor da proposição que quer defender.
4. Identifique as principais teses Pode haver vários argumentos. (…). Nesse caso,
concorrentes deve concentrar-se apenas nos dois ou três que
5. Apresente a tese que quer lhe parecem ser os mais fortes e expô-los por
defender palavras suas, tentando mostrar que são váli-
Corpo dos e que as suas premissas são verdadeiras ou,
6. Apresente os argumentos a favor
dessa proposição
do ensaio pelo menos, plausíveis. (…)

7. Apresente as principais objeções


7. Deve apresentar as principais objeções aos
ao que acabou de ser defendido seus argumentos, quer indicando possíveis
contraexemplos ao que é afirmado em alguma
8. Responda às objeções
das premissas, quer disputando a sua plausibi-
9. Tire as suas conclusões Conclusão lidade, quer questionando a validade dos pró-
prios argumentos. Deve procurar as objeções
que lhe parecem mais fortes e não escolher
1. Deve começar pelo problema. Mas, muitas apenas as mais fracas e fáceis de responder. (…).
vezes, não basta formular o mais claramente Deve (…) apresentar as objeções por palavras
possível o problema para as coisas ficarem suas, e não se limitar a citar os autores consul-
completamente claras e não haver margem tados, pois só assim mostra compreender o que
para dúvidas ou ambiguidades. Se, por exem- escreve.
plo, a pergunta é se os animais têm direitos, é
8. Responda às objeções: uma vez apresentadas
preciso dizer exatamente que direitos tem em
as objeções à sua tese, deve dizer o que há de
mente e dar exemplos concretos; deve igual-
errado com elas, ou como lhes responder.
mente deixar bem claro se está a referir-se a
todos os animais — incluindo os piolhos e as 9. Tire as suas conclusões. Resuma muito breve-
baratas — ou só a alguns. mente o seu argumento principal e exponha
as suas dúvidas, caso existam. Mesmo que se
2. Um ensaio pode ter diferentes objetivos. Se o incline mais para uma das respostas concor-
seu objetivo é oferecer razões para acreditar
rentes, não deve hesitar em apresentar os seus
numa determinada tese, então deve dizer que
pontos fracos. Se lhe parecer haver razões
é isso o que vai procurar fazer. (…)
para não tomar posição na disputa, deve, ainda
3. Deve procurar mostrar por que razão é impor- assim, apresentar essas razões. Note que, na
tante que nos ocupemos do problema de que conclusão, não deve apresentar seja o que for
se ocupa. Uma maneira de fazer isso é mostrar que não tenha sido dito anteriormente.»
o que estaríamos a perder se não o fizéssemos. Artur Polónio, «Como Escrever Um Ensaio Filosófico»,
(…) Páginas de Filosofia – http://www.paginasdefilosofia.net/
como-escrever-um-ensaio-filosofico/
4. Aqui deve, muito brevemente, apresentar as (consultado em 04/01/2022 e adaptado)
teses mais conhecidas que respondem a esse
problema. (…)

116 Editável e fotocopiável © Texto | Dúvida Metódica, 11.º ano


O ensaio filosófico

Exemplos de regras práticas para a elaboração dos ensaios filosóficos


e critérios gerais de avaliação
(sítio da APF, Ensaio Filosófico no Ensino Secundário – 7.ª edição
Regulamento do Concurso 2020/2021):

O ensaio filosófico deve assumir a forma escrita, Os ensaios serão avaliados em função dos seguin-
de acordo com as especificações que se seguem: tes critérios:
a) ter uma estrutura reconhecível, nomeada- a) exploração rigorosa do problema, tese(s), argu-
mente com a identificação clara do problema mentos e conceitos em discussão;
e da(s) tese(s) em discussão;
b) estruturação dos conteúdos;
b) ter no máximo 12 páginas A4, escritas com um
c) clareza e rigor da expressão;
processador de texto, em letra Arial, tamanho
11 e entrelinha 1,5 (as 12 páginas incluem capa, d) originalidade da discussão e da posição assu-
sumário, possíveis índices e lista de referências mida.
bibliográficas); Associação de Professores de Filosofia – https://apfilosofia.
org/wp-content/uploads/2020/10/Apf_ensaio_filosofico_7_
c) ter capa e lista de referências bibliográficas; regul_com-capa.pdf (consultado em 04/01/2022)

d) a lista de referências bibliográficas deve assu-


mir uma norma bibliográfica reconhecível
(preferencialmente, norma APA).

Editável e fotocopiável © Texto | Dúvida Metódica, 11.º ano 117


Cidadania e
Desenvolvimento
CIDADANIA
E DESENVOLVIMENTO

• Como articular Cidadania e Desenvolvimento,


Filosofia e outras disciplinas?
Tabela semelhante à que foi incluída no projeto
Dúvida Metódica no 10.º ano de escolaridade
– agora adaptada aos temas de 11.º ano – será
disponibilizada em .

• Aprender Lá Fora
Atividades da rubrica Aprender Lá Fora do manual,
complementadas por mais algumas sugestões,
disponibilizadas em em formato editável,
para permitir a sua adaptação e eventual partilha no
âmbito de atividades interdisciplinares
e/ou de Cidadania e Desenvolvimento.

© Texto | Dúvida Metódica, 11.º ano


Textos
TEXTOS COMPLEMENTARES

• Textos do Caderno do Aluno


• Outros Textos Complementares disponibilizados
em

© Texto | Dúvida Metódica, 11.º ano


Textos Complementares

CAPÍTULO 1
Epistemologia – os problemas da definição, da possibilidade e da origem
do conhecimento

Objeções à definição tradicional de conhecimento

Edmund L. Muitos filósofos aceitam a definição tradicional de conhecimento, mesmo


Gettier (1927-2021) que a tenham formulado por palavras diferentes. Contudo, alguns contes-
ċĺĕūİƧĩĻŜĺċĺ tam-na. Edmund Gettier, por exemplo, procurou mostrar que uma crença
norte-americano
ëūģÑĺêŗÑİÑĕŜ verdadeira justificada pode não ser conhecimento, apresentando con-
ëĺıđøëĕñÑùĺøıŜÑĕĺLj traexemplos, ou seja, situações em que um sujeito tem uma crença verda-
ëĺİÑŔøıÑŜťŗüŜ deira justificada, mas não tem conhecimento. Gettier procurou, portanto,
ŔÒČĕıÑŜLjǠ-ÑŗøıîÑ
VøŗñÑñøĕŗÑUūŜťĕƧëÑñÑ mostrar que essas três condições são necessárias, mas não suficientes. Ou
ĺıđøëĕİøıťĺǎǡLj seja: todo o conhecimento é uma crença verdadeira justificada, mas nem
ŔūêĩĕëÑñĺøİưƸƵƲǍ todas as crenças verdadeiras justificadas são conhecimento.
Os filósofos têm tentado resolver o problema de maneiras bastante diferentes.
Alguns filósofos tentaram identificar uma quarta condição do conhecimento.
Outros defenderam que devemos aceitar apenas justificações infalíveis, jus-
tificações de que possamos ter completa certeza. E houve ainda diversas
outras tentativas.
Um contraexemplo possível é o seguinte:

«Henry está a ver televisão numa tarde de junho. Assiste à


final masculina de Wimbledon e, na televisão, McEnroe vence
Connors; o resultado é de dois a zero e match point para McEnroe
no terceiro set. McEnroe ganha o ponto. Henry crê justificada-
mente que
1. Acabei de ver McEnroe ganhar a final de Wimbledon deste
ano, e infere sensatamente que
2. McEnroe é o campeão de Wimbledon deste ano.
No entanto, as câmaras que estavam em Wimbledon deixaram
na realidade de funcionar, e a televisão está a passar uma grava-
ção da competição do ano passado. Mas enquanto isto acontece,
McEnroe está prestes a repetir a retumbante vitória do ano pas-
sado. Portanto, a crença 2 de Henry é verdadeira, ele tem decerto
justificação para crer nela. Contudo, dificilmente aceitaríamos
que Henry conhece 2.»
Jonathan Dancy, Epistemologia Contemporânea,
Edições 70, Lisboa, 2002, pp. 41-42

122 Editável e fotocopiável © Texto | Dúvida Metódica, 11.º ano


Textos Complementares

Henry acredita que McEnroe venceu e é verdade que este venceu. Além
disso, Henry tem uma justificação adequada para a sua crença, pois viu na
televisão e não havia nenhuma indicação de que as imagens eram uma gra-
vação (e não seria razoável uma pessoa suspeitar sistematicamente, e sem
indícios, da veracidade das imagens televisivas). Contudo, é implausível que
ele tenha conhecimento, pois, apesar de a justificação ser adequada, esta
não incide no aspeto da realidade que torna a crença de Henry verdadeira e,
por isso, este acertou na verdade por sorte.

DeƧıĕîçĺtradicionaĩ
ñøëĺıđøëimento

JustĕƧcação

Conhecimento

Crença Verdade

ModiƧëaçãĺñÑñøƧıiçãĺ
tradicionalñøëĺıđøëimentĺ
ñøžĕñĺÑĺsëĺıtraexemploŜ
ñø?øttier

Condiçãĺ
JustĕƧcação extra

Conhecimento

Crença Verdade

Editável e fotocopiável © Texto | Dúvida Metódica, 11.º ano 123


Textos Complementares

CAPÍTULO 1
Epistemologia – os problemas da definição, da possibilidade e da origem
do conhecimento

Algumas ideias cartesianas

Q
uatro anos apenas depois da publi- mento” ou “epistemologia” (que deriva do
« cação do Discurso [do Método], Des-
cartes publicou – em latim, desta
termo grego episteme, que significa “conhe-
cimento”). Entre outras coisas, nesta disci-
vez – uma obra filosófica mais pormenoriza- plina trata-se de investigar qual é a justifica-
da, cujo título completo é Meditações sobre a ção última das nossas crenças. Mas o que é
Filosofia Primeira, nas quais são demonstradas isso de “justificação última”? E, já agora, o
a existência de Deus e a distinção entre a Alma que é uma crença?
Uma crença não é o mesmo que uma cren-
e o Corpo. Foi nesta obra que Descartes inven-
ça religiosa. Todas as crenças religiosas são
tou o famoso génio maligno (…).
obviamente crenças, mas muitas crenças
O génio maligno é um ser poderoso, mas tão
não são religiosas: são crenças matemáti-
perverso, que nos engana continuamente:
cas, científicas, históricas ou de senso co-
sempre que cremos ver algo, estamos a ser mum. O leitor tem a crença de que está a ler
vítimas de uma ilusão, de maneira que esse este livro e de que a Espanha é maior que
algo não existe ou é totalmente diferente do Portugal. Uma crença é apenas uma repre-
que parece. sentação, verdadeira ou falsa, que alguém
A hipótese do génio maligno torna mais ní- faz de algo.
tido um problema central de uma área da Por sua vez, a justificação última é aquele tipo
filosofia que se chama “teoria do conheci- de justificação que não depende de qualquer

124 Editável e fotocopiável © Texto | Dúvida Metódica, 11.º ano


Textos Complementares

outra. (…) Dizer apenas “sei que estou a ler enganar, é preciso que eu exista. A crença de
um livro porque é isso que vejo e sinto” não é que existo não pode ser falsa em qualquer
uma justificação última. É uma justificação, das circunstâncias em que pondero se exis-
e não é de modo algum uma má justificação, to ou não – ou em que pondero seja o que for.
mas não é uma justificação última – porque (…) a crença de que existo como ser pensan-
depende de outras crenças que, por sua vez te é, por um lado insuscetível de refutação e,
precisam também de ser justificadas. por outro, constitui – por isso mesmo – a jus-
Se lhe ocorre agora que ao raciocinar dessa tificação última de todas as nossas crenças.
maneira nunca conseguiremos parar porque Vejamos brevemente este segundo aspeto.
nunca descobriremos justificações últimas, Tome-se uma crença percetiva, como a
já está a pensar filosoficamente. Só que de que o leitor está com este livro na mão.
ainda não considerou cuidadosamente se Trata-se de uma crença muito diferente das
realmente não descobriríamos tais justifica- crenças matemáticas. Estas últimas não se
ções. O melhor a fazer é então responder a justificam recorrendo à experiência, mas
esse desafio e tentar descobri-las. Foi o que antes ao cálculo matemático: ao pensamen-
fez Descartes. to puro.
Já no que respeita às crenças percetivas, faz
O cogito sentido justificá-las recorrendo à experiên-
Descartes estava convencido de ter descober- cia percetiva: o leitor sabe que está com este
to pelo menos uma crença cuja justificação livro na mão porque é isso que sente e vê.
não depende de quaisquer outras crenças: a Mas Descartes considera que esta justifica-
crença de que ele mesmo existe. Na gíria aca- ção, apesar de perfeitamente adequada não
démica chama-se “cogito cartesiano” a esta é última – pois se formos vítimas do génio
crença, devido à expressão latina cogito, ergo maligno, o facto de parecer que o leitor vê e
sum (penso, logo existo), e o nome latino de sente o livro é compatível com a inexistên-
Descartes: Renatus Cartesius. cia do livro. O que justifica a confiança nos
sentidos terá de ser outro conjunto de con-
O raciocínio de Descartes é que mesmo sob a siderações que Descartes procura retirar do
extravagante suposição de que um génio ma- próprio cogito. Daí que Descartes pense que
ligno me engana sistematicamente, ele não a justificação última das nossas crenças, in-
me pode enganar se eu não existir (…). cluindo as percetivas, não repousa nos sen-
Sempre que vejo árvores, talvez não exis- tidos.
tam árvores na realidade; sempre que me Deste modo se vê que uma posição filo-
lembro de algo talvez se trate de uma falsa sófica aparentemente absurda – como po-
memória; quando sinto e vejo ter um cor- derá alguém crer que o conhecimento do
po com certas características talvez esteja que vemos não se baseia inteiramente nos
iludido – quem sabe se, de facto, me pareço sentidos? – não é, afinal, tão absurda assim.
com lagartixas ou besouros, e não com um Poderá ser falsa, mas é avisado começar por
símio sem pelos? compreendê-la bem para tentar então de-
Talvez tudo isto ocorra, pensa Descartes, se fender que é.»
a hipótese do génio maligno for verdadeira. Desidério Murcho, 7 Ideias Filosóficas que Toda a Gente
Mas para que todas essas ilusões possam Devia Conhecer, Bizâncio, Lisboa,
2011, pp. 18-25 (adaptado)
existir, para que o génio maligno me possa

Editável e fotocopiável © Texto | Dúvida Metódica, 11.º ano 125


Textos Complementares

CAPÍTULO 1
Epistemologia – os problemas da definição, da possibilidade e da origem
do conhecimento

O ceticismo e o irracionalismo de Hume

H
ume tem sido frequentemente segundo lugar, ter substituído a justificação
« acusado de ceticismo e de irracio-
nalismo. Qual a razão de ser des-
racional pelo hábito, uma espécie de instin-
to natural sobre o qual a razão não tem po-
tas acusações? Em primeiro lugar, o facto der. Numa palavra, ter substituído a razão
de Hume ter mostrado que não existe uma pelos instintos.
justificação racional para as nossas inferên- No entanto, Hume pensa que existem ra-
cias causais. Muitos filósofos pensam que zões para preferir a ciência à superstição.
Hume provou não haver razão para prefe- As teorias da ciência são suportadas pela
rir a ciência à superstição. Nenhuma é ra- observação e pela experiência, pela unifor-
cionalmente justificável e, por isso, não há midade da natureza, ao contrário do que
diferenças assinaláveis entre as prescrições acontece com as crenças supersticiosas.
dos médicos e as mezinhas das bruxas. Em Ele não considera, por isso, a sua filosofia

126 Editável e fotocopiável © Texto | Dúvida Metódica, 11.º ano


Textos Complementares

uma forma de irracionalismo, mas sim da- Mas, para aqueles a quem a solução natu-
quilo a que chamamos hoje naturalismo, e ralista não satisfaz, o resultado último da
não duvida de que estabelecemos relações filosofia de Hume foi ter mostrado que (…)
causais e raciocínios indutivos e de que de- não temos [tanto] conhecimento do mundo
vemos confiar nas suas conclusões. Mas [como habitualmente julgamos], seja no sen-
pensa que a causa para essa confiança não tido de verdade indubitável seja no sentido
é a razão mas sim a natureza. Ele vê nesta de crença racionalmente justificada. Daí que
necessidade natural a justificação adequa- o problema da indução esteja no centro do
da e suficiente das nossas crenças sobre o debate filosófico contemporâneo, em parti-
mundo. Embora não possamos justificar cular, em filosofia da ciência.»
racionalmente essas crenças, a natureza Álvaro Nunes, «O Empirismo de David
fez-nos de modo a termos uma propen- Hume», Crítica – https://criticanarede.com/
anunesoempirismodedavidhume.html
são para que certas experiências passadas (consultado em 12/03/2022)
nos levem inevitavelmente a ter certas
crenças sobre o futuro. É tudo o que pre-
cisamos para confiarmos na verdade des-
tas crenças e para demarcar a ciência da
superstição.

Editável e fotocopiável © Texto | Dúvida Metódica, 11.º ano 127


Textos Complementares

CAPÍTULO 2
Filosofia da ciência – os problemas da demarcação, do método, da evolução
e da objetividade da ciência

A perspetiva de Popper sobre o modo como a ciência se desenvolve

N
a verdade, as experiências podem Segundo, que a qualidade de um cientista é
« provar a superioridade da teoria
sobrevivente, mas não a sua ver-
avaliada pela fiabilidade das hipóteses que
desenvolve, as quais, sem dúvida, necessi-
dade; e a teoria sobrevivente pode, por seu tam de ser mais bem elaboradas à medida
turno, ser rapidamente ultrapassada. que se acumulam mais dados, mas que es-
O cientista, após ter percebido que é as- pera venham a manter-se como fundação
sim que as coisas se passam, vai ele próprio firme e segura para um posterior desenvol-
adotar uma atitude crítica em relação à sua vimento conceptual. Um cientista prefere
teoria. Vai preferir ser ele próprio a testá-la falar sobre os dados experimentais e consi-
e mesmo a falibilizá-la, do que deixar essa derar a hipótese como uma espécie de cons-
tarefa aos seus críticos. tructo de trabalho. Por fim, e este é o ponto
Um exemplo, de que me orgulho, é o meu importante: é profundamente lamentável e
velho amigo, o cientista e Prémio Nobel, sinal de fracasso um cientista adotar uma
Sir John Eccles. Mas posso deixar que hipótese que seja falibilizada por novos da-
Eccles fale por si: dos de forma a ter de ser posta de parte no
“Até 1945 defendi (…) a noção indutiva da seu todo.
ciência que deriva de Bacon e Mill. A maior Essa foi a minha dificuldade. Há muito que
parte dos cientistas e dos filósofos continua eu adotara uma hipótese que acabei por per-
a acreditar que este é o método científico. ceber que teria provavelmente de ser posta

128 Editável e fotocopiável © Texto | Dúvida Metódica, 11.º ano


Textos Complementares

de parte, e fiquei muito deprimido com esse livro sobre a sinapse, há dois capítulos sobre
facto. Estivera envolvido numa controvérsia a transmissão elétrica, quer excitante, quer
sobre sinapses […] acreditando nessa época inibidora.”
que a transmissão sináptica entre células Vale a pena notar que tanto Eccles como
nervosas era, em larga medida, elétrica. Ad- Dale estavam errados nas suas teorias ino-
mitia haver uma componente química len- vadoras iniciais, inovadoras em investiga-
ta, mas acreditava que a rápida transmissão ção neurológica; porque ambos pensavam
através das sinapses era elétrica. que as suas teorias eram válidas para todas
Nessa época aprendi com Popper que não as sinapses. A teoria de Dale era válida para
era uma vergonha sob o ponto de vista cien- as sinapses nas quais ambos estavam a tra-
tífico que a nossa hipótese fosse falibiliza- balhar na altura, mas não era aplicável de
da. Há muito tempo que não ouvia nada tão forma mais geral do que a teoria de Eccles.
agradável. Na verdade, fui persuadido por Os defensores de Dale parecem nunca ter
Popper a formular as minhas hipóteses de reconhecido isso; estavam demasiado segu-
transmissão sináptica elétrica excitante e ros da sua vitória sobre Eccles para percebe-
inibidora de forma tão precisa e rigorosa que rem que sofriam ambos do mesmo (presu-
convidassem ao falibilismo – e, de facto, foi mível) mal: nomeadamente, “generalização
isso que lhes aconteceu uns anos mais tarde, precipitada sem esperar por todos os dados
em grande parte pelos meus colegas e por relevantes” (o que nunca é, todavia, prati-
mim próprio, quando em 1951 começámos cável).
a fazer registos intracelulares dos motoneu- Noutro lado, na biografia para o Prémio
rónios. Graças aos ensinamentos de Popper Nobel, Eccles escreve: “Posso agora até re-
consegui aceitar alegremente a morte da jubilar com o falibilismo de uma hipótese
minha criação, que acalentara durante duas que acarinhei como criação minha, já que
décadas, e consegui de imediato contribuir esse falibilismo é um êxito científico.”
o mais que me foi possível para a história da Este último ponto é de extrema importância.
transmissão química, que era a criação de Estamos sempre a aprender milhares de coi-
Dale e Loewi. sas com o falibilismo. Aprendemos não só que
Experimentara finalmente grande poder li- uma teoria está errada, mas também porque é
bertador dos ensinamentos de Popper sobre que está errada, e acima de tudo ganhamos um
o método científico. problema novo e focado com maior precisão; e
Verificou-se uma estranha continuação. um novo problema é, como já sabemos, o ver-
Acontece que fora demasiado lesto na dadeiro ponto de partida para um novo desen-
completa rejeição das hipóteses elétricas volvimento da ciência.»
da transmissão sináptica. Os muitos tipos Karl Popper, A Vida É Aprendizagem,
de sinapses em que tinha trabalhado eram Edições 70, Lisboa, 2020,
pp. 27-30 (adaptado)
certamente químicos, mas agora são conhe-
cidas muitas sinápticas elétricas e, no meu

Editável e fotocopiável © Texto | Dúvida Metódica, 11.º ano 129


Textos Complementares

CAPÍTULO 2
Filosofia da ciência – os problemas da demarcação, do método, da evolução
e da objetividade da ciência

A perspetiva de Kuhn sobre o modo como a ciência se desenvolve

É
esta a estrutura das revoluções cien- mas sim por produzir uma mudança na con-
« tíficas: a ciência normal com um
paradigma e devotada à resolução
cepção do mundo. O livro termina com um
pensamento desconcertante: o progresso
de enigmas; seguida por graves anomalias, científico não é uma mera linha que conduz à
que levam a uma crise; e finalmente a reso- verdade. É antes um afastamento de conce-
lução da crise com um novo paradigma. Há ções e interações com o mundo que são me-
outra palavra famosa: incomensurabilidade. nos adequadas.
A ideia de que, durante uma revolução e con- Antes de Kuhn, Karl Popper (1902-1994) era
sequente mudança de paradigma, as novas o mais influente filósofo da ciência – quero
ideias e asserções não podem ser compara- dizer, o mais lido e, até certo ponto, o mais
das de forma estrita com as antigas. Mesmo aceite pelos cientistas. Popper crescera du-
que algumas das palavras que eram usadas rante a segunda revolução quântica, o que
continuem a sê-lo, o seu sentido mudou. lhe ensinou que a ciência progride através de
O que, por sua vez, levou à ideia de que não conjeturas e refutações, para usar o título de
foi por ser mais verdadeira que uma nova um dos seus livros. Popper dizia que (…) pri-
teoria foi escolhida para substituir a antiga, meiro, avançamos com conjeturas arrojadas,

130 Editável e fotocopiável © Texto | Dúvida Metódica, 11.º ano


Textos Complementares

que sejam testáveis o mais possível, e depois tão pouco com a produção de novidades de
descobrimos-lhes, inevitavelmente, as fa- grande monta, quer no plano conceptual,
lhas. As primeiras conjeturas são refutadas quer no dos fenómenos.”
e avançamos com novas conjeturas, que se Muitas das inovações da experiência ou dos
adequem aos factos. As hipóteses só poderão instrumentos não são tidas em conta no po-
ser tidas como «científicas» se forem passí- sicionamento teórico de Kuhn, portanto a
veis de serem testadas. Esta visão purista da ciência normal pode apresentar muitas ino-
ciência seria impensável antes das grandes vações, mas não no campo da teoria. E para
revoluções do início do século. o público em geral, que deseja tecnologias
e curas, as inovações que fazem com que a
Ciência normal e resolução de enigmas ciência seja admirada geralmente não são
Os pensamentos de Kuhn eram chocantes. teóricas. Essa é a razão para que as afirma-
A ciência normal consiste apenas, ensinava ções de Kuhn pareçam tão irracionais. (…)
ele, em resolver alguns enigmas deixados em Kuhn estava certo. A ciência normal não
aberto numa dada área do saber. Resolver procura a novidade. (…) A caracterização da
enigmas faz-nos pensar em palavras cruza- ciência normal como resolução de enigmas
das, puzzles, sudoku, maneiras agradáveis de
sugere que Kuhn não considerava que fosse
nos entretermos quando não queremos fazer
importante. Mas, na verdade, é o contrário:
trabalho sério. A ciência normal é isso? Parte
Kuhn considerava que a atividade científica
considerável do público científico ficou cho-
era extremamente importante e que a maior
cado, mas teve de admitir que muito do seu
parte consistia na prática da ciência normal.
trabalho quotidiano consistia nisso mesmo.
Atualmente, até os cientistas que duvidam da
Os problemas de investigação não procuram
sua teoria sobre as revoluções têm em grande
produzir novidades de monta. Uma única
conta o seu conceito de ciência normal.»
frase (…) de Kuhn resume-lhe a doutrina:
“A característica mais notável dos problemas Thomas S. Kuhn, A Estrutura das Revoluções
Científicas, Guerra e Paz, Lisboa, 2021,
da investigação normal que acabámos de ob- pp. 13-19 (citações do ensaio introdutório
servar é talvez o facto de eles se preocuparem da autoria de Ian Hacking)

Editável e fotocopiável © Texto | Dúvida Metódica, 11.º ano 131


Textos Complementares

CAPÍTULO 3
Filosofia da arte – o problema da definição de arte

St Ives Harbour, de Alfred Wallis, s/d

Teoria formalista e teoria institucional da arte

S
egundo as teorias formalistas, o que inesperada harmonia entre as cores puras,
« caracteriza a obra de arte é a sua for-
ma e não o seu caráter representativo.
as formas e dimensões de seus retângulos, o
que deve constituir uma forma significante.
Um paradigma do formalismo é a teoria pro- Característico da forma significante é que
posta por Clive Bell em 1914 com o objetivo ela produz uma emoção estética em pessoas
de defender o neo-impressionismo de pin- com sensibilidade para a arte.
tores como Paul Cézanne. Para Bell o que A teoria da forma significante foi útil como
caracteriza as artes plásticas e talvez a músi- defesa da pintura abstrata ou semi-abstrata
ca é a presença da forma significante. O con- surgida desde o final do século XIX. Mas ela
ceito de forma significante é simples, não possui defeitos sérios. Para Bell a represen-
podendo ser definido. Mas na pintura ele re- tação e o contexto não possuem relevância.
sulta da combinação de formas, linhas e co- Mas não é difícil encontrarmos exemplos de
res. Considere, por exemplo, a Composição obras de arte nas quais o elemento repre-
em Vermelho, Amarelo e Azul, de Mondrian. sentacional ou o contexto são importantes.
O que faz a singularidade dessa pintura é a Considere os autorretratos de Rembrandt,

132 Editável e fotocopiável © Texto | Dúvida Metódica, 11.º ano


Textos Complementares

ou ainda, o quadro de Géricault, A Jangada 70 anos, após a morte da esposa, decidiu


do Meduza. A composição do quadro é im- pintar barcos na madeira para afugentar a
portante, mas o que ele representa também. solidão. Casualmente, dois pintores de pas-
Nele estão retratados alguns náufragos à sagem pelo lugar gostaram das suas telas e
beira da morte, em uma jangada perdida descobriram-no como artista. Como resul-
no oceano, no momento em que é divisada tado as obras de Wallis podem ser hoje vis-
a salvação. A pintura foi inspirada por um tas em vários museus ingleses. Como disse
acontecimento verídico. Sentimos que esse um crítico, Wallis tornou-se um artista sem
quadro potencializa o drama e a esperança sequer saber que era.
humanos para além da simples representa- Há duas objeções principais à teoria institu-
ção naturalista de um acontecimento. Certo cional. A primeira é que ou os entendidos
é que não é só a composição, mas também o em arte decidem o que deve ser considerado
conteúdo simbólico que aqui se somam na uma obra de arte com base em razões ou o
produção do sentimento estético. fazem arbitrariamente. Se eles o fazem com
base em razões, essas razões constituem
A dificuldade maior com a teoria de Bell con- uma teoria da arte que não é a teoria insti-
siste, no entanto, na sua falta de conteúdo. tucional. Assim, alguém poderá dizer que
Para a questão “O que é forma significan- os quadros de Wallis apresentam excelentes
te?”, a melhor resposta parece ser: aquela combinações de cores aliadas a simplicida-
que tende a produzir no auditório um senti- de formal. Mas essa é uma maneira de dizer,
mento estético. À pergunta “O que é o sen- por exemplo, que eles possuem forma sig-
timento estético?”, a resposta parece ser: nificante. Nesse caso, a teoria institucional
aquele que é produzido pela forma signifi- colapsa em outras conceções acerca do que
cante. A teoria beira a vacuidade e a circu- é a arte. Suponhamos agora que os entendi-
laridade. dos em arte decidem o que deve ser consi-
A teoria institucional da arte surgiu na déca- derado obra de arte arbitrariamente. Ora,
da de sessenta, tendo sido sustentada por nesse caso não fica claro porque devemos
George Dickie. Essa teoria enfatiza a im- dar qualquer importância à arte. Uma obje-
portância da comunidade de conhecedores ção adicional seria a de que a teoria institu-
de arte na definição e ampliação dos limites cional é viciosamente circular. Obras de arte
daquilo a que se pode chamar arte. Dickie são definidas como objetos que são aceites
define a obra de arte como um artefato que como tais pelas pessoas que entendem de
possui um conjunto de aspetos que lhe con- arte; e as pessoas que entendem de arte são
ferem o status de candidato à apreciação das definidas como as que aceitam certos obje-
pessoas da instituição do mundo da arte. tos como sendo obras de arte.»
A importância disso pode ser ilustrada pela Cláudio F. Costa, «Teorias da Arte»,
Crítica – https://criticanarede.com/est_tarte.html
obra de Alfred Wallis. Wallis era um mari- (consultado em 12/03/2022 e adaptado)
nheiro que nada entendia de arte e que aos

Editável e fotocopiável © Texto | Dúvida Metódica, 11.º ano 133


Textos Complementares

CAPÍTULO 4
Filosofia da religião – o problema da existência de Deus

A aposta de Pascal

P
ascal fez uma análise pormenori- existir, argumentava Pascal, o nosso ganho –
« zada dos prós e dos contras do de-
ver para com Deus como se estivesse
a felicidade eterna – é infinito. Se, por outro
lado, Deus não existir, a nossa perda, ou lucro
a calcular matematicamente a sensatez de negativo, é pequena – os sacrifícios da pieda-
uma aposta. de. E, para pesar estes possíveis ganhos e per-
A sua grande inovação foi o método de pesar das, Pascal propunha que se multiplicasse
estes prós e contras, um conceito a que se dá a probabilidade de cada resultado possível
hoje o nome de esperança matemática. pela sua recompensa e se somasse tudo, for-
A esperança matemática é um importante mando uma espécie de recompensa média
conceito, não só nos jogos de azar como na ou esperada. Por outras palavras, a esperan-
tomada de decisões. Com efeito, a aposta de ça matemática do nosso lucro com a piedade
Pascal é muitas vezes considerada a funda- é metade de infinito (o ganho se Deus exis-
ção da disciplina matemática da teoria dos tir) menos metade de um número pequeno
jogos, o estudo quantitativo das estratégias (a nossa perda se Ele não existir). Pascal sa-
de decisão ótimas nos jogos. bia o suficiente sobre o infinito para saber
O raciocínio de Pascal era o seguinte. que a resposta deste cálculo era infinita, pelo
que o lucro esperado com a piedade é infini-
Admitamos que não sabemos se Deus exis-
tamente positivo. E assim, concluiu Pascal,
te ou não, e, por conseguinte, atribuamos
qualquer pessoa sensata deve seguir as leis
uma probabilidade de 50% para cada uma
de Deus. Hoje, chama-se a este argumento a
das proposições. Como pesar esta probabi-
aposta de Pascal.»
lidade na decisão de levar ou não uma vida Leonard Mlodinow, O Passeio do Bêbado,
piedosa? Se vivermos piedosamente e Deus Bizâncio, Lisboa, 2009, p. 92

134 Editável e fotocopiável © Texto | Dúvida Metódica, 11.º ano


Textos Complementares

CAPÍTULO 5
Tema/problema: A legitimidade moral da experimentação animal

Direitos dos animais e erros dos humanos

H
averá limites para o modo como no primeiro ou no segundo domínio? Ou al-
« os seres humanos podem tratar
legitimamente os animais não-
gures no meio?
Resumidamente, todos parecemos concor-
-humanos? Ou podemos tratá-los de qualquer dar que há limites para o modo como pode-
maneira que nos agrade? Se há limites, quais mos tratar legitimamente os animais não-
são? São suficientemente fortes, como al- -humanos e que estes limites surgem devido
gumas pessoas supõem, para nos levarem a à natureza dos animais, não apenas devido
ser vegetarianos e a diminuir, se não mesmo aos desejos de outros humanos de verem
eliminar, o nosso uso de animais não-huma- os animais a ser bem tratados. Isto é, esses
nos em experiências “científicas” concebi- atos são errados não apenas porque outros
das para nos beneficiar? humanos se incomodam com eles. Pensa-
(…) Há uma diferença fundamental entre ríamos que seriam igualmente errados se
aqueles objetos que podemos tratar como fossem praticados secretamente de modo
nos apetecer (exceto quando estivermos que mais ninguém da comunidade soubesse
limitados pelos interesses de outros huma- deles. Pensamos que são errados devido ao
nos) e aqueles que não podemos. As pedras que acontece ao animal.
vulgares enquadram-se no primeiro domí- Por outro lado, estamos integrados numa
nio; os humanos enquadram-se no último. cultura que usa arrogantemente animais
E os animais não-humanos? Enquadram-se para a alimentação, para o vestuário, para a

Editável e fotocopiável © Texto | Dúvida Metódica, 11.º ano 135


Textos Complementares

pesquisa no desenvolvimento de novos me- Comparem-se os seguintes casos: 1) eu es-


dicamentos, e para determinar a segurança peto o braço da minha filha com uma agu-
de produtos de higiene doméstica. lha sem ter uma razão aparente para o fazer
As pessoas comuns parecem igualmente (embora não precisemos de presumir que
pouco ou nada familiarizadas com o uso retiro daí qualquer prazer sádico); 2) sou um
extensivo de animais em experiências la- médico e vacino-a contra a tifóide. O que
boratoriais. Muitas destas experiências (…) diferencia estes casos? Em ambos espeto o
envolvem uma dor prolongada para os ani- braço da minha filha; em ambos (presuma-
mais. Por exemplo, N. J. Carlson adminis- mos) inflijo-lhe uma quantidade similar de
trou choques elétricos de alta voltagem a de- dor. Todavia, consideramos que o último
zasseis cães e descobriu que o “grupo de alta não é apenas justificável, mas possivelmen-
voltagem” ficava “ansioso” mais depressa. te obrigatório; consideramos o primeiro
(…) Ou ainda o caso de investigadores em caso sádico. Porquê? Porque consiste na in-
Harvard que puseram ratos bebés com ratos flicção de dor desnecessária. A minha filha
adultos esfomeados. Os adultos comeram não beneficia de todo com o que lhe faço.
os bebés. A conclusão dos investigadores: a Assim, a dor desnecessária é aquela que é
fome é um móbil importante nos animais. infligida num ser senciente (física e psiqui-
(Isso, é claro, é algo que aprendemos com camente sensível) quando tal não acontece
surpresa; nunca saberíamos deste facto de para o bem desse ser em particular. Esta
outro modo.) última seria uma dor necessária, porque se-
ria aquela dor que esse ser sofreria para seu
Por que os animais não devem sofrer próprio bem.
desnecessariamente Há duas premissas principais no meu argu-
Até agora tenho tentado identificar as nossas mento. A primeira é a afirmação factual de
profundas crenças sobre as restrições relati- que os animais sentem, de facto, dor. A se-
vas ao tratamento correto dos animais. Ago- gunda é a afirmação de que o potencial so-
ra é altura de tentar oferecer uma defesa po- frimento de um animal limita fortemente
sitiva do nosso entendimento comum, uma aquilo que lhe podemos justificadamente
defesa que terá implicações ainda mais radi- fazer, restringindo o modo como podemos
cais do que possamos ter suposto. Isto é, que- usá-lo legitimamente.
ro argumentar (…) que há limites rigorosos
sobre o que é moralmente permissível fazer Que os animais sentem dor
aos animais. Mais especificamente, pretendo Que os animais sentem dor parece relativa-
argumentar que todos devemos tornar-nos mente incontestável. É uma crença que to-
vegetarianos e que devemos reduzir drastica- dos partilhamos. (…)
mente, se não mesmo eliminar, o nosso uso de Pode-se dizer ainda mais. Temos mais do que
animais nos laboratórios. provas comportamentais adequadas de que
Embora haja numerosos argumentos que os animais sentem dor e de que podem sofrer.
podem ser apresentados em defesa desta po- A maior parte de nós viu um cão atropelado
sição, quero defender uma afirmação em par- por um carro, embora não tenha morrido
ticular: que devemos não infligir dor desne- imediatamente. O cão tem convulsões, san-
cessária a animais. Antes de continuar, devo gra e gane. De forma menos dramática, a
esclarecer o que quero dizer com “dor desne- maior parte de nós, num qualquer momen-
cessária”. O ponto pode ser estabelecido mais to, já pisou a cauda de um gato ou a pata de
claramente com uma analogia. um cão e testemunhou a reação do animal.

136 Editável e fotocopiável © Texto | Dúvida Metódica, 11.º ano


Textos Complementares

A reação, surpreendentemente, é como a mento que lhes destinamos? Por que razão
nossa própria reação em casos similares. Se não podemos continuar a usá-los para os
alguém pisa a minha mão, provavelmente nossos fins, sejam eles quais forem?”
eu gritarei e tentarei mexê-la. Coloquemos a questão ao contrário por um
Mas não precisamos de fazer depender o momento, e perguntemos: por que razão
nosso ponto nas provas comportamentais, pensamos que devemos poder usá-los para
embora me pareça realmente que isso é os nossos fins, considerando que eles so-
mais do que suficiente. Devemos também frem? Afinal de contas, opomo-nos firme-
notar que partilhamos estruturas anatómi- mente à inflição desnecessária de dor a se-
cas importantes com animais superiores. res humanos. Se os animais também sentem
O sistema nervoso central de um ser huma- dor, porque não devemos ter a mesma relu-
no é impressionantemente semelhante ao tância em infligir-lhes dor desnecessária?
de um chimpanzé, cão, porco, e mesmo ao Um princípio fundamental da ética é que
de um rato. Isto não é o mesmo que dizer devemos tratar casos iguais de forma igual.
que os cérebros são exatamente iguais; não Isto é, devemos tratar dois casos igualmen-
o são. O córtex cerebral nos seres humanos te, a não ser que exista alguma razão geral e
está mais desenvolvido do que na maior relevante que justifique a diferença no tra-
parte dos mamíferos (embora não evidente- tamento. (…) se dois seres sentem dor e se
mente quando comparado com um golfinho é impróprio infligir dor desnecessária a um
ou um grande primata); mas o córtex é a lo- deles, seria igualmente impróprio infligir
dor desnecessária ao outro.
calização nas nossas “funções cerebrais su-
(…) Embora as pessoas tenham outrora vis-
periores” — por exemplo, onde se encontra
to os animais como seres não-sencientes,
o pensamento, o discurso, etc. Contudo, as
como meros autómatos, isso já não aconte-
áreas do cérebro identificadas neurofisiolo-
ce. A crença de que os animais são incapazes
gicamente como os “centros de dor” são vir-
de sentir dor não é defensável à luz de todas
tualmente idênticas entre animais humanos
as provas comportamentais e científicas.
e não-humanos. De acordo com a biologia
Portanto, o passo comum é encontrar algu-
evolutiva, isto é exatamente o que é de es-
ma outra diferença que se pense distinguir
perar. Os centros de dor funcionaram bem
significativamente os humanos dos animais.
ao aumentar a sobrevivência de espécies
A mais frequentemente citada e promissora
menores, pelo que foram alterados apenas
candidata é a racionalidade ou a consciên-
ligeiramente em estágios evolutivos que se
cia de si como um ser contínuo. Os huma-
sucederam. As funções cerebrais superio-
nos, diz-se, podem raciocinar e pensar; os
res, no entanto, conduzem à sobrevivência
animais (presume-se) não podem. Mais ain-
e, assim, levaram a avanços mais dramáti-
da, a capacidade de raciocínio reflete-se na
cos no desenvolvimento cerebral. Conside- capacidade do humano de se ver a si mes-
rando tudo isto, parece inegável que muitos mo como um ser contínuo, como um ser que
animais sentem dor. tem um passado e que terá um futuro.
Concedamos por um momento que os hu-
Que eles sentem dor é moralmente manos são racionais e que os animais não;
relevante que os humanos têm uma consciência de si
“E depois?”, alguém poderá perguntar. como seres com uma existência contínua e
“Mesmo que os animais sintam realmente que os animais não. Porque haveria isso de
dor, por que motivo deverá isso limitar ou, fazer diferença? Ou, mais precisamente, por-
pelo menos, restringir seriamente o trata- que haveria de fazer tanta diferença como

Editável e fotocopiável © Texto | Dúvida Metódica, 11.º ano 137


Textos Complementares

faz? Será que o facto de sermos racionais le- dor, se não tivessem experiência da dor, en-
gitima o nosso uso de animais não-racionais tão não teriam o direito de não sofrer dor
de qualquer maneira que nos apeteça? desnecessária, independentemente de quão
Parece que não. Certamente não legitima inteligentes ou racionais fossem. Conse-
o tratamento abusivo de outros humanos. quentemente, parece moralmente inaceitá-
Alguns seres humanos têm sérios atrasos vel infligir dor desnecessária em animais.
mentais ou estão em comas irreversíveis,
e assim são tão racionais como os animais. Objeções à minha perspetiva
Contudo, pensamos que seria inapropriado
O sofrimento dos animais não é desnecessário
usar estes humanos quer para determinar os
efeitos do amoníaco na pele, quer para gre- Alguém poderia conceder tudo isto que es-
lhá-los para o jantar. Presumimos que fazê- tabeleci, e ainda assim afirmar que a nossa
-lo iria violar os seus direitos. Então porque utilização de animais é aceitável, uma vez
não devemos ter igual relutância quanto a que o sofrimento dos animais não é verda-
usar animais desta maneira? deiramente desnecessário. Afinal de contas,
Finalmente, podemos notar que as carac- a maior parte dos humanos come animais
terísticas em causa estão mais propriamen- e, portanto, consegue alimentar-se a par-
te ligadas a outros direitos que não sejam tir deles; a experimentação animal é uma
o direito de não sofrer dor desnecessária. parte significativa e vital da nossa tentativa
A racionalidade está mais ligada ao direito de descobrir curas para doenças humanas
de voto, de liberdade de expressão, etc., en- devastadoras e de proteger os humanos da
quanto a capacidade que um ser tem de ser introdução de produtos comerciais possivel-
consciente de si como tendo uma existência mente perigosos.
contínua parece mais intimamente ligada ao Sem dúvida que a utilização de animais
direito à vida. (Afinal de contas, a morte não destas maneiras beneficia por vezes os hu-
é temida por um ser que não tem consciência manos. Mas será genuinamente necessária?
de si como existindo no futuro.) Inversamen- Não é evidente que assim seja (…).
te, o direito a não sofrer dor desnecessária
parece ligado a apenas uma característica, Hugh LaFollette, «Direitos dos Animais e Erros dos
Humanos», Crítica –https://criticanarede.com/
nomeadamente a capacidade de sentir dor. hlafollettedireitosdosanimaiseerrosdoshumanos.html
Se os humanos não tivessem um centro de (adaptado)

138 Editável e fotocopiável © Texto | Dúvida Metódica, 11.º ano


Mais textos complementares serão disponibilizados na ,
em formato editável aos professores adotantes do projeto.
Ensino Digital
ENSINO DIGITAL

• Guião de Recursos Digitais


• Roteiro Aula Digital

© Texto | Dúvida Metódica, 11.º ano


Guião de Recursos Digitais

Guião de recursos digitais do projeto Dúvida Metódica

Capítulo 1
Epistemologia – os problemas da definição, da possibilidade e da origem do
conhecimento

Animação que utiliza o argumento do filme Matrix para


Animação – abordar a temática da perceção da realidade e dos sonhos,
A realidade é real? bem como a informação enganadora que os sentidos por
vezes proporcionam.

Baseada num texto de Descartes, esta animação ilustra o


Animação – Duvidar
processo de dúvidas constantes seguido pelo filósofo até
para alcançar
chegar à certeza da sua conclusão mais conhecida: Penso,
a certeza
logo existo.

Animação – Partindo de uma situação vivida num ambiente de sala de


O argumento da aula, esta animação oferece um exemplo concreto para
marca explicar o argumento da marca de Descartes.
APRESENTAÇÃO

Apresentação de uma afirmação supostamente retirada


Animação – Polígrafo
das redes sociais de um filósofo, que será validada ou
Filosófico (Descartes)
desmentida.

Apresentação de uma afirmação supostamente retirada


Animação – Polígrafo
das redes sociais de um filósofo, que será validada ou
Filosófico (Hume)
desmentida.

Apresentação – Apresentação em formato PowerPoint contendo todas as


Banco de Fotografias fotografias e cartoons existentes no manual, acompanhados
e Cartoons por legenda ou questão orientadora.

Apresentação em formato PowerPoint de todos os esquemas


Apresentação –
interativos presentes no manual (permite apresentação passo
Esquemas interativos
a passo).

Apresentação – Apresentação em formato PowerPoint sintetizando a matéria


Síntese de capítulo do capítulo.

Resumo da matéria do capítulo apresentado em formato


Áudio-resumo
áudio.

continua

142 © Texto | Dúvida Metódica, 11.º ano


continuação

«HOW DO YOU KNOW YOU EXIST?» James Zucker


3:02 Legendado em português
«DR. HOUSE, TEMPORADA 2, EPISÓDIO 24: NO REASON»
APRESENTAÇÃO

Vídeos da rubrica David Shore


«Visiona» 44:00 Legendado em português
«EPISTEMOLOGY: HUME’S SKEPTICISM AND INDUCTION, PART 1»
Daniel Greco
3:02 Legendado em português

Recursos vídeo, áudio e outros para ilustrar e complementar


Recursos dos
informação dada no manual, rapidamente e de forma estimulante
códigos QR
para os alunos.

Animação – Polígrafo Apresentação de uma afirmação supostamente retirada das redes


Filosófico (Descartes) sociais de um filósofo, que será validada ou desmentida.

Animação – Polígrafo Apresentação de uma afirmação supostamente retirada das redes


Filosófico (Hume) sociais de um filósofo, que será validada ou desmentida.

5-10 questões sobre a matéria do capítulo disponibilizadas em


Kahoot
APLICAÇÃO

formato Kahoot.

Jogo semelhante ao conhecido jogo televisivo «Quem quer


Jogo – Penso, logo ser milionário», mas em que as ajudas são substituídas pela
acerto possibilidade de aceder a uma dica de um filósofo que orientará a
resposta.

Teste interativo composto por 10 questões com correção


Teste interativo 1
automática e relatório de desempenho.

Teste interativo composto por 10 questões com correção


Teste interativo 2
automática e relatório de desempenho.

Kahoot 5-10 questões sobre a matéria do capítulo disponibilizadas em


Exclusivo do Professor formato Kahoot.

Teste interativo composto por 10 questões com correção


Teste interativo 1
automática e relatório de desempenho.

Teste interativo 2 Teste interativo composto por 10 questões com correção


Exclusivo do Professor automática e relatório de desempenho.

Permite gerar testes segundo o modelo do exame nacional,


AVALIAÇÃO

selecionando o tipo e o número de questões, o capítulo ou os


Simulador de Exames
capítulos que serão testados e a origem das questões (exames
e Testes
nacionais ou questões dos autores). Todas as questões são
disponibilizadas com solução ou cenário de resposta.

Questões-aula Banco de questões-aula em formato editável e propostas de


Exclusivo do Professor resolução.

Banco de questões de diferente tipologia em formato editável,


Banco de questões
para apoio à construção de materiais de avaliação, e propostas de
Exclusivo do Professor
resolução.

Teste
Em formato editável e com propostas de resolução.
Exclusivo do Professor

© Texto | Dúvida Metódica, 11.º ano 143


Guião de Recursos Digitais

Capítulo 2
Filosofia da ciência – os problemas da demarcação, do método, da evolução
e da objetividade da ciência

Animação de uma situação retirada da banda desenhada «Tintim


Animação – Vantagens e o Templo do Sol», onde se apresentam duas reações diferentes
de saber ciência perante um eclipse solar (mística, por parte dos indígenas, e
científica, por parte de Tintim).

Animação – O canto
dos canários
Aplicação do método das conjeturas e refutações a um caso
é aprendido
concreto.
ou transmitido
geneticamente?

Animação – Polígrafo Apresentação de uma afirmação, supostamente retirada das redes


Filosófico (Popper) sociais, de um filósofo, que será validada ou desmentida.

Animação – Polígrafo Apresentação de uma afirmação, supostamente retirada das redes


Filosófico (Khun) sociais, de um filósofo, que será validada ou desmentida.

Apresentação – Apresentação em formato PowerPoint contendo todas as


Banco de Fotografias fotografias e cartoons existentes no manual, acompanhados por
e Cartoons legenda ou questão orientadora.
APRESENTAÇÃO

Apresentação em formato PowerPoint de todos os esquemas


Apresentação –
interativos presentes no manual (permite apresentação passo a
Esquemas interativos
passo).

Apresentação – Apresentação em formato PowerPoint sintetizando a matéria do


Síntese de capítulo capítulo.

Áudio-resumo Resumo da matéria do capítulo apresentado em formato áudio.

«FILOSOFIA EM PEDACINHOS – ENTÃO? É CIÊNCIA OU NÃO?»


Universidade de Nancy
4:14 Legendado em português
«O PAÍS QUE SE SEGUE: ENTREVISTA A DAVID MARÇAL»
Fundação Francisco Manuel dos Santos
Vídeos da rubrica 7:00
«Visiona»
«COMO DARWIN REVOLUCIONOU O MUNDO»
BBC News Brasil
5:19 Legendado em português
«RADIOATIVO»
110 minutos

Recursos vídeo, áudio e outros para ilustrar e complementar


Recursos dos
informação dada no manual, rapidamente e de forma estimulante
códigos QR
para os alunos.

continua

144 © Texto | Dúvida Metódica, 11.º ano


continuação

Animação – Polígrafo Apresentação de uma afirmação supostamente retirada das redes


Filosófico (Popper) sociais de um filósofo, que será validada ou desmentida.

Animação – Polígrafo Apresentação de uma afirmação supostamente retirada das redes


Filosófico (Kuhn) sociais de um filósofo, que será validada ou desmentida.

5-10 questões sobre a matéria do capítulo disponibilizadas em


Kahoot
APLICAÇÃO

formato Kahoot.

Jogo semelhante ao conhecido jogo televisivo «Quem quer


Jogo – Penso, logo ser milionário», mas em que as ajudas são substituídas pela
acerto possibilidade de aceder a uma dica de um filósofo que orientará a
resposta.

Teste interativo composto por 10 questões com correção


Teste interativo 1
automática e relatório de desempenho.

Teste interativo composto por 10 questões com correção


Teste interativo 2
automática e relatório de desempenho.

Kahoot 5-10 questões sobre a matéria do capítulo disponibilizadas em


Exclusivo do Professor formato Kahoot.

Teste interativo composto por 10 questões com correção


Teste interativo 1
automática e relatório de desempenho.

Teste interativo 2 Teste interativo composto por 10 questões com correção


Exclusivo do Professor automática e relatório de desempenho.

Permite gerar testes segundo o modelo do exame nacional,


AVALIAÇÃO

selecionando o tipo e o número de questões, o capítulo ou os


Simulador de Exames
capítulos que serão testados e a origem das questões (exames
e Testes
nacionais ou questões dos autores). Todas as questões são
disponibilizadas com solução ou cenário de resposta.

Questões-aula Banco de questões-aula em formato editável e propostas de


Exclusivo do Professor resolução.

Banco de questões de diferente tipologia em formato editável,


Banco de questões
para apoio à construção de materiais de avaliação, e propostas de
Exclusivo do Professor
resolução.

Teste
Em formato editável e com propostas de resolução.
Exclusivo do Professor

© Texto | Dúvida Metódica, 11.º ano 145


Guião de Recursos Digitais

Capítulo 3
Filosofia da arte – o problema da definição de arte

Enquanto observamos imagens da peça escultórica «A Linha


Animação – Isto é
do Mar» do escultor Pedro Cabrita Reis, ouve-se um diálogo
Arte?!
hipotético sobre a mesma, discutindo se é ou não é arte.

Animação – Polígrafo Apresentação de uma afirmação, supostamente retirada das redes


Filosófico sociais, de um filósofo, que será validada ou desmentida.

Apresentação – Apresentação em formato PowerPoint contendo todas as


Banco de Fotografias fotografias e cartoons existentes no manual, acompanhados por
e Cartoons legenda ou questão orientadora.
APRESENTAÇÃO

Apresentação em formato PowerPoint de todos os esquemas


Apresentação –
interativos presentes no manual (permite apresentação passo a
Esquemas interativos
passo).

Apresentação – Apresentação em formato PowerPoint sintetizando a matéria do


Síntese de capítulo capítulo.

Áudio-resumo Resumo da matéria do capítulo apresentado em formato áudio.

«POR QUE A BELEZA IMPORTA?» Roger Scruton


Vídeos da rubrica Legendado em português
«Visiona» «À LUZ DA NOITE» David Wnendt
106 minutos Legendado em português

Recursos vídeo, áudio e outros para ilustrar e complementar


Recursos dos
informação dada no manual, rapidamente e de forma estimulante
códigos QR
para os alunos.

Animação – Polígrafo Apresentação de uma afirmação supostamente retirada das redes


Filosófico sociais de um filósofo, que será validada ou desmentida.

5-10 questões sobre a matéria do capítulo disponibilizadas em


Kahoot
formato Kahoot.
APLICAÇÃO

Jogo semelhante ao conhecido jogo televisivo «Quem quer


Jogo – Penso, logo ser milionário», mas em que as ajudas são substituídas pela
acerto possibilidade de aceder a uma dica de um filósofo que orientará a
resposta.

Teste interativo composto por 10 questões com correção


Teste interativo 1
automática e relatório de desempenho.

Teste interativo composto por 10 questões com correção


Teste interativo 2
automática e relatório de desempenho.

Kahoot 5-10 questões sobre a matéria do capítulo disponibilizadas em


Exclusivo do Professor formato Kahoot.
AVALIAÇÃO

Teste interativo composto por 10 questões com correção


Teste interativo 1
automática e relatório de desempenho.

Teste interativo 2 Teste interativo composto por 10 questões com correção


Exclusivo do Professor automática e relatório de desempenho.

continua

146 © Texto | Dúvida Metódica, 11.º ano


continuação

Permite gerar testes segundo o modelo do exame nacional,


selecionando o tipo e o número de questões, o capítulo ou os
Simulador de Exames
capítulos que serão testados e a origem das questões (exames
e Testes
nacionais ou questões dos autores). Todas as questões são
disponibilizadas com solução ou cenário de resposta.
AVALIAÇÃO

Questões-aula Banco de questões-aula em formato editável e propostas de


Exclusivo do Professor resolução.

Banco de questões de diferente tipologia em formato editável,


Banco de questões
para apoio à construção de materiais de avaliação, e propostas de
Exclusivo do Professor
resolução.

Teste
Em formato editável e com propostas de resolução.
Exclusivo do Professor

© Texto | Dúvida Metódica, 11.º ano 147


Guião de Recursos Digitais

Capítulo 4
Filosofia da religião – o problema da existência de Deus

Animação de um diálogo retirado da obra Némesis, de Philip


Animação – Porquê Roth, entre um pai que perdeu um filho e o seu antigo professor,
tanto mal? questionando-se sobre o sentido da vida e sobre a noção de
justiça.

Animação – Polígrafo Apresentação de uma afirmação, supostamente retirada das redes


Filosófico sociais, de um filósofo, que será validada ou desmentida.

Apresentação – Apresentação em formato PowerPoint contendo todas as


Banco de Fotografias fotografias e cartoons existentes no manual, acompanhados por
e Cartoons legenda ou questão orientadora.

Apresentação em formato PowerPoint de todos os esquemas


Apresentação –
interativos presentes no manual (permite apresentação passo a
Esquemas interativos
passo).
APRESENTAÇÃO

Apresentação – Apresentação em formato PowerPoint sintetizando a matéria do


Síntese de capítulo capítulo.

Áudio-resumo Resumo da matéria do capítulo apresentado em formato áudio.

«ANSELM & THE ARGUMENT FOR GOD» Crash Course Philosophy


5:32 minutos Legendado em português
«DEUS VISITA INDIVÍDUO»
Da série «Mixórdia de Temáticas»
Ricardo Araújo Pereira
Vídeos da rubrica 3:50 minutos
«Visiona»
«PORQUE NÃO É CRISTÃO?» Excerto da entrevista dada por
Bertrand Russell em 1959 ao canal CBC
3:25 minutos Legendado em português
«ABSOLUTELY ANYTHING» Terry Jones
85 minutos Legendado em português

Recursos vídeo, áudio e outros para ilustrar e complementar


Recursos dos
informação dada no manual, rapidamente e de forma estimulante
códigos QR
para os alunos.

Animação – Polígrafo Apresentação de uma afirmação supostamente retirada das redes


Filosófico sociais de um filósofo, que será validada ou desmentida.

5-10 questões sobre a matéria do capítulo disponibilizadas em


Kahoot
formato Kahoot.
APLICAÇÃO

Jogo semelhante ao conhecido jogo televisivo «Quem quer


Jogo – Penso, logo ser milionário», mas em que as ajudas são substituídas pela
acerto possibilidade de aceder a uma dica de um filósofo que orientará a
resposta.

Teste interativo composto por 10 questões com correção


Teste interativo 1
automática e relatório de desempenho.

Teste interativo composto por 10 questões com correção


Teste interativo 2
automática e relatório de desempenho.

continua

148 © Texto | Dúvida Metódica, 11.º ano


continuação

Kahoot 5-10 questões sobre a matéria do capítulo disponibilizadas em


Exclusivo do Professor formato Kahoot.

Teste interativo composto por 10 questões com correção


Teste interativo 1
automática e relatório de desempenho.

Teste interativo 2 Teste interativo composto por 10 questões com correção


Exclusivo do Professor automática e relatório de desempenho.

Permite gerar testes segundo o modelo do exame nacional,


AVALIAÇÃO

selecionando o tipo e o número de questões, o capítulo ou os


Simulador de Exames
capítulos que serão testados e a origem das questões (exames
e Testes
nacionais ou questões dos autores). Todas as questões são
disponibilizadas com solução ou cenário de resposta.

Questões-aula Banco de questões-aula em formato editável e propostas de


Exclusivo do Professor resolução.

Banco de questões de diferente tipologia em formato editável,


Banco de questões
para apoio à construção de materiais de avaliação, e propostas de
Exclusivo do Professor
resolução.

Teste
Em formato editável e propostas de resolução.
Exclusivo do Professor

Capítulo 5
A legitimidade da experimentação animal

Animação – E se…
seres extraterrestres Animação de uma situação hipotética, abordando a temática da
decidissem ética, senciência e sofrimento causado aos seres utilizados como
utilizar-nos em cobaias em experiências científicas.
testes laboratoriais?

«PETER SINGER: O ESTATUTO MORAL DO SOFRIMENTO»


3:29 minutos Legendado em português
«COMO TRATAR OS ANIMAIS NÃO-HUMANOS? OS ARGUMENTOS
DE PETER SINGER E CARL COHEN»
APRESENTAÇÃO

9:47 minutos Legendado em português


«RAZÕES QUE EXPLICAM A NECESSIDADE DA EXPERIMENTAÇÃO
ANIMAL»
Vídeos da rubrica 4:27 minutos Legendado em português
«Visiona»
«E SE O RECURSO À EXPERIMENTAÇÃO ANIMAL ACABASSE? QUE
ALTERNATIVAS EXISTIRIAM?»
6:29 minutos Legendado em português
«EXPERIMENTAÇÃO ANIMAL: PRÓS E CONTRAS»
3:40 minutos Legendado em português
«COBAIAS» Alex Lockwood
16:47 minutos Legendado em português

Recursos vídeo, áudio e outros para ilustrar e complementar


Recursos dos
informação dada no manual, rapidamente e de forma estimulante
códigos QR
para os alunos.

© Texto | Dúvida Metódica, 11.º ano 149


Guião de Recursos Digitais

Encontram-se disponíveis na Aula Digital


os conteúdos do Dossiê do Professor.

Os conteúdos de seguida assinalados estão dispo-


níveis em formato editável, permitindo aos colegas
a sua alteração de acordo com as suas preferências
e as necessidades específicas dos seus alunos:

• Questões-aula
• Banco de questões
• Testes (inclui teste diagnóstico) e respetivos
guiões de conteúdo para os alunos
• Cidadania e Desenvolvimento
• Textos complementares
• Relembra Ideias-chave
• Outra maneira de preparar os Exames:
um exemplo
• Prova com a estrutura do Exame Nacional
de Filosofia
• Avaliação por competências: exemplos
• Temas/problemas alternativos: Guerra justa
e Tecnociência
• Planificações
• Respostas de todas as atividades das rubricas
do manual

150 © Texto | Dúvida Metódica, 11.º ano


Guia do utilizador • Professor

Índice
I. Aula Digital – o que é e como aceder?
III. Explorar os manuais digitais
III.. Explorar os recursos exclusivos do Professo
sor
a. Dossiê do Professor
b Banco de Recursos
b.
IV. Exp
xplorar os recursos do Aluno
V. Criar
ar e editar aulas e testes interativo
vos
VI. Comun
unicar e orientar o estudo dos
s alunos
a. Comu
municar
b. Enviarr e acompanhar a realizaç ção
de trabbalhos e testes interativ
vos
c. Partilharr recursos

151
I. Aula Digital – o que é e como aceder?
A Aula Digital, disponível em auladigital.leya.com, é a plataforma de ensino e apren-
dizagem da LeYa Educação.
Aqui o Professor poderá aceder aos projetos escolares e a todos os recursos e
ferramentas digitais a eles associados.

Para explorar os recursos disponíveis na plataforma, basta: Tutorial: Registo e


acesso do Professor
1. Aceder a auladigital.leya.com;
2. Clicar em Entrar;
3. Preencher os campos de Utilizador e Palavra-Passe;
4. Clicar em Entrar.

1 2

3
4

152 © Texto | Dúvida Metódica, 11.º ano


A Aula Digital está organizada nas seguintes áreas:
As minhas salas
Área de comunicação
Biblioteca com os alunos através
Manuais e recursos digitais da criação de salas, que
a eles associados, permitem atribuição
incluindo materiais de trabalhos e testes
exclusivos do Professor. interativos (com relatório
detalhado de resultados).

Banco de Recursos Os meus testes


Pesquisa de recursos Ferramenta de construção
por tipologia, de testes interativos.
ano de escolaridade, Permite o acesso a
disciplina e/ou temas questões de testes já
curriculares. existentes e a criação de
questões personalizadas.
As questões podem incluir
imagens, áudios e fórmulas
matemáticas. Estes testes
Smart podem ser partilhados
Vídeos e sínteses, para rever o com os alunos através da
essencial da matéria, e quizzes área “As minhas salas” ou
com explicações imediatas, para exportados para Word®.
esclarecer dúvidas à medida
que elas surgem. O registo do
progresso apoia o aluno no seu As minhas aulas
estudo autónomo. Ferramenta de elaboração de sequências de
recursos disponíveis na área Biblioteca e/ou no
Banco de Recursos. Inclui ainda a possibilidade
de carregamento de recursos próprios. Estas
sequências podem ser projetadas na sala de aula
e/ou partilhadas com os alunos através da área
“As minhas salas”.

© Texto | Dúvida Metódica, 11.º ano 153


II. Explorar os manuais digitais
Na Biblioteca, estão disponíveis todos os manuais em formato digital,
assim como os recursos digitais a eles associados.

Para explorar
uma publicação
em conjunto com
os seus recursos
digitais, basta
clicar sobre a
capa.

154 © Texto | Dúvida Metódica, 11.º ano


A projeção do manual digital facilita a exploração dos conteúdos em sala de aula.
Várias ferramentas apoiam o Professor nesta tarefa:

O zoom, o ajuste Desenho livre


à largura/altura, Nota de texto
Índice do manual a vista em página Marcador de página
Índice de recursos única/dupla e o full
Todos os desenhos,
digitais screen permitem
notas e marcações
Índice de notas e ajustar a visualização
ficam automaticamente
páginas marcadas e explorar texto,
guardados e acessíveis
imagens ou esquemas
a partir de qualquer
com todo o detalhe.
dispositivo. Pesquisa

A barra e as setas É possível destacar com diferentes


de navegação cores um excerto de texto selecionado.
permitem encontrar
rapidamente uma
página específica.

Na banda lateral surge a indicação dos recursos


digitais disponíveis. Animações, vídeos,
atividades interativas ou fichas do Caderno
de Atividades, por exemplo, são algumas das
tipologias de recursos a que o Professor pode
recorrer, sem sair da página que está a projetar.

© Texto | Dúvida Metódica, 11.º ano 155


III. Explorar os recursos exclusivos do Professor
a. Dossiê do Professor

Na área Dossiê/Editáveis de cada projeto, é possível descarregar materiais


exclusivos do Professor, totalmente editáveis, tais como planificações, grelhas
de avaliação, fichas, testes ou materiais para alunos com dificuldades ou áudios.

Na pasta
Novidades serão
disponibilizados
novos materiais
ao longo do ano.

OFFLINE
Todas as publicações e recursos digitais disponíveis na Biblioteca estão
também acessíveis offline através da app Aula Digital,
em computador, tablet ou smartphone.
Versão
para download

156 © Texto | Dúvida Metódica, 11.º ano


b. Banco de Recursos

No Banco de Recursos o Professor encontra recursos digitais das suas


disciplinas, que pode usar de forma complementar ou independente do
manual escolar.
Tutorial: Explorar o
Banco de Recursos

Estes recurs
os podem se
pesquisados r
pelos temas
curriculares
ou por palav
chave. ra

Os filtros laterais ajudam a


refinar a pesquisa por tipologia
(vídeo, ficha, teste, …), ciclo, ano
ou disciplina.

Todos os recursos da área Banco de Recursos e Biblioteca


podem ser partilhados com os alunos através da área
As minhas salas ou de qualquer outra plataforma de
comunicação.

© Texto | Dúvida Metódica, 11.º ano 157


IV. Explorar os recursos do Aluno
Na área Smart, disponibilizam-se aos alunos sequências de aprendizagem
que permitem rever o essencial de cada conteúdo, testar conhecimentos
e esclarecer dúvidas. Esta área está também disponível para o Professor,
que assim poderá fazer recomendações de estudo.

Vídeos, áudios e sínteses, organizados por temas curriculares, que


ajudam a compreender a matéria.

Quizzes com explicações imediatas, que permitem esclarecer as


dúvidas. A correção automática e o registo do progresso permitem
autorregular a aprendizagem do aluno e melhorar os resultados.

Os conteúdos Smart podem também ser explorados a partir


da app Aula Digital, disponível para computador, tablet ou
smartphone, com ou sem Internet.

158 © Texto | Dúvida Metódica, 11.º ano


V. Criar e editar aulas e testes interativos
Nas áreas Os meus testes e As minhas aulas, o Professor pode
personalizar os testes e as aulas, acedendo a propostas disponíveis
na área Biblioteca, ou criar estes recursos de raiz.

Para criar um novo teste interativo com correção automática basta: Tutorial: Criar um
teste interativo
1. Entrar na área Os meus testes;
2. Clicar em Novo teste;
3. Preencher o título, as instruções e a duração do teste;
4. Adicionar questões ao teste, clicando em:
•Questão do banco – para adicionar questões disponíveis
na área Biblioteca;
• Nova questão – para criar questões que podem incluir imagens,
áudios e fórmulas matemáticas.
5. Clicar em Gravar.

Depois de adicionar
todas as questões
ao teste é possível
definir diferentes
pesos para cada
uma das questões.

© Texto | Dúvida Metódica, 11.º ano 159


Tutorial: Criar uma
Para criar uma nova aula interativa, ou seja, uma nova sequência aula interativa
pedagógica de recursos digitais, basta:
1. Entrar na área As minhas aulas;
2. Clicar em Nova aula;
3. Preencher o título, o sumário, a duração e carregar um plano
(facultativo);
4. Adicionar recursos à aula, clicando em:
• Recursos – para adicionar recursos da Biblioteca ou do Banco de Recursos;
• Páginas – para adicionar páginas de qualquer livro disponível na Biblioteca;
• Testes – para adicionar um teste interativo da Biblioteca, do Banco
de Recursos ou da área Os meus testes;
• Ficheiro – para adicionar os seus próprios recursos;
• Texto – para adicionar texto;
• Link – para adicionar links para páginas da Internet ou vídeos do YouTube.
5. Clicar em Gravar.

5
3

4
As aulas e os
testes interativos
criados pelo Professor
também podem ser
partilhados com os
alunos através da
área As minhas
salas. Os testes interativos podem
ser exportados em formato
Word®.
As aulas e os
testes interativos
existentes na
Biblioteca podem
ser copiados para
as áreas de edição
– As minhas aulas
e Os meus testes –
para serem editados
e adaptados à
realidade das suas
turmas.

160 © Texto | Dúvida Metódica, 11.º ano


Tutorial: Criar uma
VI. Comunicar e orientar o estudo sala e associar alunos

Na área As minhas salas o Professor pode comunicar


com os alunos e orientar o seu estudo, tirando partido
dos recursos que encontra na Aula Digital.

Para criar uma sala e associar alunos basta: 5


1. Entrar na área As minhas salas e
clicar em Nova sala;
2. Preencher o nome da sala;
3. Clicar em Criar Sala;
4. Clicar em Associar alunos;
5. Disponibilizar o código da sala
aos alunos (alternativamente, é
possível associar alunos introduzindo
os seus e-mails)

a. Comunicar

Na Entrada de uma sala, o Professor pode publicar informações importantes, lançar


questões/tópicos de debate ou partilhar recursos, criando um post no mural.

Os alunos podem
responder e colocar
as suas questões num
ambiente moderado
pelo Professor.

© Texto | Dúvida Metódica, 11.º ano 161


Tutorial: Enviar um
b. Enviar e acompanhar a realização de trabalhos teste
e testes interativos

A partir de uma sala o Professor pode enviar trabalhos e testes


interativos, que os alunos podem realizar de acordo com as suas
orientações.

Para enviar um teste basta:


1. No menu Testes, clicar em Novo Teste;
2. Definir as datas e as horas de início e de fim da realização do teste;
3. Clicar em Adicionar teste e selecionar o teste interativo que pretende enviar;
4. Selecionar os alunos a quem pretende enviar o teste.

3
2

Depois de concluído o teste, o Professor acede a um relatório automático individual


para cada aluno.

162 © Texto | Dúvida Metódica, 11.º ano


Tutorial: Enviar um
Para enviar um trabalho basta: trabalho
1. No menu Trabalhos, clicar em Novo Trabalho;
2. Preencher o Título e o Enunciado do trabalho;
3. Definir a data e a hora de início e de fim da realização do trabalho;
4. Indicar se o trabalho terá avaliação;
5. Selecionar os recursos de apoio à realização do trabalho;
6. Selecionar os alunos a quem pretende enviar o trabalho.

4
5
3

6
Ao longo da
realização de um
trabalho, o Professor
pode esclarecer
individualmente
as dúvidas
de cada aluno.

c. Partilhar recursos através de qualquer plataforma

Todos os recursos disponíveis na Biblioteca e no Banco de Recursos, incluindo os


recursos exclusivos do Professor, podem ser partilhados com os alunos.

Clicando no botão de partilha, disponível no cartão de identificação ou no interior do


recurso, é possível partilhá-lo através:

da área As minhas
salas.

do Google Classroom.

do Teams, do
Moodle ou de outras
plataformas de
comunicação, copiando
e colando o link.

© Texto | Dúvida Metódica, 11.º ano 163


Respostas
(Manual)
RESPOSTAS (MANUAL)

© Texto | Dúvida Metódica, 11.º ano


RESPOSTAS (MANUAL)

Capítulo 1 justificada, mas não acredita nela 4. Pôr em causa as crenças a priori,
Epistemologia – os problemas (falta-lhe, portanto, a crença). pois não foram atingidas pelo
da definição, da possibilidade 4. Não. Para descobrir que «A Maria argumento do sonho.
e da origem do conhecimento tem o cabelo castanho» é preciso 5. A. Porque nem um Deus
vê-la (presencialmente ou numa enganador consegue abalar a
fotografia colorida, por exemplo). certeza de que para pensar é
Situação inicial • pág. 7 A proposição expressa na frase só preciso existir (se há pensamento
Sugestões de resposta (o aluno pode é conhecível a posteriori. é preciso haver quem pense).
usar outras palavras) 5. Sim. Basta pensar no conceito de B. Porque é autoevidente,
1. Por exemplo: miragens; ver esfera para concluir isso. Não ter a sua verdade impõe-se à
alguém ao longe que julgamos arestas faz parte do conceito de mente de modo imediato
conhecer e descobrir depois que esfera. e sem recorrer a qualquer
estamos enganados; a ilusão de 6. Não. Um cético moderado pode raciocínio (compreendemo-la
movimento, utilizada no cinema defender essa tese, pois trata-se intuitivamente).
e na televisão. de um ceticismo localizado e 6. C.
2. Podemos duvidar (por exemplo): relativo a uma área específica (a 7. Não. Para sentir frio é preciso
das crenças que possuímos religião). O ceticismo radical é ter corpo e o génio maligno
acerca do mundo exterior; da global. podia enganar-nos quanto a isso.
existência do mundo exterior; 7. Segundo o ceticismo radical isso Contudo, «Penso que sinto frio,
do modo como agimos, já que produz uma regressão infinita: logo existo» já seria uma boa
poderemos estar a basear-nos em vamos justificando uma crença formulação.
informação falsa, etc. com outra crença num processo
3. Se o aluno defender que sim, interminável. O que faz com que EXERCITA III • pág. 40
poderá alegar que a maioria não seja produzida nenhuma 1. É uma ideia cuja verdade é
dos sonhos é confusa e muito justificação com valor epistémico. racionalmente evidente e que
diferente da vigília / que 8. A ilusão de movimento no não se confunde com nenhuma
acordamos dos sonhos / etc. cinema; durante a noite, uma outra.
Se o aluno defender que não, pessoa que bebeu álcool parece 2. Não seria coerente, pois a
poderá alegar que muitos sonhos ver um automóvel a uma grande hipótese do Deus enganador
são indistinguíveis da vigília / que distância e ele na verdade está não foi ainda suprimida e este
há sonhos dentro de sonhos / etc. muito próximo; etc. poderia enganá-lo até em
9. A ideia dos críticos do ceticismo relação à matemática.
EXERCITA I • pág. 20 radical é que é contraditório 3. Trata-se do argumento da
1. dizer que nada se sabe, pois isso marca: tenho a ideia de Deus (ser
A. Conhecimento prático. implica saber algumas coisas, perfeito), mas não a posso ter
B. Conhecimento por contacto. nomeadamente o significado das criado pois sou muito imperfeito
C. Conhecimento proposicional. palavras usadas. e a causa não pode ser inferior
D. Conhecimento prático. ao efeito. Sendo assim, essa ideia
E. Conhecimento prático. EXERCITA II • pág. 33 foi criada pelo próprio Deus, que
F. Conhecimento proposicional. 1. Para fundamentar o «edifício» do existe.
G. Conhecimento por contacto. conhecimento humano. E, desse 4. O criador perfeito (Deus) deixou
2. Quando o sujeito se procura modo, responder ao ceticismo a ideia de si mesmo em nós como
conhecer a si mesmo. radical, nomeadamente ao uma marca – como um artista a
3. argumento dos enganos assinar a obra.
A. A Luísa tem conhecimento, sensoriais e ao argumento da 5. Solipsismo: apenas eu e as
pois tem uma crença verdadeira regressão infinita da justificação. minhas próprias experiências
justificada. 2. A dúvida é metódica porque é um são reais; os objetos físicos e
B. A Teresa não tem método para alcançar a certeza. as outras mentes só existem na
conhecimento, pois tem uma Trata-se de duvidar até encontrar minha mente.
crença verdadeira, mas sem uma algo que resista à dúvida. Não é, 6. «Ideias matemáticas», «O meu
justificação adequada (o tamanho portanto, uma dúvida psicológica corpo existe».
dos nomes não tem relação com o e realmente sentida, mas sim 7. Sereia, centauro.
tamanho das coisas). uma dúvida voluntária. 8. Descartes quer dizer que, só após
C. O Carlos não tem 3. Pôr em causa todas as crenças provar a existência de Deus, a
conhecimento, pois tem acesso a posteriori, ou seja, toda a hipótese do Deus enganador, ou
a uma informação verdadeira e informação empírica. génio maligno, fica suprimida,

Editável e fotocopiável © Texto | Dúvida Metódica, 11.º ano 167


RESPOSTAS (MANUAL)

pois Deus, sendo bom não é dependem da aprendizagem real e a ideia de sereia não
enganador. Assim, podemos permitida pela experiência, corresponde a nada real,
acreditar na veracidade das seria presumível que as crianças sendo apenas um produto da
ideias claras e distintas. mostrassem conhecê-las apesar imaginação.
9. Hume, por um lado, considera de não terem ainda muita 8. A ideia de Deus é uma ideia
que o cogito não é mais certo que experiência do mundo. Mas as complexa. Como tal, não deriva
outras crenças; por outro lado, crianças não mostram conhecer diretamente da experiência, mas
considera que a dúvida metódica essas ideias. Por isso, estas não sim indiretamente. É formada a
é tão radical que, depois da existem. partir de ideias como ser sábio,
descoberta do cogito, não se ser bondoso, ser poderoso,
consegue obter outras certezas EXERCITA V • pág. 51 criador, etc. A mente humana
a menos que se recorra a ideias 1. Genericamente, o empirismo ao refletir sobre elas e juntá-las
e capacidades que tinham sido defende que a fonte principal na ideia de Deus «multiplica»
suspensas pela dúvida metódica do conhecimento humano é a as qualidades que elas
(o que seria uma incoerência). experiência e que todas as ideias representam e concebe então
10. Trata-se da objeção conhecida derivam das nossas experiências alguém que é absolutamente
como círculo cartesiano. do mundo (não havendo ideias sábio, absolutamente bondoso,
Descartes considera que Deus inatas). absolutamente poderoso, etc.
existe porque a ideia de Deus é 2. A – impressão complexa;
clara e distinta e considera que impressão externa; B – impressão EXERCITA VI • pág. 59
pode confiar na veracidade das simples; impressão externa; 1. Relações de ideias: A, C, E, F, H, J.
ideias claras e distintas porque C – impressão simples; impressão
Deus existe. Questões de facto: B, D, G, I.
externa; D – ideia complexa;
11. A – F; B – V; C – F; D – V; E – F. 2. Exemplo de verdade necessária:
E – impressão interna; F – ideia
Um quadrado tem quatro lados.
complexa; G – ideia simples;
EXERCITA IV • pág. 44 É uma verdade necessária, pois
H – impressão complexa;
se uma figura geométrica não
1. impressão externa; I – impressão
tiver quatro lados não é sequer
I: A, C, D. interna; J – ideia simples.
um quadrado. A sua negação
II: E, G. 3. Perceção é o nome que Hume dá
é contraditória. Por isso, não
III: B, F. a qualquer conteúdo da mente.
conseguimos imaginar um
2. São ideias que já nascem Tanto as impressões como as
quadrado que não tenha quatro
connosco. Compreendê-las faz ideias são perceções.
lados.
parte da nossa natureza. 4. Significa que são mais intensas
Exemplo de verdade contingente:
3. Intuição: compreensão direta e mais fortes do que as ideias.
Espanha é uma monarquia.
e imediata da verdade de algo; Ou seja: estão mais próximas
é uma compreensão num único da realidade, mostram mais É uma verdade contingente,
«momento» e não em vários detalhadamente a realidade do pois é assim, mas podia não
«momentos» / passos como que as ideias. ser assim. A sua negação
sucede nos raciocínios. 5. Hume argumenta que, quando não é contraditória. Por isso,
Dedução: é uma inferência uma pessoa tem um certo órgão conseguimos imaginar Espanha
ou raciocínio em que a partir sensorial que não funciona, não com outro regime político
de certas ideias concluímos consegue formar ideias nesse (nomeadamente com uma
uma outra; envolve vários âmbito (por exemplo, um cego de república).
«momentos» / passos. nascença não consegue formar a 3. Para Hume o conhecimento a
4. É o conhecimento a priori, pois ideia de azul ou a ideia de verde). priori é apenas uma relação de
não contém os erros e ilusões 6. Hume discordaria. Essa pessoa ideias, ou seja, uma explicitação
que se encontram muitas vezes não conseguiria formar ideias de conceitos que já possuíamos
misturados com o conhecimento relativas a sons, mas conseguiria [graças à experiência] e, por
a posteriori. formar ideias relacionadas com isso, não diz nada novo sobre o
5. Sim, porque existe conhecimento os outros sentidos. mundo.
a priori sobre o mundo, ou seja, 7. Semelhanças: são ambas ideias 4. Vemos uma pessoa mexer os
através do pensamento é possível complexas e não derivam lábios (causa) e inferimos que
adquirir conhecimentos novos diretamente de impressões; vamos ouvir a sua voz daí a um
sobre as coisas do mundo. são formadas a partir de outras instante (efeito). Vemos o chão
6. Porque se existissem ideias que ideias. Diferenças: a ideia de molhado (efeito) e inferimos que
já nascem connosco, e que não universo corresponde a algo choveu (causa).

168 Editável e fotocopiável © Texto | Dúvida Metódica, 11.º ano


RESPOSTAS (MANUAL)

5. Temos experiência de dois é pressupor aquilo que se DISCUTE • págs. 74-75


acontecimentos que ocorrem quer mostrar. Trata-se de um Proposta de alguns tópicos
um a seguir ao outro: vemos o procedimento ilegítimo (falácia orientadores do debate
fósforo a riscar a lixa e depois da circularidade ou petição de
vemos o fósforo aceso (mas não princípio). Situação 1
vemos a conexão necessária 3. Não podemos viver como 1. Porque pensa. A Bomba diz:
entre riscar a lixa e o fósforo céticos radicais: as exigências «penso, logo existo».
aceso). Se tal sequência de da vida prática impedem 2. O capitão, depois de perguntar
acontecimentos se repetir que suspendamos o juízo à Bomba como é que ela sabe
algumas vezes teremos então relativamente àquilo de que que mais coisas existem, diz «a
experiência da sua conjunção não temos certeza ou que única experiência que te está
constante. não consigamos justificar diretamente disponível são os
6. Hume considera que a ideia racionalmente, como as teus dados sensoriais e esses
de conexão necessária é a inferências indutivas e causais. dados sensoriais não passam de
cópia de um sentimento (e Se tentássemos suspender um fluxo de impulsos elétricos
não, como se poderia esperar, o juízo, provavelmente não que estimulam o teu centro
de uma impressão externa). sobreviveríamos durante muito computacional». Seguidamente,
Esse sentimento consiste na tempo. Por isso, Hume defende a Bomba tira daí a seguinte
expectativa de que a seguir a que devemos manter essas ilação: «isso quer dizer que eu
um acontecimento A vai ocorrer crenças. não posso saber ao certo como
um acontecimento B. Criamos o 4. Hume era um cético moderado, é o universo exterior.»
hábito de esperar isso e depois pois pensava que temos alguns Descartes não concordaria com
projetamos essa expectativa no conhecimentos e que devemos o capitão na medida em que
mundo (e tendemos a acreditar manter muitas crenças que pensava que, existindo Deus
que é algo que existe no próprio não conseguimos justificar (que é perfeito e não
mundo). Nessa medida, pode racionalmente. enganador), podemos confiar
dizer-se que é algo psicológico, 5. Quando se fazem cálculos na veracidade dos dados dos
algo que existe na nossa mente. matemáticos parece haver sentidos. Por outro lado, pensava
7.1 C conhecimento novo e não uma que, caso não pudéssemos
7.2 D mera explicitação de ideias. E a confiar na bondade divina,
matemática aplica-se ao mundo estaríamos na mesma situação
EXERCITA VII • pág. 68 em muitas situações, na vida que a Bomba.
1. A tentativa de justificar a quotidiana e nas ciências. 3. Se o aluno pensar que temos,
indução leva a invocar o 6. A identificação humeana poderá usar os argumentos dos
princípio da uniformidade da entre causalidade e conjunção erros sensoriais; poderá também
natureza (a natureza é regular). constante, se for aceite, leva alegar que a hipótese de a vida
Mas tal princípio é uma verdade a ter de admitir coisas falsas. ser um sonho ou de existir um
contingente (podemos imaginar Por exemplo: o dia é causa da génio maligno são logicamente
uma natureza irregular) e, noite (ou vice-versa), já que possíveis e não foram realmente
portanto, é uma questão de entre eles existe conjunção refutadas nem por Descartes,
facto. Ora, as verdades sobre constante; o mundo não teve nem por Hume; etc.
estas não podem ser descobertas uma causa, já que a criação do Se o aluno pensar que
a priori. mundo aconteceu só uma vez, não temos, poderá usar a
2. A tentativa de justificar a não havendo, portanto, uma argumentação de Descartes; ou
indução leva a invocar o conjunção constante. alegar, como Hume, que temos
princípio da uniformidade uma tendência psicológica para
da natureza (a natureza é EXERCITA VIII • pág. 70 acreditar que existe mundo
regular). Justificar esse princípio 1. B exterior e que, por razões
a posteriori seria fazer um 2. C práticas, devemos segui-la,
raciocínio deste tipo: Até agora 3. D mesmo que não tenhamos uma
a natureza tem sigo regular. 4. A justificação racional; etc.
Logo, a natureza continuará a 5. D
ser regular. Ora, esse raciocínio Situação 2
é indutivo. Recorrer a um 1. Não temos justificação para
raciocínio indutivo no processo acreditar que o Sol vai nascer
de justificação da indução amanhã, pois é um raciocínio

Editável e fotocopiável © Texto | Dúvida Metódica, 11.º ano 169


RESPOSTAS (MANUAL)

indutivo que nos leva a acreditar II – Dr. House, «Sem Motivo» reais representadas. O que
nisso e a indução não pode ser 1. Uma alucinação é uma perceção levanta uma velha questão
racionalmente justificada. sem objeto. Ou seja: vê-se ou cética: como garantir que eu
2. Não, pois essa explicação ouve-se algo que não existe próprio não estou a alucinar
científica também se baseia realmente. O Dr. House alucina, neste momento?
numa indução e a indução por exemplo, que dá um murro
não pode ser racionalmente no Dr. Wilson. III – Hume: as relações de ideias e as
justificada. 2. Por vezes, a pessoa que alucina questões de facto
3. Se o aluno concordar com Hume tem consciência que está a 1. Sabemos menos do que julgamos.
poderá reiterar os argumentos alucinar. Contudo, muitas outras 2. Dois exemplos de relações de
por ele apresentados e defender vezes, a pessoa não tem qualquer ideias: Todos os triângulos têm
que, apesar de recorrermos muito consciência de que se trata de três lados. / 2 + 2 = 4.
frequentemente a raciocínios uma alucinação e acredita que se Três características das
indutivos, estes não têm trata de algo real. Nesses casos proposições anteriores:
justificação nem a priori nem quem alucina tem crenças falsas. 1. Expressam verdades
a posteriori. 3. O Dr. House descobriu algumas necessárias / a sua negação
Se o aluno não concordar das alucinações que tinha tido envolve contradição.
com Hume poderá alegar,
através do raciocínio. Notou 2. São conhecidas através de
por exemplo, que a indução
que algumas coisas eram meras operações do pensamento
funciona geralmente bem e que,
incompatíveis com certos factos / a sua verdade ou falsidade é
se formos cuidadosos, as suas
e concluiu que não podiam independente da experiência.
conclusões podem ter um grau
ser reais. Num certo momento 3. São a priori.
de probabilidade elevado, pelo
concluiu que a conversa que Dois exemplos de questões de
que é errado dizer que não tem
estava a ter com a diretora do facto: Está a chover lá fora no
justificação.
hospital era uma alucinação, momento em que o autor do
pois esta disse recear que ele vídeo está a gravá-lo. / O autor
VISIONA • págs. 76-77
gritasse com ela tal como tinha deste vídeo é o dono de um cão.
I – Como sabes que és real? feito no dia anterior. Ora, o Dr.
1. Exemplos de informações Três características das
House já tinha percebido que proposições anteriores:
enganadoras:
essa situação dos gritos tinha 1. Expressam verdades
– confundir, ao longe, uma pessoa
sido uma alucinação. Por isso, a contingentes / a sua negação
com outra;
situação atual também era uma não envolve contradição.
– calcular que conseguimos
alucinação. 2. A sua verdade ou falsidade
apanhar uma bola em
4. Uma interpretação plausível é está dependente da experiência.
movimento, que vem em direção
que antes ele estava a alucinar. 3. São a posteriori.
a nós, e esta acabar por cair
E dentro dessa alucinação
mesmo à nossa frente, sem que a
maior, teve outras alucinações
possamos alcançar. ANALISA TEXTOS • págs. 78-81
– e quando saía delas (e tomava
Os dados dos sentidos não são
consciência de que eram Texto 1
confiáveis porque se temos
alucinações) continuava na 1. A matemática é a priori, na
a experiência destes nos
alucinação maior. Um aspeto medida em que não recorre
enganarem algumas vezes, então
que se pode discutir é se as a qualquer observação para
não podemos garantir que não
cenas do início do episódio (o Dr. chegar às suas conclusões.
nos enganem sempre.
2. O argumento dos sentidos House a discutir com os colegas (Essas conclusões são obtidas
enganadores, o argumento do o estado clínico do doente da pensando.)
sonho e o argumento do génio língua inchada e ser alvejado 2. Não, a matemática não é
maligno. duas vezes) fazem parte da corrigível pela experiência.
3. Ao argumento do génio maligno. alucinação maior ou representam «Não há qualquer experiência
4. A dúvida mais radical, introduzida acontecimentos reais. É possível que possa desmentir, por
pelo argumento do génio fazer interpretações plausíveis exemplo, que 5 + 7 = 12. Se
maligno, leva-nos a uma certeza: do episódio com ambas as adicionássemos 5 coisas a
o cogito (ou «penso, logo existo»), possibilidades. outras 7 e chegássemos a um
pois o sujeito pensante é o Seja como for, o aspeto resultado de 13, contaríamos de
suporte de todos os pensamentos, filosoficamente relevante é o novo. Se, após termos repetido
mesmo que estes possam ser facto de as alucinações serem a soma, obtivéssemos 13 coisas,
todos falsos. indistinguíveis das experiências concluiríamos que uma das

170 Editável e fotocopiável © Texto | Dúvida Metódica, 11.º ano


RESPOSTAS (MANUAL)

12 se dividira em duas ou que radicalidade da dúvida metódica, que ela pode produzir» significa
estávamos a ver a dobrar (…) ou particularmente quando inclui a que para descobrir os efeitos não
até a ficar loucos.» possibilidade de existir um génio basta olhar para uma coisa e
3. Dizer que uma crença maligno ou Deus enganador. depois pensar nela para descobrir
matemática é a priori não Essa dúvida é tão forte que que efeito provocará: é preciso
significa que esta seja inata. talvez atinja mais coisas do que ver acontecer, é preciso ter a
Algo é inato se nasceu connosco. Descartes julga: talvez a ideia de experiência de ver acontecer.
Ora, o facto de se perceber a que o suporte do pensamento é 2. «Ao contrário do sobrinho, o
priori que 5 + 7 = 12 não significa um «eu», um sujeito, possa ser tio pôde observar, num grande
que tenha nascido connosco. uma ilusão provocada por esse número de casos, primeiro
Significa apenas que agora não génio maligno. uma bola de borracha cair ao
precisamos de fazer nenhuma 3. Se o aluno pensar que a crítica chão e, depois, o salto que ela
experiência para perceber isso, de Lichtenberg põe em causa o dá. Na verdade, nunca na sua
mas apenas de pensar. Porém, caráter indubitável do cogito, experiência houve um só caso em
«é óbvio, precisamos primeiro poderá alegar que a radicalidade que uma bola de borracha tenha
de adquirir os conceitos e a da hipótese do génio maligno é sido deixada cair numa superfície
linguagem para as expressar, tal que Descartes não possa dizer dura sem saltar, ou uma bola
antes que possamos acreditar «eu penso», mas apenas algo de borracha tenha começado a
que 5 + 7 = 12». como «há pensamento»: talvez a saltar sem primeiro ter caído ou
ideia de um «eu» seja uma ilusão ter sido atirada. Segundo Hume,
Texto 2 provocada pelo génio maligno há uma “conjunção constante”
1. Porque nos leva a questionar e, nesse caso, torna-se muito entre a queda da bola e o salto
de modo global (ou radical) a incerto dizer «existo». que dá.»
veracidade das nossas crenças. Se o aluno pensar que a crítica «O tio viu uma bola de borracha
2. No filme The Matrix, as de Lichtenberg não põe em cair ao chão e saltar em muitas
pessoas iludidas tinham corpo causa o caráter indubitável ocasiões, enquanto o sobrinho
e havia a possibilidade de do cogito, poderá alegar que, só viu isso acontecer uma vez.
serem desligadas do sistema mesmo admitindo – no contexto Todavia, o tio não viu nada que
e libertadas. Na situação da dúvida – que o «eu penso» é o sobrinho não tivesse visto
imaginada por Putnam as ilegítimo e que Descartes deveria também, apenas teve mais
pessoas iludidas já não têm ter dito «há pensamento», vezes a mesma sequência de
corpo (só o cérebro) e, por isso, é continua a ser certo que existe experiências. Ambos observam
presumível que não possam ser um qualquer suporte desse que uma bola cai e depois salta –
libertadas. pensamento, mesmo que não nada mais. O tio, porém, acredita
3. A experiência mental do seja um «eu». que há uma conexão necessária
«cérebro numa cuba» proposta entre a bola cair e saltar. E isto
por Putnam é similar à hipótese Texto 4 não é alguma coisa que ele
cartesiana de a vida ser um 1. «O bebé examina-a [a bola] encontre na sua experiência; a
sonho (argumento do sonho) e de todos os ângulos, cheira-a, sua experiência é a mesma que
à hipótese de existir um Deus mete-a na boca, apalpa-a, depois a do sobrinho, só que se repetiu
enganador, pois também nos deixa-a cair. Não obstante muitas vezes.»
permite questionar se alguma das o exame cuidadoso a que 3. Hume diz que essa ideia é a cópia
nossas crenças será indubitável. submeteu a bola, o menino não é a cópia de um sentimento (e
Presumivelmente, um cérebro tem maneira de saber que, em não, como se poderia esperar,
numa cuba também poderia vez de cair suavemente no chão de uma impressão externa).
dizer «penso, logo existo». como os outros brinquedos, ela Esse sentimento consiste na
salta.» expectativa de que a seguir a
Texto 3 «Só pelo exame de uma coisa, um acontecimento A vai ocorrer
1. Lichtenberg critica o cogito diz-nos Hume constantemente, um acontecimento B. Criamos o
cartesisano, o «penso, logo não poderemos dizer quais os hábito de esperar isso e depois
existo», e a pretensão de efeitos que ela pode produzir. projetamos essa expectativa no
Descartes ter encontrado uma Só podemos determinar as suas mundo.
certeza capaz de resistir ao génio consequências em resultado da
maligno. experiência.» A expressão «Só PÕE-TE À PROVA • págs. 83-85
2. A expressão «pelo menos no pelo exame de uma coisa (…) não Grupo I
contexto da dúvida» refere-se à poderemos dizer quais os efeitos 1. A 2. D

Editável e fotocopiável © Texto | Dúvida Metódica, 11.º ano 171


RESPOSTAS (MANUAL)

3. A 6. B de termos a ideia de Deus não prática da ciência e permite a


4. B 7. A prova que Deus existe poderá, construção de instrumentos e
5. C nomeadamente, explicitar mecanismos com finalidades
objeções à argumentação específicas, como computadores
Grupo II utilizada para provar a existência e telemóveis.
1. A. Conhecimento prático de Deus, por exemplo: a 4.
B. Conhecimento proposicional circularidade e a incoerência A. V G. V
C. Conhecimento proposicional dos argumentos utilizados por
B. V H. F
D. Conhecimento prático Descartes.
C. F I. V
E. Conhecimento por contato
2. O argumento dos enganos D. V J. F
sensoriais. O conhecimento E. F K. F
Capítulo 2
baseado nos dados sensoriais é F. F L. V
Filosofia da ciência –
falível, por vezes, engana-nos. Por 5. Uso da linguagem matemática e
isso, existe a possibilidade de as
2.1 Os problemas da demarcação rejeição das autoridades como
crenças justificadas com base na e do método justificação última.
experiência poderem ser falsas.
Não podemos garantir que isso SITUAÇÃO INICIAL • pág. 87 EXERCITA II • pág. 107
não acontece. Sugestões de resposta (o aluno pode 1. Como é que se pode distinguir as
3. O argumento da regressão
usar outras palavras) teorias científicas das teorias não
infinita da justificação. Segundo
1. Os incas entendem o eclipse solar científicas?
Descartes, a refutação deste
como um castigo dos deuses 2. A verificação de uma hipótese
argumento foi realizada com
e pensam que existe o risco supõe a possibilidade de
a descoberta do cogito (uma
de o Sol desaparecer e nunca
crença básica, autoevidente, que conceber experiências que
mais reaparecer. Tintim tem
não necessita de ser justificada estabeleçam conclusivamente
uma compreensão científica do
a partir de outras crenças, a verdade ou a falsidade dessas
eclipse solar: ele sabe que é um
colocando fim ao processo de afirmações. A confirmação
fenómeno natural que por vezes
regressão). de uma hipótese supõe a
ocorre (quando a Lua se interpõe
4. possibilidade de conceber
entre a Terra e o Sol, ocultando a
1. E 5. D experiências que confirmem, com
sua luz) e que dura apenas alguns
2. B 6. A maior ou menor probabilidade, a
minutos.
3. F 7. G
2. Se o aluno pensar que não há verdade dessas afirmações.
4. C
perigo algum poderá alegar que 3. Enunciados científicos como as
5. Não substanciais / não
uma compreensão não científica leis de Kepler são universais.
informativas / não nos dão
dos eclipses, como a dos incas na Ora, a verdade destas não pode
conhecimento sobre o mundo.
BD, faz sentido no contexto da nunca ser verificada, dado que
6. Fig. 1 – Ideia complexa (anjo).
sua cultura. a observação incide sempre
Formada a partir de várias ideias
Se o aluno pensar que há num número finito de casos
(de homem e de pássaro), através
perigo poderá alegar que uma particulares. Assim sendo, os
da imaginação e do pensamento.
compreensão não científica
Fig. 2 – Ideia complexa (pássaro). enunciados científicos nunca
dos eclipses constitui uma
Formada a partir de várias ideias seriam verdadeiros.
simples (bico, patas, asas, etc.), superstição, ou seja, uma crença
4. A afirmação «Todos os metais
através do pensamento e da falsa e sem fundamento. Ora,
dilatam com o calor» é mais
memória. as superstições podem levar as
pessoas a agirem de modo nocivo falsificável do que a afirmação
7. Se o aluno defender que o
para si próprias e para os outros. «O cobre dilata com o calor»
facto de termos a ideia de Deus
porque se aplica a mais casos e,
prova que Deus existe poderá,
EXERCITA I • pág. 95 portanto, corre um risco maior de
nomeadamente, esclarecer o
1. A, D, E e G. ser refutada.
argumento da marca e responder
a uma das objeções a este 2. (Por exemplo) é um 5. Linguagem vaga e a não
argumento, defendendo as ideias conhecimento: rigoroso / submissão a testes imparciais.
em que Descartes se baseia para resultante de investigações / 6. A. Falsificável B. necessária /
provar a existência de Deus. coerente. suficiente C. a astrologia / a
Se o aluno defender que o facto 3. A tecnologia é uma aplicação psicanálise / o marxismo, etc.

172 Editável e fotocopiável © Texto | Dúvida Metódica, 11.º ano


RESPOSTAS (MANUAL)

7. A. É falsificável: se for descoberto não aumentam o grau de as que se aproximam mais da


um som com velocidade superior probabilidade de a teoria ser verdade.
à da luz, a frase será falsa. verdadeira. As teorias apenas
B. Não é falsificável: mesmo que podem ser corroboradas, isto EXERCITA VI • pág. 130
se descubram planetas sem vida é, aceites provisoriamente, 1. A verdade deve ser entendida
inteligente, isso não mostra a enquanto resistirem às tentativas como um princípio regulador da
falsidade da frase (proposições de falsificação. atividade dos cientistas, algo que
particulares não se refutam com 4. É dedutiva. Recorre-se à forma de eles aspiram alcançar (um ideal)
contraexemplos). inferência do modus tollens (ou e não uma característica das
8. A. – F; B. – F; C. – V; D. – F negação da consequente). teorias científicas.
5.1As taxas deste tipo de cancro são 2. A adaptação, através do processo
EXERCITA III · pág. 112 mais elevadas nos países com de seleção natural, dos seres
maior taxa de fumadores. Logo, vivos ao meio ambiente.
1. A tese de que o método científico
fumar causa cancro do pulmão. Sim, o progresso é cumulativo
se baseia na indução.
5.2 As taxas deste tipo de cancro porque a ciência evolui através
2. Não se deveriam basear na
não são mais elevadas nos de um processo de eliminação de
autoridade (de filósofos e
países com maior taxa de erros, as teorias atuais corrigem
cientistas do passado ou da
fumadores. Logo, fumar não
religião), mas no raciocínio lógico as falhas das teorias anteriores,
causa cancro do pulmão.
e na experiência. permitindo assim um acréscimo
3. A experimentação. de novos conhecimentos e,
Filosofia da ciência –
4. Não, referem-se todos os casos consequentemente, uma maior
semelhantes: do passado, do 2.2 Os problemas da evolução e aproximação à verdade.
presente e do futuro. da objetividade da ciência 3. Os resultados obtidos nos
testes empíricos (são estes que
EXERCITA IV • pág. 118 SITUAÇÃO INICIAL • pág. 123 falsificam ou corroboram as
1. Se uma pessoa não tiver teorias).
Sugestões de resposta (o aluno pode
conhecimentos de geologia 4. Não teríamos boas razões
usar outras palavras)
acerca das características dos para confiar nas explicações
1. A ideia da expansão do universo
diferentes tipos de rochas científicas dos fenómenos e no
não é fácil de entender. É, aliás,
(por exemplo, se não souber funcionamento de objetos que
contraintuitiva, pois coloca
a diferença entre rochas resultam da sua aplicação. O
em causa a informação que
sedimentares e magmáticas), facto de as teorias terem resistido
os nossos sentidos nos dão:
ao observar uma rocha não às tentativas de falsificação não
quando olhamos para o céu este
poderá identificar de que tipo de é uma justificação racional para
parece-nos estático e sempre
rocha se trata ou explicar a sua as considerarmos fiáveis, pois
igual. Durante muitos séculos
formação. no futuro podem vir a revelar-se
os homens acreditaram nestas
2. Não. Os resultados dos testes falsas.
ideias. Todavia, a utilização de
falsificam ou corroboram os 5. A dificuldade em aceitar ideias
telescópios poderosos permitiu
enunciados científicos. Mesmo novas e analisar os factos numa
mostrar a sua falsidade.
quando estes são corroborados, outra perspetiva, segundo
2. O motivo, de acordo com o
poder-se-ão descobrir erros no físico Vítor Cardoso, foi o facto Popper, pode ser ultrapassada
futuro. desse astrofísico não ser muito através da análise imparcial dos
conhecido nem prestigiado junto dados e da discussão crítica.
EXERCITA V • pág. 121 da comunidade científica. Foi isso
1. D, A, C, B que explicou que não tenha sido EXERCITA VII • pág. 137
2. C, B, A dada, na altura, a devida atenção 1. A comunidade científica é
3.1 A conclusão inferida foi que o à sua teoria. constituída pelos especialistas
registo fotográfico de um buraco 3. Segundo o físico Vítor Cardoso, de cada ciência. Estes têm uma
negro era «uma confirmação o esclarecimento de um assunto formação semelhante e seguem
poderosa» da teoria da na ciência é possível através do o mesmo paradigma nas suas
relatividade geral de Einstein. confronto de argumentos lógicos investigações.
3.2 Segundo Popper, um enunciado e da discussão pública. 2. A existência de um paradigma e
científico nunca pode ser É esta atitude crítica que permite de uma comunidade científica.
confirmado por dados distinguir, com clareza, as teorias 3. Quando um enigma não é
observacionais, ou seja, estes corretas das incorretas, ou seja, resolúvel pelo paradigma

Editável e fotocopiável © Texto | Dúvida Metódica, 11.º ano 173


RESPOSTAS (MANUAL)

dominante, isso é visto como 6. paciente (a Ana) e recolhe


uma anomalia pontual que é ou 1. Os fenómenos passam a ser dados acerca da sua situação,
ignorada, ou explicada como interpretados a partir de tentando encontrar a causa dos
um erro individual dos cientistas pressupostos metodológicos, problemas de que ela se queixa.
ou uma falha dos instrumentos científicos e filosóficos O seu diagnóstico baseia-se
utilizados. Outras vezes, os diferentes. nesses dados. Isso corresponde
cientistas criam hipóteses para 2. Utilizam-se linguagens às práticas típicas da ciência:
tentar salvar o paradigma aceite. distintas em cada um dos procura de evidências empíricas,
4. Adotam uma atitude de paradigmas e, por isso, o aplicação dos conhecimentos
questionamento, de crítica em mesmo conceito pode passar existentes de modo a resolver um
relação ao paradigma dominante a ter outro significado no novo problema, etc.
/ a tradição estabelecida, paradigma; por exemplo: a O curandeiro utiliza conceitos
procurando alternativas. noção de luz foi explicada de vagos e sem fundamento, como
5. A. paradigmas diversas maneiras, consoante o «os ciclos naturais do corpo» e
B. ciência normal paradigma em causa. «crença em energias positivas».
C. anomalias 7. Ciência extraordinária. Crítica os médicos (que têm
D. extraordinária uma formação que ele não tem),
6. EXERCITA IX • pág. 151 mas fá-lo através de afirmações
A. V G. V incorretas (não é verdade que os
1. Dois dos seguintes critérios:
B. F H. F médicos se limitem a prescrever
exatidão; consistência; alcance;
C. F comprimidos: a médica de
I. V simplicidade; fecundidade.
D. V família da Ana apenas receita
J. F 2. Kuhn defende que existem
E. V um sedativo suave e diz que
critérios objetivos para a escolha
F. F K. F o principal é ela mudar de
de teorias. Todavia, a forma
hábitos). Isso corresponde às
como são aplicados é subjetiva.
EXERCITA VIII • pág. 143 práticas típicas da pseudociência:
/ A ciência é influenciada por
1. Os paradigmas em competição ausência de rigor, desvalorização
diversos fatores, pessoais,
são incomensuráveis, apresentam das evidências empíricas, etc.
ideológicos, económicos (entre
formas distintas de ver a 2. Se o aluno concordar com
outros), que são extracientíficos.
realidade, não são comparáveis Popper poderá, por exemplo,
3. O que o espetador vê na
e, portanto, não se pode afirmar alegar que não é racional aceitar
figura depende do seu ponto
que um seja melhor do que mais do que um critério, pois
de vista: da atenção dada à
outro. Por isso, quando há uma caso contrário a distinção entre
forma ou ao fundo (preto). Algo
mudança de paradigma ocorre ciência e não ciência tornar-se-á
semelhante acontece com os
uma quebra ou rutura: a visão do arbitrária. Outra possibilidade:
cientistas quando observam
mundo até aí aceite é substituída poderá defender que o critério de
a realidade: consoante o
por outra totalmente diferente. falsificabilidade é incontornável,
paradigma orientador, fazem
Não se podem considerar essas pois exprime aspetos essenciais
uma interpretação diferente
mudanças como um progresso. da ciência, como o confronto
da realidade (veem coisas que
2. Revoluções científicas. Não são com os factos e a valorização da
antes não viam e até os mesmos
frequentes na história da ciência, capacidade explicativa.
objetos passam a ser vistos de
são episódios raros – na maior Se o aluno não concordar com
forma diferente). Popper poderá, por exemplo,
parte do tempo as atividades dos
4. A negação do progresso alegar que a complexidade e a
cientistas inserem-se na ciência
cumulativo ao longo da história diversidade das investigações
normal.
da ciência. científicas tornam inviável o
3. A ciência normal.
4. A. Revolução científica. 5. recurso a um único critério
B. Anomalias. 5.1 D 5.3 A de cientificidade, pelo que é
C. Ciência normal. 5.2 C 5.4 B legítimo aceitar mais do que um
D. Paradigma. critério.
E. Incomensurabilidade. DISCUTE • págs. 154-155
F. Pré-ciência Proposta de alguns tópicos Situação 2
G. Ciência extraordinária orientadores do debate 1.
H. Paradigma • A teoria do Big Bang. Tese:
I. Crise Situação 1 O universo teve um início e está em
5. A um argumento por analogia. 1. A médica conversa com a expansão.

174 Editável e fotocopiável © Texto | Dúvida Metódica, 11.º ano


RESPOSTAS (MANUAL)

• A teoria do estado estacionário. através da tentativa de falsificar rapidamente. Bebam líquidos


Tese: O universo existe as teorias, recorrendo a testes quentes de 5 em 5 minutos pois
eternamente, passando a matéria a empíricos, e da análise e o vírus aloja-se no esófago,
existir espontaneamente no vácuo discussão crítica das teorias a bebida mata o vírus e
do espaço. realizada entre os especialistas. empurra-o para o estômago não
2. Os dados favoráveis à teoria do o deixando ir para os pulmões.»
Big Bang foram os seguintes: II – Covid-19: mitos e factos https://observador.pt/factchecks/
– «a observação da radiação científicos fact-check-novo-coronavirus-
cósmica de fundo de micro- 1. morre-em-temperaturas-acima-
ondas, que foi deixada pela A. Para ser científica, uma dos-23-graus/
história mais antiga do universo»; explicação precisa de (indicar Podem encontrar-se muitos
– «a existência de quasares duas destas características): outros exemplos nos links
(objetos quase estelares) e de • ser racional; seguintes:
galáxias de rádio só em regiões • basear-se no raciocínio lógico; • https://observador.pt/especiais/
distantes dele, mostrando que o • poder ser submetida a testes; as-33-informacoes-falsas-
universo remoto era diferente do • apresentar evidências ou provas sobre-o-coronavirus-que-ja-
que é hoje». consistentes que sustentem as encontramos-nas-redes-sociais/
3. Resposta aberta. ideias defendidas; • https://www.rtp.pt/noticias/
O aluno poderá defender que a • utilizar uma linguagem rigorosa; mundo/teorias-e-mitos-sobre-
perspetiva mais plausível sobre a • poder conter erros ou falhas; a-origem-do-novo-coronavirus_
evolução da ciência • submeter-se à análise crítica n1218948
– é a de Popper, justificando a dos especialistas de uma dada
sua posição com base nas ideias área científica.
III – Como as ideias de Darwin
e argumentos a favor desta B. Características das explicações
revolucionaram o mundo
teoria ou, então, explicitando pseudocientíficas, por
1. Uma perspetiva religiosa (o
e refutando as objeções à exemplo (indicar duas destas
criacionismo) considerava que
perspetiva de Popper; características): utilizam uma
o homem tinha sido criado por
– é a de Kuhn, justificando a sua linguagem vaga e imprecisa;
Deus (tal como é, sem ter sofrido
posição com base nas ideias pretendem explicar tudo (não é
e argumentos a favor desta qualquer tipo de mudança ao
possível imaginar situações em
teoria ou, então, explicitando longo do tempo).
que possam ser falsa); não podem
e refutando as objeções à 2.
ser submetidas a testes empíricos
perspetiva de Kuhn. 2.1 Fatores exteriores à atividade
e, muitas vezes, baseiam-se em
– não é nem a de Kuhn, nem a de científica: sociais e ideológicos
autoridades não qualificadas
Popper, apresentando objeções (a sociedade da época era
(gurus).
a estas duas teorias ou, então, contrária à ideia de que as
2. As condições são: recolher
uma outra explicação alternativa informação junto dos espécies, incluindo a humana,
da evolução da ciência, especialistas (ou autoridades tivessem sofrido um processo
devidamente fundamentada. reconhecidas) no assunto; biológico evolutivo).
procurar fontes credíveis, 2.2 Período da ciência normal.
VISIONA • págs. 156-157 como sites de divulgação Neste período, segundo Kuhn,
I – Popper aos pedacinhos – Então? científica oficiais (nacionais ou a comunidade científica não
É ciência ou não? internacionais), por exemplo a questiona o paradigma vigente:
1. Segundo Popper, para ser DGS (Direção Geral de Saúde) ou a maioria dos cientistas adota
científica uma investigação deve a OMS (Organização Mundial de um ponto de vista acrítico,
basear-se em teorias que sejam Saúde). descartando os problemas ou
refutáveis ou falsificáveis. 3. Exemplos de explicações as ideias que põem em causa
2. De acordo com Popper, a pseudocientíficas relacionadas o paradigma aceite, tentando
astrologia não é uma ciência com a pandemia do Covid-19: preservá-lo a todo o custo,
pelas seguintes razões: • a crença de que o coronavírus mesmo contra as evidências
– faz previsões vagas; se cura com alho ou outras empíricas.
– as explicações apresentadas mezinhas. 3.
justificam qualquer facto que • «O vírus só se propaga 3.1 A origem e a evolução das
aconteça na realidade, não são, em temperaturas baixas. espécies.
portanto, falsificáveis. Temperaturas acima dos 3.2 A evolução (ou o
3. As ciências empíricas progridem 23º não propaga e morre desenvolvimento) da ciência.

Editável e fotocopiável © Texto | Dúvida Metódica, 11.º ano 175


RESPOSTAS (MANUAL)

IV – Filme: «Radioativo» inicialmente, têm uma atitude as conjeturas são submetidas


1. Marie Curie descobriu, em hostil e desvalorizam o trabalho a rigorosos testes empíricos, e
conjunto com o seu marido científico de Marie Curie, que o que importa é a análise
Pierre Curie, a radioatividade e apenas pelo facto de ser mulher; imparcial dos resultados obtidos
dois novos elementos químicos: • as ideias morais e e a discussão pública feita pelos
o «polónio» e o «rádio». Estas conservadoras existentes em outros cientistas, entre outros
descobertas tiveram uma relação às mulheres fizeram aspetos.
importância fundamental não com que a vida privada de
apenas no desenvolvimento da Marie Curie, a sua relação
ANALISA TEXTOS • págs. 158-161
Física e da Química, mas também amorosa com o físico Paul
Langevin (que era casado) fosse Texto 1
no da Medicina. A prática médica
exposta nos jornais e tivesse 1. Popper nega que a ciência se
adotou, progressivamente, o
sido difamada publicamente baseie na indução: os cientistas
rádio no tratamento de certas
em manifestações realizadas à não começam com observações.
doenças (de pele e tumores
porta de sua casa. Ou seja: a ciência não precisa
malignos). Além disso, como se
3. Uma passagem do filme que da indução. Por isso, ainda que,
mostra no final do filme, durante
se relaciona com o conceito de como defendeu Hume, a indução
a 1.ª Guerra Mundial (1914-1918),
os aparelhos com raios X foram ciência apresentado por Popper: não tenha justificação, isso
utilizados pelos médicos, na as descrições científicas da não compromete a ciência, ou
frente de guerra, como meios decomposição de substâncias seja, não diminui o seu carácter
de diagnóstico. Marie Curie químicas feita em laboratório racional e objetivo.
(do processo de experimentação) 2. Raciocínio dedutivo.
trabalhou como voluntária, na
e do modo como as hipóteses, 3. Segundo Popper, não se pode
linha da frente da guerra, para
que levaram à descoberta da provar que uma teoria científica
garantir o funcionamento dos
radioatividade, foram sendo é verdadeira. Mesmo que a
aparelhos de radiologia.
corroboradas. teoria tenha resistido a diversas
Em 1903, o Prémio Nobel da
Uma passagem do filme que tentativas de falsificação
Física foi atribuído a Marie Curie
se relaciona com o conceito continua a ser uma conjetura
(partilhado com Pierre Curie e
de pseudociência apresentado (mantendo-se sempre a
Henri Becquerel) pela descoberta
por Popper: Marie Curie e o possibilidade de um dia vir a ser
da radioatividade. Em 1911, Marie
seu marido visitam Loie Fuller, refutada). Pode-se dizer que está
Curie recebeu o Prémio Nobel da
uma espiritualista que afirma corroborada, mas não que está
Química pela sua descoberta dos
conseguir comunicar com os provada.
novos elementos, o polónio e o
mortos através dos seus poderes
rádio. 4. Duas das seguintes:
mentais.
2. Fatores sociológicos: Os «sistemas de crenças como a
4. Se o aluno defender que é a
• na sociedade da época, as astrologia são irremediavelmente
teoria de Kuhn que permite
mulheres não tinham acesso vagos, de tal maneira que se
compreender melhor os
a uma carreira profissional torna impossível mostrar que
contributos de Curie para o
na área da investigação estão claramente errados».
desenvolvimento da ciência,
científica, nem ao ensino «A astrologia pode prever que
poderá alegar, por exemplo,
universitário: (Marie Curie foi a os escorpiões irão prosperar
que as ideias inovadoras de
primeira mulher a ensinar na nas suas relações pessoais
Marie Curie correspondem a
universidade da Sorbonne, após um período revolucionário, que à quinta-feira, mas, quando
a morte do marido); há uma atitude conservadora são confrontados com um
• a sociedade discriminava dos defensores do paradigma escorpião cuja mulher o
Marie Curie pelo facto de ela anterior e que há uma influência abandonou numa quinta-feira,
ser estrangeira e não seguir os de critérios não objetivos na é natural que os defensores
valores e as normas aceites para avaliação das ideias científicas de da astrologia respondam que,
as mulheres. Marie Curie, entre outros aspetos. considerando todas as coisas, o
Fatores ideológicos: Se o aluno defender que é a fim do casamento provavelmente
• desigualdade de género – teoria de Popper que permite acabou por ser melhor.»
considerava-se que as mulheres compreender melhor os «Nada forçará alguma vez os
não eram dotadas para a contributos de Curie para o astrólogos a admitir que a sua
investigação científica, como desenvolvimento da ciência, teoria está errada.»
os homens. A universidade é poderá alegar, por exemplo, que «A teoria [a astrologia] apresenta-
dominada por homens que, é utilizado o método crítico, que se em termos tão imprecisos que

176 Editável e fotocopiável © Texto | Dúvida Metódica, 11.º ano


RESPOSTAS (MANUAL)

nenhumas observações efetivas Os primeiros pensam que, Conclusão: Logo, os continentes


poderão falsificá-la.» embora os dados possam não não estiveram unidos como um
ser suficientes, há critérios puzzle no passado.
Texto 2 objetivos para proceder à 3.1 Os cientistas mudam as teorias
1. Primeira razão: a desconfiança avaliação e seleção das teorias e os instrumentos usados na
que algumas pessoas têm em em competição. Os segundos investigação, por exemplo.
relação aos cientistas e à ciência, defendem que os dados empíricos 3.2 Algumas das razões que
provocada principalmente pela não permitem a escolha das explicam a aceitação do novo
descoberta de algumas fraudes teorias, já que nunca são neutros, paradigma são exteriores
científicas. a sua interpretação depende à ciência, tais como fatores
Segunda razão: muitas pessoas sempre de um enquadramento ideológicos, sociais e
sofrem de doenças sem cura ou teórico (e até podem ser usados psicológicos.
cuja cura é muito cara, o que para sustentar teorias diferentes). Estas razões são, portanto,
as leva a ficar desapontadas subjetivas, pois a sua aplicação
com a ciência médica. Por PÕE-TE À PROVA • págs. 163-165 não é imparcial, mas depende
outro lado, algumas terapias Grupo I de cada pessoa, a qual irá
pseudocientíficas por vezes 1. C 5. B selecionar e dar prioridade a
proporcionam um alívio 2. A 6. A razões diferentes na avaliação
momentâneo devido ao efeito de 3. C 7. D dos paradigmas.
placebo. 4. D 8. A 3.3 Não. A avaliação e a escolha
2. As três grandes amigas da das teorias científicas depende
pseudociência são a fraude Grupo II apenas de critérios objetivos
científica, a corrupção científica 1. Está mais próximo da perspetiva e imparciais: os resultados dos
e a ciência ideologicamente indutivista do método científico, testes e a discussão crítica das
enviesada (pois geram ceticismo porque tem como ponto de teorias.
e desconfiança acerca da ciência partida os dados empíricos (a 4. À semelhança do que acontece
e das suas práticas). realização de várias observações) nas figuras ambíguas, também
3. Precisamos da autoridade e se baseia num raciocínio na ciência a interpretação da
epistémica porque nenhuma indutivo (generalização). realidade depende do paradigma.
pessoa pode estudar e avaliar A partir de uma amostra limitada Paradigmas diferentes dão acesso
cientificamente todos os assuntos (o que o Bernardo observou da a visões distintas dos objetos.
e porque muitas pessoas não têm sua condição física e dos seus 5. A negação, ao longo da história
conhecimentos nem capacidade amigos), infere-se uma conclusão da ciência, do progresso
para seguir o raciocínio da universal: «qualquer pessoa cumulativo.
ciência, da matemática e da melhora a sua saúde através do 6. Se o aluno defender que a
medicina. exercício físico». perspetiva falsificacionista
2.1 A hipótese estaria corroborada é satisfatória poderá,
Texto 3 se fossem encontrados fósseis nomeadamente, alegar que a
1. A perspetiva racionalista e a idênticos (do mesmo tipo) em ciência evolui através de um
historicista. zonas costeiras de continentes processo de eliminação de erros,
2. A ciência avança com base que hoje se encontram afastados. em que as teorias do presente,
em argumentação e indícios A hipótese (ou conjetura) seria ao corrigirem as anteriores e
empíricos sólidos. aceite provisoriamente, dado responderem a novos problemas,
3. Não. A mudança de teorias, que resistiu às tentativas de se aproximam mais da verdade.
segundo os historicistas, é falsificação. Se o aluno defender que a
comparável a uma conversão, não 2.2 Premissa 1: Se, há cerca de perspetiva falsificacionista
é inteiramente racional. Ainda 250 milhões de anos, todos os não é satisfatória poderá,
que existam critérios objetivos continentes estivessem unidos nomeadamente, explicitar
para a escolha das teorias, a como um puzzle, então os fósseis algumas das objeções ao ponto
aplicação destes é subjetiva e encontrados em zonas costeiras de vista de Popper ou alegar
depende de fatores exteriores à de continentes, hoje muito que a ciência evolui de forma
ciência, psicológicos, ideológicos afastados entre si, seriam do progressiva nos períodos da
e sociais, por exemplo. mesmo tipo. ciência normal e de forma
4. O papel dos dados na escolha Premissa 2: Não foram descontínua nos períodos em que
entre teorias rivais é diferente encontrados fósseis do mesmo existem revoluções científicas
para os racionalistas e para os tipo nas zonas costeiras de devido à incomensurabilidade
historicistas. continentes hoje afastados. dos paradigmas.

Editável e fotocopiável © Texto | Dúvida Metódica, 11.º ano 177


RESPOSTAS (MANUAL)

Capítulo 3 propriedades ou características informações que permitam a sua


Filosofia da arte – o problema da intrínsecas que permitem distingui- clarificação. A forma significante
definição de arte -las de outros objetos que não são é a propriedade existente em
arte. certos objetos que é capaz de
B. Não existe uma essência da arte. causar a emoção estética. E, por
SITUAÇÃO INICIAL • pág. 167
As obras de arte distinguem-se dos outro lado, a emoção estética
Sugestões de resposta (o aluno pode objetos que não são arte devido a é a emoção provocada pela
usar outras palavras) características contextuais (e não apreciação dos objetos que
1. Os motivos poderão ser, por intrínsecas). possuem uma forma significante.
exemplo: protestar contra o gasto C. É possível indicar condições 5. Premissa 2: A obra de arte não
elevado de dinheiro público para necessárias e suficientes da arte e, tem forma significante e não
a aquisição da obra; considerar por isso, é possível defini-la. desperta emoção estética.
que aquele objeto não é artístico 5.1 B Conclusão: Logo, a obra de arte
e é de mau gosto, chamando, 5.2 D não é autêntica.
deste modo, a atenção para isso; 5.3 C Ou a outra possibilidade:
razões ideológicas e políticas; etc. Premissa 2: A obra de arte é
2. O artista considera que EXERCITA II • pág. 193 autêntica.
cabe aos artistas decidir que Conclusão: Logo, a obra de arte
1. As pinturas que permitem ilustrar
objetos podem ou não ser arte, tem forma significante e desperta
a arte como representação são a
independentemente de isso ir emoção estética.
A e a B.
contra o gosto estabelecido
A é uma representação simbólica
ou não ser compreendido pela EXERCITA III • pág. 205
e B é uma representação
opinião pública. Além disso, ele
imitativa. C é uma obra abstrata 1. São extrínsecas, pois dizem
sugere que a sua escultura é uma
e, em princípio, não constitui respeito ao contexto institucional
obra de arte, pois esta continuará
uma representação. em que estas estão enquadradas:
a ser contemplada e apreciada 2. «um sistema do mundo da arte»
pelas gerações futuras durante A. F D. V (este é uma instituição social
muito tempo. B. V E. F onde as pessoas desempenham
3. Resposta aberta. O aluno poderá C. F F. V papéis específicos e seguem
considerar que um objeto é uma 3. Duas objeções: certas regras estabelecidas de
obra de arte se: • A arte como expressão e modo informal).
• representar a realidade; clarificação de emoções é 2.
• expressar certos sentimentos; demasiado restritiva, já que Premissa: Se o titular deste objeto
• tiver uma forma cativante; deixa de parte obras que são não tem a intenção de que ele
• os «especialistas» disserem que consideradas artísticas, algumas seja encarado como o foram as
é arte; delas obras-primas, e que obras de arte anteriores, então
• outras possibilidades de constituem contraexemplos este objeto não é uma obra de
resposta. à teoria, pois não exprimem arte.
nenhuma emoção ou exprimem Premissa: Este objeto é uma obra
EXERCITA I • págs. 176-177 emoções, mas em condições de arte.
1. diferentes das defendidas por Conclusão: Logo, o titular deste
A. Escultura Collingwood. objeto tem a intenção de que ele
B. Literatura (poesia) • O estado de espírito dos artistas seja encarado como o foram as
C. Literatura (romance) é muitas vezes desconhecido obras de arte anteriores.
D. Arquitetura e não é possível o público ter 3. Sim. Porque o autor do graffiti
E. Pintura acesso imaginativo às emoções não tem o direito de propriedade
F. Fotografia expressas pelo seu criador; por sobre a parede em que fez o seu
G. Cinema isso, a apreciação das obras de desenho [Mas, se mesmo assim, o
H. Música arte deve ser independente dos graffiti é uma obra de arte, então
2. D sentimentos do artista. o direito de propriedade não é
3. Teorias essencialistas: a 4. A circularidade relaciona- uma condição necessária].
representacional, a expressivista se com o facto de os termos 4.1 Diria que o diretor do museu
e a formalista. «emoção estética» e «forma público agiu corretamente
4. significante» serem definidos um ao permitir a exposição. Para
A. Existe uma essência da arte. em função do outro, sem que defender esta posição, poderia
Nas obras de arte estão presentes se introduzam quaisquer outras evocar que as obras do artista

178 Editável e fotocopiável © Texto | Dúvida Metódica, 11.º ano


RESPOSTAS (MANUAL)

eram artefactos criados por definidas no guião de trabalho público. De acordo com a teoria
ele para serem apresentados (fornecido pelo professor). formalista, também poderão ser
a um público do mundo da 2. Pertinência e adequação obras de arte caso se descubra
arte (com alguma preparação das imagens ilustrativas dos que têm forma significante (e
para apreciar e compreender conceitos, dos exemplos e provocam emoção estética).
os objetos artísticos em causa) contraexemplos das teorias. E é plausível que um defensor
numa instituição adequada para 3. Relevância, adequação e clareza da teoria formalista pense que
esse efeito, um museu público. dos conteúdos filosóficos é assim, pois é notório que as
Além disso, o estatuto artístico selecionados e explicitados. fotografias foram tiradas tendo
das obras foi reconhecido por 4. Correção da terminologia em conta aspetos formais (o
diversas pessoas do mundo da filosófica utilizada. enquadramento, a luz, a cor, etc.)
arte: o artista, o diretor do museu 5. Pensamento crítico e criativo. e não apenas temáticos.
público e o consultor artístico. 6. Cooperação entre os membros 3. Resposta aberta. O aluno poderá
4.2 Os críticos poderiam do grupo e relacionamento defender que as fotografias de
argumentar, por exemplo, interpessoal. Vivian Maier:
que a noção de mundo da 7. Estrutura e organização do • são arte, justificando a sua
arte é vaga e pouco clara. produto final partilhado em posição com base nas ideias
Ninguém sabe ao certo o que formato digital. e argumentos a favor de uma
é o mundo da arte, como é que 8. Correção linguística e clareza do das teorias estudadas, ou então
alguém faz parte dele, quem discurso. refutando as objeções a uma
faz parte dele e quais são as dessas teorias.
suas regras e procedimentos, DISCUTE • págs. 212-213 • não são arte, justificando a sua
ao contrário do que acontece Proposta de alguns tópicos posição com base nas ideias
noutras instituições, como as orientadores do debate e argumentos a favor de uma
universidades, por exemplo. das teorias estudadas, ou então
E que, portanto, o facto de o Situação 1 refutando as objeções a uma
diretor e o consultor terem 1. Segundo a teoria histórica, as dessas teorias.
aceitado fazer a exposição não fotografias de Vivian Maier são
significa que se tratasse de obras obras de arte. Observando as Situação 2
de arte. fotografias, é notório que ela 1. Segundo a teoria institucional,
teve a intenção séria de que estas as pinturas de Betsy poderão
EXERCITA IV • pág. 209 fossem vistas do mesmo modo ser consideradas obras de arte
1. que as obras de arte do passado, se algum membro do mundo da
A–4e6 D–7e8 pois, apesar de provavelmente arte pensar que devem sê-lo e
B – 1, 5 e 6 E–3 ela não pensar explicitamente lhes atribuir o estatuto de obras
C–2e6 em coisas como exposições candidatas à apreciação. Mas,
artísticas, essa intenção se assim for, serão arte devido
Atividade a realizar em grupo transparece nas próprias à pessoa responsável pela sua
Os critérios de correção e de fotografias (por exemplo, no exibição (alguém pertencente ao
avaliação serão definidos pelo enquadramento cuidado e nos mundo da arte) e não por causa
professor em função do modo conhecimentos de técnicas da Betsy, a autora das pinturas.
como irá ser estruturado o trabalho fotográficas que revelam). 2. As pinturas da Betsy não podem
dos alunos (se, por exemplo, 2. De acordo com a teoria ser consideradas arte segundo
tem uma componente escrita e representacional, as fotografias essas três teorias: a teoria da
oral), das competências a avaliar, de Vivian Maier são obras representação alega que não
dos conteúdos programáticos de arte, pois é manifesto representam nada (pois são
selecionados e dos critérios de que representam diversas completamente aleatórias);
avaliação definidos pelo grupo situações da existência humana, a teoria expressivista alega
disciplinar (aprovados pelo nomeadamente da vida em que, uma vez que Betsy não
Conselho Pedagógico da escola). grandes cidades. Na perspetiva é um animal racional, nunca
Eis algumas sugestões de da teoria expressivista, trata- poderia ter os estados mentais
parâmetros de avaliação que podem se de obras de arte, porque é necessários à expressão e
ser utilizados (a cotação a atribuir a defensável que contemplá-las clarificação de emoções; a
cada um deles é deixada ao critério permita expressar e clarificar teoria formalista alega que,
do professor): determinados sentimentos, uma vez que os «trabalhos»
1. Cumprimento das tarefas tanto da autora como do de Betsy são aleatórios, não

Editável e fotocopiável © Texto | Dúvida Metódica, 11.º ano 179


RESPOSTAS (MANUAL)

parecem suscetíveis de possuir as 3.2 De acordo com essa conceção, é consolação e harmonia no meio
propriedades formais capazes de arte qualquer artefacto (criado do caos e do sofrimento, pois
produzir emoções estéticas. ou não pelo artista) que seja faz-nos ter a experiência de que
3. Resposta aberta. reconhecido por alguém do meio existe algo superior que tem
4. O aluno poderá defender que a artístico (galeristas, críticos de valor em si e está fora da vida
pintura de Betsy: arte ou artistas, por exemplo). quotidiana.
• é arte, justificando a sua 4. • Nenhum deles tem razão:
posição com base nas ideias 4.1 Justifica dizendo que é pelo pode-se discordar da posição
e argumentos a favor de uma facto de ela, enquanto artista, do filósofo Roger Scruton
das teorias estudadas, ou então considerar este objeto como e também da posição dos
refutando as objeções a uma uma obra de arte. defensores da arte conceptual,
dessas teorias. A resposta pode enquadrar-se na desde que se apresentem
• não é arte, justificando a sua teoria institucional e na teoria objeções e contraexemplos às
posição com base nas ideias histórica. teses e aos argumentos que
e argumentos a favor de uma 4.2 figurativa / realista. ambos utilizam para sustentar
das teorias estudadas, ou então 5. A crítica é que a arte conceptual os seus pontos de vista.
refutando as objeções a uma contribuiu para o descrédito
dessas teorias. II – Filme «À luz da noite»
da arte e a desvalorização da
beleza. 1. As razões que fazem com
VISIONA • págs. 214-215
A resposta à crítica de Roger que o celeiro, pintado
I – Análise e discussão de diferentes Scruton é que não nos devemos geometricamente em tons de
definições de arte preocupar com o que a arte amarelo, possa ser uma obra de
1. Segundo Roger Scruton, o deve ser. São as pessoas ligadas arte são as seguintes:
objetivo da arte era a beleza. A ao meio artístico (o mundo da • Nils é um artista reconhecido e
razão de ser assim é que a beleza arte, na linguagem da teoria ele decidiu considerar aquele
era considerada um valor tão institucional) que definem o que objeto como candidato a
importante como a verdade e é arte e o que não é. Deste modo, apreciação. Por essa razão,
a bondade. Acreditava-se que a a noção de arte torna-se mais trata-se de uma obra de
contemplação de obras de arte
abrangente, permitindo que se arte. Além disso, este objeto
podia fornecer uma espécie de
passe a incluir coisas que antes foi avaliado por pessoas
consolação para o sofrimento, a
não eram consideradas arte e a que trabalham no Instituto
tristeza e o caos que, por vezes, a
apreciar como sendo belas coisas Norueguês das Artes e
vida humana contém. Contribuía,
que se pensava não serem. pertencem também ao mundo
em suma, para entender melhor a
6. Três cenários possíveis de da arte, e estas decidiram
natureza humana e dar sentido à
resposta: atribuir-lhe o estatuto de obra
vida.
• Os defensores da arte de arte, colocando-o num
2. Duchamp pôs em causa duas
conceptual têm razão: qualquer roteiro cultural para visitantes
ideias: i) a arte como um meio
coisa pode ser arte, desde que e subsidiando o artista. Esta
para transmitir a beleza ou
seja original e leve as pessoas justificação enquadra-se na
outros valores espirituais e ii) o
artista como alguém que tem a pensar. O artista não deve teoria institucional.
de possuir, além da criatividade, cultivar ou procurar a beleza, • Nils, que é o proprietário do
várias aptidões e técnicas para mas mostrar o mundo tal como celeiro, tem a intenção (firme
que as suas obras possuam, por ele é – imperfeito e caótico – e e duradoura) de que este
exemplo no caso da pintura, não um mundo ideal. seja encarado como o foram
certas características visuais. • O filósofo Roger Scruton obras de arte do passado
Em alternativa, Duchamp tem razão: a arte conceptual reconhecidas pela tradição
defende que o mais importante esgota-se na descrição verbal histórica. O artista apresenta
é a originalidade, e esta não e põe em causa a criatividade, ideias para justificar as escolhas
requer habilidades especiais. o conhecimento e a habilidade. que faz ao nível da cor, da
O objetivo do artista deve ser Ora, desvalorizar a beleza, forma e das técnicas a utilizar
captar a atenção e a imaginação substituí-la pelo feio ou pelo útil na sua obra e evidencia, de
do espetador através de qualquer conduz a um deserto espiritual, forma persistente, a pretensão
meio ou artefacto. à desolação e à alienação. A de que esta seja apreciada
3. beleza das obras de arte é um como o foram obras de arte
3.1 Um exemplo possível: Artist´s escape ideal, um lugar onde anteriores. Esta justificação
Shit, de Piero Manzoni. o ser humano pode encontrar enquadra-se na teoria histórica.

180 Editável e fotocopiável © Texto | Dúvida Metódica, 11.º ano


RESPOSTAS (MANUAL)

2. Ela vê as seguintes semelhanças emoção estética nos exageradamente os seus


entre o apartamento, pequeno e espetadores. sentimentos em palco, através
claustrofóbico que partilha com 5. Resposta aberta: de gestos e palavras, é suscitar
a sua família, e o quadro abstrato Indicação de dois aspetos mais emoções no público. Deste
de Mondrian: o quadrado apreciados ou menos apreciados ponto de vista, por exemplo, é
vermelho do quadro corresponde no filme e justificação do ponto «meritório que o facto de uma
ao espaço ocupado pela cama de vista apresentado. atriz, quando está a interpretar
dos pais na casa (o de maior 6. Resposta aberta: uma cena patética, (…) chorar
dimensão); o retângulo azul (de • Defesa de uma posição pessoal: lágrimas verdadeiras», se ela
menor dimensão) corresponde ao o filme é uma obra de arte / o pretende levar os espetadores a
espaço das camas da irmã e dela, filme não é uma obra de arte. sentirem tristeza.
e o retângulo amarelo (o mais • Fundamentação da posição Se pelo contrário, a atividade
pequeno de todos) corresponde à pessoal defendida, mobilizando teatral for entendida como arte
saída para a escada de incêndio uma ou mais teorias estudadas autêntica, o objetivo dos atores
– a pequena varanda, onde todos sobre a definição de arte. é expressar as suas próprias
podem ir respirar. emoções e, ao manifestá-las,
3. Um exemplo de uma resposta ANALISA TEXTOS • págs. 216-220 tomar consciência delas e
possível (existem outras): Texto 1 compreendê-las com clareza,
O quadro de Mondrian pode 1. Artes identificadas por fazendo com que o seu
constituir um contraexemplo à Aristóteles: poesia, comédia e auditório também vivencie uma
teoria expressivista, já que não música. experiência semelhante à sua.
exprime nem clarifica quaisquer Os diferentes tipos de arte Como é dito no texto: «Nesse
emoções. diferem quanto aos meios caso, o que distingue a atriz não
O quadro de Mondrian pode ser utilizados (as palavras, os é a sua habilidade para chorar
utilizado para ilustrar a teoria sons, os gestos, por exemplo), lágrimas verdadeiras, mas antes a
formalista, caso possua forma aos assuntos abordados sua capacidade para tornar claro,
significante e provoque emoção (incidem em coisas diferentes: a si e ao seu público, aquilo sobre
estética no espetador que o acontecimentos, ações humanas o qual chora.»
contempla. ou emoções, por exemplo) e 2. Porque o autor defende que a
4. No início do filme, os professores às diferentes maneiras de os arte deve ser uma expressão
avaliam de forma negativa a abordar. clarificadora das emoções. Deste
pintura de Frances, dizendo 2. O termo grego mímesis, embora modo, o objetivo do verdadeiro
que «não funciona», criticam seja geralmente traduzido por artista não é exteriorizar, de
aspetos relacionados com a imitação (que significa uma forma excessiva, as suas emoções
forma, a cor e a incapacidade representação o mais fiel possível para se sentir aliviado, mas sim
de esta exprimir ideias ou da realidade) deve ser entendido tentar identificá-las e entendê-
emoções. No final do filme, as num sentido mais abrangente, las, levando o seu auditório
apreciações que eles fazem das pois Aristóteles dá exemplos de a fazer o mesmo. A atividade
pinturas são opostas: elogiam a artes em que a representação artística envolve um processo
luz, a expressividade da figura não pode ser apenas imitativa, mental de identificação e
retratada e a capacidade de como a música. Assim sendo, compreensão das emoções do
suscitar certas emoções estéticas representar algo, numa atividade artista (e, através dele, do seu
no espetador e de o levar a artística, implica que um objeto público) e não a vivência intensa
experienciar emoções e clarificar (a obra) está em vez do original, ou a exteriorização exagerada
ideias enquanto as contempla. remete para este. Pode-se fazer de certas emoções. Quando é
As ideias expressas pelos isso de forma realista ou apenas apenas isto que acontece, não se
professores ao avaliarem as simbólica. Neste último caso, trata de arte genuína, mas mero
pinturas realizadas na Noruega não tem de ser semelhante ao entretenimento.
podem relacionar-se com as original. 3. Por exemplo, telenovelas e filmes
teorias seguintes (entre outras): 3. A arte é representação. de ação / aventuras. Os objetivos
• representacional, pois é Pode enquadrar-se na teoria são, entre outros, suscitar as
retratada uma figura feminina representacional. emoções do auditório.
de forma realista;
• formalista, já que as obras Texto 2 Texto 3
possuem forma significante e, 1. Na atividade teatral como ofício, 1. Problema central da estética:
por isso, suscitam claramente o objetivo dos atores, ao exporem encontrar uma propriedade ou

Editável e fotocopiável © Texto | Dúvida Metódica, 11.º ano 181


RESPOSTAS (MANUAL)

característica particular comum propriedade comum: a forma 2. É classificativa, porque descreve


às obras de arte e que permita significante. as propriedades relacionais ou
distingui-las de todos os objetos contextuais que certos objetos
que não são arte ou, dito por Texto 4 têm de possuir para serem obras
outras palavras, responder à 1. Em termos simples, o mundo de arte, independentemente
pergunta «O que é a arte?» da arte diz respeito às pessoas da avaliação que delas se faz.
2. Não. Cada obra produz em que de modos diferentes estão Portanto, o problema do valor
cada observador uma emoção interessadas na arte. O «mundo não é tematizado. Para o autor
diferente, isto é, cada pessoa da arte» engloba vários «sistemas do texto (tal como o defensor
sente a obra de uma maneira do mundo da arte», ou seja, em da teoria institucional do texto
pessoal. Todavia, ainda que cada uma das áreas da arte, 4), importa somente encontrar
sejam emoções particulares, por exemplo na pintura ou no as características distintivas que
dizem respeito a um mesmo teatro, existe uma organização fazem com que certos objetos
tipo de emoção: a emoção e existem atividades ou práticas sejam arte.
estética provocada pela forma estabelecidas com papéis 3. As definições de arte propostas
significante, presente em todos específicos para os artistas e por Levinson (no texto) e
os objetos que são obras de arte para o público, entre outros por Dickie são ambas não
(que as define como tal e lhes dá essencialistas, pois procuram
intervenientes. Estes «sistemas
valor). as condições necessárias e
do mundo da arte» «persistem ao
3. Segundo o autor do texto, a suficientes capazes de definir
longo do tempo», com alterações,
forma significante corresponde a arte que não se baseiam em
pois são «um empreendimento
às «linhas e cores combinadas características observáveis
cultural contínuo». Por isso,
de um modo particular, certas nas próprias obras, mas no
os pintores ou os atores, de
formas e relações de formas, contexto onde estas são criadas e
períodos históricos diferentes
que suscitam as nossas emoções apresentadas.
ou em contextos culturais
estéticas». As várias obras de Contudo, são propostas
distintos, seguem algumas regras
arte referidas no texto – apesar diferentes na medida em que
convencionais próprias da sua
de serem objetos com formas a de Dickie é institucional e
atividade artística, tal como o sublinha a importância do
físicas diferentes e representarem
público, por exemplo, age de mundo da arte (uma instituição
realidades distintas, pois todas
forma diferente numa galeria de social constituída por muitos
elas provocam emoções estéticas
pintura ou num teatro. indivíduos) para conferir a uma
nos observadores sensíveis e
2. As características que a obra de obra o estatuto de candidato
preparados – têm em comum
arte deve possuir são: à apreciação; enquanto a
o facto de terem uma forma
• ser um artefacto produzido por proposta de Levinson é histórico-
significante.
4. A circularidade relaciona-se com um artista; intencional e põe a tónica na
o facto de os termos «emoção • ser apresentado a um público intenção de um individuo (ou
estética» e «forma significante» do mundo da arte que tem indivíduos, no caso das obras
serem definidos um em função alguma preparação para coletivas) de que a sua obra seja
do outro, sem que se introduzam compreender o objeto, encarada similarmente às obras
quaisquer outras informações enquadrando-o num (ou mais do passado.
que permitam a sua clarificação. sistemas) do mundo da arte.
Elementos do texto: 3. Definir arte no sentido PÕE-TE À PROVA • págs. 222-225
Forma significante – «São, em classificativo significa identificar Grupo I
cada um dos casos, as linhas e o que faz de algo uma obra 1. B 5. D
cores combinadas de um modo de arte, independentemente 2. C 6. C
particular, certas formas e da avaliação que dela se faz. 3. B 7. B
relações de formas, que suscitam Para o autor interessa apenas 4. A 8. A
as nossas emoções estéticas.» estabelecer as características do
Emoção estética – «(…) a que pode ser considerado arte, Grupo II
existência de um tipo particular isto é, só a questão da definição 1.1 A pintura de Frith, segundo o
de emoção, provocada por de arte deve abordada, sendo autor do texto, não é uma obra
obras de arte visuais, emoção a questão do valor da arte um de arte porque não possui uma
causada por todos os géneros de problema distinto deste. forma significante – «linha e cor
arte visual (pinturas, esculturas, servem para relatar historietas,
edifícios, vasos, gravuras, Texto 5 sugerir ideias e mostrar os
têxteis, etc.)» que possuem uma 1. Teoria histórica. costumes e comportamentos

182 Editável e fotocopiável © Texto | Dúvida Metódica, 11.º ano


RESPOSTAS (MANUAL)

de uma época» – e não suscita Ninguém sabe ao certo o que males poderem conduzir a bens
emoção estética. é o mundo da arte, como é que maiores.
É uma «pintura descritiva». alguém faz parte dele, quem faz 2. O aluno poderá apresentar, por
Na sua obra, este pintor parte dele e quais são as suas exemplo, esboços do argumento
não procura despertar a regras e os seus procedimentos, cosmológico (Deus é necessário
sensibilidade estética do ao contrário do que acontece para explicar a existência do
espetador, pinta para informar noutras instituições, como as mundo, etc.) ou do argumento
acerca de comportamentos da universidades, por exemplo. teleológico (Deus é necessário
sua época. 4. Se o aluno defender que a teoria para explicar a complexidade
1.2 A tese defendida por Clive Bell: histórica é satisfatória, poderá, da natureza, etc.). Mas,
a arte é forma significante. nomeadamente, alegar que a naturalmente, poderá também
Elemento do texto onde está definição histórica e intencional referir outros argumentos,
presente, de modo implícito, de Levinson nomeadamente o chamado
essa tese: «As formas e as • consegue explicar porque é que argumento moral (Deus é
relações das formas não eram, chamamos arte a coisas tão necessário para descobrirmos as
para Frith, objetos de emoção diferentes; verdades morais, etc.).
estética, eram antes meios para • não é tão restritiva como as
sugerir emoção e transmitir teorias essencialistas nem tão
EXERCITA I • pág. 234
ideias.» vaga e arbitrária como a teoria
1.3 Não. Na perspetiva de Clive Bell, institucional. 1.1 D 1.3 B
a conceção expressivista não dá Se o aluno defender que a teoria 1.2 A
conta da verdadeira natureza histórica não é satisfatória, 2. Sugestões de resposta (estas
da arte. Para ele, o que importa poderá, nomeadamente, alegar ou outras com significado
é a forma significante provocar que a definição histórica e equivalente):
emoções estéticas no espetador. intencional de Levinson a) acreditar/ter fé
Tudo o resto – o conteúdo, a • não proporciona uma boa b) de Deus ou de deuses
existência ou não de expressão definição de arte; c) reverência/adoração
clarificadora de emoções do • não consegue resolver o d) que admite a existência de um
artista e do público, por exemplo problema da primeira obra de só Deus
– é irrelevante. arte; e) o cristianismo
2. Na representação imitativa • não justifica devidamente f) o islamismo (ou o judaísmo)
procura-se reproduzir, de forma supostas condições necessárias,
fiel, o original, imitando-o como o direito de propriedade. Canção Into My Arms, de Nick Cave
tal como ele é na realidade,
I don’t believe in an interventionist
enquanto na representação Capítulo 4
God
simbólica utiliza-se algo que Filosofia da religião – o problema But I know, darling, that you do
está em vez do original e remete da existência de Deus But if I did I would kneel down and ask
para este. Por exemplo: a pintura
Him
de Frith referida na questão 1.1
é uma representação imitativa; SITUAÇÃO INICIAL • pág. 227 Not to intervene when it came to you
o desenho de uma águia Sugestões de resposta (o aluno pode Not to touch a hair on your head
representa o S. L. Benfica, mas usar outras palavras) To leave you as you are
não é esse clube. 1. Se o aluno pensar que se deve And if He felt He had to direct you
3.1 Concordaria com a decisão, duvidar da existência de Deus, Then direct you into my arms
tomada pelo diretor, de expor poderá referir que existem muitos Into my arms, O Lord
a obra, já que se trata de um males no mundo (nomeadamente Into my arms, O Lord
artefacto produzido por um doenças como a poliomielite e a Into my arms, O Lord
artista para ser apresentado a morte de crianças inocentes) que Into my arms
um público do mundo da arte um Deus bom e todo-poderoso
(com alguma preparação para poderia impedir. Pelo que, se não And I don’t believe in the existence of
compreender o objeto em causa) impede, talvez não exista. angels
numa instituição adequada para Se o aluno pensar que não se But looking at you I wonder if that’s
esse efeito, uma galeria de arte deve duvidar da existência de true
moderna. Deus, poderá referir que talvez But if I did I would summon them
3.2 Uma objeção possível seria a Deus tenha razões para permitir together
seguinte: a noção de mundo os males que há no mundo, And ask them to watch over you
da arte é vaga e pouco clara. nomeadamente o facto de alguns To each burn a candle for you

Editável e fotocopiável © Texto | Dúvida Metódica, 11.º ano 183


RESPOSTAS (MANUAL)

To make bright and clear your path 3. 4. A objeção da ilha perfeita


And to walk, like Christ, in grace and A. V D. V apresentada por Gaunilo
love B. F E. F pretende mostrar que a aceitação
And guide you into my arms C. F da ideia contida no argumento
4. As coisas artificiais, criadas pelo ontológico tem implicações
Into my arms, O Lord
ser humano, são feitas para absurdas e, portanto, é ela
Into my arms, O Lord concretizar uma determinada mesma absurda.
Into my arms, O Lord finalidade, isto é, têm uma 5. O argumento ontológico baseia-
Into my arms função. Por exemplo, o garfo -se na definição de Deus como
And I believe in Love pode ser utilizado quando se o ser mais grandioso ou perfeito
come ou cozinha os alimentos. em que se pode pensar. Porém,
And I know that you do too
Segundo o argumento há a possibilidade de não existir
And I believe in some kind of path
teleológico, o mesmo acontece nenhum ser mais grandioso ou
That we can walk down, me and you
com as coisas do mundo natural perfeito em que se pode pensar,
So keep your candlew burning
– como as mãos, cuja finalidade tal como não existe um número
And make her journey bright and pure
é agarrar os objetos –, que que seja o maior de todos.
That she will keep returning
foram criadas com uma forma
Always and evermore adequada ao seu propósito. EXERCITA IV • pág. 256
Into my arms, O Lord 5. Não se deve ao acaso, mas sim
1. Exemplos do mal natural:
Into my arms, O Lord ao facto de o organismo humano
epidemias e terramotos.
Into my arms, O Lord ter sido concebido por um ser
Exemplos do mal moral:
Into my arms
inteligente – Deus –, que o criou
homicídios e pedofilia.
de modo que as suas partes
2. A bondade divina e a
funcionassem harmoniosamente
3.1 A conceção teísta de Deus. omnipotência.
e de forma a alcançar certas
3.2 Indicar uma destas 3. Uma teodiceia é uma resposta
finalidades como a gestação.
características: intervém no ao problema do mal, uma
6. A, D e F.
universo / guia as pessoas / é um explicação da razão ou razões
ser pessoal (a quem podemos que levam Deus a permitir a
EXERCITA III • pág. 249
dirigir pedidos). ocorrência deste.
1. O argumento ontológico é a
3.3 Rezar e acender velas. 4. Deus é perfeito e por isso criou o
priori porque as suas premissas
melhor mundo que era possível
podem justificar-se recorrendo
EXERCITA II • pág. 243 criar. Os males existentes têm
somente ao pensamento, sem
uma razão de ser: contribuem
1. Dados da experiência: precisarmos de informação
para um bem maior (mesmo que
• No universo existem inúmeras proveniente da experiência.
muitas vezes não saibamos qual
coisas (seres humanos, animais, 2.
é).
plantas, rochas, objetos A. perfeição/todas as qualidades;
5. O facto de Deus ter dotado os
fabricados pelos seres humanos, máximo
seres humanos de livre-arbítrio e
por exemplo). B. existência; superior; perfeito/
de estes, por vezes, escolherem
• As coisas do universo têm algo maior do que o qual nada
realizar ações erradas.
causas. pode ser pensado
6. O aluno poderá referir, por
• As coisas do universo não são C. exista na realidade e não
exemplo, que o mal existente
apenas no pensamento
causas de si mesmas. no mundo é excessivo e
3.1 A qualidade (ou o atributo) da
2. Se existisse uma regressão injustificado, pois é maior do que
existência é entendida como
infinita, recuar-se-ia num o necessário para promover o
uma qualidade entre outras.
processo sem fim de causas e desenvolvimento do caráter.
3.2 É deduzida da ideia de Deus.
efeitos, mas pode-se observar E, portanto, existe mal gratuito e
3.3. Não, pois a existência do
que tal não acontece na automóvel é contingente e por sem sentido.
natureza. Assim, segundo Tomás isso não pode ser inferida a 7.1 B 7.3 D
de Aquino, para o próprio priori da ideia que dele temos (a 7.2 C
processo causal existir, temos existência de Deus é necessária
de admitir a existência de uma e por isso pode ser inferida a EXERCITA V • pág. 261
causa primeira, pois é esta que priori da sua definição como o 1. Pascal pretende mostrar que
possibilita a existência das causas ser mais grandioso em que se há boas razões (práticas,
seguintes e das intermédias pode pensar). existenciais) para acreditar em

184 Editável e fotocopiável © Texto | Dúvida Metódica, 11.º ano


RESPOSTAS (MANUAL)

Deus: ganha-se mais em ter essa em Deus «mecanicamente, Situação 2


crença do que em não a ter. sem raciocinar», pois pode-se 1. Existem muitos males no mundo.
2. Aos ateus e aos agnósticos. justificar racionalmente essa Um exemplo possível é a
3. Para Pascal, não é possível provar crença recorrendo: perseguição, a tortura e a morte
racionalmente a existência de • ao argumento cosmológico: de cristãos no Japão do século
Deus, pois isso ultrapassa os para explicar a existência XVII. O problema do mal pode ser
limites do pensamento humano. do mundo precisamos de enunciado do seguinte modo:
A via de acesso a Deus é a fé e pressupor uma causa primeira Será que a existência de mal
não a razão. – que é Deus – para evitar uma no mundo nos dá indícios
No entanto, os seres humanos regressão infinita das causas); suficientemente fortes para não
podem socorrer-se do raciocínio • ou ao argumento teleológico: acreditar em Deus? Ou seja,
para compreender que é uma vez que na natureza se existe tanto mal, não será
preferível aceitar a crença em existem inúmeros exemplos de improvável existir um Deus
Deus do que rejeitá-la, já que as desígnio em coisas destituídas omnipotente, omnisciente e
vantagens de o fazer, do ponto de de inteligência, é preciso admitir sumamente bom?
vista prático, são superiores. a existência de um criador 2. Uma resposta possível é dizer
4. À utilidade, porque se baseia inteligente – que é Deus – que que Deus permite o mal para
numa ponderação dos ganhos lhes conferiu esse desígnio; que os seres humanos tenham
e das perdas, defendendo que a • ou ainda ao argumento livre-arbítrio. O martírio dos
escolha deve recair na opção que ontológico: uma vez que a ideia cristãos japoneses, por exemplo,
produz mais benefícios: apostar que temos de Deus é a ideia do resulta das escolhas efetuadas
na existência de Deus. por alguns dirigentes políticos
ser mais grandioso que se pode
5. As pessoas não podem decidir japoneses da época.
conceber, é preciso admitir que
ter fé, já que esta não depende Outra resposta consiste em
ele existe pois, se não existisse,
inteiramente de fatores defender que Deus permite que
poder-se-ia imaginar um ser
conscientes ou voluntários, como existam males pois sem estes não
ainda mais grandioso, o que
a repetição de certas ações. haveria oportunidade para os
seria contraditório.
seres humanos desenvolverem
6. Talvez que se tratava de alguém 3. Aos argumentos cosmológico
virtudes importantes como,
interesseiro, pouco sincero, sem e teleológico poderia, por
por exemplo, a coragem
uma fé genuína que fosse digna exemplo, objetar que, mesmo
e a esperança. No caso da
de apreço. admitindo hipoteticamente que
perseguição aos cristãos
provam o que querem provar (a
japoneses, estes tiveram de fazer
DISCUTE • págs. 264-267 existência de uma causa primeira
escolhas morais fundamentais,
Proposta de alguns tópicos e de um criador inteligente,
o que levou alguns a agir de
orientadores do debate respetivamente), não precisamos modo heroico, sendo ainda hoje
de admitir a existência do Deus recordados pela sua coragem e
Situação 1 teísta (pois a causa primeira e o fidelidade aos seus princípios.
1. A relação é a seguinte: Pascal criador inteligente poderiam ser Uma outra resposta sublinha
considerava que uma pessoa uma entidade sem os predicados que os males que ocorrem são
que «aposte» na existência de teístas). Ao argumento ontológico compensados por bens maiores,
Deus deve tentar viver como os poderia, nomeadamente, objetar embora nem sempre consigamos
crentes e realizar as ações que que a definição de Deus como perceber quais são estes, devido
estes realizam (rezar, praticar o ser mais grandioso que se às nossas limitações cognitivas.
a caridade, etc.); ora, o stárets pode conceber é incorreta, pois 3. Relativamente à resposta
aconselha a senhora a viver podemos sempre imaginar um baseada no livre-arbítrio, um
de um modo que podemos ser ainda mais grandioso, não crítico do teísmo pode alegar que
chamar cristão (na Bíblia pede-se havendo nenhuma grandeza esta incide apenas no mal moral
para amar o próximo): «Tente máxima concebível). e não explica o mal natural.
amar o seu próximo, ativa e 4. O aluno poderá defender várias Um crítico do teísmo pode alegar,
incansavelmente. À medida perspetivas: o teísmo (recorrendo contra a resposta baseada na
que fizer progressos no amor, aos argumentos estudados); ideia de que o mal contribui para
ir-se-á convencendo também o deísmo; o agnosticismo ou a edificação do caráter, que os
da existência de Deus e da o ateísmo (tendo em conta o males existentes no mundo são
imortalidade da alma». problema do mal ou as objeções excessivos e que se conseguia
2. Um teísta poderia tentar mostrar aos argumentos a favor da esse efeito com um menor
que não é preciso acreditar existência de Deus). número de males.

Editável e fotocopiável © Texto | Dúvida Metódica, 11.º ano 185


RESPOSTAS (MANUAL)

Quanto à resposta que considera 2. Um tsunâmi na Ásia e a dor de filosófica desses argumentos
que os males são compensados costas de Franklin Ribeiro. não incide geralmente na
por bens maiores, um crítico do 3. A incorreção reside no facto de validade ou invalidade, mas sim
teísmo, mesmo reconhecendo «Deus», após ter sido criticado na verdade ou falsidade das
que temos limitações cognitivas, por Franklin Ribeiro por ter premissas. Portanto, Russel devia
pode alegar que dificilmente permitido um tsunâmi na Ásia, se ter dito que, na sua opinião, esses
certos males podem ser ter defendido referindo o livre- argumentos não são sólidos.
compensados por coisas boas arbítrio. Ora, a defesa do livre-
que ocorram (como talvez seja arbítrio incide apenas no mal IV – Filme: «Uma Comédia
o caso do sofrimento causado a moral, deixando de fora males Intergaláctica»
bebés por certas doenças). naturais como os tsunâmis. 1. Característica: a omnipotência.
4. Se o aluno considerar que a Os extraterrestres resolvem
posição teísta é mais plausível, III – Porque não é crente? atribuir esse poder a Neil pelas
poderá justificá-la com as 1. Russel defende que é desonesto seguintes razões: foi aprovada,
respostas teístas estudadas (ou aceitar uma crença porque é pelo Conselho Intergaláctico,
outras). Se o aluno considerar útil, e não porque é verdadeira. a destruição do planeta Terra.
que a posição teísta não é mais Por isso, considera errado Porém, antes de esta decisão ser
plausível, poderá defender a concretizada, os extraterrestres
aceitar as crenças religiosas por
sua perspetiva com as objeções
razões práticas, nomeadamente irão realizar ainda um teste. Este
estudadas (ou outras).
por permitirem obter certos consiste em escolher, ao acaso,
benefícios. Ora, Pascal alega um terráqueo (que foi Neil),
VISIONA • págs. 268-269
precisamente que a «aposta» dar-lhe superpoderes e verificar
I – O argumento ontológico na existência de Deus é mais como ele os vai usar. Se ele fizer
1. Com que argumentos se pode compensadora, em termos o Bem, a Terra será poupada.
defender a existência de Deus? práticos e existenciais, do que a Se ele fizer o Mal, a Terra será
2. Anselmo recorre a um argumento «aposta» na sua inexistência. destruída.
dedutivo. Russel refere também, embora 2. Primeiro, ele utiliza os seus
3. A consequência é que de modo muito breve, que nunca poderes para realizar os seus
Deus, se é uma ideia que
decidiu deixar de ser crente: desejos pessoais e dos amigos.
existe no pensamento, tem
não foi uma decisão, mas sim Depois, para corrigir os males
necessariamente de existir
uma descoberta (depois de que atingem a humanidade em
também na realidade. Caso
analisar os dogmas cristãos geral.
não fosse assim, Deus não seria
e de ter concluído que não O aluno deve expressar a sua
perfeito.
tinham fundamento). Tal pode posição pessoal quanto ao
4. Crítica de Gaunilo ao argumento
ser relacionado com uma das modo como usaria poderes
ontológico: se seguirmos a
objeções à «aposta de Pascal»: sobre-humanos, fundamentando
mesma linha de raciocínio
as crenças não são algo que de forma adequada a posição
que Anselmo, usando um
dependa da nossa vontade e, defendida.
argumento com a mesma
portanto, não podemos decidir 3. As objeções ao argumento
estrutura, poderemos concluir
que existe uma ilha perfeita acreditar em algo (mas sim cosmológico e ao argumento
(ou qualquer outra coisa que descobrir que acreditamos). teleológico. No primeiro
eu possa imaginar). Se a ideia 2. A afirmação não é rigorosa, na argumento, mesmo admitindo
de uma ilha perfeita existe no medida em que os argumentos a a necessidade de uma causa
meu pensamento, também teria favor da existência de Deus (ou, primeira, não se prova que
de existir na realidade, caso pelo menos, algumas das suas esta tenha de ser o Deus teísta:
contrário já não seria a ilha mais versões) podem ser formulados poderia ser uma entidade
perfeita que se pode pensar. através de formas de inferência poderosa, mas com outras
Resposta de Anselmo a essa reconhecidamente válidas, como características, por exemplo,
objeção: o argumento ontológico por exemplo o modus ponens seres extraterrestres como os
aplica-se apenas a seres e o modus tollens. Mesmo um retratados no filme. O mesmo
necessários e só há um: Deus. ateu pode reconhecer isso. acontece no segundo argumento,
Contudo, tal não implica que supondo que se prova a
II – O homem que foi visitado por esses argumentos sejam sólidos necessidade de um criador
Deus (um argumento para ser sólido inteligente, a quem se deve o
1. De acordo com o teísmo, Deus é tem de ser válido e ter premissas desígnio e a harmonia de todas
incorpóreo e transcendente. verdadeiras). A discussão as coisas existentes no universo,

186 Editável e fotocopiável © Texto | Dúvida Metódica, 11.º ano


RESPOSTAS (MANUAL)

esse ser pode não ser o Deus Textos 2 e 3 superficiais e pouco significativas)
teísta, mas, por exemplo, seres 1. Uma maneira possível de explicar e sem estas não haveria
extraterrestres como os do filme. o argumento de Paley: «crescimento moral e espiritual»
4. Ao tentar corrigir alguns dos Objetos como os relógios e (sem a necessidade de escolher
males existentes no mundo os telemóveis são demasiado entre bens e males significativos
(como, por exemplo: as pessoas complexos para terem surgido e sem a necessidade de enfrentar
deixarem de morrer), Neil verifica por acaso, pois são constituídos obstáculos, as pessoas não teriam
que as consequências das por diversas partes interligadas e oportunidade de desenvolver
suas correções são ainda mais que trabalham articuladamente qualidades como a coragem e a
desastrosas do que os males já para um mesmo desígnio (ou tenacidade).
existentes e, por isso, volta atrás, finalidade). Parece óbvio que 2. Hick responde à crítica de que
repondo a realidade como era tiveram de ser criadas por o sofrimento é excessivo e
anteriormente. alguém, por um ser consciente e arbitrário alegando que, caso
Estas passagens do filme inteligente. Coisas da natureza o sofrimento fosse menor e
podem-se relacionar com a como os olhos e o cérebro são previsível, seria entendido
teodiceia de Leibniz, porque nos também muito complexas e como um castigo merecido
mostram como a existência de constituídas por diversas partes para os sofredores e levaria
certos males é necessária para interligadas e que parecem as pessoas a agir bem por
um bem maior. E, portanto, se trabalhar para um mesmo fim. medo das consequências – e
tentássemos corrigir ou eliminar Parece também improvável que isso não constituiria, portanto,
tenham surgido por acaso. Se uma oportunidade para o
determinados males, o resultado
reconhecemos a necessidade «crescimento moral e espiritual».
final poderia ser pior.
5. O aluno poderá defender, de um criador inteligente
para os relógios e telemóveis, PÕE-TE À PROVA • págs. 275-277
nomeadamente, que Niels
prefere viver com incerteza também devemos reconhecer Grupo I
e outras limitações próprias a necessidade de um criador 1. B 5. B
dos seres humanos a ter inteligente para os olhos e para 2. D 6. D
o cérebro (e para a natureza 3. A 7. C
superpoderes. Só assim ele
em geral). Quem pode ser esse 4. C 8. A
poderá estabelecer relações
criador? Deus. Logo, Deus existe.
autênticas com as pessoas de
2. Para Darwin não existe desígnio Grupo II
quem gosta; tendo poderes
na natureza: «a seleção natural 1.1 Mal natural.
sobre-humanos haverá sempre
explica a complexidade dos 1.2 Omnipotência e suprema
a possibilidade de as pessoas
organismos vivos sem recorrer bondade
pensarem que ele as está a
ao propósito ou ao desígnio», o 1.3 Não se atribui livre-arbítrio
manipular, obrigando-as a
«desígnio da natureza é, portanto, aos corços (ao contrário dos
obedecer aos seus desejos.
um desígnio aparente e não seres humanos), pelo que não
real». A consequência dessa ideia podemos imputar o terrível
ANALISA TEXTOS • págs. 270-273
para a questão da existência de sofrimento do corço a um mau
Texto 1 Deus é que enfraquece bastante uso do livre-arbítrio.
1. Não se deve confundir a filosofia o argumento teleológico, 2.
da religião com a história da pois o facto de se conseguir Todos os «seres sensíveis» têm
religião nem com a teologia. essa explicação através de causas.
Deve também distinguir-se da «um processo natural cego» Nenhum «ser sensível» é causa de
antropologia, da sociologia e da parece tornar desnecessária si próprio.
psicologia, que também podem a intervenção de um criador A cadeia de causas dos «seres
estudar a religião. inteligente, isto é, de um deus. sensíveis» não pode regredir até
2. A filosofia da religião estuda ao infinito.
a justificação das crenças Texto 4 Logo, tem de existir uma primeira
religiosas, ou seja, discute se as 1. Segundo Hick, Deus permite que causa – que é Deus.
razões a favor e contra as crenças as pessoas possam fazer mal 3. Podemos inferir a partir da
religiosas são ou não plausíveis. umas às outras, pois de outra definição de Deus como o ser
A filosofia da religião é exterior maneira não haveria «escolhas mais grandioso em que se pode
à religião, pois não implica que morais importantes» (ou seja, pensar.
as pessoas que a ela se dedicam se não houvesse sofrimento 4.1 A teoria da evolução das
sejam religiosas. envolvido, as escolhas seriam espécies, de Charles Darwin,

Editável e fotocopiável © Texto | Dúvida Metódica, 11.º ano 187


RESPOSTAS (MANUAL)

enfraquece o argumento estatuto moral, são meios e essas experiências já que, de


teleológico, porque permite não fins em si mesmos. E, por acordo com o princípio da maior
compreender a existência e isso, ao contrário dos seres felicidade, o mais importante
o desenvolvimento dos seres humanos, podem ser utilizados seria o bem-estar geral produzido
vivos (e o seu funcionamento na investigação científica; na maioria das pessoas afetadas
ajustado) sem recorrer a um • Carl Cohen: apesar de os pela ação (seria possível salvar
criador inteligente, isto é, a seres humanos terem muitas milhares de pessoas).
Deus. Esta teoria explica a obrigações para com os Se o aluno concordar com a
complexidade dos seres vivos e animais, estes não têm resposta de Peter Singer, poderá
dos seus órgãos através de um direitos. Quem tem direitos são argumentar a partir das ideias da
mecanismo cego – a seleção somente os seres humanos, ética utilitarista.
natural e a sobrevivência dos enquanto agentes morais. Se o aluno discordar da
seres mais adaptados ao meio Portanto, é legítimo utilizar resposta de Peter Singer, poderá
– uma causalidade natural e os animais para alguns fins argumentar a partir das ideias da
impessoal. com interesse para os seres ética deontológica de Kant.
4.2 A resposta teísta é que Deus humanos, nomeadamente em
criou o próprio mecanismo da experiências científicas úteis e 3. Se admitirmos a tese de Tom
evolução. relevantes. Regan, segundo a qual os
5. Na resposta de opinião, o aluno Na resposta a esta questão, animais têm direitos absolutos (à
deverá: se o aluno considerar a
semelhança dos seres humanos,
• Enunciar, de forma clara e experimentação animal como
são fins em si e não devem ser
inequívoca, a sua posição crueldade, poderá defender
tratados como meros meios),
pessoal face ao problema da argumentos baseados nas ideias
então é errado causar-lhes dor e
existência de Deus. de:
instrumentalizá-los.
• Identificar e explicar pelo • Bentham ou de Peter Singer:
Umas das implicações práticas da
menos um argumento a favor da é preciso reconhecer estatuto
aceitação desta tese seria que as
posição defendida. moral aos animais não-
experiências, utilizando animais
• Explicar uma objeção a esse -humanos, devido ao facto
(seja qual for a sua finalidade e
argumento, respondendo à de serem sencientes (terem
mesmo que visem salvar vidas
objeção. a capacidade de sentir dor
humanas), são moralmente
• Confrontar diferentes ideias ou prazer). Não ter em conta
erradas e deveriam ser proibidas.
estudadas. os interesses dos animais
Carl Cohen não concordaria com
• Utilizar corretamente a constitui uma manifestação
de «especismo» (uma a proibição de toda e qualquer
terminologia filosófica.
discriminação injustificada com experimentação animal, pois
base na espécie e equivalente considera que os animais não têm
Capítulo 5
ao racismo); direitos, pois estes dizem respeito
A legitimidade da apenas aos seres humanos. Por
• Tom Regan: tal como os seres
experimentação animal isso, não é moralmente errado
humanos, animais como os
mamíferos e as aves são usar os animais não-humanos
DISCUTE • pág. 295 sujeitos-de-uma-vida, ou seja, em experiências que beneficiem
Proposta de alguns tópicos estão conscientes do mundo e os seres humanos em aspetos
orientadores do debate podem sentir dor. São alguém e importantes, como a saúde, por
não apenas algo. Por isso, têm exemplo.
1. direito a serem tratados com O aluno poderá concordar com
1.1 Na resposta a esta questão, respeito e é errado causar-lhes o ponto de vista de Tom Regan,
se o aluno considerar a dor. Sendo assim, não devemos argumentando a favor das ideias
experimentação animal como usá-los em experiências deste filósofo. Ou, em alternativa,
um mal necessário, poderá científicas. discordar da posição de Tom
defender argumentos baseados 2. A resposta utilitarista de Regan, recorrendo a argumentos
nas ideias de: Peter Singer é afirmativa: os baseados nas ideias de Carl
• Kant: os animais não têm cientistas deveriam realizar Cohen ou de Kant.

188 Editável e fotocopiável © Texto | Dúvida Metódica, 11.º ano


Outros
Tema/problema Eutanásia

OUTROS

• Notas explicativas (remissões do manual)


• Tipologia de questões
• Relembra Ideias-chave de 10.º ano
• Outra Maneira de Preparar os Exames: um exemplo
• Prova com a estrutura do Exame Nacional de
Filosofia
• Avaliação por competências: exemplos
• Temas/problemas alternativos:
Guerra justa e Tecnociência (em )
• Planificações (em )
• Bibliografia e Webgrafia

© Texto | Dúvida Metódica, 11.º ano


Notas explicativas

Notas explicativas

Capítulo 1 Pág. 35

Pág. 11 Por outras palavras:

Essas formas de conhecimento são indepen- «Olhando para o seu próprio “eu”, que é tudo
dentes e em muitos casos ocorrem separada- o que lhe resta neste momento, Descartes
mente. Mas há também casos em que podem descobre que tem uma ideia de perfei-
relacionar-se umas com as outras de diversos ção. Argumenta, então, que uma tal ideia
modos. Vejamos três maneiras dessa relação implica uma causa. No entanto, a coisa que
ocorrer. originou essa ideia deve possuir tanta “rea-
lidade” como ela, e isso inclui a perfeição.
1. Existem problemas cuja resolução implica
Isto implica que só uma causa perfeita, isto
o recurso aos vários tipos de conhecimento
é, Deus, pode servir. Por isso, Deus existe e
(por exemplo, uma cirurgia).
legou-nos a ideia de perfeição como um sinal
2. O saber-fazer e o conhecimento por con- inato da sua ação nas nossas mentes, assim
tacto podem muitas vezes dar origem a como um artesão deixa a sua marca gravada
conhecimentos proposicionais, nomeada- no seu trabalho.»
mente quando as pessoas tentam descrever
Simon Blackburn, Pense – Uma Introdução à Filosofia,
o que sabem fazer e explicam verbalmente Gradiva, Lisboa, 2001, p. 42.
como se faz ou quando tentam descre-
ver algo que aprenderam por experiência,
quando tentam pôr em palavras algo conhe-
cido através do contacto sensorial. Capítulo 2

3. Podemos ter um conhecimento mera- Pág. 92


mente proposicional relativamente a certas Albert Einstein escreveu uma vez sobre
coisas, mas depois desenvolver em relação a Galileu que «ele é o pai da Física moderna
elas conhecimentos práticos e por contacto, – aliás, de toda a ciência moderna». As suas
numa tentativa de aplicar e concretizar esse palavras faziam eco das do filósofo Bertrand
conhecimento proposicional. Russell, que também considerava Galileu «o
maior dos fundadores da ciência moderna».
Einstein acrescentou que «a descoberta e a
Pág. 22 utilização por Galileu do raciocínio cientí-
Para Descartes, só as crenças de cuja verdade fico» foi «uma das realizações mais impor-
não é possível duvidar são conhecimento. As tantes da história do pensamento humano».
crenças apenas prováveis não são conheci- (…) Por meio da sua obstinada e pioneira
mento (tal como, obviamente, as falsas). Por insistência em que o livro da natureza estava
isso, a sua epistemologia pode ser conside- «escrito na linguagem da Matemática» e a
rada infalibilista. O infalibilismo é uma pers- sua fusão bem-sucedida da experimentação,
petiva epistemológica de acordo com a qual da idealização […] e da quantificação, Galileu
uma crença só é conhecimento se tiver uma [transformou] uma simples coleção de rela-
justificação infalível. tos vagos, verbais e nublosos numa magnífica
obra que incluía (sempre que o conheci-
mento contemporâneo o permitia) teorias

© Texto | Dúvida Metódica, 11.º ano 191


Notas explicativas

matemáticas rigorosas. No âmbito dessas dos cientistas, imagina-se a existência de cons-


teorias, as observações, as experiências e o pirações para as eliminar.»
raciocínio tornaram-se os únicos métodos Carl Sagan, Um Mundo Infestado de Demónios,
aceitáveis de descobrir factos sobre o mundo Gradiva, Lisboa, 1997, pp. 29 e 36 (adaptado)
e de investigar novas relações na natureza.
Como Max Born, vencedor do prémio Nobel
da Física em 1954, disse: «A atitude científica Pág. 109
e os métodos de investigação experimental A noção de teoria científica e exemplos de
e teórica têm sido os mesmos ao longo de afirmações empíricas e de princípios gerais.
séculos, desde Galileu, e continuarão a ser.»
«(…) Qualquer teoria científica implica uma
Mario Livio, Galileu e a Busca da Verdade,
Edições Marcador, Lisboa, 2021, pp. 16-17
série de afirmações, que variam entre afirma-
ções empíricas sobre fenómenos particulares
observáveis (quer a olho nu, quer mediante
Pág. 106 instrumentos científicos), valores que pode-
mos medir e princípios gerais. Eis um exem-
«É natural que as pessoas experimentem
plo de uma afirmação empírica sobre um
vários sistemas de crença para verem o que
acontecimento particular em física: “o corpo
mais convém para as ajudar. E, se estivermos
cai à velocidade de 40 km/h”. Um exemplo
muito desesperados, dispomo-nos a aban-
de um princípio geral é o princípio da inércia,
donar o que pode ser considerado o fardo
segundo o qual um corpo conservará uma
pesado do ceticismo. A pseudociência dirige-
velocidade constante, a menos que sobre ele
-se a necessidades emocionais fortíssimas que
atue uma força cuja resultante não seja nula.
a ciência muitas vezes deixa sem resposta.
Fornece fantasias sobre poderes pessoais O que esperamos de uma teoria científica
que não temos e que desejaríamos possuir. num domínio específico é uma explicação
Em algumas das suas manifestações oferece coerente e sistemática da razão por que
a satisfação da fome espiritual, curas para alguns factos ocorrem como ocorrem, e uma
doenças, promessas de que a morte não é o maneira fundamentada de prever os factos
fim. Tranquiliza-nos, garantindo que ocupa- que ocorrerão no futuro.»
mos um lugar central e importante no cosmo.
Lisa Bortolotti, Introdução à Filosofia da Ciência,
Edições Gradiva, Lisboa,2013, pp. 107-108
A pseudociência difere da ciência. A ciência ali-
menta-se de erros que vai eliminando um por
um. Formula hipóteses para poder refutá-las.
Pág. 112
Submete uma sucessão de hipóteses alterna-
tivas à experiência e à observação. A ciência «O exemplo de Neptuno ilumina por que
avança às apalpadelas e em passos hesitantes razão temos a hipótese da “matéria negra”. A
em direção a um conhecimento mais perfeito. velocidade de rotação das estrelas nos braços
A pseudociência é precisamente o oposto. As exteriores de uma galáxia em disco deveria
hipóteses são muitas vezes formuladas sem ser maior do que a das estrelas mais próximas
exatidão para serem invulneráveis perante do centro galáctico, mas a observação – feita
qualquer experimentação suscetível de as pela astrónoma Vera Rubin – mostrou que não
refutar, pelo que não podem ser invalidadas. há grande diferença. Combine-se isto com o
Os seus praticantes jogam à defesa e são cau- valor conhecido da gravidade, e a conclusão é
telosos (…). Quando as hipóteses pseudocientí- que, tanto quanto parece, há uma vasta quan-
ficas não conseguem despertar o entusiasmo tidade de massa por detetar na galáxia visível,

192 © Texto | Dúvida Metódica, 11.º ano


Notas explicativas

e à sua volta. Outras observações – por exem- menores deu azo a muitos dos principais
plo, agregados de galáxias que deveriam afas- problemas que ocuparam a investigação
tar-se, a menos que algo as mantenha juntas, e astronómica normal de muitos sucessores de
os efeitos de lente gravitacional, em que a luz Ptolomeu (…). Dada uma certa discrepância,
vinda de mais longe do que uma dada galá- os astrónomos eram inevitavelmente leva-
xia se curva num grau maior do que aquele dos a eliminá-la, fazendo um determinado
que explicaria com base na sua massa visível ajustamento do sistema dos círculos com-
– fazem pesar a sugestão de que uma imensi- postos de Ptolomeu. Mas à medida que o
dade de 95 por cento da matéria do universo é tempo passava, um homem que olhasse para
energia e matéria negra, uma substância que o resultado líquido do esforço de investiga-
não interage com o campo eletromagnético e, ção normal de muitos astrónomos poderia
portanto, não pode ser vista nem detetada de observar que a complexidade da astronomia
qualquer outra maneira.» estava a aumentar mais rapidamente do que
A. C. Grayling, As Fronteiras do Conhecimento, a sua exatidão e que uma discrepância corri-
O que Sabemos Hoje Sobre Ciência, História da Mente, gida num ponto reaparecia, quase de certeza,
Edições 70, Lisboa, 2021, p. 143
num outro. (…) O próprio Copérnico escreveu
Pág. 126 que a tradição astronómica que ele herdou
não tinha criado senão um monstro. Pelos
«O realismo filosófico é um ponto de vista
começos do século XVI, um número cada vez
sobre o que existe e o que podemos conhe-
maior dos melhores astrónomos europeus
cer, e, por conseguinte, tem uma componente
reconhecia que o paradigma astronómico
ontológica e uma componente epistemoló-
falhava na aplicação aos seus problemas
gica. De acordo com a tese ontológica de um
tradicionais. Este reconhecimento foi um
realista representativo, há objetos e proprie-
pré-requisito para Copérnico ter rejeitado o
dades no mundo independentes da mente,
paradigma ptolemaico e ter procurado um
objetos e propriedades que estariam no
novo.»
mundo mesmo que não houvesse uma mente
para pensar neles. De acordo com a tese epis- Thomas S. Kuhn, A Estrutura das Revoluções Científicas,
Editora Guerra & Paz, Lisboa, 2009, pp 120-21
temológica, podemos ter acesso a esses obje-
tos e propriedades, e as representações que
deles fazemos, ainda que falíveis, não são sis- Movimento de precessão

tematicamente enganadoras.»
Eixo de
Lisa Bortolotti, Introdução à Filosofia da Ciência, rotação
Gradiva, Lisboa, 2013, p. 181 Direção da
23,5 o transição

Pág. 135
«Quer no caso das posições planetárias, quer Eixo vertical Movimento
da eclítica de rotação
no da precessão dos equinócios, as predi-
ções feitas com base no sistema de Ptolomeu A precessão dos equinócios é um dos vários
movimentos realizados pela Terra e corresponde
nunca conseguiram estar em conformidade ao deslocamento circular efetuado pelo planeta
com as melhores observações disponíveis. em torno do eixo de sua eclíptica. Por analogia,
compara-se esse movimento ao de um pião
A redução ulterior dessas discrepâncias desequilibrado ou levemente inclinado.

© Texto | Dúvida Metódica, 11.º ano 193


Notas explicativas

Capítulo 3 «Claro que a teoria da evolução de Darwin


não refuta de forma alguma a existência de
Pág. 194
Deus – na verdade, muitos cristãos aceitam-
Uma objeção que se pode fazer à posição -na como a melhor explicação de como as
de Weitz é que a liberdade e a criatividade plantas, os animais e os seres humanos se tor-
artísticas que, segundo ele, explicam o cará- naram no que são hoje: eles acreditam que
ter aberto do conceito de arte podem ser vis- Deus criou o próprio mecanismo da evolução.
tas como características definidoras da arte. Contudo, a teoria de Darwin enfraquece, de
Outra objeção, desenvolvida pelas teorias facto, a força do argumento do desígnio, uma
não-essencialistas, é que as características vez que explica os mesmos efeitos sem men-
definidoras da arte podem existir, mas não cionar Deus como causa. A existência desta
onde Weitz as procurou (isto é, podem existir, teoria acerca do mecanismo da adaptação
mas não ser intrínsecas às obras). biológica impede o argumento do desígnio
de constituir uma demonstração conclusiva
da existência de Deus.»
Nigel Warburton, Elementos Básicos de Filosofia,
Capítulo 4 Gradiva, Lisboa, 2007, p. 33
Pág. 230
Esta perspetiva sobre a fé é a chamada pers- Pág. 251
petiva tradicional, que é defendida por inú-
meros filósofos (Agostinho, Tomás de Aquino, Um outro argumento contra a existência de
Locke, Plantinga, Swinburne, etc.): «Nesta Deus é o chamado argumento da ocultação
perspetiva a fé em Deus consiste em ter um divina, defendido nomeadamente por J. L.
estado doxástico de crença que Deus existe, Schellenberg. Este sustenta o seguinte:
bem como em ter outras atitudes não-do- «Um Deus bom e amoroso, como se pretende
xásticas, tal como a confiança em Deus ou que seja o Deus teísta, não se ocultaria ao
comprometimento com Deus. Assim, (…) a ponto de pessoas razoáveis, sinceras e bem-
fé envolve tanto a crença num determinado -intencionadas não acreditarem nele. Ora, é
conteúdo proposicional como um compro- um facto que existem pessoas razoáveis, sin-
misso de viver à luz disso.» ceras e bem-intencionadas que não acredi-
tam em Deus. Por isso, pode concluir-se que
Contudo, para outros autores (como Daniel
Deus não existe.»
McKaughan e Louis Pojman), «a fé não
envolve a crença, mas antes atitudes de con-
fiança ou de grande esperança em Deus». Pode distinguir-se entre o problema lógico
Domingos Faria, «O Que É a Fé Em Deus? Uma Defesa da do mal e o problema indiciário do mal. O
Perspectiva Tradicional», Revista Brasileira de Filosofia da
problema lógico do mal pode formular-se do
Religião, 3(2), 2018, pp. 150-159 (Recuperado de
https://periodicos.unb.br/index.php/rbfr/article/ seguinte modo: Será possível compatibilizar
view/17368 e consultado em 02/04/2022) logicamente os inúmeros males existentes
no nosso mundo com a existência de Deus?
(Duas proposições são incompatíveis se não é
Pág. 242
possível serem ambas verdadeiras – podendo
O texto de Warburton continua do seguinte uma ser verdadeira e a outra falsa ou ambas
modo, explicando uma ideia que por vezes é falsas.) A discussão do problema lógico do mal
abordada nas aulas: consiste em apurar se é ou não compatível

194 Editável e fotocopiável © Texto | Dúvida Metódica, 11.º ano


Notas explicativas

admitir a existência do Deus teísta (omni- prova conclusiva, torna essa existência pouco
potente, sumamente bom, etc.) e admitir a provável, ou seja: dá-nos razões para preferir
existência de mal. Alguns filósofos tentaram o ateísmo ao teísmo. Vamos estudar unica-
mostrar que essa incompatibilidade existe e mente o problema indiciário.
outros esforçaram-se por mostrar que não
existe, na medida em que Deus pode ter boas
razões para permitir o mal. Contudo, mesmo Pág. 255
que a existência de Deus e de mal sejam
Esta resposta de Leibniz tem eco numa res-
logicamente compatíveis, isso não resolve o
posta mais contemporânea conhecida como
problema do mal nem encerra a discussão,
teísmo cético (defendida, por exemplo, por
pois permanece em aberto o problema do
Stephen Wykstra), segundo a qual mesmo
mal na chamada forma indiciária (por vezes
que pareça haver mal gratuito, não podemos
também designada probatória ou probabilís-
tica). Mesmo que não haja incompatibilidade garantir que este seja realmente gratuito,
lógica entre a ideia de que Deus existe e a pois as nossas limitações cognitivas impe-
ideia de que há mal no mundo, parece haver dem-nos de compreender adequadamente
pelo menos uma certa tensão. O problema as motivações de Deus. Deus pode ter razões
indiciário do mal consiste na afirmação de para permitir um determinado mal que nos
que existem indícios que tornam implausível parece gratuito e nós desconhecermos essas
a existência de Deus ou, mais exatamente, do razões. O facto de não conseguirmos imagi-
Deus teísta. Mesmo não se tratando de uma ná-las não significa que não existam.

Editável e fotocopiável © Texto | Dúvida Metódica, 11.º ano 195


Tipologia de questões

Tipologia de questões

Questões de seleção

Questões de Questões de Questões de


Questões de associação
escolha múltipla verdadeiro / falso completamento

O aluno deve selecionar A resposta implica A resposta envolve A resposta implica o


a resposta correta numa categorizar como o preenchimento de estabelecimento de
lista de alternativas verdadeira ou falsa espaços em branco uma correspondência
em que algumas são cada uma das frases numa frase ou texto, entre os elementos
erradas. O mais habitual de um dado conjunto. mediante a seleção de de dois conjuntos,
é haver uma única A classificação da entre um conjunto de de acordo com as
resposta certa, mas resposta costuma ser elementos fornecido. instruções dadas. O
é possível construir dicotómica (certa ou A classificação da número de elementos
questões com mais do errada, sem níveis resposta é geralmente dos dois conjuntos pode
que uma resposta certa. intermédios). dicotómica (certa ou ser diferente, para que
errada, sem níveis a última resposta não
Por exemplo: Por exemplo: intermédios). possa ser dada por
exclusão das outras.
Qual é o problema que Indica se as seguintes Por exemplo: A classificação da
é filosófico? afirmações são resposta é geralmente
verdadeiras ou falsas. Completa a frase dicotómica (certa ou
A. Há algum país do apresentada com uma errada, sem níveis
mundo em que A. O determinismo das seguintes palavras: intermédios).
seja proibido não moderado é universais, pessoais,
acreditar em Deus? uma teoria individuais. Por exemplo:
B. Acreditar em Deus incompatibilista.
ajuda as pessoas a B. De acordo com o O subjetivismo defende Relaciona
viver melhor? libertismo, todas as que não há verdades adequadamente os
C. Será que Deus existe? ações humanas são morais___________. elementos do conjunto
D. Como se escreve livres. 1 e os elementos do
ͪDeusͫ em chinês? C. O determinismo conjunto 2.
radical considera que
os nossos desejos Conjunto 1
são causalmente A. Ética deontológica.
determinados. B. Ética utilitarista.

Conjunto 2
i) Princípio da maior
felicidade.
ii) Imperativo
categórico.
iii) Autonomia da
vontade.

Editável e fotocopiável © Texto | Dúvida Metódica, 11.º ano 197


Tipologia de questões

Questões de construção

Questões de resposta curta Questões de resposta restrita Questões de resposta extensa

Solicitam uma resposta direta: a Como o nome sugere, estas Exigem respostas desenvolvidas
apresentação de uma palavra, de questões não exigem um grande (por vezes incluem um conjunto
uma expressão, de uma frase, etc. desenvolvimento. Podem de instruções de realização
Geralmente, a classificação da solicitar uma explicação, uma destinadas a orientar as
resposta é dicotómica (certo ou justificação, uma distinção, etc.; respostas). Podem – por exemplo
errado, sem níveis intermédios). ou seja, uma tarefa circunscrita. – solicitar uma comparação
A resposta deve ser rigorosa e entre teorias diferentes ou a
Por exemplo: direta, mas completa. Neste tipo exposição de um tema, mas
de questões, a opinião do aluno o mais comum em Filosofia é
Qual é a disciplina filosófica em não costuma ser solicitada. pedirem ao aluno que tome
que se estuda o problema da posição em relação a um certo
existência de Deus? Por exemplo: problema e que a defenda
argumentativamente.
A partir de um exemplo, explica
o que é um problema empírico. Por exemplo:

«Uma vida sem pensar não


é digna de ser vivida por um
homem.»
Platão, Apologia de Sócrates, 38 a.

Concordas com Platão? Porquê?

Este quadro é uma adaptação de um documento intitulado


«TIPOLOGIA DE ITENS» publicado no site do IAVE – Instituto de
Avaliação Educativa.
Pode aceder ao documento aqui:
https://iave.pt/wp-content/uploads/2020/03/IAVE_
TipologiaDeItens2019.pdf
(consultado em 26/01/2022)
Nesse documento são referidos tipos de questões que não incluímos
no quadro por serem pouco frequentes em Filosofia. Optámos por
usar, em vez de «itens», a palavra «questões», por ser mais habitual.
Os exemplos são da nossa autoria.
Os autores

198 Editável e fotocopiável © Texto | Dúvida Metódica, 11.º ano


Relembra Ideias-chave de 10.º ano

Relembra ideias-chave sobre O que é a Filosofia?

O que estuda a Filosofia? Como surgem as questões filosóficas?


• Problemas ou questões deste • As questões filosóficas nascem
género: Teremos livre-arbítrio? da própria existência humana
O que distingue uma ação correta e relacionam-se com situações
de uma ação incorreta? O que é a que até no dia-a-dia se podem
arte? Será que Deus existe? encontrar.

A natureza das questões filosóficas • Todas as pessoas têm crenças


filosóficas, mas algumas não têm
• As questões filosóficas distinguem-se
consciência disso e não analisam
das questões não filosóficas
nem discutem essas crenças,
na medida em que são básicas,
arriscando-se por isso a acreditar
conceptuais e muito controversas: em falsidades.
são básicas, pois a sua resposta • A Filosofia implica uma atitude crítica
condiciona a resposta a questões de análise e discussão das
mais específicas sobre o mesmo ideias que a generalidade das pessoas
assunto; aceita de modo acrítico.
são conceptuais, pois não são As aulas de Filosofia
empíricas, ou seja, não podem
• Nas aulas de Filosofia espera-se
ser adequadamente respondidas que os alunos analisem e discutam
através da experiência, mas sim criticamente as teorias estudadas
do pensamento; e argumentem para justificar
são controversas, pois há as suas opiniões acerca
desacordos persistentes e dos problemas filosóficos.
duradouros entre os especialistas As disciplinas filosóficas
acerca da melhor maneira de
• Na Filosofia existem várias
lhes responder, existindo por isso
disciplinas, tais como: Lógica,
diversas tentativas de resposta.
Metafísica, Ética, Filosofia política,
• Devido a isso, a Filosofia é diferente Epistemologia, Filosofia da ciência,
quer das ciências empíricas quer Filosofia da arte e Filosofia
da Matemática. da religião.

>ĕĩĺŜĺƧÑ analisa e debate questões

• básicas
Requer • conceptuais
pensamento crítico • controversas

Editável e fotocopiável © Texto | Dúvida Metódica, 11.º ano 199


Relembra Ideias-chave de 10.º ano

Relembra Ideias-chave de 10.º ano


Relembra ideias-chave sobre Lógica (formal e informal)

Proposições • As proposições categóricas são aquelas em


que se afirma ou nega um predicado de
• A Lógica é o estudo da argumentação
um sujeito. Podem incluir quantificadores
e fornece «ferramentas» úteis para a
(palavras como todos, nenhum e alguns,
reflexão e para os debates filosóficos.
que indicam a quantidade de elementos
• Uma proposição é o pensamento da classe referida).
literalmente expresso por uma frase
• Quanto à qualidade, as proposições
declarativa. As frases não declarativas
categóricas podem ser afirmativas ou
(perguntas, ordens, exclamações, etc.)
negativas. Quanto à quantidade, podem
não expressam proposições, pois não têm
ser universais, particulares ou singulares.
valor de verdade (não são verdadeiras
Combinando os dois critérios obtém-se
nem falsas).
seis tipos de proposições.
• Frases declarativas diferentes podem
• Essas proposições – tal como os
expressar a mesma proposição.
argumentos – têm uma formulação
• Quando uma frase é ambígua, pode canónica (a maneira padrão de as
expressar proposições diferentes. exprimir, considerada mais explícita e
• Há diversos tipos de proposições. clara do que as numerosas alternativas).

Lógica proposicional • A formalização de proposições


• A lógica proposicional estuda as compostas requer atenção ao
proposições formadas através do uso âmbito das conetivas (isto é, à sua
de conetivas proposicionais: não, e, maior ou menor abrangência). Para o
ou, se… então, se e só se. distinguir recorre-se aos parênteses.
• Essas conetivas, aplicadas a • Uma negação inverte o valor de
proposições simples, permitem verdade da proposição inicial.
formar novas proposições Uma conjunção só é verdadeira se
(compostas): negação, conjunção, ambas as conjuntas forem verdadeiras.
disjunção, condicional, bicondicional. Uma disjunção inclusiva só é falsa se
ambas as disjuntas forem falsas.
• A forma lógica de uma proposição
Uma disjunção exclusiva é
(e de um argumento) é a sua
verdadeira se as disjuntas tiverem
estrutura. Para a detetar, substitui-
valores de verdade diferentes e falsa
-se as palavras por símbolos:
se estes forem iguais.
as proposições simples são
Uma condicional só é falsa se
representadas pelas variáveis
a antecedente for verdadeira
proposicionais e as conetivas pelas
e a consequente for falsa.
constantes lógicas.

200 Editável e fotocopiável © Texto | Dúvida Metódica, 11.º ano


Relembra Ideias-chave de 10.º ano

• A negação de uma proposição inverte o seu


valor de verdade.
• Uma singular negativa nega uma singular
afirmativa; uma particular negativa nega uma
A contrariedade E
universal afirmativa; uma particular afirmativa
nega uma universal negativa. E vice-versa.

co
nt
ra
• O quadrado da oposição apresenta diversas

di
to
subalternidade

subalternidade
rie
relações lógicas entre as proposições

da
de
universais e particulares: contraditoriedade

de
(entre UA/PN e UN/PA); contrariedade (entre

da
rie
UA/UN); subcontrariedade (entre PA/PN).

to
di
• A relação de contraditoriedade corresponde

ra
nt
co
à negação, pois as contraditórias têm
sempre valores de verdade diferentes. O
I subcontrariedade
A contrariedade e a subcontrariedade são
distintas da negação, pois as contrárias podem
ser ambas falsas e as subcontrárias podem ser
ambas verdadeiras.

Uma bicondicional é verdadeira • Um argumento é um conjunto de


se as suas partes tiverem valores proposições, em que se pretende
de verdade iguais e falsa se estes que uma delas (a conclusão) seja
forem diferentes. defendida ou apoiada pela(s) outra(s)
• A antecedente de uma condicional – a(s) premissa(s).
constitui uma condição suficiente • Num argumento na forma canónica,
e a consequente uma condição a conclusão surge no final,
necessária. antecedida da palavra «Logo»; mas na
• Numa bicondicional cada uma comunicação real os argumentos têm
das suas partes constituintes apresentações variadas, podendo a
é, simultaneamente, condição conclusão surgir no início ou no meio.
suficiente e condição necessária. • Alguns argumentos, os entimemas,
• As condições de verdade de cada têm premissas subentendidas e a
conetiva, bem como de formas sua clarificação e reconstituição
proposicionais compostas, podem canónica exige que estas sejam
ser apresentadas através de tabelas identificadas e explicitadas.
de verdade. • A formalização dos argumentos
recorre ao símbolo ‫ ׵‬, que representa
o «Logo».

Editável e fotocopiável © Texto | Dúvida Metódica, 11.º ano 201


Relembra Ideias-chave de 10.º ano

Argumentos dedutivos e falácias formais


• Os argumentos dedutivos são válidos • Algumas formas argumentativas válidas
ou inválidos (e não verdadeiros ou são frequentes e têm nomes próprios:
falsos). São válidos quando há um nexo negação dupla, modus ponens, modus
lógico entre as premissas e a conclusão tollens, contraposição, silogismo
e, portanto, esta se segue daquelas; hipotético, silogismo disjuntivo
isto é, quando é impossível terem e leis de De Morgan.
premissas verdadeiras e conclusão • As falácias são argumentos incorretos
falsa. Essa validade depende apenas que podem parecer corretos a quem
da forma do argumento. pouco souber de Lógica.
• Um argumento pode ser válido e ser Se a incorreção residir na forma lógica
mau, isto é, constituído por proposições do argumento, são falácias formais.
falsas. Um argumento sólido é um Se residir no conteúdo, são falácias
argumento dedutivo válido e com informais.
premissas verdadeiras (o que assegura
• As falácias da afirmação da
a verdade da conclusão). consequente e da negação da
• A validade dos argumentos dedutivos, antecedente são falácias formais
depois de devidamente formalizados, frequentes (devido à sua enganadora
pode ser testada através de tabelas semelhança com o modus ponens e o
de validade. modus tollens).

Argumentos não dedutivos e falácias informais

• Os argumentos não dedutivos incluem os • Ao contrário do que sucede com a validade


argumentos indutivos (generalização e dos argumentos dedutivos estudados, a força
previsão), os argumentos por analogia e dos argumentos não dedutivos depende do seu
os argumentos de autoridade. conteúdo e não da sua forma lógica.
• Nos argumentos não dedutivos, a verdade das
• Numa generalização conclui-se que, se algumas
premissas não garante a verdade da conclusão,
coisas possuem uma certa característica, então
apenas a torna mais ou menos provável. Por
todas as coisas dessa classe possuem essa
grande que seja essa probabilidade, nunca se
característica.
pode excluir completamente a possibilidade de
as premissas serem verdadeiras e a conclusão • Numa previsão conclui-se que, se algumas
falsa. Quando essa probabilidade é elevada, coisas possuem uma certa característica, então
o argumento é forte; quando é baixa, o as próximas coisas dessa classe que forem
argumento é fraco. observadas também a possuirão.

202 Editável e fotocopiável © Texto | Dúvida Metódica, 11.º ano


Relembra Ideias-chave de 10.º ano

• As generalizações e previsões fortes


obedecem a critérios: baseiam-se numa
amostra grande e representativa e não
ignoram informações que possam contrariar
o seu conteúdo. Se não obedecerem, tornam-
-se falácias: generalização precipitada,
amostra não representativa e previsão
inadequada.
• Num argumento por analogia defende-se
que, se duas coisas são semelhantes em • Na falácia ad hominem, em vez de se criticar
alguns aspetos, é provável que também as ideias de alguém, criticam-se aspetos
sejam semelhantes noutros. pessoais que nada têm a ver com o assunto
e que não são pertinentes para justificar a
• Os argumentos por analogia fortes obedecem
conclusão.
a critérios: baseiam-se numa quantidade
significativa de semelhanças, estas não são • A falácia do espantalho ocorre quando
irrelevantes e não existem diferenças que distorcemos as ideias do nosso interlocutor
inviabilizem a conclusão. Se não obedecerem, para que pareçam ridículas (faz-se uma
tornam-se falácias da falsa analogia. caricatura), tornando mais fácil a sua
rejeição.
• Num argumento de autoridade defende-
-se que uma ideia é verdadeira porque • A falácia ad populum consiste em defender
foi enunciada por uma autoridade que uma ideia é verdadeira simplesmente
(um especialista). porque é defendida pela maioria das pessoas,
como se a maioria tivesse sempre razão.
• Os argumentos de autoridade fortes
obedecem a critérios: a autoridade não deve • Na falácia do apelo à ignorância conclui-se
ser anónima, deve ser realmente especialista que algo é verdadeiro por não se ter provado
no assunto, deve estar de acordo com que é falso; ou que é falso porque não se
as outras autoridades da área e deve ser provou que é verdadeiro.
imparcial. Se não obedecerem, tornam-se na • Na falácia da falsa relação causal considera-
falácia do apelo ilegítimo à autoridade. -se erradamente que, pelo facto de um certo
• Na falácia da derrapagem alega-se que acontecimento ocorrer a seguir a outro,
uma certa proposição é inaceitável porque existe entre eles uma relação de causa
a sua aceitação conduziria a uma cadeia e efeito.
de implicações que se vão tornando • Na falácia da petição de princípio dá-se como
progressivamente mais inaceitáveis – mas adquirido aquilo que se quer provar, ou seja,
também mais implausíveis. a conclusão é pressuposta nas premissas.
• Num falso dilema alega-se que as duas • Estas treze últimas falácias são informais,
alternativas de uma disjunção são as únicas pois os erros de que padecem residem no
possíveis, quando na realidade há mais. conteúdo dos argumentos.

Editável e fotocopiável © Texto | Dúvida Metódica, 11.º ano 203


Relembra Ideias-chave de 10.º ano

Relembra ideias-chave sobre Determinismo e liberdade na ação humana

O problema do livre-arbítrio

• Formulação do problema: as ações humanas • As crenças no livre-arbítrio e no determinismo


serão realmente livres? Será a crença no são muito comuns e estão pressupostas em
livre-arbítrio compatível com a crença no inúmeras situações da vida.
determinismo? • O problema filosófico do livre-arbítrio resulta
• Livre-arbítrio: liberdade da vontade ou do facto de essas duas crenças poderem,
a liberdade de escolher (é diferente da eventualmente, ser incompatíveis: talvez o
liberdade política). facto de as ações serem determinadas as
• Determinismo: ideia de que todos os impeça de ser livres.
acontecimentos, incluindo as ações humanas, • Estudámos três respostas a este problema:
são o efeito de causas anteriores e das leis da o determinismo radical, o libertismo e o
natureza. determinismo moderado.

O determinismo radical
• Tese: não existe livre-arbítrio e nenhuma ação • O livre-arbítrio é uma ilusão provocada pelo facto
humana é livre. de as pessoas não se aperceberem das inúmeras
• As ações humanas, como qualquer outro causas que determinam os seus desejos e as suas
acontecimento, são determinadas por causas ações.
anteriores. • Dado que as ações não são livres, os agentes não
• O determinismo radical é uma perspetiva têm genuína responsabilidade (não são realmente
incompatibilista, já que considera o determinismo culpados pelo que fazem de mal e não têm mérito
e o livre-arbítrio incompatíveis. O facto de as ações pelas boas ações).
serem determinadas por causas anteriores impede- • Mas, apesar de não existir genuína responsabilidade,
-as de ser livres. as pessoas devem ser responsabilizadas, pois desse
• Qualquer ação é o efeito necessário e inevitável modo promove-se as boas ações e dissuade-se
das causas prévias determinantes e, por isso, o as más ações.
agente só podia ter feito o que fez – não há nunca
possibilidades alternativas de ação.
• As causas que determinam o comportamento Objeções ao determinismo radical
humano, dada a complexidade deste, são • Sem livre-arbítrio a vida não teria sentido,
muito variadas: biológicas, sociais e culturais, porque seríamos como robôs, programados
psicológicas, entre outras. para fazer o que fazemos.
• Muitas descobertas científicas – da Biologia, • É implausível que o livre-arbítrio seja uma
Psicologia, Sociologia, etc. – apoiam as ideias ilusão, pois isso não promoveria a adaptação
do determinismo radical; por isso, negar o da espécie humana ao meio ambiente e o seu
determinismo é anticientífico e pouco racional. sucesso evolutivo.

204 Editável e fotocopiável © Texto | Dúvida Metódica, 11.º ano


Relembra Ideias-chave de 10.º ano

O libertismo
• Tese: existe livre-arbítrio e algumas ações são • Argumento da responsabilidade: caso não exista
livres (as que não são causalmente determinadas). livre--arbítrio, não é possível existir responsabilidade
• O libertismo é uma perspetiva incompatibilista, (nem culpa nem mérito), visto que as pessoas podem
pois considera que o determinismo e o livre- ser responsabilizadas somente pelas ações de que
arbítrio não podem coexistir. são autoras; mas, como isso é implausível, devemos
concluir que o livre-arbítrio existe.
• Ao agir livremente, o agente autodetermina-se:
age independentemente de quaisquer causas • Para Sartre, a liberdade é fundamental, pois ela é que
anteriores e a única causa da ação é a própria faz dos seres humanos aquilo que são. A existência
decisão (causalidade do agente). precede a essência: ao contrário dos objetos, os
seres humanos não têm à partida uma natureza
• O livre-arbítrio envolve a existência de
que estabeleça aquilo que são e o que devem fazer.
possibilidades alternativas de ação: realizamos
Isso tem de ser escolhido por eles. Assim sendo, não
uma certa ação, mas podíamos ter realizado
podemos recusar a necessidade de escolher. Mesmo
outras.
nas circunstâncias mais difíceis, podemos pelo
• Argumento da experiência: temos experiência do menos optar pela maneira como vamos reagir aos
livre--arbítrio (= sentimo-nos livres) e, portanto,
problemas. Daí que sejamos, totalmente responsáveis
este existe. Ou seja: muitas vezes, ao fazer
pela nossa vida e pelas nossas ações.
escolhas, não sentimos nenhuma espécie de
constrangimento, nenhuma força a controlar-nos e
a obrigar-nos a fazer algo. Objeções ao libertismo
• A ideia de que há ações sem causas anteriores
é implausível e anticientífica, pois em
qualquer ação que analisemos encontramos
sempre essas causas.
O determinismo moderado
• Podemos não sentir ou não reconhecer os
• Tese: existe livre-arbítrio e algumas ações são livres. efeitos provocados pelas causas anteriores e,
• O determinismo moderado é compatibilista, mesmo assim, estas existirem e determinarem
pois considera compatíveis o determinismo a nossa vontade. Portanto, o argumento da
e o livre-arbítrio – todas as ações são determinadas experiência não prova a existência de livre-
por causas anteriores, mas algumas são livres. arbítrio.
• Redefinição do conceito de livre-arbítrio: não é
o poder de agir independentemente de causas
anteriores, mas sim o poder de fazer o que Objeções ao determinismo moderado
queremos e não fazer o que não queremos. • Não permite compreender realmente a
• Uma ação é livre se é realizada sem compulsões existência de autênticas possibilidades
ou coações (internas ou externas) e se deriva alternativas de ação, pois se uma ação tem
das crenças e desejos do agente. sempre causas anteriores que o agente não
controla, não parece possível fazer algo de
• Essas crenças e desejos têm causas anteriores que o
diferente e alternativo.
agente não controla, mas isso não retira liberdade à
ação (desde que o agente não tenha sido compelido). • A redefinição do conceito de livre-arbítrio é
simplista e não resolve o problema, pois os
• Assim, existem possibilidades alternativas de ação:
incompatibilistas não duvidam de que por
o agente faz uma coisa, mas podia ter feito outras,
vezes seja possível agir sem coações e não é
caso o tivesse desejado.
isso que discutem entre eles, mas sim se um
agente pode ou não autodeterminar-se.

Editável e fotocopiável © Texto | Dúvida Metódica, 11.º ano 205


Relembra Ideias-chave de 10.º ano

Relembra ideias-chave sobre O problema da natureza dos juízos morais

Os valores
O subjetivismo

• Os valores são princípios ou ideais que • Tese defendida: Todos os juízos de valor
inspiram aquilo que as pessoas pensam, morais são subjetivos, pois a sua verdade
querem e fazem. ou falsidade depende de sentimentos e
preferências pessoais. Estes juízos não são
• Existem valores de diversos tipos: éticos ou
objetivamente verdadeiros ou falsos, mas
morais, estéticos, religiosos, etc.
sim verdadeiros para as pessoas que os
aceitam e falsos para quem os rejeita.
Juízos de facto, juízos de valor e Ética • Na Ética não há verdades objetivas e
universais. Assim, se uma pessoa disser que
• Os juízos de facto são descritivos (por exemplo: algo é certo e outra disser que é errado,
«O Papa pensa que o aborto é errado») e os terão ambas razão.
juízos de valor são avaliativos (por exemplo:
• Os defensores do subjetivismo alegam
«Não devemos tratar mal os animais»).
que este favorece a liberdade individual e
Os juízos de valor morais são normativos:
promove a tolerância entre pessoas com
dizem (explícita ou implicitamente) como
opiniões morais diferentes.
o sujeito pensa que as coisas deviam ser.
• A ética ou moral diz respeito à diferença entre
o certo e o errado, entre o que devemos
e não devemos fazer.
• Problema da natureza dos juízos de valor Objeções ao subjetivismo
morais: • Se o subjetivismo fosse verdadeiro,
Será que estes juízos são subjetivos, seria ilegítimo censurar ações
culturalmente relativos ou objetivos? claramente erradas, como as
dos assassinos e dos pedófilos.
• O problema da natureza dos juízos de Mas, como é muito implausível
valor morais leva-nos a refletir sobre o não poder censurar assassinos e
multiculturalismo: muitas das sociedades pedófilos, pode concluir-se que o
atuais são constituídas por pessoas e grupos subjetivismo não é verdadeiro.
com culturas diferentes. Como lidar com essas
diferenças culturais? • Se o subjetivismo fosse verdadeiro,
os debates morais não fariam
sentido. Mas, como é muito
implausível que os debates morais
não façam sentido, pode concluir-
-se que o subjetivismo não é
verdadeiro.

206 Editável e fotocopiável © Texto | Dúvida Metódica, 11.º ano


Relembra Ideias-chave de 10.º ano

O objetivismo

O relativismo • Tese defendida: Alguns juízos de valor morais


são objetivos porque a sua verdade ou falsidade não
• Tese defendida: Todos os juízos de valor
depende de uma certa perspetiva (é independente
morais são culturalmente relativos, pois
dos sentimentos da pessoa e da sua cultura),
a sua verdade ou falsidade depende da
mas baseia-se em razões: um juízo moral será
cultura de cada sociedade. Estes juízos
objetivamente verdadeiro se puder ser justificado
não são objetivamente verdadeiros com boas razões e falso se isso não for possível.
ou falsos: são verdadeiros para as
sociedades que os aceitam e falsos para • Boas razões são razões informadas (baseadas em factos
e conhecimentos) e imparciais (não tendenciosas, isto é,
as que os rejeitam.
tentam ter em conta todas as pessoas envolvidas).
• Os relativistas acentuam a importância
• Quando uma pessoa discorda de um juízo moral
da diversidade cultural e dela inferem
verdadeiro está enganada – tal como estaria se dissesse
que não se pode dizer que os valores
que «2 + 7 = 11». Portanto, há assuntos em relação aos
e costumes de uma sociedade são
quais se pode dizer, de forma plausível, que já sabemos
melhores ou piores do que os de outra.
o que é moralmente verdadeiro (por exemplo, «É errado
• Não existe nenhum critério objetivo e maltratar as crianças»).
universal para determinar quem tem
• Contudo, há assuntos mais difíceis em que existe
razão, pois qualquer critério traz a marca controvérsia mesmo entre os especialistas. Por exemplo:
da cultura onde surgiu e não é válido «O aborto é sempre errado» ou «O aborto é correto
para as pessoas de outras culturas. Por em certas situações» – não podemos garantir qual
isso, na Ética não há verdades objetivas e desses dois juízos é verdadeiro. Ainda assim, um deles é
universais. verdadeiro e o outro falso: é por isso que esse problema
• As pessoas de cada sociedade devem é tão debatido, para tentar descobrir a verdade objetiva
viver de acordo com os seus costumes acerca dele.
e abster-se de criticar os costumes das • Os defensores do objetivismo alegam que este,
outras sociedades. Se criticarem estarão a contrariamente ao subjetivismo e ao relativismo,
ser intolerantes e etnocêntricas. favorece a tolerância entre pessoas de sociedades
diferentes, já que cria condições para um autêntico
• Os defensores do relativismo alegam que
debate intercultural.
este favorece a tolerância entre pessoas
de sociedades diferentes.
Objeções ao objetivismo
• Se alguns juízos morais fossem objetivos seria de
Objeções ao relativismo
esperar que não existissem tantas divergências éticas;
• É possível criticar construtivamente, numa porém, como estas existem em grande quantidade,
perspetiva de diálogo intercultural, alguns pode concluir-se que nenhum juízo moral é objetivo
costumes de outras sociedades, sem ser e que o objetivismo não é uma boa teoria.
intolerante e etnocêntrico.
• Nas ciências as divergências tendem a ser
• Se o relativismo fosse verdadeiro, a ideia ultrapassadas graças aos métodos científicos de
de progresso moral não teria sentido. prova. Todavia, na Ética não existem esses métodos
Contudo, como é implausível não haver e por isso os desacordos éticos são intermináveis – o
progresso moral, pode concluir-se que o que mostra que não há verdades éticas objetivas e
relativismo não é verdadeiro. que o objetivismo não é uma boa teoria.

Editável e fotocopiável © Texto | Dúvida Metódica, 11.º ano 207


Relembra Ideias-chave de 10.º ano

Relembra ideias-chave sobre O problema do critério da moralidade das ações

O problema da
fundamentação da moral A ética de Kant

Como distinguir uma ação moralmente • É deontológica (= ética do dever): defende que há
ações que é sempre errado realizar e outras que
correta de uma ação moralmente incorreta?
é sempre correto realizar – independentemente
Que critério ético permite fazer essa
das suas consequências serem positivas ou
distinção? Estudámos duas respostas
negativas. Há deveres morais absolutos; estes
diferentes: a ética deontológica de Immanuel devem ser incondicionalmente cumpridos – sem
Kant e o utilitarismo de John Stuart Mill. exceções – e são válidos para todos
os seres humanos ou quaisquer outros seres
racionais (caso existam).

A ética de Stuart Mill • Na maioria das situações, os agentes não usam o


princípio da utilidade (o princípio supremo) para
• O utilitarismo é consequencialista: as ações são avaliar diretamente ações particulares, orientando-
moralmente corretas ou incorretas em função -se por regras de ação (os princípios subordinados),
das suas consequências: se estas forem favoráveis, como por exemplo «não se deve roubar»,
a ação é correta; caso sejam desfavoráveis, consagradas pela experiência de vida e «aprovadas»
é incorreta. O motivo é irrelevante. pelo princípio da utilidade. Quando, por vezes,
• Fundamento da moral (ou o princípio ético essas regras não funcionam (se forem seguidas
supremo): o princípio da utilidade. É um critério produzir-se-á, ao contrário do que é habitual, mais
ético que permite distinguir o certo do errado: infelicidade do que felicidade) justifica-se – de
as consequências são favoráveis na medida acordo com o princípio da utilidade – abrir
em que trazem felicidade (ou bem-estar) às uma exceção.
pessoas envolvidas e são desfavoráveis se trazem • A felicidade é o bem último, já que é a finalidade
infelicidade. última das ações humanas e tem valor intrínseco:
• Essa felicidade deve ser calculada com desejamo-la por si mesma, enquanto as outras
imparcialidade, pois não se trata só da felicidade coisas são desejadas como meios para as alcançar.
do agente, mas da felicidade do maior número
possível de pessoas envolvidas na ação.
• O utilitarismo de Mill é hedonista: a felicidade
consiste no prazer e na ausência de dor.
• Os prazeres são qualitativamente diferentes: Objeções à teoria ética de Mill
os intelectuais são superiores aos corporais e • Dificuldades de cálculo: podemos enganar-
proporcionam, por isso, mais felicidade a um ser -nos a calcular as consequências e considerar
benéficas ações que afinal são prejudiciais.
racional. As pessoas que conhecem bem ambos
preferem sempre os intelectuais aos corporais. • Violação de direitos individuais: o utilitarismo
parece justificar ações que intuitivamente
• Não há deveres absolutos: geralmente, roubar consideramos erradas, pois desrespeitam
é incorreto, pois habitualmente provoca mais os direitos de algumas pessoas (ainda que
infelicidade do que felicidade; contudo, pode haver maximizem a felicidade).
situações em que roubar maximiza a felicidade e
• Excesso de exigência: impor ao agente que seja
nesse caso será uma ação correta (sendo então imparcial na consideração da sua felicidade (e
possível dizer que os fins justificaram os meios). das pessoas próximas) e da felicidade das outras
pessoas é errado e impossível de cumprir.

208 Editável e fotocopiável © Texto | Dúvida Metódica, 11.º ano


Relembra Ideias-chave de 10.º ano

• A razão é a fonte do imperativo categórico • O verbo «dever» nem sempre é usado num sentido
(ou lei moral). moral. Há deveres que derivam do imperativo
• O imperativo categórico é o princípio supremo hipotético (que nos diz que meios devemos usar se
da moralidade: constitui um critério que permite queremos atingir certos resultados) e, por isso, não
distinguir as ações corretas das incorretas e, têm caráter moral. Aplicação possível do imperativo
portanto, o que devemos fazer do que não devemos hipotético: «Se não queres perder a confiança dos
fazer. amigos, então não deves mentir». Aplicando o
imperativo categórico, diremos simplesmente: «Não
• O imperativo categórico pode ser expresso
mintas» ou «Não mintas, porque é errado».
na fórmula da lei universal e na fórmula da
humanidade. • A boa vontade caracteriza-se pela autonomia, ou
seja, obedece à razão e ao imperativo categórico (ou
• Fórmula da lei universal: devemos realizar apenas
lei moral) – o que significa que obedece a si mesma.
as ações que decorram de máximas (= regras)
que possam ser transformadas em leis universais • Se a vontade for influenciada por interesses pessoais
(= seguidas por todas as pessoas nas mesmas ou sentimentos revela, pelo contrário, heteronomia
circunstâncias). – o que significa que não obedece a si mesma, mas a
algo exterior à razão.
• Fórmula da humanidade: não devemos realizar
ações em que as pessoas (os outros e nós próprios) • Uma vontade autónoma é uma vontade livre
sejam consideradas apenas meios e não fins em e, portanto, ser livre é obedecer à lei moral (ou
si mesmas. Um ser humano não deve ser tratado imperativo categórico). Ou seja: para Kant não basta
como um mero meio ou instrumento ao serviço dos escolher para ser realmente livre, é preciso escolher
outros, pois é um ser racional capaz de fazer as suas o que é moralmente correto.
próprias escolhas e, como tal, a sua autonomia deve • Somos seres racionais e, por isso, quando a nossa
ser respeitada. vontade obedece à razão e se submete à lei moral,
• A boa vontade é o bem supremo (ou último), pois obedecemos a nós próprios – é como se cada um de
é intrínseca e ilimitadamente boa – e não pode nós fosse um legislador moral e coautor da lei moral.
ser usada para o mal. Temos uma boa vontade
quando o nosso querer é determinado pela razão e
obedecemos ao imperativo categórico.
• Uma pessoa dotada de boa vontade realizará ações Objeções à ética de Kant
por dever (cujo motivo é fazer o que é correto,
• Conflito de deveres: existem casos em que há
cumprir o dever pelo dever – têm valor moral) e deveres incompatíveis e, se forem absolutos,
não realizará ações contrárias ao dever (erradas ou não sabemos o que fazer.
imorais), nem sequer ações meramente conformes
• O imperativo categórico afinal é
ao dever (estão de acordo com o dever, não são consequencialista: o teste da universalização
motivadas pelo respeito ao dever – não têm valor das máximas mostra se as consequências, de
moral). todos seguirem uma certa ação, são boas ou
más.
• As ações meramente conformes ao dever podem ser:
motivadas por interesse / com «intenção egoísta» • É errado negar a importância moral dos
sentimentos: alguns deles são centrais na
(desejo de consequências benéficas para o agente) e
vida humana e têm caráter moral, pelo que é
motivadas por «inclinações imediatas» (sentimentos inadequado retirá-los da esfera ética.
positivos como a compaixão e a simpatia).

Editável e fotocopiável © Texto | Dúvida Metódica, 11.º ano 209


Relembra Ideias-chave de 10.º ano

Relembra ideias-chave sobre O problema da organização de uma sociedade justa

O problema da justiça distributiva

• Formulação do problema: qual é a maneira


correta de distribuir a riqueza numa Teoria de Rawls
sociedade? As desigualdades sociais e
• Posição original: situação hipotética
económicas serão justas ou injustas? (caracterizada pela igualdade de direitos
• Principais respostas ao problema da justiça entre todos os participantes) em que as
distributiva: pessoas têm a possibilidade de escolher
o igualitarismo – todas as desigualdades os princípios da justiça que irão reger a
sociais e económicas são injustas; sociedade.
devemos promover a igualdade; • Véu de ignorância: a pessoa que está
o libertarismo de Nozick – as na posição original desconhece as suas
desigualdades sociais e económicas só características individuais e a posição
são injustas se resultarem de atividades social que vai ocupar na sociedade. Esta
ilegítimas; devemos promover a liberdade; situação garante a imparcialidade na
liberalismo igualitário de Rawls – as escolha dos princípios da justiça e faz com
desigualdades sociais e económicas são que qualquer pessoa opte por princípios
injustas se não contribuírem para uma semelhantes.
melhoria dos que estão em pior situação; • Os princípios da justiça: princípio da
devemos promover um equilíbrio entre a liberdade igual; princípio da oportunidade
liberdade e a igualdade; justa e princípio da diferença.
o comunitarismo de Sandel – as • O primado do princípio da liberdade
desigualdades sociais e económicas muito implica uma rejeição do utilitarismo,
acentuadas são injustas, principalmente pois este é compatível com sacrifícios
porque prejudicam a comunidade como da liberdade que permitam maximizar a
um todo. felicidade global.
• Justificação para a escolha dos princípios
da justiça:
o argumento da posição original e a
regra maximin: não sabendo que lugar
iriam ocupar na sociedade, as pessoas
tenderiam a ser imparciais e não
escolheriam princípios não igualitários
que depois as poderiam prejudicar;

210 Editável e fotocopiável © Texto | Dúvida Metódica, 11.º ano


Relembra Ideias-chave de 10.º ano

o argumento moral: a distribuição Por outro lado, as pessoas que


da riqueza e das oportunidades foram beneficiadas pelos acasos
não se deve basear em fatores sociais e naturais têm obrigação
moralmente arbitrários. Por isso, de partilhar a sorte que tiveram,
é preciso salvaguardar a situação contribuindo para a sociedade
das pessoas que tiveram azar na através do pagamento de impostos
lotaria social e na lotaria natural. significativos.

Críticas a Rawls

Nozick
• A distribuição da
riqueza defendida por Sandel
Rawls, nomeadamente o facto de • Algumas das ideias da teoria de Rawls,
ser incondicional e independente em particular o método por ele usado para
das atitudes das pessoas, promove identificar os princípios da justiça (a posição original
a desresponsabilização de quem é e o véu de ignorância), baseiam-se num eu abstrato,
ajudado e é injusta para as pessoas desenraizado da sociedade, o que além de não
empenhadas e cumpridoras, pois obriga- corresponder à realidade (o eu encontra-se sempre
-as a financiar pessoas que escolheram situado e inserido na comunidade), não é desejável.
não se esforçar.
• O bem tem prioridade sobre o justo, ao contrário do
• As pessoas têm direitos muito que Rawls defende, porque só a partir de uma certa
fortes à liberdade e à propriedade, conceção de bem se pode avaliar se as escolhas e as
e a tributação, tendo em vista a ações são ou não justas.
redistribuição da riqueza, viola esses
direitos. • O bem é a vida boa. Esta só é alcançável na
convivência com os outros, quando existe a
• A tributação é equivalente a possibilidade de uma pessoa desenvolver as
trabalho forçado, constitui uma capacidades (ou virtudes) próprias dos seres
instrumentalização das pessoas e uma racionais como a reflexão crítica, o debate público,
negação da sua autonomia moral. a entreajuda e a participação na vida política e
• O único estado que se pode justificar e cívica, entre outras.
que não viola os direitos das pessoas é o • Os princípios de justiça não podem ser
estado mínimo, reduzido a funções como fundamentados a partir de um cálculo baseado
a polícia, os tribunais e o exército (ou no interesse egoísta (como Rawls – segundo
seja, sem funções sociais). Sandel – pretende), mas sim a partir dos benefícios
que trazem para a comunidade, por exemplo, a
diminuição das desigualdades ou da segregação
entre grupos e classes sociais, o aumento da
solidariedade e da coesão social. Estes são objetivos
que qualquer sociedade democrática deve procurar
alcançar para ser justa.

Editável e fotocopiável © Texto | Dúvida Metódica, 11.º ano 211


Outra Maneira de Preparar os Exames: um exemplo

Atividades para realizar em grupo


Sugestões: a partir de um guião, exploração de um pequeno vídeo e realização de um debate.
Conteúdos: Lógica (tipos de proposições) e o Problema da natureza dos juízos morais
Tipo de trabalho: colaborativo (grupos de 3 a 4 alunos)

Guião de análise do vídeo

1 . Enuncia a frase declarativa, apresentada no vídeo, que corresponde à tese defendida pelo
relativismo moral.

2 . Considera as seguintes afirmações:

A. As regras morais são relativas à cultura.

B. Nenhum costume é melhor do que outro.

2.1 Classifica o tipo de proposição (A, E, I ou O) presente nas alíneas anteriores, indicando
a quantidade e qualidade.

2.2 Como se podem negar as proposições expressas nas alíneas A e B?

3 . Dá dois exemplos, referidos no vídeo, que permitam ilustrar a diversidade cultural.

4 . Segundo alguns antropólogos, não se deve criticar as regras morais e os hábitos sociais
das diferentes culturas. Como justificam eles esta tese?

5 . Qual é o exemplo, apresentado no vídeo, que permite colocar uma objeção ao relativismo
moral? Que conclusão é retirada, no vídeo, da análise deste exemplo?

6 . O que significa dizer «a verdade é relativa»?

Editável e fotocopiável © Texto | Dúvida Metódica, 11.º ano 213


Outra Maneira de Preparar os Exames: um exemplo

Sugestão de problemas para debater (no grupo-turma)

• Na tua opinião, o certo ou o errado dependerão apenas da cultura? Porquê?


• Na tua opinião, será correto ou incorreto criticar certas regras morais e hábitos sociais de
outras culturas? Porquê?

Proposta de resolução do guião de análise do vídeo

1 . Tese do relativismo moral: «O que é correto numa cultura pode ser errado noutra.»

2.

2.1

A. Proposição tipo A: Universal/Afirmativa.

B. Proposição tipo E: Universal/Negativa.

2.2

A. Algumas regras morais não são relativas à cultura.

B. Alguns costumes são melhores do que outros.

3 . Dois exemplos: as diferentes perspetivas (consoante a época histórica, o país e a cultura)


que já existiram e existem em relação à tourada e à escravatura.

4 . Justificação da tese defendida: uma cultura não deve ser avaliada de fora, a partir de
um ponto de vista diferente, já que as suas práticas respondem a situações e tradições
específicas. Julgar negativamente uma cultura com base noutras culturas, consideradas
superiores, corresponde a uma atitude etnocêntrica e leva à imposição de certos valores e
regras morais.

5 . O exemplo refere-se ao que aconteceu durante o nazismo: era considerado correto, do


ponto de vista moral e legal, maltratar e matar judeus, homossexuais e ciganos.
Conclui-se o seguinte: se admitirmos o relativismo moral, não teremos condições para
criticar práticas desumanas e distingui-las das que respeitam os direitos humanos.

6 . Significa que não há verdades objetivas e universais («válidas» para todas as pessoas), mas
apenas verdades particulares. O que é verdade para algumas pessoas é falso para outras –
e nenhuma tem mais razão do que outra.

Nota relativa ao debate


Em princípio, a discussão de um dos problemas sugeridos conduzirá à discussão do outro. Caso
isso não ocorra naturalmente, o professor poderá, se assim o entender, propor o outro problema.

214 Editável e fotocopiável © Texto | Dúvida Metódica, 11.º ano


Prova com a estrutura do Exame Nacional de Filosofia

Prova com a estrutura do Exame Nacional de Filosofia


10.o e 11.o anos
Duração: 120 minutos | Tolerância: 30 minutos

1. A Filosofia é uma atividade crítica na medida em que


(A) diz mal de todas as ideias.
(B) implica uma atitude dogmática.
(C) promove a avaliação cuidadosa das ideias.
(D) nos leva a aceitar todos os pontos de vista.

2. Qual é a frase falsa?


(A) Os juízos de valor são geralmente controversos e os juízos de facto são sempre consensuais.
(B) Enquanto um juízo de valor constitui uma avaliação, um juízo de facto é descritivo.
(C) Os juízos de valor morais são geralmente normativos.
(D) Os juízos de facto visam dizer como as coisas são e não como deveriam ser.

3. De acordo com a Rosa, não devemos acreditar no Ricardo quando ele defende que as drogas
prejudicam a saúde, uma vez que ele por vezes consome drogas. A Rosa
(A) incorreu na falácia do espantalho, pois o facto de o Ricardo por vezes consumir drogas não
implica que a sua tese seja falsa.
(B) incorreu na falácia ad hominem, pois o facto de o Ricardo por vezes consumir drogas não
implica que a sua tese seja falsa.
(C) não incorreu em nenhuma falácia, pois o Ricardo é incoerente e isso é criticável.
(D) incorreu na falácia do falso dilema, pois pressupõe que as pessoas ou não consomem drogas
ou são incoerentes.

4. A falácia do apelo à ignorância ocorre quando se conclui que uma certa ideia é falsa porque
(A) não se provou que é verdadeira.
(B) é rejeitada pela maioria das pessoas.
(C) é defendida por alguém que é ignorante.
(D) tem consequências muito implausíveis.

5. Segundo Kant, se a vontade não for determinada por interesses alheios mas sim pelos interes-
ses pessoais do agente será
(A) autónoma.
(B) heterónoma.
(C) necessariamente má.
(D) necessariamente boa.

Editável e fotocopiável © Texto | Dúvida Metódica, 11.º ano 215


Prova com a estrutura do Exame Nacional de Filosofia

6. Robert Nozick critica a teoria de John Rawls por esta defender


(A) a igualdade de liberdades e direitos entre os cidadãos.
(B) a primazia do direito à propriedade privada face a outros direitos.
(C) a maximização da felicidade dos cidadãos como critério de escolha dos princípios da justiça.
(D) uma organização da sociedade que promove uma redistribuição não voluntária dos
rendimentos.

7. Para Michael Sandel, o bem tem prioridade sobre o justo, na medida em que
(A) o Estado deve ser neutral relativamente às diversas conceções de bem que se podem
encontrar na sociedade.
(B) é com base no justo que se deve avaliar se as escolhas são ou não boas.
(C) os princípios da justiça devem ser escolhidos a coberto de um véu de ignorância.
(D) é com base no bem que se deve avaliar se as escolhas são ou não justas.

8. De acordo com a chamada definição tradicional de conhecimento, uma crença será um conhe-
cimento se
(A) for verdadeira e estiver justificada.
(B) estiver justificada e fundamentada.
(C) for verdadeira e plausível.
(D) se o sujeito tiver uma convicção forte e profunda de que é verdadeira.

9. Um cético radical é alguém que defende a seguinte tese:


(A) O conhecimento humano não está justificado.
(B) O conhecimento só é possível em poucas áreas de estudo e na condição de haver rigor nas
investigações.
(C) O conhecimento não é possível em nenhuma área de estudo, independentemente do modo
como organizarmos as nossas investigações.
(D) O conhecimento é possível em todas as áreas de estudo desde que os procedimentos adotados
pelo sujeito sejam epistemicamente virtuosos.

10. Segundo Descartes, a principal fonte do conhecimento humano é


(A) a experiência.
(B) a razão.
(C) a dúvida.
(D) a certeza.

11. O Frederico nunca tinha comido toranjas. Um dia foi passar o fim de semana a casa de um
amigo e experimentou comer uma. Era amarga. No dia seguinte comeu mais duas e constatou
que também eram amargas.

Indica a que conclusões poderá o Frederico chegar se raciocinar indutivamente e fizer


A. uma previsão.
B. uma generalização.

216 Editável e fotocopiável © Texto | Dúvida Metódica, 11.º ano


Prova com a estrutura do Exame Nacional de Filosofia

12. Lê o texto seguinte:


«A teoria institucional de Dickie tem dificuldade em lidar com casos de arte adventícia, uma
categoria heterogénea de arte produzida por pessoas alheias ao mundo da arte. Coleções de
obras desse género incluem, por exemplo, desenhos e pinturas de doentes mentais e outras
pessoas que não se viam como artistas e que não tinham quaisquer conhecimentos acerca da
arte. Em contrapartida, a teoria histórico-intencional de Levinson explica como as obras de
artistas genuinamente adventícios podem também ser arte.»
Nigel Warburton, O Que É a Arte?,
Bizâncio, Lisboa, 2007, pp. 117 e 131 (adaptado)

12.1 Segundo o texto, a teoria institucional tem dificuldade em explicar o estatuto artístico das
obras adventícias e a teoria histórica não. Porquê?
12.2 De acordo com a teoria formalista, que atributo deverá ter uma obra adventícia para ser
considerada artística?

13. Na tua opinião, se uma pessoa formar a crença de que Deus existe através de um procedimento
como a aposta de Pascal, terá uma crença justificada? Porquê?

14. De acordo com Stuart Mill, o agente, ao determinar o que é correto ou incorreto, deve ser
imparcial. Porque pensa ele desse modo?

15. De acordo com o critério da verificação, como se pode distinguir entre ciência e não ciência?

16. O livre-arbítrio é geralmente considerado um bem precioso. Contudo, alguns filósofos pensam
que não existe livre-arbítrio.
16.1 Uma tentativa de mostrar que o facto de haver mal no mundo não é incompatível com a
existência de Deus pressupõe a existência de livre-arbítrio. Explica essa tentativa.
16.2 Na tua opinião, existe livre-arbítrio? Porquê?

Editável e fotocopiável © Texto | Dúvida Metódica, 11.º ano 217


RESOLUÇÃO DA PROVA COM A ESTRUTURA DO EXAME NACIONAL DE FILOSOFIA

1. C 5. B 9. C 15. De acordo com o critério da verificação, proposto


2. A 6. D 10. B pelo positivismo lógico, uma teoria é científica
3. B 7. D se poder ser empiricamente verificável, ou seja,
4. A 8. A se puder ser testada e desse modo se comprovar
11. conclusivamente a sua verdade ou falsidade.
A. A próxima toranja que comer será amarga. / As (As críticas a esse critério levaram alguns
próximas toranjas que comer serão amargas. positivistas lógicos a substituir a verificabilidade
B. Todas as toranjas são amargas. pela confirmabilidade e a comprovação conclusiva
12.1 A dificuldade da teoria institucional prende-se pela comprovação probabilística.)
com o facto de defender que, para algo ser arte, 16.1 A ideia é que o mal que há no mundo resulta das
tem de ser considerado arte por alguém do mundo escolhas livres dos agentes e que não teria sido
da arte. Ora, como os autores dessas obras não se possível, nem mesmo para um Deus omnipotente,
viam como artistas, segundo a teoria institucional a criação de agentes livres e simultaneamente
as suas obras só podem ser consideradas artísticas incapazes de agir erradamente. Por isso, a
a partir do momento em que são descobertas e existência de mal no mundo é o «preço» a pagar
reconhecidas por alguém do mundo da arte. Mas, para possuirmos livre-arbítrio e não é, por isso,
intuitivamente, parece fazer sentido dizer que esses incompatível com a existência de Deus.
autores eram artistas sem consciência de o serem. 16.2 Se o aluno pensar que não existe livre-arbítrio
A teoria histórica não tem essa dificuldade, pois poderá defender a sua posição invocando o
Levinson admite que a intenção de que a obra seja determinismo radical: todas as nossas ações são
vista como o foram as obras do passado por vezes determinadas por causas anteriores às nossas
seja inconsciente ou, pelo menos, não seja explícita. decisões, causas sobre as quais não temos qualquer
12.2 Deverá ter forma significante. controlo, e, portanto, não há espaço para a escolha
13. Se o aluno pensar que a crença em Deus obtida livre.
através da aposta de Pascal é justificada poderá Se o aluno pensar que existe livre-arbítrio poderá
alegar que se trata de uma justificação prática: defender a sua posição invocando o libertismo:
não há provas racionais nem da existência nem algumas das nossas ações são livres pois não são
da inexistência de Deus, mas não se trata de uma determinadas por causas anteriores às nossas
crença absurda – e o argumento de Pascal mostra decisões; enquanto seres racionais conseguimos
que a nossa vida será melhor se a tivermos. autodeterminarmo-nos.
Se o aluno pensar que a crença em Deus obtida O aluno poderá também defender a existência de
através da aposta de Pascal não é justificada livre-arbítrio invocando o determinismo moderado:
poderá invocar as objeções à aposta de Pascal, todas as nossas ações são determinadas por causas
nomeadamente a ideia de que as crenças não são anteriores às nossas decisões, mas algumas são
voluntárias. livres pois são realizadas porque as queremos
14. Para Mill uma ação é correta ou incorreta em realizar e não porque tenhamos sido coagidos
função das suas consequências relativamente à (interna ou externamente) a fazê-lo.
felicidade das pessoas envolvidas. O agente deve
ser imparcial para que, ao avaliar as consequências,
não tenha em conta apenas a sua própria
felicidade, mas sim a felicidade de todas as pessoas
envolvidas e para que a felicidade de cada uma
delas tenha o mesmo peso.

218
Processo cognitivo Nível inferior Nível intermédio Nível superior
REPRODUZIR APLICAR CRIAR Peso
(operações mentais) N.º de
Identificar/Reconhecer Interpretar/Analisar Avaliar/Ponderar (cotação)
itens
Comparar/Distinguir Relacionar/Aplicar Extrapolar/Criticar/
Competências e domínios Expor/Explicitar Justificar/Inferir Tomar posição

A – Análise e interpretação
1.2
1. O problema da natureza dos juízos morais
1.1 Formulação do problema 1.3 50 pontos
1.2 Relevância do problema 1.1 1.4 5 25%
1.3 A distinção entre juízos de facto e de valor. Os juízos morais como juízos de valor.
1.4 A tese, os argumentos e as objeções de diferentes perspetivas sobre o problema
2
da natureza dos juízos morais: subjetivismo, relativismo e objetivismo.
B – Rigor concetual e teórico
1. O problema da natureza dos juízos morais
1.1 Formulação do problema 60 pontos
1.6 3.1
1.2 Relevância do problema 4 30%
1.3 A distinção entre juízos de facto e de valor. Os juízos morais como juízos de valor. 5. 3.2
1.4 A tese, os argumentos e as objeções de diferentes perspetivas sobre o problema
da natureza dos juízos morais: subjetivismo, relativismo e objetivismo.
C – Problematização e pensamento crítico
1. O problema da natureza dos juízos morais
1.1 Formulação do problema 30 pontos
1.5
1.2 Relevância do problema 2 15%
1.3 A distinção entre juízos de facto e de valor. Os juízos morais como juízos de valor. 7.
Avaliação por Competências – Um exemplo: Matriz específica do teste

1.4 A tese, os argumentos e as objeções de diferentes perspetivas sobre o problema


da natureza dos juízos morais: subjetivismo, relativismo e objetivismo.
D – Raciocínio lógico e argumentação
1. O problema da natureza dos juízos morais
1.1 Formulação do problema 4.1
60 pontos

Editável e fotocopiável © Texto | Dúvida Metódica, 11.º ano


1.2 Relevância filosófica do problema 4.2 3
30%
1.3 A distinção entre juízos de facto e de valor. Os juízos morais como juízos de valor. 6.
1.4 A tese, os argumentos e as objeções de diferentes perspetivas sobre o problema
da natureza dos juízos morais: subjetivismo, relativismo e objetivismo.
Número de itens 3 9 2 14
Cotação 30 pontos 140 pontos 30 pontos 200 pontos
Peso 15% 70% 15% 100%

Nota: Foi utilizado o rácio ½ – a pontuação mais baixa a atribuir é 10 pontos e a mais elevada 20 pontos.

219
Avaliação por Competências – Um exemplo: Enunciado do teste e critérios de classificação

Teste de Filosofia – 10.º ano

Aluno:

Duração: 90 minutos / Data: / / Professor/a:

Tema: O problema da natureza dos juízos morais


1 . Lê, atentamente, o texto e responde às questões seguintes, selecionando a alternativa
correta.
«Em 1966, uma rapariga de dezassete anos chamada Fauziya Kassindja chegou ao
Aeroporto Internacional de Newark e pediu asilo. Tinha fugido do seu país natal, o Togo,
pequena nação do oeste africano, para escapar ao que ali as pessoas chamam “excisão”.
Fauziya Kassindja era a mais jovem de cinco filhas de uma família muçulmana devota.
O seu pai, proprietário de uma bem-sucedida empresa de camionagem, opunha-se à
excisão, e tinha capacidade de se opor à tradição por causa da sua riqueza. As suas
primeiras quatro filhas casaram sem ser mutiladas. Mas quando Fauziya tinha dezasseis
anos, ele morreu subitamente. Fauziya ficou então sob a tutela do avô, que ajustou para
ela um casamento e se preparava para a submeter à excisão. Fauziya ficou aterrorizada e
a mãe e a irmã mais velha ajudaram-na a fugir. A mãe tendo ficado sem recursos, teve de
pedir desculpas formais e submeter-se à autoridade do patriarca que ofendeu.
Entretanto, na América, Fauziya foi detida durante dois anos enquanto as autoridades
decidiam o que fazer. Por fim, foi-lhe concedido asilo, mas não sem antes se tornar o
centro de uma controvérsia sobre a forma como devemos encarar as práticas culturais de
outros povos. Uma série de artigos no New York Times favoreceu a ideia de que a excisão
é uma prática bárbara e merecedora de condenação. Outros observadores mostraram-se
relutantes (…) – vive e deixa viver, afirmaram; afinal de contas, é provável a nossa cultura
parecer igualmente estranha para eles.»
James Rachels, Elementos de Filosofia Moral,
Gradiva, Lisboa, 2004, pp. 47-48

1.1 O texto relaciona-se com a seguinte questão filosófica:


A. Como se caracterizam as tradições ligadas ao casamento em diferentes países do
mundo?
B. Quais as consequências da excisão na saúde física e mental das mulheres?
C. O que leva certos povos a seguir a tradição da excisão?
D. Como devemos encarar as práticas culturais de outros povos?

1.2 Os defensores da atribuição de asilo a Fauziya argumentam com base em ideias que
se enquadram
A. no objetivismo.
B. no subjetivismo.
C. no relativismo.
D. nas tradições do Togo.

220 Editável e fotocopiável © Texto | Dúvida Metódica, 11.º ano


Avaliação por Competências – Um exemplo: Enunciado do teste e critérios de classificação

1.3 Quem critica a atribuição de asilo a Fauziya argumenta com base em ideias que se
enquadram
A. nas tradições norte-americanas. C. no subjetivismo.
B. no relativismo. D. no objetivismo.

1.4 A afirmação «A excisão é uma prática bárbara e merecedora de condenação»


pressupõe que os juízos morais podem ser
A. particulares e subjetivos. C. universais e objetivos.
B. individuais e objetivos. D. universais e subjetivos.

1.5 Uma objeção às teses pressupostas nas afirmações «Vive e deixa viver» e «É provável
a nossa cultura parecer igualmente estranha para eles» é:
A. As pessoas de cada sociedade devem viver de acordo com os seus costumes e
abster-se de criticar os costumes dos outros.
B. Criticar certos costumes supõe a superioridade de uma cultura em relação a outras.
C. Criticar certos costumes supõe que todas as culturas têm igual valor.
D. Criticar certos costumes não implica ser etnocêntrico ou intolerante.

1.6 É errado obrigar Fauziya a seguir as tradições do Togo.


Este é um juízo
A. moral. C. religioso.
B. estético. D. descritivo.

2 . O cartoon seguinte apela, de forma implícita, para diversos valores. Indica dois deles.

FIG. 1

Editável e fotocopiável © Texto | Dúvida Metódica, 11.º ano 221


Avaliação por Competências – Um exemplo: Enunciado do teste e critérios de classificação

3 . Analisa, atentamente, as figuras e os exemplos nas alíneas seguintes.

FIG. 2 e 3ǝ>ĺťĺČŗÑƧÑŜñÑĕıžÑŜçĺñĺÑŔĕťĻĩĕĺLjøİ°ÑŜđĕıČťĺıLj,œLjŔĺŗÑŔĺĕÑıťøŜñø#ĺıÑĩñ–ŗūİŔıĺñĕÑƯƵǓƯưǓƱƯƱư

a) A Maria Albertina, a propósito da invasão do Capitólio nos EUA, argumentou a favor


da existência de limites à divulgação de mensagens nas redes sociais, nomeadamente
aquelas que incitem ao recurso à violência como forma de protesto.
b) Apesar de todos os defeitos que possa ter, a democracia é preferível a qualquer outra
forma de governação.
c) Nos EUA, os apoiantes de Donald Trump, em protesto pela vitória eleitoral do Joe
Biden, invadiram o Capitólio.
d) É errado não aceitar a vontade da maioria dos eleitores.

3.1 Identifica a alínea ou alíneas em que é expresso um juízo de facto. Justifica.


3.2 Identifica a alínea ou alíneas em que é expresso um juízo de valor. Justifica.

4 . Considera o seguinte argumento: «A caça de veados é correta, porque eu, Joaquim Silva,
gosto de os caçar e tenho um enorme prazer nisso.»
4.1 Um subjetivista consideraria que a premissa justifica a conclusão desse argumento?
Porquê?
4.2 Um relativista consideraria que a premissa justifica a conclusão desse argumento?
Porquê?

5 . Se admitirmos que a verdade ou a falsidade dos juízos morais é independente do ponto de


vista individual ou cultural, estaremos a considerar correta que teoria ética?

6 . Quem adota uma atitude etnocêntrica concordará com a tese de que todas as culturas
têm igual valor? Justifica.

7 . Na tua opinião, para combater a intolerância existente nas sociedades multiculturais será
mais eficaz recorrer aos argumentos do subjetivismo, do relativismo ou do objetivismo?
Na resposta deves:
– indicar claramente a posição que defendes;
– explicar os argumentos apresentadas pelo menos por uma das perspetivas;
– justificar a posição defendida.

FIM DO ENUNCIADO DO TESTE

222 Editável e fotocopiável © Texto | Dúvida Metódica, 11.º ano


Avaliação por Competências – Um exemplo: Enunciado do teste e critérios de classificação

Cotações

Cotação
Itens
(em pontos)

1.1 10 pontos
1.2 10 pontos
1.3 10 pontos
1.4 10 pontos
1.5 10 pontos
1.6 10 pontos
2. 10 pontos
3.1 20 pontos
3.2 20 pontos
4.1 20 pontos
4.2 20 pontos
5. 10 pontos
6. 20 pontos
7. 20 pontos

14 itens 200 pontos

Competências avaliadas Itens

A – Análise e interpretação 1.1/1.2/1.3/1.4/2.

B – Rigor concetual e teórico 1.6/3.1/3.2/5.

C – Problematização e 1.5/7.
pensamento crítico

D – Raciocínio lógico e argumentação 4.1/4.2/6.

Editável e fotocopiável © Texto | Dúvida Metódica, 11.º ano 223


Avaliação por Competências – Um exemplo: Enunciado do teste e critérios de classificação

Número de itens, cotação, tipologia dos itens, nível de dificuldade das operações mentais
envolvidas, competências avaliadas e nível de dificuldade previsto

Nível de dificuldade Nível de


Tipologia dos Competências
Itens Cotação das operações dificuldade
itens avaliadas
mentais envolvidas previsto

1.1 10 pontos EM* Nível inferior A Fácil


1.2 10 pontos EM Nível intermédio A Médio
1.3 10 pontos EM Nível intermédio A Médio
1.4 10 pontos EM Nível intermédio A Médio
1.5 10 pontos EM Nível superior C Difícil
1.6 10 pontos EM Nível inferior B Fácil
2. 10 pontos RC Nível intermédio A Médio
3.1 20 pontos RR** Nível intermédio B Médio
3.2 20 pontos RR Nível intermédio B Médio
4.1 20 pontos RR Nível intermédio D Médio
4.2 20 pontos RR Nível intermédio D Médio
5. 10 pontos RC*** Nível inferior D Fácil
6. 20 pontos RR Nível intermédio D Médio
7. 20 pontos RE**** Nível superior C Difícil

14 Total *EM (Escolha Nível inferior – 3


A – Análise e Médio
itens 200 múltipla): 6 Nível intermédio – 8 interpretação
pontos **RR (Resposta Nível superior – 2 B – Rigor concetual e Fácil
restrita): 5 teórico
***RC (Resposta C – Problematização Difícil
curta): 2 e pensamento crítico
****RE D – Raciocínio lógico
(Resposta e argumentação
extensa): 1

224 Editável e fotocopiável © Texto | Dúvida Metódica, 11.º ano


Avaliação por Competências – Um exemplo: Enunciado do teste e critérios de classificação

CRITÉRIOS ESPECÍFICOS DE CLASSIFICAÇÃO


Os alunos podem usar palavras e expressões diferentes das que se encontram nestes critérios
específicos, desde que estas se relacionem com o solicitado nos itens e a resposta se encontre
devidamente fundamentada.

1. ................................................................................................................................................................. 10 = 6  60 pontos
1.1 D
1.2 A
1.3 B
1.4 C
1.5 D
1.6 A
2. .................................................................................................................................................................................... 10 pontos
Cenário de resposta
Indicação de dois dos valores que se seguem (ou outros que se possam relacionar com a
figura): saúde / solidariedade / altruísmo / etc.
3.
3.1 ............................................................................................................................................................................ 20 pontos
Cenário de resposta
Identificação das alíneas que expressam juízos de facto: a) e c).
Justificação: estes exemplos correspondem a juízos de facto porque são descritivos – no
primeiro é-nos dada uma informação sobre um facto (o que a Maria Albertina disse), tal
como no segundo, que descreve uma situação concreta que ocorreu nos EUA.

Descritores do nível de desempenho no domínio


Níveis Pontuação
específico da disciplina

Identifica corretamente ambas as alíneas.


3 Justifica de forma completa, sem imprecisões ou elementos 20
incorretos.

Identifica corretamente ambas as alíneas.


2 Justifica de forma incompleta, com imprecisões ou elementos 10
incorretos.

1 Identifica corretamente ambas as alíneas, mas não justifica. 5

Editável e fotocopiável © Texto | Dúvida Metódica, 11.º ano 225


Avaliação por Competências – Um exemplo: Enunciado do teste e critérios de classificação

3.2 ............................................................................................................................................................................ 20 pontos


Cenário de resposta
Identificação das alíneas que expressam juízos de valor: b) e d).
Justificação: ambos os exemplos expressam uma avaliação, faz-se uma apreciação e
expressa-se uma preferência de natureza política.

Descritores do nível de desempenho no domínio


Níveis Pontuação
específico da disciplina

Identifica corretamente ambas as alíneas.


3 Justifica de forma completa, sem imprecisões ou elementos 20
incorretos.

Identifica corretamente ambas as alíneas.


2 Justifica de forma incompleta, com imprecisões ou elementos 10
incorretos.

1 Identifica corretamente ambas as alíneas, mas não justifica. 5

4.
4.1 ............................................................................................................................................................................ 20 pontos
Cenário de resposta
Reponde sim.
Justificação: um subjetivista consideraria que a premissa justifica a conclusão do argu-
mento, pois defende a tese de que os juízos morais, como «A caça de veados é correta», se
baseiam em sentimentos e preferências pessoais, como «Eu gosto de os caçar».

Descritores do nível de desempenho no domínio


Níveis Pontuação
específico da disciplina

Responde corretamente sim.


3 Justifica de forma completa, sem imprecisões ou elementos 20
incorretos.

Responde corretamente sim.


2 Justifica de forma incompleta, com imprecisões ou elementos 10
incorretos.

1 Responde corretamente sim, mas não justifica. 5

226 Editável e fotocopiável © Texto | Dúvida Metódica, 11.º ano


Avaliação por Competências – Um exemplo: Enunciado do teste e critérios de classificação

4.2 ............................................................................................................................................................................ 20 pontos


Cenário de resposta
Reponde não.
Justificação: um relativista consideraria que não justifica, pois defende a tese de que os
juízos morais, como «A caça de veados é correta», se baseiam nos padrões culturais da
sociedade do agente e não em sentimentos e preferências pessoais, como «Eu gosto de os
caçar».

Descritores do nível de desempenho no domínio


Níveis Pontuação
específico da disciplina

Responde corretamente não.


3 Justifica de forma completa, sem imprecisões ou elementos 20
incorretos.

Responde corretamente não.


2 Justifica de forma incompleta, com imprecisões ou elementos 10
incorretos.

1 Responde corretamente não, mas não justifica. 5

5. .................................................................................................................................................................................... 10 pontos
Responde o objetivismo.

6. .................................................................................................................................................................................... 20 pontos
Cenário de resposta
Reponde não.
Justificação: alguém que adote uma atitude etnocêntrica considera que as culturas não
têm igual valor, existem umas superiores e outras inferiores, e que estas últimas podem
ser avaliadas negativamente (subjugadas, discriminadas ou mesmo exterminadas). Dai que,
em nome de ideias etnocêntricas, se possam defender práticas como a escravatura ou o
racismo.

Descritores do nível de desempenho no domínio


Níveis Pontuação
específico da disciplina

Responde corretamente não.


3 Justifica de forma completa, sem imprecisões ou elementos 20
incorretos.

Responde corretamente não.


2 Justifica de forma incompleta, com imprecisões ou elementos 10
incorretos.

1 Responde corretamente não, mas não justifica. 5

Editável e fotocopiável © Texto | Dúvida Metódica, 11.º ano 227


Avaliação por Competências – Um exemplo: Enunciado do teste e critérios de classificação

7. .................................................................................................................................................................................... 20 pontos
Cenário de resposta
Indicação clara de uma das perspetivas (subjetivismo, relativismo ou objetivismo).
Justificação da posição defendida.
Explicação dos argumentos utilizados por uma das seguintes perspetivas:

Se o aluno pensar que os argumentos do subjetivismo são mais eficazes para combater a
intolerância poderá alegar, nomeadamente, que as ideias do subjetivismo promovem a
liberdade individual e o respeito pelas opiniões alheias, pois incluem a tese de que cada
indivíduo pode ter a sua verdade ética e que nenhum tem mais razão do que outro.
Se o aluno pensar que os argumentos do relativismo são mais eficazes para combater
a intolerância poderá alegar, nomeadamente, que as ideias do relativismo promovem
o respeito pelas crenças e costumes dos outros povos, pois incluem a tese de que cada
sociedade pode ter a sua verdade ética e que nenhuma tem mais razão do que outra.
Se o aluno pensar que os argumentos do objetivismo são mais eficazes para combater
a intolerância poderá alegar, nomeadamente, que as ideias do objetivismo promovem
um genuíno diálogo intercultural, bem como efetivos debates entre os indivíduos, pois
permitem distinguir, imparcialmente, as crenças e os comportamentos que são toleráveis
dos que não são.

Descritores do nível de desempenho no domínio


Níveis Pontuação
específico da disciplina

Apresenta claramente a sua posição.


3 Argumenta a favor da sua posição. 20
A resposta é clara e não contém elementos incorretos.

Apresenta claramente a sua posição.


2 Argumenta a favor da sua posição. 10
A resposta contém imprecisões ou elementos incorretos.

Não refere a sua posição, mas menciona corretamente


1 conteúdos relacionados com o problema OU refere a sua 5
posição, mas não justifica ou justifica incorretamente.

FIM DOS CRITÉRIOS DE CORREÇÃO

228 Editável e fotocopiável © Texto | Dúvida Metódica, 11.º ano


Avaliação por Competências – Um exemplo: Guião de um trabalho de grupo por competências

Guião de um trabalho colaborativo


(destinado a alunos e a professores)
Conteúdo das AE – O problema da natureza dos juízos morais: análise e discussão de tradições
de culturas diferentes

Guião do trabalho a realizar pelos alunos


Analisa, atentamente, os textos, as fotos e o vídeo que se encontram nos links seguintes e rea-
liza as tarefas apresentadas.

Notícia A (Público) Notícia B (Visão)


«A fotógrafa egípcia Somaya «Em várias regiões do Paquistão,
Abdelrahman tinha 10 anos o casamento entre crianças é ainda
quando foi submetida ao corte. comum. Milhares de meninas são
No Dia Internacional da Tolerância obrigadas a casar para resolver
Zero à Mutilação Genital problemas entre famílias. Agora,
Feminina, partilha a sua história e apresentamos a história de Saima,
ĺŔŗĺģøťĺǥ…øŗİÑıøıť°ĺūıñǦLj uma menina de 13 anos a quem o pai
em que retrata outras vítimas escolheu um noivo de 36.»
deste procedimento, no Egipto.»

Tarefas

1 . Explica, por palavras tuas, em que consistem as tradições das notícias A e B (da mutilação
genital feminina no Egipto e do casamento das meninas no Paquistão) e os motivos que
levam muitas pessoas a aceitá-las.

2 . Identifica um valor e uma norma moral em que se baseia a tradição


A. da mutilação genital feminina no Egipto;
B. do casamento das meninas no Paquistão.

3 . Constrói um juízo de facto e um juízo de valor que se relacionem com as tradições


descritas nas duas notícias.

4 . Como avaliaria um defensor do relativismo estas duas tradições? Que crítica lhe poderia
fazer quem discordasse dele?

5 . Como avaliaria um defensor do objetivismo estas duas tradições? Que objeção lhe poderia
colocar um relativista cultural?

6 . Na tua opinião, os juízos morais expressos nas frases: «A prática da mutilação genital
feminina é moralmente errada» e «O casamento de meninas no Paquistão é uma tradição
condenável do ponto de vista ético» serão verdadeiros ou falsos?
Justifica.

Editável e fotocopiável © Texto | Dúvida Metódica, 11.º ano 229


Avaliação por Competências – Um exemplo: Guião de um trabalho de grupo por competências

7 . Compara as tradições culturais relacionadas com o casamento nestes dois países (Egipto e
Paquistão) com as existentes em Portugal.
7.1 Que conclusões podes tirar quanto à questão da igualdade de género?
7.2 Como é que, na tua opinião, se pode mudar a situação das mulheres nesses dois países
e em Portugal?

8 . Debate, no grupo-turma, o seguinte problema – as tarefas de organização do debate


ficaram a cargo dos alunos, sob a orientação do(a) professor(a):
Como se devem encarar tradições culturais diferentes? Poder-se-á justificar ou não a
rejeição de tradições baseadas na discriminação das mulheres?

Orientações / informações dadas aos alunos


1 . Objetivos do trabalho (o que se pretende que tu faças)
• Seleciona informações dos vários suportes (notícias / fotos / vídeos indicados pela
professora), identificando as que são relevantes para o tema em estudo.
• Analisa e sintetiza informações, apresentando-as por palavras tuas.
• Aplica conceitos, argumentos e objeções das várias perspetivas filosóficas estudadas na
análise dos dois problemas referidos nas notícias.
• Discute a relatividade ou a objetividade dos juízos morais, as suas implicações
filosóficas e práticas.
• Debate (no grupo-turma) os problemas da natureza dos juízos morais.
• Analisa criticamente, aplicando os conhecimentos filosóficos adquiridos, as soluções
apresentadas para problemas atuais, propondo alternativas.
• Defende uma posição pessoal em relação ao problema da natureza dos juízos morais.
2 . Procedimentos a adotar ao longo da realização do trabalho
• Antes de realizares as tarefas propostas, revê com os teus colegas de grupo as noções
de valor, juízo de valor, juízo de facto, juízo moral e as teses, argumentos e objeções às
três teorias que estudaste.
• Ao longo do trabalho, esclarece as tuas dúvidas com os teus colegas e com o(a)
professor(a) (na aula, no Google Meet ou no Google Drive, inserindo comentários num
documento do Google Docs).
• Disponibiliza, no Google Drive a resolução das Tarefas (até à questão 7), utilizando o
Google Docs.
• Tem em conta os comentários / sugestões / correções que o(a) professor(a) faz e
introduz alterações que te permitam melhorar as respostas dadas.
• Coloca as respostas às Tarefas (até à questão 7) num suporte digital à tua escolha (por
exemplo: o Google Slides, Prezi, Canva, Padlet, SlideDog, entre outros) e constrói uma
apresentação com os teus colegas.
• Prepara e treina com os teus colegas de grupo a apresentação oral das Tarefas e, ao
fazê-la, procura responder às questões colocadas pelos outros colegas de turma e pela
professora.
• Durante o debate (na turma, em grande grupo), reflete sobre as tuas próprias ideias,
confrontando-as com pontos de vista diferentes dos teus e procurando perceber quem
apresenta as melhores razões para justificar as várias posições acerca dos problemas em
discussão.

230 Editável e fotocopiável © Texto | Dúvida Metódica, 11.º ano


Avaliação por Competências – Um exemplo: Guião de um trabalho de grupo por competências

Competências a desenvolver neste trabalho


1 . Competências específicas de Filosofia (das Aprendizagens Essenciais)
• Identificar problemas filosóficos, distinguindo-os dos não filosóficos e justificando a sua
relevância.
• Esclarecer conceitos filosóficos através da sua exemplificação.
• Relacionar os conceitos filosóficos nucleares, permitindo a compreensão dos
problemas, teses e argumentos das várias teorias.
• Defender teses, teorias e argumentos filosóficos.
• Determinar as implicações filosóficas e práticas de uma tese ou teoria.
• Avaliar as teorias e os argumentos, apresentando objeções e contraexemplos.
• Discutir as teses, argumentos e contra-argumentos das várias perspetivas filosóficas.
• Analisar criticamente as soluções apresentadas para problemas atuais, aplicando os
conhecimentos filosóficos adquiridos e propondo alternativas.
• Defender posições pessoais, com clareza e rigor, a propósito das teorias filosóficas
estudadas.
2 . Competências transversais
• Pesquisar (em grupo), em meios diversificados, informação relevante, manifestando
sentido crítico na seleção adequada de contributos.
• Aquisição de informação e conhecimento.
• Envolvimento em tarefas de síntese, de planificação, de revisão, de monitorização e na
promoção do estudo autónomo.
• Respeito por diferenças de características, crenças ou opiniões.
• Interrogar-se sobre o seu próprio conhecimento prévio, saber questionar uma situação e
organizar questões para terceiros.
• Aprofundamento do pensamento crítico e analítico.
• Desenvolvimento da criatividade.
• Identificação dos pontos fracos e fortes das suas aprendizagens, a consideração do
retorno dos pares e dos professores para melhoria ou aprofundamento de saberes.
• Colaboração com outros, apoio a terceiros, melhoria ou aprofundamento das suas
ações.
• Desenvolvimento da consciência da cidadania e da necessidade de intervenção crítica
em diversos contextos e espaços.
3 . Competências do Perfil dos Alunos
A. Linguagens e Textos; B. Informação e Comunicação; C. Raciocínio e Resolução de
Problemas; D. Pensamento Crítico e Pensamento Criativo; E. Relacionamento Interpessoal;
F. Desenvolvimento Pessoal e Autonomia; G. Bem-estar e Saúde; H. Sensibilidade Estética e
Artística; I. Saber Científico, Técnico e Tecnológico.

Meios utilizados na recolha de dados


Grelha de registo da observação direta (elaborada no decurso da realização das tarefas nas
aulas presenciais ou online no Google Meet).
Resolução das tarefas (até à questão 7) disponibilizada no Google Docs e os comentários inse-
ridos no texto escrito (perguntas dos alunos e respostas dos alunos às observações / correções
/ sugestões da professora).

Editável e fotocopiável © Texto | Dúvida Metódica, 11.º ano 231


Avaliação por Competências – Um exemplo: Guião de um trabalho de grupo por competências

Apresentação escrita em suporte digital (a definir pelos alunos).


Apresentação oral realizada pelos alunos das tarefas (até à questão 7).
Grelha de registo da observação direta da participação dos alunos no debate.
Questionário (recorrendo aos formulários do Google Drive, de autoavaliação e heteroavaliação
do trabalho de grupo realizado) preenchido pelos alunos.

Tipo de retorno dado aos alunos pelo(a) professor(a)


O feedback acerca do desempenho é dado oralmente (e registado numa grelha online).
No feed up e feed forward é utilizada a Drive e o Google Docs (inserção, sob a forma de comen-
tário escrito em certas partes do texto dos alunos, de observações, correções e sugestões de
melhoria).

Avaliação dos dados recolhidos / Avaliação formativa


Os alunos serão informados, antes do início do trabalho, dos conhecimentos, capacidades e
atitudes em que incidirá a avaliação formativa. Esta basear-se-á na seguinte informação (reco-
lhida ao longo da realização do trabalho):
• observação e registo das atitudes/comportamentos evidenciados pelos alunos durante a
realização das tarefas;
• intervenções dos alunos, colocando dúvidas e questões (oralmente ou por escrito) ao(à)
professor(a);
• resolução escrita das tarefas no Google Docs;
• reformulações ou melhorias resultantes dos comentários / sugestões que o(a) professor(a)
fez no decurso do trabalho.

Avaliação sumativa
A avaliação sumativa basear-se-á na informação que se segue:
• componente escrita: resolução das tarefas (até à questão 7) apresentada num suporte
digital (à escolha dos alunos).
• desempenho oral: na apresentação oral do trabalho (até à questão 7) e a qualidade das
intervenções dos alunos no debate (na turma em grande grupo).

Nota:
Das orientações fornecidas aos alunos, antes do início do trabalho, fará parte uma informação
acerca do tipo de avaliação (formativa e sumativa) e dos critérios de avaliação específicos,
nomeadamente da componente oral e escrita. Nestas irão ser tidos em consideração aspetos
como, por exemplo, a adequação e correção dos conteúdos filosóficos; a capacidade de análise
e de síntese; a aplicação de conceitos, argumentos e teorias filosóficas à discussão dos proble-
mas; o pensamento crítico; a criatividade; a utilização de terminologia filosófica adequada; a
estruturação, articulação e clareza do discurso (escrito ou oral), entre outros.
Na avaliação dos alunos aplicar-se-ão os critérios gerais de avaliação aprovados na escola /
/ agrupamento e no grupo disciplinar de Filosofia.

232 Editável e fotocopiável © Texto | Dúvida Metódica, 11.º ano


TEMAS/PROBLEMAS ALTERNATIVOS:
GUERRA JUSTA E TECNOCIÊNCIA
disponíveis em
PLANIFICAÇÕES (anual e por tema) disponíveis em
Bibliografia / Webgrafia

Bibliografia do projeto
A. C. Grayling, As Fronteiras do Conhecimento, Filosofia – Uma Introdução por Disciplinas,
O Que Sabemos Hoje Sobre Ciência, Edições Org. Pedro Galvão, Edições 70, 2012
70, 2021 Fiódor Dostoiévski, Os Irmãos Karamázov,
A. J. Ayer, Linguagem, Verdade e Lógica, Editorial Presença, 2002
Editorial Presença, 1991 Francis Bacon, Nova Atlântida, A Grande
Aires Almeida e Desidério Murcho, Janelas Instauração, Edições 70, 2008
para a Filosofia, Gradiva, 2014 George Dickie, Introdução à Estética, Bizâncio,
Aires Almeida e Desidério Murcho, Textos e 2008
Problemas de Filosofia, Plátano, 2006 Gottfried Leibniz, Discurso de Metafísica,
Aires Almeida, O Que É a Arte – O Essencial, Edições 70, 1995
Plátano, 2019 Hélade, Antologia da Cultura Grega, Faculdade
Albert Einstein e Leopold Infield, A Evolução de Letras da Universidade de Coimbra, 1990
da Física, De Newton até à Teoria dos Quanta, Henry Gleitman e outros, Psicologia, F. C.
Edição Livros do Brasil, s/d Gulbenkian, 2009
Aristóteles, Poética, F. C. Gulbenkian, 2018 Hilary Putnam, Razão, Verdade e História,
Bryan Magee, Os Grandes Filósofos, Presença, Publicações Dom Quixote, 1992
1989 Immanuel Kant, Fundamentação da Metafísica
Carl Cohen, Os Animais Têm Direitos? – dos Costumes, Edições 70, 2014
Perspetivas e Argumentos, Dinalivro, 2011 Jerrold Levinson, Investigações Estéticas
Carl Sagan, Um Mundo Infestado de Demónios, – Ensaios de Filosofia da Arte, Edições
Gradiva, 1997 Afrontamento, 2020
Carlo Rovelli, A Realidade Não É o Que Parece, John Locke, Ensaio sobre o Entendimento
Contraponto, 2019 Humano, F. C. Gulbenkian, 1999
Clive Bell, Arte, Edições Texto & Grafia, 2009 Jorge Buescu, O Mistério do Bilhete de
Dan O'Brien, Introdução à Teoria do Identidade e Outras Histórias, Gradiva, 2014
Conhecimento, Gradiva, 2013 Jorge Dias de Deus, Ciência, Curiosidade e
David Hume, Diálogos Sobre Religião Natural, Maldição, Gradiva, 1986
Edições 70, 2005 Karl Popper, A Lógica da Pesquisa Científica,
David Hume, Tratado da Natureza Humana, Cultrix, 1974
Fundação Calouste Gulbenkian, 2012 Karl Popper, A Vida É Aprendizagem, Edições
David Hume, Tratados Filosóficos I – 70, 2020
Investigação sobre o Entendimento Humano, Karl Popper, Conjeturas e Refutações,
INCM, 2002 Almedina, 2003
David Marçal, Pseudociência, Fundação Lisa Bortolotti, Introdução à Filosofia da
Francisco Manuel dos Santos, 2014 Ciência, Gradiva, 2008
Desidério Murcho, 7 Ideias Filosóficas Que Matt Ridley, Genoma, Gradiva, 2001
Toda a Gente Devia Conhecer, Bizâncio, 2011 Nigel Warburton, Elementos Básicos de
Desidério Murcho, A Existência de Deus – Filosofia, Gradiva, 1998
O Essencial, Plátano, 2020 Nigel Warburton, O Que É a Arte?, Bizâncio,
Dicionário Escolar de Filosofia, Org. Aires 2007
Almeida, Plátano, 2009, Peter Singer, Ética Prática, Gradiva, 2000
Eric Chaline, Dilemas para o Filósofo de
Bancada, Jacarandá Editora, 2020

© Texto | Dúvida Metódica, 11.º ano 237


Bibliografia / Webgrafia

Rebecca Goldstein, Incompletude – A Simon Blackburn, Dicionário de Filosofia,


Demonstração e o Paradoxo de Kurt Gödel, Gradiva, 1997
Gradiva, 2009 Simon Blackburn, Pense – Uma Introdução à
René Descartes, «Les Principes de la Filosofia, Gradiva, 2001
Philosophie», in Oeuvres de Descartes IX, Vrin, Steven Nadler e Ben Nadler, Hereges!, Gradiva,
1996 2018
René Descartes, Discurso do Método, Edições Thomas S. Kuhn, A Estrutura das Revoluções
70, 1991 Científicas, Guerra & Paz, 2009
René Descartes, Meditações sobre a Filosofia Timothy Williamson, Filosofar, Da Curiosidade
Primeira, Livraria Almedina, 1985 Comum ao Raciocínio Lógico, Gradiva, 2019
René Descartes, Méditations Métaphysiques, Tom Regan, Os Animais Têm Direitos? –
Flammarion, 1979 Perspetivas e Argumentos, Dinalivro, 2011
René Descartes, Princípios da Filosofia, Edições Tomás de Aquino, «Suma Teológica», Escola
70, 2018 Superior de Teologia São Lourenço dos
Richard Swinburne, Será que Deus Existe?, Brindes, Universidade de Caxias do Sul e outras
Gradiva, 1998 instituições, 1980
Rudolf Carnap, An Introduction to the Vítor Moura, Arte em Teoria – Uma Antologia
Philosophy of Science, Dover, 1995 de Estética, Edições Húmus, 2009
Shusaku Endo, Silêncio, Dom Quixote, 2019 William L. Rowe, Introdução à Filosofia da
Religião, Verbo, 2011

Webgrafia do projeto
Os endereços abaixo foram consultados em abril de 2022 e os Autores e a Editora não se
responsabilizam sobre a alteração dos seus conteúdos em data posterior à indicada.
45 Graus Crítica
https://quarentaecincograus.libsyn.com/ https://criticanarede.com/

8-Bit Philosophy Dicionário de Filosofia Moral e Política


https://www.wisecrack.co/8-bit-philosophy http://www.dicionariofmp-ifilnova.pt/

Antony Gormley Dicionário Escolar de Filosofia


https://www.antonygormley.com/ https://criticanarede.com/dicionario.htm

Art Works for Change diferencial


https://www.artworksforchange.org/ https://diferencial.tecnico.ulisboa.pt/

Associação de Professores de Filosofia Documania TV


https://apfilosofia.org/ https://www.documaniatv.com/

Ciência Viva Dúvida Metódica


https://www.cienciaviva.pt/ http://duvida-metodica.blogspot.com/

238 © Texto | Dúvida Metódica, 11.º ano


Bibliografia / Webgrafia

ESA Portal do astrónomo


https://esamultimedia.esa.int/ https://portaldoastronomo.org/

Humane Society International Público


https://www.hsi.org/news-media/about/ https://www.publico.pt/

Internet Encyclopedia of Philosophy RTP Ensina


https://iep.utm.edu/ https://ensina.rtp.pt/

Jornal de Filosofia Stanford Encyclopedia of Philosophy


jornaldefilosofia-diriodeaula.blogspot.com/ https://plato.stanford.edu/

Lockwood Film TED-ED


https://lockwoodfilm.com/ https://ed.ted.com/

LOGOSFERA Termos de Arte – TATE


http://filosofialogos.blogspot.com/ https://www.tate.org.uk/art/art-terms

LPDA VIMEO
https://www.lpda.pt/ https://vimeo.com/

MULT Visão
https://blog.multcomercial.com.br/ https://visao.sapo.pt/

No Jardim da Filosofia Wikipédia


https://blog.criticanarede.com/2015/03/no- https://pt.wikipedia.org/
jardim-da-filosofia.html

Wiley Online Library


Páginas de Filosofia https://onlinelibrary.wiley.com
http://www.paginasdefilosofia.net/

© Texto | Dúvida Metódica, 11.º ano 239

Você também pode gostar